You are on page 1of 89

1

vii. Issuance of Decree


1. When judgment becomes final

G.R. No. 77770. December 15, 1988.
ATTY. JOSE S. GOMEZ, DELFINA GOMEZ ESTRADA, ENRIQUITA GOMEZ OXCIANO,
BENITA GOMEZ GARLITOS, REYNALDO GOMEZ ESPEJO, ARMANDO GOMEZ,
ERLINDA GOMEZ GUICO, EUGENIA GOMEZ CALICDAN, AZUCENA GOMEZ
ORENCIA, TEODORO S. GOMEZ, JR., and ALEJO S. GOMEZ (now deceased)
represented by his wife, LETICIA Y. GOMEZ, and children, namely, MARGIE
GOMEZ GOB, JACINTO Y. GOMEZ, ALEJO Y. GOMEZ, JR., and MARY ANN Y.
GOMEZ, petitioners, vs. HON. COURT OF APPEALS, HON. PEDRO G. ADUCAYEN,
Judge Regional Trial Court, San Carlos City (Pangasinan) Branch LVI, HON.
CHIEF, LAND REGISTRATION COMMISSION, Quezon City, Metro Manila, and
SILVERIO G. PEREZ, Chief, Division of Original Registration, Land Registration
Commission, Quezon City Metro Manila
D E C I S I O N
PADILLA, J p:
The present case originated with the filing by petitioners on 30 August 1968 in the Court of First Instance
(now Regional Trial Court) of San Carlos City, Pangasinan, of an application for registration of several lots
situated in Bayambang, Pangasinan. Cdpr
The lots applied for were Lots Nos. 1, 2, 3, 4, 5, 6, 7, 8, 9,10, 11 and 12 of Plan Psu-54792 Amd.-2. The
lots were among those involved in the case of Government of the Philippine Islands vs. Abran, 1 wherein
this Court declared Consolacion M. Gomez owner of certain lots in Sitio Poponto, Bayambang,
Pangasinan. Petitioners are the heirs of Teodoro Y. Gomez (father of Consolacion) who, together with
Consolacion's son, Luis Lopez, inherited from her parcels of land when Consolacion Gomez died intestate.
Petitioners alleged that after the death of Teodoro Y. Gomez, they became the absolute owners of the
subject lots by virtue of a Quitclaim executed in their favor by Luis Lopez. The lots (formerly portions of
Lots 15, 16, 34 and 41 covered by Plan Ipd-92) were subdivided into twelve lots Lots Nos. 1, 2, 3, 4,
5, 6, 7, 8, 9, 10, 11 and 12. The subdivision plan was duly approved by the Bureau of Lands on 30
November 1963. Petitioners agreed to allocate the lots among themselves.
After notice and publication, and there being no opposition to the application, the trial court issued an
order of general default. On 5 August 1981, the court rendered its decision adjudicating the subject lots
in petitioners' favor. 2
On 6 October 1981, the trial court issued an order 3 expressly stating that the decision of 5 August 1981
had become final and directed the Chief of the General Land Registration Office to issue the
corresponding decrees of registration over the lots adjudicated in the decision of 5 August 1981.
On 11 July 1984, respondent Silverio G. Perez, Chief of the Division of Original Registration, Land
Registration Commission (now known as the National Land Titles and Deeds Registration Administration),
submitted a report to the court a quo stating that Lots 15, 16, 34 and 41 of Ipd-92 were already covered
by homestead patents issued in 1928 and 1929 and registered under the Land Registration Act. He
recommended that the decision of 5 August 1981, and the order of 6 October 1981 be set aside.
Petitioners opposed the report, pointing out that no opposition was raised by the Bureau of Lands during
the registration proceedings and that the decision of 5 August 1981 should be implemented because it
had long become final and executory.
After hearing, the lower court rendered a second decision on 25 March 1985 setting aside the decision
dated 5 August 1981 and the order dated 6 October 1981 for the issuance of decrees. 4 Petitioners
moved for reconsideration but the motion was denied by respondent judge on 6 August 1985 for lack of
merit. 5
Petitioners filed a petition for certiorari and mandamus with this Court which in turn referred the petition
to the Court of Appeals. 6
On 17 September 1986, the appellate court rendered judgment, 7 dismissing the petition and stating,
among others, thus
"In resum, prior to the issuance of the decree of registration, the 138
respondent Judge has still the power and control over the decision he
rendered. The finality of an adjudication of land in a registration or
cadastral case takes place only after the expiration of the one-year period
after entry of the final decree of registration (Afalla vs. Rosauro, 60 Phil.
622; Valmonte vs. Nable, 85 Phil. 256; Capio vs. Capio, 94 Phil. 113).
When the respondent Judge amended his decision after the report of the
respondent officials of the Land Registration office had shown that
homestead patents had already been issued on some of the lots,
respondents cannot be faulted because land already granted by
homestead patent can no longer be the subject of another registration
(Manalo vs. Lukban, et al., 48 Phil. 973).
"WHEREFORE, in view of the foregoing, We resolve to DISMISS the
petition for lack of merit.
"SO ORDERED."
Petitioners' motion for reconsideration was denied by the appellate court in its Resolution dated 10 March
1987. 8 Hence, this recourse.
Several issues are raised by petitioners in this petition. The more important issues before the Court are:
(a) whether or not respondent Judge had jurisdiction to issue the decision of 26 March 1985 which set
aside the lower court's earlier decision of 5 August 1981 and the order of 6 October 1981; (b) whether or
not the respondents Acting Land Registration Commissioner and Engr. Silverio Perez, Chief, Division of
Original Registration, Land Registration Commission, have no alternative but to issue the decrees of
registration pursuant to the decision of 5 August 1981 and the order for issuance of decrees, dated 6
October 1981, their duty to do so being purely ministerial; (c) whether or not "the law of the case" is the
decision in Government of the Philippine Islands v. Abran, supra, which held that the lands adjudicated to
Consolacion Gomez were not public lands, and therefore they could not have been acquired by holders of
homestead titles as against petitioners herein.

It is not disputed that the decision dated 5 August 1981 had become final and executory. Petitioners
vigorously maintain that said decision having become final, it may no longer be reopened, reviewed,
much less set aside. They anchor this claim on section 30 of P.D. No. 1529 (Property Registration
Decree) which provides that, after judgment has become final and executory, the court shall forthwith
issue an order to the Commissioner of Land Registration for the issuance of the decree of registration
and certificate of title. Petitioners contend that section 30 should be read in relation to section 32 of P. D.
1529 in that, once the judgment becomes final and executory under section 30, the decree of registration
must issue as a matter of course. This being the law, petitioners assert, when respondent Judge set aside
in his decision, dated 25 March 1985, the decision of 5 August 1981 and the order of 6 October 1981 he
clearly acted without jurisdiction.
Petitioners' contention is not correct. Unlike ordinary civil actions, the adjudication of land in a cadastral
2
or land registration proceeding does not become final, in the sense of incontrovertibility until after the
expiration of one (1) year after the entry of the final decree of registration. 9 This Court, in several
decisions, has held that as long as a final decree has not been entered by the Land Registration
Commission (now NLTDRA) and the period of one (1) year has not elapsed from date of entry of such
decree, the title is not finally adjudicated and the decision in the registration proceeding continues to be
under the control and sound discretion of the court rendering it. 10
Petitioners contend that the report of respondent Silverio Perez should have been submitted to the court
a quo before its decision became final. But were we to sustain this argument, we would be pressuring
respondent land registration officials to submit a report or study even if haphazardly prepared just to
beat the reglementary deadline for the finality of the court decision. As said by this Court in De los Reyes
vs. de Villa: 11
"Examining section 40, we find that the decrees of registration must be
stated in convenient form for transcription upon the certificate of title and
must contain an accurate technical description of the land. This requires
technical men. Moreover, it frequently occurs that only portions of a parcel
of land included in an application are ordered registered and that the limits
of such portions can only be roughly indicated in the decision of the court.
In such cases amendments of the plans and sometimes additional surveys
become necessary before the final decree can be entered. That can hardly
be done by the court itself; the law very wisely charges the Chief Surveyor
of the General Land Registration Office with such duties (Administrative
Code, section 177)."
Thus, the duty of respondent land registration officials to render reports is not limited to the
period before the court's decision becomes final, but may extend even after its finality but not
beyond the lapse of one (1) year from the entry of the decree. LLpr
Petitioners insist that the duty of the respondent land registration officials to issue the decree is purely
ministerial. It is ministerial in the sense that they act under the orders of the court and the decree must
be in conformity with the decision of the court and with the data found in the record, and they have no
discretion in the matter. However, if they are in doubt upon any point in relation to the preparation and
issuance of the decree, it is their duty to refer the matter to the court. They act, in this respect, as
officials of the court and not as administrative officials, and their act is the act of the court. 12 They are
specifically called upon to "extend assistance to courts in ordinary and cadastral land registration
proceedings." 13
The foregoing observations resolve the first two (2) issues raised by petitioners.
Petitioners next contend that "the law of the case" is found in Government of the Philippine Islands vs.
Abran, et al., supra, where it was decided by this Court that the lands of Consolacion M. Gomez, from
whom petitioners derive their ownership over the lots in question, were not public lands. A reading of the
pertinent and dispositive portions of the aforesaid decision will show, however, that the lots earlier
covered by homestead patents were not included among the lands adjudicated to Consolacion M. Gomez.
The decision states:
"With respect to the portions of land covered by homestead certificates of
title, we are of opinion that such certificates are sufficient to prevent the
title to such portion from going to appellants aforesaid, for they carry with
them preponderating evidence that the respective homesteaders held
adverse possession of such portions, dating back to 1919 or 1920,
accordingly to the evidence, and the said appellants failed to object to that
possession in time." (Emphasis supplied)
"Wherefore, modifying the judgment appealed from, it is hereby ordered
that the lots respectively claimed by Agustin V. Gomez, Consolacion M.
Gomez, and Julian Macaraeg, be registered in their name, with the
exclusion of the portions covered by the homestead certificates . . ."
(Emphasis supplied.) 14
The report of respondent land registration officials states that the holders of the homestead
patents registered the lots in question in the years 1928 and 1929. The decision in Government of
the Philippine Islands vs. Abran was promulgated on 31 December 1931. Hence, the subject lots
are specifically excluded from those adjudicated by the aforesaid decision to Consolacion M.
Gomez. prLL
It is a settled rule that a homestead patent, once registered under the Land Registration Act, becomes
indefeasible and incontrovertible as a Torrens title, and may no longer be the subject of an investigation
for determination or judgment in cadastral proceeding. 15
The aforecited case of Government vs. Abran, therefore, is not "the law of the case", for the lots in
question were not private lands of Consolacion M. Gomez when homestead patents were issued over
them in 1928-1929. There is sufficient proof to show that Lots 15, 16, 34 and 41 of Ipd-92 were already
titled lands way back in 1928 and 1929 as shown by Annexes "A, "B", "C" and "D" of respondents'
Memorandum. 16
Lastly, petitioners claim that if the decision of 5 August 1981 of the lower court is sustained, the
homestead title holders may still vindicate their rights by filing a separate civil action for cancellation of
titles and for reconveyance in a court of ordinary civil jurisdiction. Conversely, the same recourse may be
resorted to by petitioners. "(T)he true owner may bring an action to have the ownership or title to land
judicially settled, and if the allegations of the plaintiff that he is the true owner of the parcel of land
granted as free patent and described in the Torrens title and that the defendant and his predecessor-in-
interest were never in possession of the parcel of land and knew that the plaintiff and his predecessor-in-
interest have been in possession thereof be established, then the court in the exercise of its equity
jurisdiction, without ordering the cancellation of the Torrens title issued upon the patent, may direct the
defendant, the registered owner, to reconvey the parcel of land to the plaintiff who has been found to be
the true owner thereof." 17
WHEREFORE, the petition is DENIED. The appealed decision of the Court of Appeals is AFFIRMED. Costs
against the petitioners-appellants.
SO ORDERED.


























3











G.R. No. 159595. January 23, 2007.
REPUBLIC OF THE PHILIPPINES, petitioner, vs. LOURDES ABIERA NILLAS,
respondent.
D E C I S I O N
TINGA, J p:
The central question raised in this Petition for Review is whether prescription or laches may bar a petition
to revive a judgment in a land registration case. It is a hardly novel issue, yet petitioner Republic of the
Philippines (Republic) pleads that the Court rule in a manner that would unsettle precedent. We deny
certiorari and instead affirm the assailed rulings of the courts below.
The facts bear little elaboration. On 10 April 1997, respondent Lourdes Abiera Nillas (Nillas) filed a
Petition for Revival of Judgment with the Regional Trial Court (RTC) of Dumaguete City. It was alleged
therein that on 17 July 1941, the then Court of First Instance (CFI) of Negros Oriental rendered a
Decision Adicional in Expediente Cadastral No. 14, captioned as El Director De Terrenos contra Esteban
Abingayan y Otros. 1 In the decision, the CFI, acting as a cadastral court, adjudicated several lots,
together with the improvements thereon, in favor of named oppositors who had established their title to
their respective lots and their continuous possession thereof since time immemorial and ordered the Chief
of the General Land Registration Office, upon the finality of the decision, to issue the corresponding
decree of registration. 2 Among these lots was Lot No. 771 of the Sibulan Cadastre, which was
adjudicated to Eugenia Calingacion (married to Fausto Estoras) and Engracia Calingacion, both residents
of Sibulan, Negros Oriental. 3
Nillas further alleged that her parents, Serapion and Josefina A. Abierra, eventually acquired Lot No. 771
in its entirety. By way of a Deed of Absolute Sale dated 7 November 1977, Engracia Calingacion sold her
undivided one-half (1/2) share over Lot No. 771 to the Spouses Abierra, the parents of Nillas. On the
other hand, the one-half (1/2) share adjudicated to Eugenia Calingacion was also acquired by the
Spouses Abierra through various purchases they effected from the heirs of Eugenia between the years
1975 to 1982. These purchases were evidenced by three separate Deeds of Absolute Sale all in favor of
the Spouses Abierra. 4
In turn, Nillas acquired Lot No. 771 from her parents through a Deed of Quitclaim dated 30 June 1994.
Despite these multiple transfers, and the fact that the Abierra spouses have been in open and continuous
possession of the subject property since the 1977 sale, no decree of registration has ever been issued
over Lot No. 771 despite the rendition of the 1941 CFI Decision. Thus, Nillas sought the revival of the
1941 Decision and the issuance of the corresponding decree of registration for Lot No. 771. The records
do not precisely reveal why the decree was not issued by the Director of Lands, though it does not
escape attention that the 1941 Decision was rendered a few months before the commencement of the
Japanese invasion of the Philippines in December of 1941. ESCacI
No responsive pleading was filed by the Office of the Solicitor General (OSG), although it entered its
appearance on 13 May 1997 and simultaneously deputized the City Prosecutor of Dumaguete City to
appear whenever the case was set for hearing and in all subsequent proceedings. 5
Trial on the merits ensued. The RTC heard the testimony of Nillas and received her documentary
evidence. No evidence was apparently presented by the OSG. On 26 April 2000, the RTC rendered a
Decision 6 finding merit in the petition for revival of judgment, and ordering the revival of the 1941
Decision, as well as directing the Commissioner of the Land Registration Authority (LRA) to issue the
corresponding decree of confirmation and registration based on the 1941 Decision.
The OSG appealed the RTC Decision to the Court of Appeals, arguing in main that the right of action to
revive judgment had already prescribed. The OSG further argued that at the very least, Nillas should
have established that a request for issuance of a decree of registration before the Administrator of the
LRA had been duly made. The appeal was denied by the appellate court in its Decision 7 dated 24 July
2003. In its Decision, the Court of Appeals reiterated that the provisions of Section 6, Rule 39 of the
Rules of Court, which impose a prescriptive period for enforcement of judgments by motion, refer to
ordinary civil actions and not to "special" proceedings such as land registration cases. The Court of
Appeals also noted that it would have been especially onerous to require Nillas to first request the LRA to
comply with the 1941 decision considering that it had been established that the original records in the
1941 case had already been destroyed and could no longer be reconstructed.
In the present petition, the OSG strongly argues that contrary to the opinion of the Court of Appeals, the
principles of prescription and laches do apply to land registration cases. The OSG notes that Article 1144
of the Civil Code establishes that an action upon judgment must be brought within ten years from the
time the right of action accrues. 8 Further, Section 6 of Rule 39 of the 1997 Rules of Civil Procedure
establishes that a final and executory judgment or order may be executed on motion within five (5) years
from the date of its entry, after which time it may be enforced by action before it is barred by statute of
limitations. 9 It bears noting that the Republic does not challenge the authenticity of the 1941 Decision,
or Nillas's acquisition of the rights of the original awardees. Neither does it seek to establish that the
property is inalienable or otherwise still belonged to the State.
The OSG also extensively relies on two cases, Shipside Inc. v. Court of Appeals 10 and Heirs of Lopez v.
De Castro. 11 Shipside was cited since in that case, the Court dismissed the action instituted by the
Government seeking the revival of judgment that declared a title null and void because the judgment
sought to be revived had become final more than 25 years before the action for revival was filed. In
Shipside, the Court relied on Article 1144 of the Civil Code and Section 6, Rule 39 of the 1997 Rules of
Civil Procedure in declaring that extinctive prescription did lie. On the other hand, Heirs of Lopez involved
the double registration of the same parcel of land, and the subsequent action by one set of applicants for
the issuance of the decree of registration in their favor seven (7) years after the judgment had become
final. The Court dismissed the subsequent action, holding that laches had set in, it in view of the
petitioners' omission to assert a right for nearly seven (7) years. cEDIAa
Despite the invocation by the OSG of these two cases, there exists a more general but definite
jurisprudential rule that favors Nillas and bolsters the rulings of the lower courts. The rule is that "neither
laches nor the statute of limitations applies to a decision in a land registration case." 12
The most extensive explanation of this rule may be found in Sta. Ana v. Menla, 13 decided in 1961,
wherein the Court refuted an argument that a decision rendered in a land registration case wherein the
decree of registration remained unissued after 26 years was already "final and enforceable." The Court,
through Justice Labrador, explained:
We fail to understand the arguments of the appellant in support of the
assignment [of error], except insofar as it supports his theory that after a
decision in a land registration case has become final, it may not be
enforced after the lapse of a period of 10 years, except by another
proceeding to enforce the judgment or decision. Authority for this theory is
the provision in the Rules of Court to the effect that judgment may be
enforced within 5 years by motion, and after five years but within 10
years, by an action (Sec. 6, Rule 39). This provision of the Rules
refers to civil actions and is not applicable to special proceedings,
4
such as a land registration case. This is so because a party in a
civil action must immediately enforce a judgment that is secured
as against the adverse party, and his failure to act to enforce the
same within a reasonable time as provided in the Rules makes
the decision unenforceable against the losing party. In special
proceedings[,] the purpose is to establish a status, condition or
fact; in land registration proceedings, the ownership by a person
of a parcel of land is sought to be established. After the
ownership has been proved and confirmed by judicial declaration,
no further proceeding to enforce said ownership is necessary,
except when the adverse or losing party had been in possession
of the land and the winning party desires to oust him therefrom.
Furthermore, there is no provision in the Land Registration Act similar to
Sec. 6, Rule 39, regarding the execution of a judgment in a civil action,
except the proceedings to place the winner in possession by virtue of a
writ of possession. The decision in a land registration case, unless the
adverse or losing party is in possession, becomes final without any further
action, upon the expiration of the period for perfecting an appeal. . . .
. . . There is nothing in the law that limits the period within which
the court may order or issue a decree. The reason is . . . that the
judgment is merely declaratory in character and does not need to
be asserted or enforced against the adverse party. Furthermore,
the issuance of a decree is a ministerial duty both of the judge
and of the Land Registration Commission; failure of the court or
of the clerk to issue the decree for the reason that no motion
therefor has been filed can not prejudice the owner, or the person
in whom the land is ordered to be registered. 14
The doctrine that neither prescription nor laches may render inefficacious a decision in a land registration
case was reiterated five (5) years after Sta. Ana, in Heirs of Cristobal Marcos, etc., et al. v. De Banuvar,
et al. 15 In that case, it was similarly argued that a prayer for the issuance of a decree of registration
filed in 1962 pursuant to a 1938 decision was, among others, barred by prescription and laches. In
rejecting the argument, the Court was content in restating with approval the above-cited excerpts from
Sta. Ana. A similar tack was again adopted by the Court some years later in Rodil v. Benedicto. 16 These
cases further emphasized, citing Demoran v. Ibanez, etc., and Poras 17 and Manlapas and Tolentino v.
Llorente, 18 respectively, that the right of the applicant or a subsequent purchaser to ask for the
issuance of a writ of possession of the land never prescribes. 19

Within the last 20 years, the Sta. Ana doctrine on the inapplicability of the rules on prescription and
laches to land registration cases has been repeatedly affirmed. Apart from the three (3) cases mentioned
earlier, the Sta. Ana doctrine was reiterated in another three (3) more cases later, namely: Vda. de
Barroga v. Albano, 20 Cacho v. Court of Appeals, 21 and Paderes v. Court of Appeals. 22 The doctrine
of stare decisis compels respect for settled jurisprudence, especially absent any compelling argument to
do otherwise. Indeed, the apparent strategy employed by the Republic in its present petition is to feign
that the doctrine and the cases that spawned and educed it never existed at all. Instead, it is insisted
that the Rules of Court, which provides for the five (5)-year prescriptive period for execution of
judgments, is applicable to land registration cases either by analogy or in a suppletory character and
whenever practicable and convenient. 23 The Republic further observes that Presidential Decree (PD)
No. 1529 has no provision on execution of final judgments; hence, the provisions of Rule 39 of the 1997
Rules of Civil Procedure should apply to land registration proceedings.
We affirm Sta. Ana not out of simple reflex, but because we recognize that the principle enunciated
therein offers a convincing refutation of the current arguments of the Republic.
Rule 39, as invoked by the Republic, applies only to ordinary civil actions, not to other or extraordinary
proceedings not expressly governed by the Rules of Civil Procedure but by some other specific law or
legal modality such as land registration cases. Unlike in ordinary civil actions governed by the Rules of
Civil Procedure, the intent of land registration proceedings is to establish ownership by a person of a
parcel of land, consistent with the purpose of such extraordinary proceedings to declare by judicial fiat a
status, condition or fact. Hence, upon the finality of a decision adjudicating such ownership, no further
step is required to effectuate the decision and a ministerial duty exists alike on the part of the land
registration court to order the issuance of, and the LRA to issue, the decree of registration.
The Republic observes that the Property Registration Decree (PD No. 1529) does not contain any
provision on execution of final judgments; hence, the application of Rule 39 of the 1997 Rules of Civil
Procedure in suppletory fashion. Quite the contrary, it is precisely because PD No. 1529 does not
specifically provide for execution of judgments in the sense ordinarily understood and applied in civil
cases, the reason being there is no need for the prevailing party to apply for a writ of execution in order
to obtain the title, that Rule 39 of the 1997 Rules of Civil Procedure is not applicable to land registration
cases in the first place. Section 39 of PD No. 1529 reads:
SEC. 39.Preparation of Decree and Certificate of Title. After the
judgment directing the registration of title to land has become final, the
court shall, within fifteen days from entry of judgment, issue an order
directing the Commissioner to issue the corresponding decree of
registration and certificate of title. The clerk of court shall send, within
fifteen days from entry of judgment, certified copies of the judgment and
of the order of the court directing the Commissioner to issue the
corresponding decree of registration and certificate of title, and a
certificate stating that the decision has not been amended, reconsidered,
nor appealed, and has become final. Thereupon, the Commissioner shall
cause to be prepared the decree of registration as well as the original and
duplicate of the corresponding original certificate of title. The original
certificate of title shall be a true copy of the decree of registration. The
decree of registration shall be signed by the Commissioner, entered and
filed in the Land Registration Commission. The original of the original
certificate of title shall also be signed by the Commissioner and shall be
sent, together with the owner's duplicate certificate, to the Register of
Deeds of the city or province where the property is situated for entry in his
registration book. EHTADa
The provision lays down the procedure that interposes between the rendition of the judgment and the
issuance of the certificate of title. No obligation whatsoever is imposed by Section 39 on the prevailing
applicant or oppositor even as a precondition to the issuance of the title. The obligations provided in the
Section are levied on the land court (that is to issue an order directing the Land Registration
Commissioner to issue in turn the corresponding decree of registration), its clerk of court (that is to
transmit copies of the judgment and the order to the Commissioner), and the Land Registration
Commissioner (that is to cause the preparation of the decree of registration and the transmittal thereof to
the Register of Deeds). All these obligations are ministerial on the officers charged with their
performance and thus generally beyond discretion of amendment or review.
The failure on the part of the administrative authorities to do their part in the issuance of the decree of
registration cannot oust the prevailing party from ownership of the land. Neither the failure of such
applicant to follow up with said authorities can. The ultimate goal of our land registration system is
geared towards the final and definitive determination of real property ownership in the country, and the
imposition of an additional burden on the owner after the judgment in the land registration case had
attained finality would simply frustrate such goal.
Clearly, the peculiar procedure provided in the Property Registration Law from the time decisions in
land registration cases become final is complete in itself and does not need to be filled in. From
another perspective, the judgment does not have to be executed by motion or enforced by action within
the purview of Rule 39 of the 1997 Rules of Civil Procedure.
Following these premises, it can even be posited that in theory, there would have been no need for
Nillas, or others under similar circumstances, to file a petition for revival of judgment, since revival of
judgments is a procedure derived from civil procedure and proceeds from the assumption that the
5
judgment is susceptible to prescription. The primary recourse need not be with the courts, but with the
LRA, with whom the duty to issue the decree of registration remains. If it is sufficiently established before
that body that there is an authentic standing judgment or order from a land registration court that
remains unimplemented, then there should be no impediment to the issuance of the decree of
registration. However, the Court sees the practical value of necessitating judicial recourse if a significant
number of years has passed since the promulgation of the land court's unimplemented decision or order,
as in this case. Even though prescription should not be a cause to bar the issuance of the decree of
registration, a judicial evaluation would allow for a thorough examination of the veracity of the judgment
or order sought to be effected, or a determination of causes other than prescription or laches that might
preclude the issuance of the decree of registration. IcESaA
What about the two cases cited by the Republic, Shipside and Heirs of Lopez? Even though the Court
applied the doctrines of prescription and laches in those cases, it should be observed that neither case
was intended to overturn the Sta. Ana doctrine, nor did they make any express declaration to such effect.
Moreover, both cases were governed by their unique set of facts, quite distinct from the general situation
that marked both Sta. Ana and the present case.
The judgment sought belatedly for enforcement in Shipside did not arise from an original action for land
registration, but from a successful motion by the Republic seeking the cancellation of title previously
adjudicated to a private landowner. While one might argue that such motion still arose in a land
registration case, we note that the pronouncement therein that prescription barred the revival of the
order of cancellation was made in the course of dispensing with an argument which was ultimately
peripheral to that case. Indeed, the portion of Shipside dealing with the issue of prescription merely
restated the provisions in the Civil Code and the Rules of Civil Procedure relating to prescription, followed
by an observation that the judgment sought to be revived attained finality 25 years earlier. However, the
Sta. Ana doctrine was not addressed, and perhaps with good reason, as the significantly more extensive
rationale provided by the Court in barring the revival of judgment was the fact that the State no longer
held interest in the subject property, having divested the same to the Bases Conversion Development
Authority prior to the filing of the action for revival. Shipside expounds on this point, and not on the
applicability of the rules of prescription.
Notably, Shipside has attained some measure of prominence as precedent on still another point, relating
to its pronouncements relating to the proper execution of the certification of non-forum shopping by a
corporation. In contrast, Shipside has not since been utilized by the Court to employ the rules on
prescription and laches on final decisions in land registration cases. It is worth mentioning that since
Shipside was promulgated in 2001, the Court has not hesitated in reaffirming the rule in Sta. Ana as
recently as in the middle of 2005 in the Paderes case.
We now turn to Heirs of Lopez, wherein the controlling factual milieu proved even more unconventional
than that in Shipside. The property involved therein was the subject of two separate applications for
registration, one filed by petitioners therein in 1959, the other by a different party in 1967. It was the
latter who was first able to obtain a decree of registration, this accomplished as early as 1968. 24 On the
other hand, the petitioners were able to obtain a final judgment in their favor only in 1979, by which time
the property had already been registered in the name of the other claimant, thus obstructing the
issuance of certificate of title to the petitioners. The issues of prescription and laches arose because the
petitioners filed their action to enforce the 1979 final judgment and the cancellation of the competing title
only in 1987, two (2) years beyond the five (5)-year prescriptive period provided in the Rules of Civil
Procedure. The Court did characterize the petitioners as guilty of laches for the delay in filing the action
for the execution of the judgment in their favor, and thus denied the petition on that score. IHSTDE

Heirs of Lopez noted the settled rule that "when two certificates of title are issued to different persons
covering the same land in whole or in part, the earlier in date must prevail . . . ," and indeed even if the
petitioners therein were somehow able to obtain a certificate of title pursuant to the 1979 judgment in
their favor, such title could not have stood in the face of the earlier title. The Court then correlated the
laches of the petitioners with their pattern of behavior in failing to exercise due diligence to protect their
interests over the property, marked by their inability to oppose the other application for registration or to
seek enforcement of their own judgment within the five (5)-year reglementary period.
Still, a close examination of Heirs of Lopez reveals an unusual dilemma that negates its application as
precedent to the case at bar, or to detract from Sta. Ana as a general rule for that matter. The execution
of the judgment sought for belated enforcement in Heirs of Lopez would have entailed the disturbance of
a different final judgment which had already been executed and which was shielded by the legal
protection afforded by a Torrens title. In light of those circumstances, there could not have been a
"ministerial duty" on the part of the registration authorities to effectuate the judgment in favor of the
petitioners in Heirs of Lopez. Neither could it be said that their right of ownership as confirmed by the
judgment in their favor was indubitable, considering the earlier decree of registration over the same
property accorded to a different party. The Sta. Ana doctrine rests upon the general presumption that the
final judgment, with which the corresponding decree of registration is homologous by legal design, has
not been disturbed by another ruling by a co-extensive or superior court. That presumption obtains in
this case as well. Unless that presumption is overcome, there is no impediment to the continued
application of Sta. Ana as precedent. 25
We are not inclined to make any pronouncements on the doctrinal viability of Shipside or Heirs of Lopez
concerning the applicability of the rules of prescription or laches in land registration cases. Suffice it to
say, those cases do not operate to detract from the continued good standing of Sta. Ana as a general
precedent that neither prescription nor laches bars the enforcement of a final judgment in a land
registration case, especially when the said judgment has not been reversed or modified, whether
deliberately or inadvertently, by another final court ruling. This qualifier stands not so much as a newly-
carved exception to the general rule as it does as an exercise in stating the obvious.
Finally, the Republic faults the Court of Appeals for pronouncing that the 1941 Decision constituted res
judicata that barred subsequent attacks to the adjudicates' title over the subject property. The Republic
submits that said decision would operate as res judicata only after the decree of registration was issued,
which did not happen in this case. We doubt that a final decision's status as res judicata is the impelling
ground for its very own execution; and indeed res judicata is more often invoked as a defense or as a
factor in relation to a different case altogether. Still, this faulty terminology aside, the Republic's
arguments on this point do not dissuade from our central holding that the 1941 Decision is still
susceptible to effectuation by the standard decree of registration notwithstanding the delay incurred by
Nillas or her predecessors-in-interest in seeking its effectuation and the reasons for such delay, following
the prostracted failure of the then Land Registration Commissioner to issue the decree of registration. In
this case, all that Nillas needed to prove was that she had duly acquired the rights of the original
adjudicates her predecessors-in-interest-in order to entitle her to the decree of registration albeit still
in the names of the original prevailing parties who are her predecessors-in interest. Both the trial court
and the Court of Appeals were satisfied that such fact was proven, and the Republic does not offer any
compelling argument to dispute such proof. cCESaH
WHEREFORE, the Petition is DENIED. No pronouncement as to costs.
SO ORDERED.









6




4. Writ of Possession and Writ of Demolition
G.R. No. L-25660. February 23, 1990.
LEOPOLDO VENCILAO, MAURO RENOBLAS, TELESFORO BALONDIA, FELIX ABANDULA,
FAUSTO GABAISEN, ISIDORO ELIVERA, RAYMUNDO BONGATO, MARTIN ROLLON,
EUSTAQUIO MEDANA, DOROTEO ELIVERA, FRANCISCO PAGAURA, MACARIO GEPALAGO,
GREGORIO ITAOC, ALEJANDRO RENOBLAS, SIMEON BARBARONA, GREGORIO RENOBLAS,
FRANCISCO ASOY, TEOFILA GUJELING, FABIAN VILLAME, VICENTE OMUSORA, PEDRO
BALORIA, GREGORIO ITAOC, TERESITA ITAOC, FAUSTINO ITAOC, FORTUNATO ITAOC,
FLORENTINA GEMENTIZA, RESTITUTA OMUSORA, ZOILA OMUSORA, FELISA OMUSORA,
ROBERTO HAGANAS, FELISA HAGANAS, FERMIN HAGANAS, VICTORIANO HAGANAS,
JULIA SEVILLA, ROMAN MATELA, MARCELA MATELA, DELFIN MATELA, PELAGIO MATELA,
ROBERTA MATELA, PROCOPIO CABANAS and SERAFINA CABANAS, plaintiff-appellants, vs.
TEODORO VANO, JOSE REYES, ROSARIO REYES, SALUD OGILVE BELTRAN, AMALIA R.
OGILVE, FLORA VDA. DE COROMINAS, JESUSA REYES, LOURDES COROMINAS MUNOZ,
JUAN COROMINAS, LOURDES C. SAMSON CEBALLOS, SOLEDAD C. SAMSON RAMA,
DOLORES V. GARCES FALCON, JAIME GARCES, JOAQUIN REYES, and PEDRO RE. R.
LUSPO, defendants-appellees.
G.R. No. L-32065. February 23, 1990.
LEOPOLDO VENCILAO, SOFRONIO ROLLON, AURELIO ELIVERA, FRANCISCO PAGAORA,
MARTIN ROLLON, GRACIANO MAHINAY, GERARDO ELIVERA, GREGORIO ITAOC, ISIDRO
ELIVERA, DEMOCRITO ELIVERA, FAUSTO GABAISIN, ALBINO RENOBLAS, EUSTAQUIO
MENDANIA, SIMEON BARBARONA, TELESFORO BALONDA, FELIX ABANDOLA, SATURNINA
GEPILAGO, TEOFILA GOHILING, TOMAS REAMBONANSA, MARCOS HAGANAS, PASTOR
ASNA and MAURO RENOBLAS, petitioners, vs. HONORABLE PAULINO S. MARQUEZ, Judge,
Court of First Instance of Bohol, Branch 1, and MARIANO OGILVE, et. al., respondents.
G.R. No. L-33677. February 23, 1990.
LEOPOLDO VENCILAO, SOFRONIO ROLLON, AURELIO ELIVERA, FRANCISCO PAGAORA,
MARTIN ROLLON, GRACIANO MAHINAY, GERARDO ELIVERA, GREGORIO ITAOC, ISIDRO
ELIVERA, DEMOCRITO ELIVERA, FAUSTO GABAISIN, ALBINO RENOBLAS, EUSTAQUIO
MENDANIA, SIMEON BARBARONA, TELESFORO BALONDA, FELIX ABANDOLA, SATURNINA
GEPILAGO, TEOFILA GOHILING, TOMAS REAMBONANSA, MARCOS HAGANAS, PASTOR
ASNA and MAURO RENOBLAS, petitioners, vs. HONORABLE PAULINO S. MARQUEZ, Judge,
Court of First Instance of Bohol, Branch 1, The Provincial Sheriff, Province of Bohol, and
MARIANO OGILVE, et. al., respondents.

D E C I S I O N
MEDIALDEA, J p:
On February 7, 1974, We resolved to allow the consolidation of these three cases, considering that they
involve the same parties and parcels of land: (1) G.R. No. L-25660 this is an appeal from the order of
the Court of First Instance of Bohol (now Regional Trial Court) 1 dated May 12, 1964 dismissing the
cases of some of the plaintiffs-appellants and its order dated August 25, 1965 denying the motion for
reconsideration and the motion to declare the defendants-appellees in default; (2) G.R. No. L32065
this is a petition for certiorari of the order of the Court of First Instance of Bohol 2 dated May 14, 1970
directing the execution of its prior order dated May 6, 1969 finding petitioners guilty of contempt; (3)
G.R. No. L-33677 this is a petition for certiorari with mandamus and prohibition of the order of the
Court of First Instance of Bohol dated June 2, 1971 directing the demolition of the houses of the
petitioners.
On February 15, 1988, We resolved to require the parties to manifest whether or not they are still
interested in prosecuting these cases, or supervening events have transpired which render these cases
moot and academic or otherwise substantially affect the same. On March 25, 1988, the petitioners filed
an ex parte manifestation that they are still very much interested in the just prosecution of these cases.
The antecedent facts are as follows:
G.R. No. 25660
On April 1, 1950, the heirs of the late Juan Reyes filed an application for registration of the parcels of
land allegedly inherited by them from Juan Reyes, in Land Registration Case No. 76, L.R.C. Record No. N-
4251. On July 26, 1951, administratrix Bernardina Vda. de Luspo filed an amended application for
registration. After hearing, the land was registered under Original Certificate of Title No. 400 (pp. 84-85,
Record on Appeal; p. 7, Rollo).

On October 9, 1962, a complaint for reconveyance of real properties with damages and preliminary
injunction, Civil Case No. 1533, (pp. 2-19, Record on Appeal; p. 7, Rollo) was filed by plaintiffs-appellants
before the Court of First Instance of Bohol. It was alleged that they are the lawful owners of their
respective parcels of land including the improvements thereon either by purchase or inheritance and have
been in possession publicly, continuously, peacefully and adversely under the concept of owners for more
than thirty (30) years tacked with the possession of their predecessors-in-interest. However, those
parcels of land were included in the parcels of land applied for registration by the heirs of Juan Reyes,
either by mistake or fraud and with the intention of depriving them of their rights of ownership and
possession without their knowledge, not until the last part of 1960 when the defendants-appellees,
through their agents, attempted to enter those parcels of land claiming that they now belong to the heirs
of Juan Reyes. To the complaint, the defendants-appellees moved to dismiss on two grounds (pp. 19-22,
Record on Appeal; p. 7, Rollo), namely: (1) for lack of cause of action and (2) the cause of action is
barred by prior judgment. LibLex
On July 20, 1963, the court a quo issued an order denying defendants-appellees' motion to dismiss (pp.
29-30, Record on Appeal; p. 7, Rollo). However, acting on the motion to set aside such order (pp. 31-32,
Record on Appeal; p. 7, Rollo), on May 12, 1964, the same court issued another order reversing itself
partially (p. 56, Record on Appeal; p. 7, Rollo), the dispositive portion of which reads:
"WHEREFORE, the cases herein of the plaintiffs Alejandro Renoblas, Fausto
Cabaisan, Fabian Villame, Gregorio Ita-oc, Faustino Ita-oc, Fortunato Ita-
oc, Roberto Haganas, Felisa Haganas, Fermin Haganas, Victoriano
Haganas, Julia Sevilla, Ramon Matela, Roberto Matela, Procopio Cabaas
and Vicente Amosora are hereby dismissed on the ground of res adjudicata
with these plaintiffs paying proportionately eighteenth-forty one (18/41) of
the costs, but the petition to dismiss the case of the rest of the plaintiffs is
hereby denied.
"SO ORDERED."
7
On May 28, 1964, the plaintiffs-appellants whose cases were dismissed filed a motion for reconsideration
(pp. 57-58, Record on Appeal; p. 7, Rollo). On July 24, 1964, the plaintiffs-appellants whose cases were
not dismissed filed a motion to declare the defendants-appellees in default for failure to file their answer
with the time prescribed by law (pp. 68-75, Record on Appeal; p. 7, Rollo). On the other hand,
defendants-appellees filed their opposition to the motion for reconsideration praying that the complaint
as regards the rest of the plaintiffs-appellants be likewise dismissed (pp. 75-80, Record on Appeal; p. 7
Rollo).
On August 25, 1965, the court a quo issued an order in connection therewith (pp. 82-98, Record on
Appeal; p. 7, Rollo) denying all motions.
The case is now before Us with the following as assignments of errors (p. 3, Brief for the Plaintiffs-
Appellants; p. 9, Rollo), to wit:
"I
"THE TRIAL COURT ERRED IN DISMISSING THE CASES OF THE
PLAINTIFFS-APPELLANTS WHOSE NAMES ARE ALREADY MENTIONED
ABOVE ON THE ALLEGED GROUND THAT THEIR CASES ARE BARRED BY A
PRIOR JUDGMENT OF RES ADJUDICATA.
"II
"THE TRIAL COURT ERRED IN DENYING THE MOTION OF THE
PLAINTIFFS-APPELLANTS WHOSE CASES ARE NOT DISMISSED TO
DECLARE THE DEFENDANTS-APPELLEES IN DEFAULT FOR HAVING
FAILED TO FILE THEIR ANSWER WITHIN THE TIME PRESCRIBED BY
LAW."
On August 12, 1966, a resolution was issued by this Court dismissing the appeal as regards the second
issue because the order appealed from was merely interlocutory, hence, not appealable (pp. 35-38,
Rollo).
On August 17, 1988, petitioners Alex Abandula, Mauro Renoblas, Simeon Barbarona, Fabian Villame,
Macario Gepalago, Eustaquio Medana, Julia Sevilla, Gregorio Itaoc, Francisco Asoy and Martin Rollon filed
a motion to withdraw their appeal on the ground that they are now the absolute owners and possessors
of their respective parcels of land subject of Civil Case No. 1533.
The appeal is not impressed with merit.
The plaintiffs-appellants claim that no evidence was presented by the defendants-appellees that they
(plaintiffs-appellants) were notified of the date of the trial on the merits of the application for registration
nor were they given copies of the decision of the trial court. Likewise, they contend that res judicata is
not applicable in an action for reconveyance. cdrep
The allegations that no evidence was presented by the defendants-appellees that plaintiffs-appellants
were notified of the date of the trial on the merits of the application for registration nor were they given
copies of the decision of the trial court are new issues. It is a well-settled rule that, except questions on
jurisdiction, no question will be entertained on appeal unless it has been raised in the court below and it
is within the issues made by the parties in their pleadings (Cordero vs. Cabral, G.R. No. 36789, July 25,
1983, 123 SCRA 532). The other contention that res judicata is not applicable in an action for
reconveyance is not plausible. The principle of res judicata applies to all cases and proceedings, including
land registration and cadastral proceedings (Republic vs. Estenzo, G.R. No. L-35376, September 11,
1980, 99 SCRA 65; Paz vs. Inandan, 75 Phil. 608; Penaloza vs. Tuazon, 22 Phil. 303).
It is a settled rule that a final judgment or order on the merits, rendered by a court having jurisdiction of
the subject matter and of the parties, is conclusive in a subsequent case between the same parties and
their successors in interest litigating upon the same thing and issue, regardless of how erroneous it may
be. In order, therefore, that there may be res judicata, the following requisites must be present: (a) The
former judgment must be final; (b) it must have been rendered by a court having jurisdiction of the
subject matter and of the parties; (c) it must be a judgment on the merits; and (d) there must be,
between the first and the second actions, identity of parties, of subject matter, and of cause of action
(San Diego vs. Cardona, 70 Phil. 281; Ramos vs. Pablo, G.R. No. 53692, Nov. 26, 1986, 146 SCRA 24).
The underlying philosophy of the doctrine of res judicata is that parties should not be permitted to litigate
the same issue more than once and when a right or fact has been judicially tried and determined by a
court of competent jurisdiction, so long as it remains unreversed, it should be conclusive upon the parties
and those in privity with them in law or estate (Sy Kao vs. Court of Appeals, G.R. No. 61752, Sept. 28,
1984, 132 SCRA 302). The doctrine of res judicata is an old axiom of law, dictated by wisdom and
sanctified by age, and is founded on the broad principle that it is to the interest of the public that there
should be an end to litigation by the same parties and their privies over a subject once fully and fairly
adjudicated. Interest republicae ut sit finis litium (Carandang vs. Venturanza, G.R. No. L-41940, Nov. 21,
1984, 133 SCRA 344). To ignore the principle of res judicata would be to open the door to endless
litigations by continuous determination of issues without end (Catholic Vicar Apostolic of the Mountain
Province vs. Court of Appeals, et al., G.R. Nos. 80294-95, Sept. 21, 1988, 165 SCRA 515).
Thus, when a person is a party to a registration proceeding or when notified he does not want to
participate and only after the property has been adjudicated to another and the corresponding title has
been issued files an action for reconveyance, to give due course to the action is to nullify registration
proceedings and defeat the purpose of the law.
In dismissing the cases of some of the petitioners, the court a quo meticulously discussed the presence
of all the elements of res judicata (pp. 36-38; pp. 42-54, Record on Appeal; p. 7; Rollo):
"There is no question that in that Registration Proceedings, LRC Record
No. N-4251, Land Registration Case No. N-76, the Court of First Instance
of the province of Bohol had jurisdiction of the subject matter, that said
court had rendered a judgment on the merit that was terminated in the
Court of Appeals since December, 1958, and that decision is now final with
a decree of registration over the parcels of land described in the
application issued to the applicants.
"The subject matter (the parcels of land) now claimed by the plaintiffs in
this case at bar are the same, or at least part of the parcels already
adjudicated registration in that registration case to the persons, some of
them are made defendants in this case before us. The cause of action
between the two cases are the same, ownership of these parcels of land,
though the forms of action are different, one is an ordinary Land
Registration and the other is reconveyance.
'It is settled that notwithstanding the
difference in the form of two actions, the doctrine of
res adjudicata will apply where it appears that the
parties in effect were litigating for the same thing. A
party can not, by varying the form of action, escape
the effects of res adjudicata (Aguirre vs. Atienza, L-
10665, Aug. 30, 1958; Geronimo vs. Nava., No. L-
12111, Jan. 31, 1959; Labarro vs. Labateria, et al., 28
O.G. 4479).
'Well settled is the rule that a party can
not by varying the form of action, or adopting a
different method of presenting his case, escape the
operation of the principle that one and the same
cause of action shall not be twice litigated between
the same parties or their privies.' (Francisco vs. Blas,
et al., No. L-5078; Cayco, et al., vs. Cruz, et al., No.
L-12663, Aug. 21, 1959).
'Accordingly, a final judgment in an
8
ordinary civil action, determining the ownership of
certain lands is res adjudicata in a registration
proceeding where the parties and property are the
same as in the former case (Paz vs. Inandan, 75 Phil.
608; Pealoza vs. Tuason, 22 Phil. 303).'

"xxx xxx xxx
"But are there identities of parties in this case before us and the former
registration proceedings? Identity of parties means that the parties in the
second case must be the same parties in the first case, or at least, must be
successors in interest by title subsequent to the commencement of the
former action or proceeding, or when the parties in the subsequent case
are heirs (Chua Tan vs. Del Rosario, 57 Phil. 411; Martinez vs. Franco, 51
Phil. 487; Romero vs. Franco, 54 Phil. 744; Valdez, et al. vs. Penida, No. L-
3467, July 30, 1951).
"xxx xxx xxx
"Returning our attention to the case at bar, and
with in mind the principles of res adjudicata above-quoted, we
noticed that many of the plaintiffs were not oppositors in the
former registration case, but many are children of the former
oppositors. In such a case we have to determine the case of
every plaintiff, if the former decision in the land registration
case is conclusive and binding upon him.
"xxx xxx xxx
"The defendants had proven that the adjoining owners and claimants of
the parcels of land object of registration proceeding had been notified
when the land was surveyed. These persons notified according to the
surveyor's certificate, Exhibit "B" were as follows: Cipriano Samoya, Fausto
Baguisin, Silveria Pahado, Enojario Laroda, Alejandro Renoblas, Heirs of
Gregorio Lofraco, Julian Villame, Pedro Itaoc, Adriano Toloy, Bartolome
Omosura, Marcelina Asilon, Gregorio Baguinang, et al., Nicolas Omosura,
Simon Lagrimas, et al., Martin Quinalayo, Gorgonio Baquinang, Demetrio
Asolan, Catalino Orellena, Heirs of Catalina Palves, Manuel Mondano, Angel
Mondano, Victoriano Balolo, Eugenio del Rosario, Verinici Bayson, Felomino
Ruiz, Apolonio Horbeda, and Mun. of Calape.
"The following persons were notified by the Chief of the Land Registration
Office of the initial hearing (Exhibit "J") of the registration proceedings
enjoining them to appear on June 16, 1952, at 8:30 a.m., before the Court
of First Instance of Bohol to show cause why the prayer of said application
should not be granted: the Solicitor General, the Director of Lands, the
Director of Public Works and the Director of Forestry, Manila; the Provincial
Governor, the Provincial Fiscal and the District Engineer, Tagbilaran, Bohol;
the Municipal Mayor, Gorgonio Baguinang, Demetrio Azocan, Catalino
Orellena, Manuel Mondano, Angel Mondano, Victoriano Bolalo, Eugenio del
Rosario, Verinici Bayson, Filomeno Ruiz, Apolonio Horboda, the Heirs of
Gregorio Lofranco, Julian Villame, Pedro Itaoc, Adriano Toloy, Bartolome
Omosora, Marcelina Asilom, Gregorio Baguinang, Nicolas Omosora, Simon
Lagrimas and Martin Quinalayo, Calape, Bohol; the heirs of Catalino
Polvos, Fausto Baguisin, Cipriano Samoya, Silveria Pohado, Enojario
Laroda, Alejandro Renoblas and Leoncio Barbarona, Antequera, Bohol.
"And after the application had been filed and published in accordance with
law the following persons represented by Atty. Conrado D. Marapao filed
opposition to that registration proceeding: Felipe Cubillo, Simon Lagrimas,
Simeon Villame, Felix Lacorte, Victor Omosura, Germana Gahil, Anastacio
Orillosa, Enerio Omosura, Valeriano Tuloy, Cipriano Sanoya, Pablo
Dumagdag, Andres Reimbuncia, Roman Reimbuncia, Cledonio Cabanas,
Moises Cabanas, Calixto Gohiting, Gervasio Sevilla, Pedro Omosura, Daniel
Itaoc, Luis Omosura, Bartolome Omosura, Nicasio Omosora, Calixto Sevilla,
Teodora Omosora, Jose Sabari, Silverio Lacorte, Silverio Tuloy, Gertrudes
Sevilla, Teodora Sevilla, Magno Orillosa, Gervacia Sevilla, Marcos Hagonos,
Eleuterio Pandas, Pablo Omosora, Fabian Villame, Teodoro Omosora,
Magdalina Asilom, Mauricio Matela, Marciano Ordada, Eusebio Omosora,
and Gregorio Repelle (Exhibit "E"), Atty. Juna V. Balmaseda in
representation of the Bureau of Lands, and Asst. Fiscal Norberto M.
Gallardo in representation of the Municipality of Calape.
"Plaintiffs Mauro Renoblas and Gregorio Renoblas are children of plaintiff
Alejandro Renoblas. Plaintiff Telesforo Balanda is son-in-law of Alejandro,
being the husband of Juliana Renoblas, daughter of Alejandro. Plaintiff
Alejandro Renoblas was not one of the oppositors in the registration
proceedings, but he was notified of the initial hearing of that registration
case and by the surveyor that surveyed the land object of registration
(Exhibit J-Movant). Therefore, the decision of the land registration
proceeding is binding upon him and his case is dismissed on the ground of
res adjudicata with costs.
"xxx xxx xxx
"Plaintiff Fausto Cabaisan was notified by the surveyor and that notice of
the initial hearing. And though he was not an oppositor, the former land
registration proceeding is binding on him. Therefore, this case is dismissed
in so far as Fausto Cabaisan is concerned with costs.
"xxx xxx xxx
"Plaintiffs Gregorio Ita-oc, Teresita Ita-oc, Faustino Ita-oc and Fortunato
Ita-oc are children of Daniel Ita-oc, one of the oppositors in the
registration proceedings. They claim parcel No. 10 described in paragraph
2 of the complaint. Gregorio Ita-oc testified that his land was inherited by
said plaintiffs' mother from her father, Pio Sevilla. The evidence on record
(Exhibits J-3, J-4, J-5). However (sic), shows that the land is declared in
the name of Daniel Ita-oc, a former oppositor in the registration case.
Hence, these plaintiffs are successors-in-interest of Daniel Ita-oc, and,
therefore, are bound by the decision in that registration case. Their case,
therefore, is dismissed, with costs.
"Plaintiffs Roberto Haganas, Felisa Haganas, Fermin Haganas and
Victoriano Haganas are children of Marcos Haganas, a former oppositor in
the registration case. Marcos testified that his claim before was only two
hectares, while the claim of his children is seven hectares, which come
from his wife, not from him. These plaintiffs claim two parcels, one under
Tax Declaration No. R-4452, and Tax Declaration No. R-8456. It appears
that Tax Declaration No. R-4452 (Exhibit M) is in the name of Marcos
Haganas and the land described under Tax Declaration No. R-8456 was
bought by the spouses Marcos Haganas and Tomasa Sevilla from Gertrudis
Sevilla in 1956 (Exhibit M-3), who was an oppositor in the registration
proceeding. Therefore, plaintiffs Roberto Haganas, Felisa Haganas, Fermin
Haganas, and Victoriano Haganas are successors-in-interest to properties
in which the decision in the registration case is conclusive and binding to
their predecessors-in-interest. Hence, their case here is dismissed with
costs.
9
"Plaintiff Julia Sevilla is the wife of Marcelo Matela, who was the oppositor
in the registration proceedings. Plaintiffs Roman Matela, Marcela Matela,
Delfin Matela, and Roberta Matela are their children. She has no son by
the name of Pelagio. Julia testified that the land now claimed by her
children came from her father Pio Sevilla. The land that was claimed by
Mauricio Matela as oppositor was in his name under Tax Declaration No.
5099. This is the same land now claimed by plaintiffs Julia Sevilla, Ramon
Matela, Marcela Matela, Delfin Matela, and Roberta Matela (Exhibit 0-4).
These plaintiffs are successors-in-interest of Mauricio Matela, who is bound
by the decision in that land proceeding wherein he was the oppositor.
Therefore, the case of these plaintiffs are dismissed with costs.
"Plaintiff Procopia Cabaas was the wife of Andres Reambonancia,
oppositor in the land registration proceedings. She claims parcel No. 20
described in paragraph 2 of the complaint bearing Tax Declaration No. R-
8121. It appears that this land is declared in the name of Andres
Reambonancia (Exhibit N-3) who, as oppositor in the land registration
case, is bound by the decision of that case. Therefore, the case of plaintiff
Procopia Cabaas, as successor-in-interest to Andres Reambonancia, is
hereby dismissed, with costs.
"Plaintiff Vicente Amosora is the son of Enerio Amosora and Florencia
Gahil, both oppositors in the former registration case. The land claimed by
plaintiff Vicente Amosora is described as parcel No. 24 of paragraph 2 of
the complaint under Tax Declaration No. R-6107, under the name of his
father Enerio Amosora. Since Enerio Amosora was an oppositor in the
former land registration of which this land was a part, the decision of that
land registration case is conclusive and binding not only to Enerio
Amosora, but also to his successor-in-interest, plaintiff Vicente Amosora,
whose case therefore, is dismissed with costs."
G.R. No. L-32065
Upon the death of administratrix Bernardina Vda. de Luspo, Transfer Certificate of Title No. 3561 was
issued in the name of Pedro R. Luspo and Transfer Certificate of Title No. 3562 was issued in the name
of several persons (p. 36, Rollo).
A writ of possession dated November 6, 1959, a first alias writ of possession dated January 6, 1961, and
a second alias writ of possession dated July 2, 1966 were issued by the trial court against the petitioners.
A sample of the guerilla-like, hide and seek tactics employed by the petitioners was proved by the official
report of the deputy sheriff dated January 21, 1960. Another evidence of petitioners' refusal to sign and
to vacate was a certification dated July 22, 1966 and the Sheriff's return dated October 25, 1966.
On March 29, 1967, a petition for contempt was filed by Mariano Ogilve, who is one of the registered
owners of the parcel of land covered by Transfer Certificate of Title No. 3562, against the petitioners for
refusing to vacate the land occupied by them and for refusing to sign the Sheriff's return.
On May 6, 1969, the court a quo issued a resolution, the dispositive portion of which reads (p. 47, Rollo):
"FOR ALL THE FOREGOING CONSIDERATION, make it of record that
Procopia Reambonansa voluntarily left the land and dropped out from the
case; the charge of contempt against Alejandro Renoblas (who died) is
dismissed and each of the remaining 22 respondents are hereby found
guilty of contempt under Sec. 3-b of Rule 71 and are hereby sentenced
each to pay a fine of One Hundred Pesos, authorizing the Constabulary
Detachment at or near Candungao, Calape, Bohol to collect the same and
to transmit the money to the Clerk of this Court, with subsidiary
imprisonment in case of insolvency at the rate of one day for every P2.50
or fraction of a day, the said Constabulary Detachment to effect the
commitment if any of them is unable to pay the fine. The fingerprints of
each of these 22 respondents shall also be taken by the constabulary and
filed with the record of this case.

"It is so ordered."
On June 4, 1969, the petitioners filed a motion for reconsideration of the aforestated resolution whereas
Ogilve filed an opposition thereto.
On February 14, 1970, the motion for reconsideration was denied. On March 18, 1970, another motion
for reconsideration was filed by petitioners on the ground of pendency of the action for reconveyance in
Civil Case No. 1533 and their appeal in G.R. No. L-25660. On May 14, 1970, the court a quo ordered the
proper officers to actually execute the resolution dated May 6, 1969. cdphil
Hence, the present petition.
Petitioners raise the following issues:
I
THAT THE SAID RESPONDENT JUDGE ERRED IN ISSUING A WRIT OF
POSSESSION WITHOUT ANY COMPLAINT FILED IN COURT FOR FORCIBLE
ENTRY AND DETAINER, NOR FOR RECOVERY OF OWNERSHIP AND
POSSESSION OF THE PARCELS OF LAND IN QUESTION AGAINST THE
HEREIN PETITIONERS.
II
THAT THE HONORABLE RESPONDENT JUDGE ERRED IN ISSUING A WRIT
OF POSSESSION AGAINST THE PETITIONERS HEREIN, WHO WERE NOT
PARTIES TO THE REGISTRATION PROCEEDING AND WHO WERE NOT
DEFEATED OPPOSITORS OF THE SAID APPLICATION FOR
REGISTRATION.
The petition is impressed with merit.
Petitioners contend that they were not claimants-oppositors nor defeated oppositors in the said land
registration case, as their names do not appear in the amended application for registration; that they
have occupied the subject parcels of land for more than thirty (30) years which began long before the
filing of the application for registration; and that after the hearing of the registration case, they continued
in possession of the said land.
In a registration case, the judgment confirming the title of the applicant and ordering its registration in
his name necessarily carried with it the delivery of possession which is an inherent element of the right of
ownership. The issuance of the writ of possession is, therefore, sanctioned by existing laws in this
jurisdiction and by the generally accepted principle upon which the administration of justice rests
(Romasanta, et. al. vs. Platon, 34 O.G. No. 76; Abulocion, et. al. vs. CFI of Iloilo, et al., 100 Phil. 554
[1956]). A writ of possession may be issued not only against the person who has been defeated in a
registration case but also against anyone unlawfully and adversely occupying the land or any portion
thereof during the land registration proceedings up to the issuance of the final decree (Demorar vs.
Ibaez, et al., 97 Phil 2 [1955]). LLpr
The petitioners' contention that they have been in possession of the said land for more than thirty (30)
years which began long before the filing of the application for registration and continued in possession
after the hearing of the registration case, worked against them. It was a virtual admission of their lack of
defense. Thus, the writs of possession were properly issued against them.
However, We do not subscribe to the ruling of the court a quo that petitioners are guilty of contempt.
Under Section 3 (d) of Rule 19, Rules of Court, if the judgment be for the delivery of the possession of
10
real property, the writ of execution must require the sheriff or other officer to whom it must be directed
to deliver the possession of the property, describing it, to the party entitled thereto. This means that the
sheriff must dispossess or eject the losing party from the premises and deliver the possession thereof to
the winning party. If subsequent to such dispossession or ejectment the losing party enters or attempts
to enter into or upon the real property, for the purpose of executing acts of ownership or possession, or
in any manner disturbs the possession of the person adjudged to be entitled thereto, then and only then
may the loser be charged with and punished for contempt (Quizon vs. Philippine National Bank, et al., 85
Phil. 459). According to this section, it is exclusively incumbent upon the sheriff to execute, to carry out
the mandates of the judgment in question, and in fact, it was he himself, and he alone, who was ordered
by the trial judge who rendered that judgment, to place the respondents in possession of the land. The
petitioners in this case had nothing to do with that delivery of possession, and consequently, their refusal
to effectuate the writ of possession, is entirely officious and impertinent and therefore could not hinder,
and much less prevent, the delivery being made, had the sheriff known how to comply with his duty. It
was solely due to the latter's fault, and not to the disobedience of the petitioners, that the judgment was
not duly executed. For that purpose, the sheriff could even have availed himself of the public force, had it
been necessary to resort thereto (see United States v. Ramayrat, 22 Phil. 183). prcd
G.R. No. L-33677
On March 22, 1971, Mariano Ogilve filed a Motion for a Writ of Demolition which was granted by the trial
court on April 5, 1971 (pp. 42-43, Rollo) against those who were adjudged guilty of contempt. On April
29, 1971, the petitioners filed an urgent motion for reconsideration of said order. On June 2, 1971, the
trial court issued another order, the dispositive portion of which reads (p. 48, Rollo):
"WHEREFORE, in the absence of writ of preliminary injunction Deputy
Provincial Sheriff Pedro Aparece must not only take P.C. soldiers with him
but also carpenters to effect the demolition, the carpenters being at the
expense of the Luspo.
"IT IS SO ORDERED."
Hence, the present petition.
The issue here is whether or not the respondent judge acted without or in excess of his jurisdiction, or
with grave abuse of discretion and thus excluded the herein petitioners from the use and enjoyment of
their right to which they are entitled when he (respondent judge) issued the order of demolition on April
5, 1971 and again on June 2, 1971 (p. 107, Rollo).
On July 14, 1971, this Court issued a temporary restraining order (p. 51, Rollo). LexLib
The petition is not impressed with merit.
The petitioners allege that the respondent judge cannot issue a writ of demolition pending the resolution
of G.R. No. L-32065.
We rule that the petition in G.R. No. L-32065 was not a bar to the issuance of the writ of demolition. It is
significant to note that the subject matter of the petition in G.R. No. L-32065 is the order dated May 14,
1970 directing the execution of the prior order dated May 6, 1969 finding petitioners guilty of contempt
and not the writs of possession themselves. Thus, the respondent Judge correctly issued the writs of
demolition. In Meralco vs. Mencias, 107 Phil 1071, We held:
"[I]f the writ of possession issued in a land registration proceeding implies
the delivery of possession of the land to the successful litigant therein
(Demorar vs. Ibaez, 97 Phil. 72; Pasay Estate Company vs. Del Rosario,
et al., 11 Phil. 391; Manlapas vs. Llorente, 48 Phil. 298), a writ of
demolition must, likewise, issue, especially considering that the latter writ
is but a complement of the former which, without said writ of demolition,
would be ineffective.
xxx xxx xxx
"[The issuance of the writ of demolition] is reasonably necessary to do
justice to petitioner who is being deprived of the possession of the lots in
question, by reason of the continued refusal of respondent . . . to remove
his house thereon and restore possession of the premises to petitioner.
ACCORDINGLY, judgment is hereby rendered as follows:
1)In G.R. No. L-25660, the appeal is DENIED and the orders of the Court of First Instance dated May 12,
1964 and August 25, 1965 are AFFIRMED; the motion to withdraw the appeal of some of the plaintiffs-
appellants is GRANTED;
2)In G.R. No. L-32065, the petition is GRANTED and the resolution of the Court of First Instance dated
May 14, 1970 is SET ASIDE; and
3)In G.R. No. L-33677, the petition is DISMISSED and the order of the Court of First Instance dated June
2, 1971 is AFFIRMED. The temporary restraining order is LIFTED.
SO ORDERED.




















11
5. Possession of third persons after issuance of final decree, Remedy
6. Consequence of Refusal to Vacate

vii. When OCT takes effect
G.R. No. 123346. March 31, 2009.
MANOTOK REALTY, INC. and MANOTOK ESTATE CORPORATION, petitioners, vs.
CLT REALTY DEVELOPMENT CORPORATION, respondent.
G.R. No. 134385. March 31, 2009.
ARANETA INSTITUTE OF AGRICULTURE, INC., petitioner, vs. HEIRS OF JOSE B.
DIMSON, REPRESENTED BY HIS COMPULSORY HEIRS: HIS SURVIVING SPOUSE,
ROQUETA R. DIMSON AND THEIR CHILDREN, NORMA AND CELSA TIRADO,
ALSON AND VIRGINIA DIMSON, LINDA AND CARLOS LAGMAN, LERMA AND
RENE POLICAR, AND ESPERANZA R. DIMSON; AND THE REGISTER OF DEEDS OF
MALABON, respondents.
R E S O L U T I O N
TINGA, J p:
In the Court's Resolution dated 14 December 2007, 1 the Court constituted a Special Division of the
Court of Appeals to hear the instant case on remand. The Special Division was composed of three
Associate Justices of the Court of Appeals, with Justice Josefina Guevara-Salonga as Chairperson; Justice
Lucas Bersamin as Senior Member; and Associate Justice Japar B. Dimaampao as Junior Member. We
instructed the Special Division to proceed as follows:
The Special Division is tasked to hear and receive evidence, conclude the
proceedings and submit to this Court a report on its findings and
recommended conclusions within three (3) months from finality of this
Resolution.
In ascertaining which of the conflicting claims of title should prevail, the
Special Division is directed to make the following determinations based on
the evidence already on record and such other evidence as may be
presented at the proceedings before it, to wit: ICASEH
i.Which of the contending parties are able to trace back their
claims of title to OCT No. 994 dated 3 May 1917?
ii.Whether the imputed flaws in the titles of the Manotoks
and Araneta, as recounted in the 2005 Decision, are borne by
the evidence? Assuming they are, are such flaws sufficient to
defeat the claims of title of the Manotoks and Araneta?
iii.Whether the factual and legal bases of 1966 Order of
Judge Muoz-Palma and the 1970 Order of Judge Sayo are
true and valid. Assuming they are, do these orders establish
a superior right to the subject properties in favor of the
Dimsons and CLT as opposed to the claims of Araneta and
the Manotoks?
iv.Whether any of the subject properties had been the
subject of expropriation proceedings at any point since the
issuance of OCT No. 994 on 3 May 1917, and if so what are
those proceedings, what are the titles acquired by the
Government and whether any of the parties is able to trace
its title to the title acquired by the Government through
expropriation.
v.Such other matters necessary and proper in ascertaining
which of the conflicting claims of title should prevail.
WHEREFORE, the instant cases are hereby REMANDED to the Special
Division of the Court of Appeals for further proceedings in accordance with
Parts VI, VII and VIII of this Resolution.
SO ORDERED. 2
The Special Division proceeded to conduct hearings in accordance with the Resolution. The parties to
these cases, namely CLT Realty Development Corporation (CLT), Manotok Realty Inc. and Manotok
Estate Corporation (the Manotoks), the Heirs of Jose B. Dimson (Heirs of Dimson), and Araneta Institute
of Agriculture, Inc. (Araneta), were directed by the Special Division to present their respective evidence
to the Court of Appeals. Thereafter, the Special Division rendered a 70-page Report 3 (Report) on 26
November 2008. The Special Division submitted the sealed Report to this Court. EHaCID
Before taking action on the Report itself, we dispose of a preliminary matter. On February 17, 2009, the
Manotoks filed a motion beseeching that copies of the report be furnished the parties "so that they may
submit their comments and objections thereon in accord with the principle contained in Sec. 10, Rule 32
of the Rules of Court". We deny the motion.
It is incorrect to presume that the earlier referral of these cases to the Court of Appeals for reception of
evidence was strictly in accordance with Rule 32. Notably, Section 1 of said Rule authorizes the referral of
the case to a commissioner "by written consent of both parties", whereas in the cases at bar, the Court
did not endeavor to secure the consent of the parties before effectuating the remand to the Court of
Appeals. Nonetheless, our earlier advertence to Rule 32 remains proper even if the adopted procedure
does not hew strictly to that Rule, owing to our power under Section 6, Rule 135 to adopt any suitable
process or mode of proceeding which appears conformable to the spirit of the Rules to carry into effect
all auxiliary processes and other means necessary to carry our jurisdiction into effect. HEaCcD
Moreover, furnishing the parties with copies of the Sealed Report would not serve any useful purpose. It
would only delay the promulgation of the Court's action on the Sealed Report and the adjudication of
these cases. In any event, the present Resolution quotes extensively from the sealed Report and
discusses its other substantive segments which are not quoted.
The Report is a commendably exhaustive and pellucid analysis of the issues referred to the Special
Division. It is a more than adequate basis for this Court to make the following final dispositions in these
cases.
I.
We adopt the succeeding recital of operative antecedents made by the Special Division in its Report:
THE PROCEDURAL ANTECEDENTS
DIMSON v. ARANETA
12
CA-G.R. CV. NO. 41883 & CA-G.R. SP No. 34819
[SC-G.R. No. 134385]
On 18 December 1979, DIMSON filed with the then Court of First Instance
["CFI"] of Rizal a complaint for Recovery of Possession and Damages
against ARANETA. On 7 May 1980, DIMSON amended his complaint and
included Virgilio L. Enriquez ["ENRIQUEZ"] as his co-plaintiff.
In said Amended Complaint, DIMSON claimed that he is the absolute
owner of a 50-hectare land located in Bo. Potrero, Malabon, Metro Manila
covered by TCT No. R-15169, [Lot 25-A-2] of the Caloocan Registry of
Deeds. Allegedly, DIMSON had transferred the subject property to
ENRIQUEZ by way of an absolute and irrevocable sale on 14 November
1979. Unfortunately though, DIMSON and ENRIQUEZ discovered that the
subject property was being occupied by ARANETA wherein an "agricultural
school house" is erected and that despite repeated demands, the latter
refused to vacate the parcel of land and remove the improvements
thereon. IECcaA
ARANETA, for its part, refuted said allegations and countered that it is the
absolute owner of the land being claimed by DIMSON and that the real
properties in the Araneta Compound are "properly documented and validly
titled". It maintained that it had been in possession of the subject parcel of
land since 1974. For this reason, the claims of DIMSON and ENRIQUEZ
were allegedly barred by prescription.
During the trial, counsel for ARANETA marked in evidence, among others,
certifications from the Land Registration Commission attesting that TCTs
Nos. 13574 and 26538, covering the disputed property, are in the names
of ARANETA and Jose Rato, respectively. ARANETA also offered TCT No.
7784 in evidence to prove that it is the registered owner of the land
described therein.
On 28 May 1993, the trial court rendered a Decision upholding the title of
DIMSON over the disputed property . . .
Undaunted, ARANETA interposed an appeal to the Court of Appeals,
docketed as CA-G.R. CV No. 41883, which was later consolidated with CA-
G.R. SP No. 34819 in view of the inter-related issues of the two cases.
In its 30 May 1997 Decision, the Court of Appeals, in CA-G.R. CV No.
41883, sustained the RTC Decision in favor of DIMSON finding that the
title of ARANETA to the disputed land is a nullity. In CA-G.R. SP No. 34819,
the Court of Appeals likewise invalidated the titles of ARANETA, relying on
the Supreme Court ruling in Metropolitan Waterworks and Sewerage
System v. Court of Appeals, which declared null and void the certificates of
title derived from OCT No. 994 registered on 3 may 1917. It was also held
that ARANETA failed to sufficiently show that the Order sought to be
nullified was obtained through extrinsic fraud that would warrant the
annulment thereof. SIHCDA
Dissatisfied still, ARANETA filed a Motion for Reconsideration and/or New
Trial espousing therein as basis for its entreaty the various letters from
different government agencies and Department Order No. 137 of the
Department of Justice, among others.
On 16 July 1998, the various Motions of ARANETA were denied by the
Court of Appeals. Nonetheless, the Court ordered DIMSON to maintain
status quo until the finality of the aforesaid judgment.
Consequently, ARANETA filed a petition before the Supreme Court.
Refuting the factual finding of the trial court and the Court of Appeals,
ARANETA contended that there is only one OCT 994 covering the Maysilo
Estate issued on 3 May 1917 pursuant to the Decree No. 36455 issued by
the Court of Land Registration on 19 April 1917 and added that there were
subsequent certifications issued by the government officials, notably from
the LRS, the DOJ Committee Report and the Senate Committees' Joint
Report which attested that there is only one OCT 994, that which had been
issued on 3 May 1917. AHDTIE
CLT v. MANOTOK
CA-G.R. CV. No. 45255
[SC-G.R. No. 123346]
On 10 August 1992, CLT filed with the Regional Trial Court ["RTC"] A
COMPLAINT FOR Annulment of Transfer Certificates of Title, Recovery of
Possession and Damages against the MANOTOKS and the Registry of
Deeds of Metro Manila District II (Calookan City, Metro Manila)
["CALOOCAN RD"].
In its Complaint, CLT alleged that it is the registered owner of Lot 26 of
the Maysilo Estate located in Caloocan City and covered by Transfer
Certificate of Title No. T-177013, a derivative title of OCT No. 994. As a
basis of its proprietary claim, CLT averred that on 10 December 1988, it
had acquired Lot 26 from its former registered owner, Estelita I. Hipolito
["HIPOLITO"], by virtue of a Deed of Sale with Real Estate Mortgage.
HIPOLITO's title was, in turn, a direct transfer from DIMSON, the
registered owner of TCT No. 15166, the latter having acquired the same
by virtue of a Court Order dated 13 June 1966 issued by the Court of First
Instance of Rizal in Civil Case No. 4557. CAaDSI

On the other hand, the MANOTOKS maintained the validity of their titles,
which were all derivatives of OCT No. 994 covering over twenty (20)
parcels of land located over a portion of Lot 26 in the Maysilo Estate. In
substance, it was contented that the title of CLT was an offspring of an
ineffective grant of an alleged undisputed portion of Lot 26 by way of
attorney's fees to its predecessor-in-interest, Jose B. Dimson. The
MANOTOKS, in this connection, further contended that the portion of Lot
26, subject of the present controversy, had long been disposed of in favor
of Alejandro Ruiz and Mariano Leuterio and hence, there was nothing more
in said portion of Lot 26 that could have been validly conveyed to Dimson.
Tracing the legitimacy of their certificates of titles, the MANOTOKS alleged
that TCT No. 4210, which cancelled OCT No. 994, had been issued in the
names of Alejandro Ruiz and Mariano Leuterio on September 1918 by
virtue of an Escritura De Venta executed by Don Tomas Arguelles and Don
Enrique Lopes on 21 August 1918. TCT No. 4210 allegedly covered an
approximate area of 19,565.43 square meters of Lot 26. On even date,
TCT No. 4211 was transferred to Francisco Gonzales on the strength of an
Escritura de Venta dated 3 March 1920 for which TCT No. T-5261,
covering an area of 871,982 square meters was issued in the name of one
Francisco Gonzales, married to Rufina Narciso.
13
Thereafter, TCT No. T-35485, canceling TCT No. T-5261, was issued to
Rufina Narcisa Vda. de Gonzales which was later replaced with the names
of Gonzales six (6) children. The property was then subdivided and as a
result of which, seven (7) certificates of titles were issued, six (6), under
the names of each of the children while the remaining title was held by all
of them as co-owners.
Eventually, the properties covered by said seven certificates of title were
expropriated by the Republic of the Philippines. These properties were
then later subdivided by the National Housing Authority ["NHA"], into
seventy-seven (77) lots and thereafter sold to qualified vendees. As it
turned out, a number of said vendees sold nineteen (19) of these lots to
Manotok Realty, Inc. while one (1) lot was purchased by the Manotok
Estate Corporation.
During the pre-trial conference, the trial court, upon agreement of the
parties, approved the creation of a commission composed of three
commissioners tasked to resolve the conflict in their respective titles.
Accordingly, the created Commission convened on the matter in dispute.
On 8 October 1993, Ernesto Erive and Avelino San Buenaventura
submitted an exhaustive Joint Final Report ["THE MAJORITY REPORT"]
finding that there were inherent technical infirmities or defects on the face
of TCT No. 4211, from which the MANOTOKS derived their titles (also on
TCT No. 4210), TCT No. 5261 and TCT No. 35486. Teodoro Victoriano
submitted his Individual Final Report ["THE MINORITY REPORT"] dated 23
October 1993.
After the conduct of a hearing on these reports, the parties filed their
respective comments/objections thereto. Upon order of the trial court, the
parties filed their respective memoranda. caHCSD
Adopting the findings contained in the Majority Report, the RTC, on 10
May 1994, rendered a Decision, in favor of CLT and ordered, among
others, the cancellation of the certificates of title issued in the name of the
MANOTOKS.
The MANOTOKS elevated the adverse RTC Decision on appeal before the
Court of Appeals. In its Decision dated 28 September 1995, the Court of
Appeals affirmed the RTC Decision, except as to the award of damages
which was deleted. The MANOTOKS then moved for reconsideration, but
said motion was denied by said appellate court in its Resolution dated 8
January 1996. After the denial of their Motion for Reconsideration, the
MANOTOKS filed a Petition for Review before the Supreme Court.
PROCEEDINGS BEFORE THE SUPREME COURT
Before the Supreme Court, the Petitioners for Review, * separately filed by
the MANOTOKS, ARANETA and Sto. Nio Kapitbahayan Association, Inc.,
["STO. NIO"], were consolidated.
Also submitted for consideration of the Supreme Court were the report of
the Fact Finding Committee dated 28 August 1997 and the Senate
Committee Report No. 1031 dated 25 May 1998 which concluded that
there was only one OCT No. 994 issued, transcribed and registered on 3
May 1917.
THE SUPREME COURT DECISION
In its Decision dated 29 November 2005 ["THE SUPREME COURT 2005
DECISION"], the Supreme Court, through its Third Division, affirmed the
RTC Decision and Resolutions of the Court of Appeals, which declared the
titles of CLT and DIMSON as valid.
In invalidating the respective titles of the MANOTOKS and ARANETA, the
Supreme Court, in turn, relied on the factual and legal findings of the trial
courts, which had heavily hinged on the imputed flaws in said titles.
Considering that these trial court findings had been affirmed by the Court
of Appeals, the Supreme Court highlighted the fact that the same were
accorded the highest degree of respect and, generally, should not be
disturbed on appeal.
Emphasis was also made on the settled rule that because the Supreme
Court was not a trier of facts, it was not within its function to review
factual issues and examine, evaluate or weigh the probative value of the
evidence presented by the parties.
THE SUPREME COURT RESOLUTION
Expectedly, the MANOTOKS and ARANETA filed their respective Motions
for Reconsideration of the Supreme Court 2005 Decision.
Resolving said motions for reconsideration, with the Office of the Solicitor
General ["OSG"] intervening on behalf of the Republic, the Supreme Court,
in its Resolution of 14 December 2007 ["THE SUPREME COURT 2007
RESOLUTION"] reversed and nullified its 2005 Decision and categorically
invalidated OCT No. 994 dated 19 April 1917, which was the basis of the
propriety claims of CLT and DIMSON. However, the Supreme Court
resolved to remand the cases to this Special Division of the Court of
Appeals for reception of evidence. EHCDSI
To guide the proceedings before this Special Division of the Court of
Appeals, the Supreme Court made the following binding conclusions:
"First, there is only one OCT 994. As it appears on the
record, that mother title was received for transcription by the
Register of Deeds on 3 May 1917, and that should be the
date which should be reckoned as the date of registration of
the title. It may also be acknowledged, as appears on the
title, that OCT No. 994 resulted from the issuance of the
decree of registration on (19) * April 1917, although such
date cannot be considered as the date of the title or the date
when the title took effect.
Second. Any title that traces its source to OCT No. 994 dated
(19) April 1917 is void, for such mother title is inexistent. The
fact that the Dimson and CLT titles made specific reference
to an OCT No. 994 dated (19) April 1917 casts doubt on the
validity of such titles since they refer to an inexistent OCT.
This error alone is, in fact, sufficient to invalidate the Dimson
and CLT claims over the subject property if singular reliance
is placed by them on the dates appearing on their respective
titles.
Third. The decision of this Court in MWSS v. Court of Appeals
and Gonzaga v. Court of Appeals cannot apply to the cases
at bar, especially in regard to their recognition of an OCT No.
994 dated 19 April 1917, a title which we now acknowledge
14
as inexistent. Neither could the conclusions in MWSS or
Gonzaga with respect to an OCT No. 994 dated 19 April 1917
bind any other case operating under the factual setting the
same as or similar to that at bar. 4
II.
The parties were afforded the opportunity to present their evidence before the Special Division. The
Report names the evidence submitted to the Special Division for its evaluation: ACHEaI
CLT EVIDENCE
In its Offer of Evidence, [ 5 ] CLT adopted the documentary exhibits and
testimonial evidence of witnesses submitted in the case filed by CLT
against STO. NIO in Civil Case No. C-15491, ["CLT-STO NIO CASE"].
These pieces of evidence include, among others, the Majority and Minority
Reports, the Formal Offer of Evidence in the presentation of the evidence-
in-chief and rebuttal evidence in the CLT-STO NIO CASE consisting of
various certificates of titles, plans by geodetic engineer, tax declarations,
chemistry report, specimen signatures and letters of correspondence.
MANOTOKS EVIDENCE
The MANOTOKS sought admission of the following evidence: Senate and
DOJ Committee Reports; certificates of title issued to them and their
vendees/assignees, i.e., Republic of the Philippines, the Gonzalezes,
Alejandro Ruiz and Mariano Leuterio, Isabel Gil del Sola and Estelita
Hipolito; deeds of absolute sale; contracts to sell; tax declarations and real
property tax receipts; the Formal Officer of Evidence of Philville
Development & Housing Corporation; ["PHILVILLE"], in Civil Case No.
15045; this Court of Appeals' Decision in CA-G.R. CV. No. 52606 between
CLT and PHILVILLE; the Orders of Judge Palma dated 13 June 1966 and
16 August 1966 in Case No. 4557 and the billing statements of SSHG Law
Office. They also submitted in evidence the Affidavits and Supplemental
Affidavits of Rosa R. Manotok and Luisa T. Padora; Affidavits of Atty. Felix
B. Lerio, Atty. Ma. P.G. Ongkiko and Engineer Jose Marie P. Bernabe; a
copy of a photograph of BM No. 9; certified true copy of coordinates and
reference point of L.M. No. 1 and BM No. 1 to 10 of Piedad Estate and TCT
No. 177013 of CLT. [ 6 ] SCDaHc
DIMSON EVIDENCE
In their Consolidated Formal Offer of Evidence, [ 7 ] DIMSON submitted the previous decisions and
resolutions passed relative to these cases, various certifications of different government agencies, OCT
994, subdivision plan of Lot 25-A-2, observations of Geodetic Engineer Reggie P. Garcia showing the
relative positions of properties within Lot 25-A; the Novation of Contract/Deed of Sale and Mortgage
dated 15 January 1948 between Rato, Don Salvador Araneta and Araneta Institute of Agriculture; copies
of various certificates of titles to dispute some of the titles held by ARANETA; several letter-requests and
official receipts.

ARANETA EVIDENCE
ARANETA, in turn, offered in evidence various certificates of title,
specifically, OCT No. 994, TCT No. 8692; TCT No. 21857; TCT No. 26538;
TCT No. 26539; TCT No. (7784)-738 and TCT No. 13574. It also marked in
evidence the certified true copies of Decree No. 36577; the DOJ and
Senate Reports; letters of correspondence to the Land Registration
Commission and the Register of Deeds of Malabon City; survey plans of
Lot 25-A and TCT r-15169 of Dimson and; the affidavit of Engineer Felino
M. Cortez and his curriculum vitae. ARANETA also offered the certified true
copy of TCT No. 6196 in the name of Victoneta, Inc.; TCT No. 13574 in the
name of ARANETA; certifications issued by Atty. Josephine H. Ponciano,
Acting Register of Deeds of Malabon city-Navotas; certified true copy of
Judge Palma's Order dated 16 August 1966 in Case No. 4557; Circular No.
17 (which pertains to the rules on reconstitution of titles as of 19 February
1947) and its official receipt and; the owner's duplicate copy of OCT No.
994. [ 8 ] 9 DHETIS
III.
We now turn to the evaluation of the evidence engaged in by the Special Division. To repeat, the Special
Division was tasked to determine the following issues based on the evidence:
i.Which of the contending parties are able to trace back their claims to
Original Certificate of Title (OCT) No. 994 dated 3 May 1917:
ii.Whether the respective imputed flaws in the titles of the Manotoks and
Araneta, as recounted in the Supreme Court 2005 Decision,
are borne by the evidence. Assuming they are, are such
flaws sufficient to defeat said claims?
iii.Whether the factual and legal bases of the 1966 Order of Judge Muoz-
Palma and the 1970 Order of Judge Sayo are true and valid.
Assuming they are, do these orders establish a superior right
to the subject properties in favor of the Dimsons and CLT as
opposed to the claims of the Araneta and the Manotoks?
iv.Whether any of the subject properties had been the subject of
expropriation proceedings at any point since the issuance of
OCT No. 994 on 3 May 1917, and if so, what are those
proceedings, what are the titles acquired by the Government,
and is any of the parties able to trace its title acquired by the
government through expropriation? DaTICE
v.Such other matters necessary and proper in ascertaining which of the
conflicting claims of title should prevail.
The ultimate purpose of the inquiry undertaken by the Court of Appeals was to ascertain which of the
four groups of claimants were entitled to claim ownership over the subject properties to which they
claimed title thereto. One set of properties was disputed between CLT and the Manotoks, while the other
set was disputed between Araneta and the Heirs of Dimson.
As can be gleaned from the Report, Jose Dimson was able to obtain an order in 1977 issued by Judge
Marcelino Sayo of the Court of First Instance (CFI) of Caloocan City on the basis of which he was able to
register in his name properties belonging to the Maysilo Estate. Judge Sayo's order in turn was sourced
from a 1966 Order issued by Judge (later Supreme Court Associate Justice) Cecilia Muoz-Palma of the
CFI of Rizal. Dimson's titles reflected, as their mother title, OCT No. 994 dated 19 April 1917. 10 Among
these properties was a fifty (50)-hectare property covered by Transfer Certificate of Title (TCT) No.
151169, which apparently overlapped with the property of Araneta covered by TCT No. 13574 and
26538. 11 Araneta was then and still is in possession of the property. The Araneta titles state, as their
mother title, OCT No. 994 dated 3 May 1917. Consequently, Dimson filed an action for recovery of
possession against Araneta.
Another property in Dimson's name, apparently taken from Lot 26 of the Maysilo Estate, was later sold to
Estelita Hipolito, who in turn sold the same to CLT. Said property was registered by CLT under TCT No.
T-177013, which also reflected, as its mother title, OCT No. 994 dated 19 April 1917. 12 Said property
claimed by CLT encroached on property covered by titles in the name of the Manotoks. The Manotoks
traced their titles to TCT Nos. 4210 and 4211, both issued in 1918 and both reflecting, as their mother
15
title, OCT No. 994 dated 3 May 1917. acHDTA
It is evident that both the Heirs of Dimson and CLT had primarily relied on the validity of OCT No. 994
dated 19 April 1917 as the basis of their claim of ownership. However, the Court in its 2007 Resolution
held that OCT No. 994 dated 19 April 1917 was inexistent. The proceedings before the Special Division
afforded the Heirs of Dimson and CLT alike the opportunity to prove the validity of their respective claims
to title based on evidence other than claims to title the inexistent 19 April 1917 OCT No. 994. Just as
much was observed by the Special Division:
Nonetheless, while the respective certificates of title of DIMSON and CLT
refer to OCT 994 issued on 19 April 1917 and that their previous
postulations in the present controversies had been anchored on the
supposed validity of their titles, that which emanated from OCT 994 of 19
April 1917, and conversely the invalidity of the 3 May 1917 OCT 994, the
Supreme Court has yet again allowed them to substantiate their claims on
the basis of other evidentiary proofs:
Otherwise stated, both DIMSON and CLT bear the onus of proving in this
special proceedings, by way of the evidence already presented before and
such other forms of evidence that are not yet of record, that either there
had only been an error in the course of the transcription or registration of
their derivative titles, or that other factual and legal bases existed to
validate or substantiate their titles aside from the OCT No. 994 issued on
19 April 1917. 13
Were they able to discharge such burden?
A.
We begin with the Heirs of Dimson. The Special Division made it clear that the Heirs of Dimson were
heavily reliant on the OCT No. 994 dated 19 April 1917.
[DIMSON], on the strength of Judge Sayo's Order dated 18 October dated
18 October 1977, was issued separate certificates of title, i.e., TCT Nos.
15166, 15167, 15168 and 15169, covering portions of the Maysilo Estate.
Pertinently, with respect to TCT No. 15169 of DIMSON, which covers Lot
25-A-2 of the said estate, the following were inscribed on the face of the
instrument.
"IT IS FURTHER CERTIFIED that said land was originally
registered on the 19th day of April in the year nineteen
hundred and seventeen in the Registration Book of the Office
of the Register of Deeds of Rizal, Volume NA page NA, as
Original Certificate of Title No. 994 pursuant to Decree No.
36455 issued in L.R.C. Case No. 4429 Record No. ______
This Certificate is a transfer from Original Certificate of Title
No. 994/NA, which is cancelled by virtue hereof in so far as
the above-described land is concerned. [ 14 ] HCIaDT
From the above accounts, it is clear that the mother title of TCT No.
15169, the certificate of title of DIMSON covering the now disputed Lot 25-
A-2, is OCT No. 994 registered on 19 April 1917. Manifestly, the certificate
of title issued to DIMSON, and as a matter of course, the derivative title
later issued to CLT, should both be voided inasmuch as the OCT which
they emanated had already been declared inexistent. 15
The Special Division noted that the Heirs of Dimson did not offer any explanation why their titles reflect
the erroneous date of 19 April 1917. At the same time, it rejected CLT's explanation that the transcription
of the erroneous date was a "typographical error".
As can be gleaned from the records, both DIMSON and their successor-in-
interest CLT, had failed to present evidence before this Court to prove that
there had been a mere typographical error in the transcription of their
respective titles with regard to the date of registration of OCT No. 994.
CLT specifically harps on this assertion that there had only been a
typographical error in the transcription of its title. [ 16 ] On the other
hand, while DIMSON had refused to categorically assert that there had
been such a typographical error causing the invalidity of their title, their
failure to proffer any reason or argument which would otherwise justify
why their title reflects 19 April 1917 and not 3 May 1917 leads this Court
to conclude that they simply had no basis to support their proprietary
claim.
Thus, without proffering any plausible explanation as to what led to the
erroneous entry of the registration dated of OCT 994, DIMSON are left
without any recourse but to substantiate their claim on the basis of other
evidence not presented during the proceedings below, which would
effectively prove that they had a valid proprietary claim over the disputed
properties. This is specifically true because DIMSON had previously placed
reliance on the MWSS doctrine to prove the validity of their title. 17
Absent such explanation, the Heirs of Dimson were particularly constrained to rely on the 1977 Order of
Judge Sayo, which was allegedly sourced from the 1966 Order of Judge Muoz Palma. On that issue, the
Special Division made the following determinations: cDTSHE
It should be recalled that in their appellee's brief in CA-G.R. CV No. 41883,
therein appellee Jose Dimson specifically denied the falsity of TCT No. R-
15169 alleging that the contention "is already moot and can be determined
by a controlling decision". [ 18 ] Jose Dimson expounded on his reliance
as follows:
"In Metropolitan Waterworks & Sewerage System (for brevity
MWSS) case, Jose B. Dimson's (as private respondent) title
TCT No. 15167 issued for Lot 28 on June 8, 1978 derived
from OCT No. 994 registered on April 19, 1917, is
overlapping with MWSS title TCT No. 41028 issued on July
29, 1940 derived from the same OCT 994, registered on May
3, 1917.
(Same facts in the case at bar; Jose B. Dimson' (plaintiff-
appellee) title TCT No. R-15169 issued for Lot 25-A-2, on
June 8, 1978, is overlapping with defendant-appellant's title
TCT Nos. 13574 and 21343, not derived from OCT No. 994."
[ 19 ]

So viewed, sans any proof of a mechanical error in the transcription or
annotation on their respective certificates of title, the present inquiry then
hinges on whether the Order dated 13 June 1966 issued by then Judge
Cecilia Muoz-Palma of the Court of First Instance of Rizal in Civil Case No.
4557 ["PALMA ORDER"] and Judge Sayo's Order dated 18 October 1977
["SAYOS 18 OCTOBER 1977 ORDER"], can be validated and authenticated.
It is so since the brunt of the proprietary claims of both DIMSON and CLT
has its roots on said Orders.
Perforce, in consideration of the foregoing, this leads Us to the THIRD
ISSUE as presented by the Supreme Court, to wit: IcHDCS
16
"Whether the factual and legal bases of Palma's 13
June 1966 Order and Sayo's 18 October 1977 Order
are true and valid. Assuming they are, do these orders
establish a superior right to the subject properties in
favor of the Dimsons and CLT as opposed to the
claims of Araneta and the Manotoks?"
As it is, in contending that their certificates of title could be validly traced
from the 3 May 1917 OCT No. 994, DIMSON point out that their title was
issued pursuant to a court order issued by Judge Palma in Case No. 4557
and entered in the memorandum of Encumbrance of OCT No. 994.
DIMSON also insist that TCT Nos. 8692, 21857 and 26538 were mere
microfilmed or certified copies and, therefore, inadmissible. Lastly,
DIMSON reiterated the flaws and irregularities which voided the titles of
the ARANETA in the previous proceedings and focused on the burden of
ARANETA to present evidence to defeat their titles.
The foregoing contentions of DIMSON find to factual and legal basis. * As
we see it, Sayo's 18 October 1977 Order, which apparently confirmed
Palma's 13 June 1966 Order, raised serious questions as to the validity of
the manner by which it was arrived at.
It is worthy to note that as early as 25 August 1981, counsel for the
ARANETA applied for a subpoena duces tecum addressed to the Clerk of
Court of CFI Pasig for the production of the records of LRC Case No. 4557
for purposes of determining the genuineness and authenticity of the
signature of Judge Palma and also of her Order granting the confirmation.
A certain Atty. Contreras, Officer-in-Charge of the said court, appeared and
manifested in open court that the records pertaining to the petition for
Substitution of names of Bartolome Rivera, et al. could no longer be
located inasmuch as they had passed hands from one court to another.
What is perplexing to this Court is not only the loss of the entire records of
Case No. 4557 but the admission of Judge Sayo that he had not seen the
original of the Palma Order. Neither was the signature of Judge Palma on
the Order duly proven because all that was presented was an unsigned
duplicate copy with a stamped notation of "original signed". Equally
perplexing is that while CFI Pasig had a Case No. 4557 on file, said file
pertained not to an LRC case but to a simple civil case. [ 20 ] Thus:
"Atty. Directo:
The purpose of this subpoena duces tecum is to present your Honor the
Order Order (sic) of Judge Palma in order to determine the
genuineness and authenticity of the signature of Judge
Palma in this court order and which order was a basis of a
petition in this court to be confirmed. That is the reason why
we want to see the genuineness of the signature of Judge
Palma. EACTSH
COURT:
No signature of Judge Palma was presented in this court. It was a
duplicate copy not signed. There is a stamp only of original
signed.
Atty. Directo:
That is the reason why we want to see the original.
Court:
I did not see the original also. When the records of this case was brought
here, I checked the records, there were so many pages
missing and the pages were re-numbered but then I saw the
duplicate original and there is a certification of a woman clerk
of Court, Atty. Molo. EcHIDT
Atty. Directo:
That is the reason why we want to see this document, we are surprised
why it is missing.
Court:
We are surprised also. You better ask Judge Muoz Palma.
Atty. Contreras:
May I make of record that in verifying our records, we found in our original
vault LRC application no. N-4557 but the applications were
certain Feliciano Manuel and Maria Leao involving Navotas
property because I was wondering why they have the same
number. There should be only one.
Atty. Directo:
Aside from that, are there other cases of the same number?
Atty. Contreras:
No, there should be only number for a particular case; that must be a
petition after decree record.
Atty. Ignacio:
This 4557 is not an LRC Case, it is a simple civil case.
xxx xxx xxx
Moreover, both the MANOTOKS and ARANETA insist that Palma's 13 June
1966 Order had been recalled by a subsequent Order dated 16 August
1966, ["RECALL ORDER"], [ 21 ] wherein the trial court dismissed the
motion filed by DIMSON on the court's findings that ". . . whatever portion
of the property covered by OCT 994 which has not been disposed of by
the previous registered owners have already been assigned and
adjudicated to Bartolome Rivera and his assignees, as a result of which
there is no portion that is left to be given to the herein supposed assignee
Jose Dimson".
However, We are reluctant to recognize the existence and due execution
of the Recall Order considering that its original or even a certified true
copy thereof had not been submitted by either of the two parties relying
on it despite having been given numerous opportunities to do so. STaIHc
Be that as it may, even if We are to consider that no Recall Order was ever
issued by then Judge Palma, the validity of the DIMSON titles over the
properties in the Maysilo Estate becomes doubtful in light of the fact that
the supposed "share" went beyond what was actually due to Jose Dimson
under the Compromise Agreement with Rivera. It should be recalled that
17
Palma's 13 June 1966 Order approved only the conveyance to Jose Dimson
of "25% of whatever share of Bartolome Rivera has over Lots 25, 26, 27,
28-B and 29 of OCT 994 . . . subject to availability of undisposed portion of
the said lots." [ 22 ]
In relation to this, We find it significant to note the observations contained
in the Senate Committee Report No. 1031 that, based on the assumption
that the value of the lots were equal, and "(C)onsidering that the share of
Maria de la Concepcion Vidal was only 1-189/1000 percent of the Maysilo
Estate, the Riveras who claimed to be the surviving heirs of Vidal will
inherit only 197,405.26 square meters (16,602,629.53 m2 x 1.1890%) or
19.7 hectares as their share. [ 23 ] Even if we are to base the 25% of Jose
Dimson on the 19.7 hectares allotted to the Riveras, it would appear that
Jose Dimson would only be entitled to more or less five (5) hectares of the
Maysilo Estate. Obviously, basing only on TCT No. 15169 of Dimson which
covered a land area of 50 hectares (500,000 square meters), [ 24 ] it is
undisputable that the total properties eventually transferred to Jose
Dimson went over and beyond his supposed 25% share.
What is more, Palma's 13 June 1966 Order specifically required that ". . .
whatever title is to be issued herein in favor of Jose Dimson, the same
shall be based on a subdivision plan duly certified by the Land Registration
Commission as correct and in accordance with previous orders issued in
this proceedings, said plan to be submitted to this court for final approval.
Interestingly however, despite such requirement, DIMSON did not submit
Survey Plan LRC (GLRO) Rec. No. 4429 SWO-5268 which allegedly was the
basis of the segregation of the lands, if only to prove that the same had
been duly approved and certified correct by the Land Registration
Commission. What was submitted before the RTC and this Court was only
the Subdivision Plan of Lot 25-A-2 which notably does not bear the stamp
of approval of the LRC. Even an inspection of the exhibit for CLT does not
bear this Survey Plan, which could have, at the very least, proven the
authenticity of the DIMSON title.
Indeed, We find the absence of this piece of evidence as crucial in proving
the validity of the titles of DIMSON in view of the allegation of contending
parties that since the survey plan upon which the land titles were based
contained the notation "SWO", meaning that the subdivision plan was only
a product of a "special work order", the same could not have passed the
LRC. Neither was it duly certified by the said office. 25 ISTDAH
In addition, the Special Division took note of other irregularities attending Dimson's TCT No. R-15169.
[Firstly], OCT No. 994 showed that Lot 25-A of the Maysilo Estate was
originally surveyed on "September 8-27, 1911, October 4-21 and
November 17-18, 1911". Yet, in said TCT No. R-15169, the date of the
original survey is reflected as "Sept. 8-27, 1911" and nothing more. [ 26 ]
The variation in date is revealing considering that DIMSON's titles are all
direct transfers from OCT No. 994 and, as such, would have faithfully
adopted the mother lot's data. Unfortunately, no explanation for the
variance was ever offered.
Equally worthy of consideration is the fact that TCT No. 15169 indicates
that not only was the date of original registration inexistent, but the
remarks thereon tend to prove that OCT No. 994 had not been presented
prior to the issuance of the said transfer certificate. This manifest from the
notations "NA" on the face of DIMSON's title meaning, "not available". It
bears emphasizing that the issuance of a transfer certificate of title to the
purchaser without the production of the owner's duplicate is illegal
(Rodriguez v. Llorente, 49 Phil. 826) and does not confer any right to the
purchaser (Philippine National Bank vs. Fernandez, 61 Phil. 448 [1935]).
The Registrar of Deeds must, therefore, deny registration of any deed or
voluntary instrument if the owner's duplicate is not presented in
connection therewith. (Director of Lands vs. Addison, 40 Phil. 19 [1926];
Hodges vs. Treasurer of the Phil. 50 Phil. 16 [1927]. [ 27 ]

In has also been held that, in cases where transfer certificates of title
emanating from one common original certificate of title were issued on
different dates to different persons or entities covering the same land, it
would be safe to conclude that the transfer certificate issued at an earlier
date along the line should prevail, barring anomaly in the process of
registration. [ 28 ] Thus, "(w)here two certificates purport to include the
same land, the earlier in date prevails. . . . . In successive registration,
where more than one certificate is issued in respect of a particular estate
or interest in land, the person is deemed to hold under the prior certificate
who is the holder or whose claim is derived directly from the person who
was the holder of the earliest certificate issued in respect thereof. . . ." [
29 ]
xxx xxx xxx
Still another indication of irregularity of the DIMSON title over Lot No. 25-A
is that the issuance of the Sayo Order allegedly confirming the Palma
Order was in itself suspect. Gleaning from the records, DIMSON filed the
Motion only on 10 October 1977, or eleven (11) years after obtaining the
supposed sanction for the issuance of titles in this name. Besides, what
was lodged by Jose Dimson before the sala of then Judge Palma was not a
simple land registration case wherein the only purpose of Jose Dimson was
to establish his ownership over the subject parcels of land, but, as
reflected in the Palma Order, the subject of the case was the confirmation
of Jose Dimson's claim over the purported rights of Rivera in the disputed
properties. The case did not partake of the nature of a registration
proceeding and thus, evidently did not observe the requirements in land
registration cases. Unlike in a land registration case, therefore, Jose
Dimson needed to file an action before Judge Sayo to seek "confirmation"
of Palma's Order dated 13 June 1966.
So viewed the general rule proscribing the application of laches or the
statute of limitations in land registration cases, [ 30 ] as well as Section 6,
Rule 39 of the Rules of Court, in relation to its provisions on revival of
judgment applies only to ordinary civil actions and not to other or
extraordinary proceedings such as land registration cases, is clearly not
applicable in the present case. The legal consequences of laches as
committed by DIMSON and their failure to observe the provisions of Rule
39 should, therefore, find application in this case and thus, the
confirmation of DIMSON's title, if any, should fail.
Parenthetically, the allegations of DIMSON would further show that they
derive the validity of their certificates of title from the decreased Jose
Dimson's 25% share in the alleged hereditary rights of Bartolome Rivera
["RIVERA"] as an alleged grandson of Maria Concepcion Vidal ["VIDAL"].
However, the records of these cases would somehow negate the rights of
Rivera to claim from Vidal. The Verification Report of the Land Registration
Commission dated 3 August 1981 showed that Rivera was 65 years old on
17 May 1963 (as gathered from the records of Civil Case Nos. 4429 and
4496). [ 31 ] It can thus be deduced that, if Rivera was already 65 years
old in 1963, then he must have been born around 1898. On the other
18
hand, Vidal was only nine (9) years in 1912; hence, she could have been
born only on 1905. This alone creates an unexplained anomalous, if not
ridiculous, situation wherein Vidal, Rivera's alleged grandmother, was
seven (7) years younger than her alleged grandson. Serious doubts existed
as to whether Rivera was in fact an heir of Vidal, for him to claim a share
in the disputed portions of the Maysilo Estate. 32
These findings are consonant with the observations raised by Justice Renato Corona in his Concurring
and Dissenting Opinion on our 2007 Resolution. To wit: TcDaSI
TCT No. T-177013 covers Lot 26 of the Maysilo Estate with an area of
891,547.43 sq. m. It was a transfer from TCT No. R-17994 issued in the
name of Estelita I. Hipolito. On the other hand, TCT No. R-17994 was a
transfer from TCT No. R-15166 in the name of Jose B. Dimson which, in
turn, was supposedly a direct transfer from OCT No. 994 registered on
April 19, 1917.
Annotations at the back of Hipolito's title revealed that Hipolito acquired
ownership by virtue of a court order dated October 18, 1977 approving the
compromise agreement which admitted the sale made by Dimson in her
favor on September 2, 1976. Dimson supposedly acquired ownership by
virtue of the order dated June 13, 1966 of the CFI of Rizal, Branch 1 in
Civil Case No. 4557 awarding him, as his attorney's fees, 25% of whatever
remained of Lots 25-A, 26, 27, 28 and 29 that were undisposed of in the
intestate estate of the decedent Maria de la Concepcion Vidal, one of the
registered owners of the properties covered by OCT No. 994. This order
was confirmed by the CFI of Caloocan in a decision dated October 13,
1977 and order dated October 18, 1977 in SP Case No. C-732.
However, an examination of the annotation on OCT No. 994, particularly
the following entries, showed:
AP-6665/0-994 Venta: Queda cancelado el presente
Certificado en cuanto a una extencion superficial de 3,052.93
metros cuadrados y 16,512.50 metros cuadrados, y descrita
en el lote no. 26, vendida a favor de Alejandro Ruiz y
Mariano P. Leuterio, el primer casado con Deogracias
Quinones el Segundo con Josefa Garcia y se ha expedido el
certificado de Titulo No. 4210, pagina 163, Libro T-22.
Fecha del instrumento Agosto 29, 1918
Fecha de la inscripcion September 9, 1918
10.50 AM
AP-6665/0-994 Venta: Queda cancelado el presente
Certificado el cuanto a una extencion superficial de
871,982.00 metros cuadrados, descrita en el lote no. 26,
vendida a favor de Alejandro Ruiz y Mariano P. Leuterio, el
primer casado con Deogracias Quinones el segundo con
Josefa Garcia y se ha expedido el certificado de Titulo No.
4211, pagina 164, Libro T-22. IcTEaC
Fecha del instrumento Agosto 25, 1918
Fecha de la inscripcion September 9, 1918
10:50-AM
Based on the description of Lot No. 26 in OCT No. 994, it has an area of
891,547.43 sq. m. which corresponds to the total area sold in 1918
pursuant to the above-cited entries. Inasmuch as, at the time the order of
the CFI of Rizal was made on June 13, 1966, no portion of Lot No. 26
remained undisposed of, there was nothing for the heirs of Maria de la
Concepcion Vidal to convey to Dimson. Consequently, Dimson had nothing
to convey to Hipolito who, by logic, could not transmit anything to CLT.
Moreover, subdivision plan Psd-288152 covering Lot No. 26 of the Maysilo
Estate described in Hipolito's certificate of title was not approved by the
chief of the Registered Land Division as it appeared to be entirely within
Pcs-1828, Psd-5079, Psd-5080 and Psd-15345 of TCT Nos. 4210 and 4211.
How Hipolito was able to secure TCT No. R-17994 was therefore
perplexing, to say the least.
All these significant facts were conveniently brushed aside by the trial and
appellate courts. The circumstances called for the need to preserve and
protect the integrity of the Torrens system. However, the trial and
appellate courts simply disregarded them. 33
The Court thus adopts these findings of the Special Division on the validity of Jose Dimson's titles, which
he obtained consequent to the 1977 Order of Judge Sayo. Consequently, we cannot give due legal
recognition to any and all titles supposedly covering the Maysilo Estate obtained by Dimson upon the
authority of either the purported 1966 Order of Judge Muoz-Palma or the 1977 Order of Judge Sayo.
B.
Indubitably, as between the titles of ARANETA and the MANOTOKS and their predecessors-in-interest, on
one hand, and those of DIMSON, on the other, the titles held by ARANETA and the MANOTOKS must
prevail considering that their titles were issued much earlier than the titles of the latter.
Our findings regarding the titles of Jose Dimson necessarily affect and even invalidate the claims of all
persons who seek to derive ownership from the Dimson titles. These include CLT, which acquired the
properties they laid claim on from Estelita Hipolito who in turn acquired the same from Jose Dimson. Just
as much was concluded by the Special Division as it evaluated CLT's claims. DEIHAa
For its part, CLT contended that even at the trial court level, it maintained
that there was only one OCT No. 994 from where its claim emanates. It
argued that its case against the MANOTOKS, including that of STO. NIO,
was never decided based on the doctrines laid down in Metropolitan
Waterworks and Sewerage System v. Court of Appeals [ 34 ] and Heirs of
Gonzaga v. Court of Appeals. [ 35 ]
Before this Special Division, CLT insists that the MANOTOKS failed to
submit "new" competent evidence and, therefore, dwelling on the alleged
flaws of the MANOTOK's titles, "the findings and conclusions of the court-
appointed commissioners as adopted by the trial court, then upheld by the
Honorable Court in its Decision dated 28 September 1995 and finally
affirmed in the Supreme Court's Decision dated 29 November 2005,
therefore stand, as there is no reason to disturb them".
Furthermore, CLT contends that the Orders of Judge Palma and Judge
Sayo are no longer open to attack in view of their finality. Lastly, CLT
asserts that the properties covered by the MANOTOKS' titles and those
covered by the expropriation proceedings did not property pertain to and
were different from Lot 26 owned by CLT. Thus, it maintains that the
MANOTOKS cannot use as basis for the validity of their titles the
expropriation undertaken by the Government as a means of staking their
claims.
19
To restate, CLT claims the 891,547.43 square meters of land covered by
TCT No. T-177013 [ 36 ] located in Malabon, Caloocan City and
designated as "Lot 26, Maysilo Estate, LRC Swo-5268". TCT No. T-177013
shows that its mother titles is OCT No. 994 registered on 19 April 1917.
Tracing said claim, Estelita Hipolito executed a Deed of Sale with Real
Estate Mortgage in favor of CLT on 10 December 1988. By virtue of this
transfer, Hipolito's TCT No. R-17994 [ 37 ] was cancelled and in lieu
thereof, CLT's TCT No. 223677/R-17994 of TCT No. R-17994. Hipolito, on
the other hand, was a transferee of the deceased Dimson who was
allegedly the registered owner of the subject land on the basis of TCT No.
15166. SIcTAC

In view of the foregoing disquisitions, invalidating the titles of DIMSON,
the title of CLT should also be declared a nullity inasmuch as the nullity of
the titles of DIMSON necessarily upended CLT's propriety claims. As earlier
highlighted, CLT had anchored its claim on the strength of Hipolito's title
and that of DIMSON's TCT No. 15166. Remarkably and curiously though,
TCT No. 15166 was never presented in evidence for purposes of tracing
the validity of titles of CLT. On this basis alone, the present remand
proceedings remain damning to CLT's claim of ownership.
Moreover, considering that the land title of CLT carried annotations
identical to those of DIMSON and consequently included the defects in
DIMSON's title, the fact that whatever typographical errors were not at
anytime cured by subsequent compliance with the administrative
requirements or subjected to administrative correction bolsters the
invalidity of the CLT title due to its complete and sole dependence on the
void DIMSON title. 38
IV.
The task of the Special Division was not limited to assessing the claims of the Heirs of Dimson and CLT.
We likewise tasked the Special Division to ascertain as well the validity of the titles held by the Manotoks
and Araneta, titles which had been annulled by the courts below. Facially, these titles of the Manotoks
and Araneta reflect, as their valid mother title, OCT No. 994 dated 3 May 1917. Nonetheless, particular
issues were raised as to the validity of the Manotok and Araneta titles independent of their reliance on
the 3 May 1917 OCT No. 994 vis--vis the inexistent 19 April 1917 OCT No. 994.
A.
We begin by evaluating the Araneta titles. The Special Division quoted the observations of the trial court,
which upheld Dimson's claim over that of Araneta, citing the following perceived flaws of TCT Nos. 26538
and 26539, from which Araneta derived its titles, thus: ISAcHD
Let us now examine TCT 26538 and TCT 26539 both in the name of Jose
Ma. Rato from where defendant was said to have acquired TCT 13574 and
TCT 7784 now TCT 21343 in the name of Araneta and the other
documents related thereto:
1)Perusal of TCT 26538 shows that its Decree No. and Record No. are both
4429. In the same vein, TCT 26539 also shows that it has Decree No. 4429
and Record No. 4429.
However, Decree No. 4429 was issued by the Court of First Instance,
Province of Isabela (Exhibit I) and Record No. 4429, issued for Ordinary
Land Registration Case, was issued on March 31, 1911 in CLR No. 5898,
Laguna (Exhibit 8, 8-A Bartolome Rivera et al.)
How then could TCT No. 26538 and TCT No. 26539 both have Decree No.
4429 and Record No. 4429, which were issued in Court of First Instance,
Province of Isabela and issued in Laguna, respectively.
2)TCT No. 26538 and TCT No. 26539 in the name of Jose Ma. Rato are not
annotated in the Original Certificate of Title 994, where they were said to
have originated.
3)The Escritura de Incorporacion de Philippine Land Improvement
Company (Exhibit I) executed on April 8, 1925 was only registered and
was stamped received by the Office of the Securities and Exchange
Commission only April 29, 1953 when the Deed of Sale & Mortgage was
executed on August 23, 1947 (Exh. 5 defendant) and the Novation of
Contract, Deed of Sale and Mortgage executed on November 13, 1947
(Exh. M). So, that when Philippine Land Improvement was allegedly given
a special power of attorney by Jose Ma. Rato to represent him in the
execution of the said two (2) documents, the said Philippine Land
Improvement Company has not yet been duly registered.
4)TCT 26538 and 26538 and TCT 26539 both in the name of Jose Ma.
Rato, both cancel 21857 which was never presented in Court if only to
have a clear tracing back of the titles of defendant Araneta.
5)If the subject matter of the Deed of Sale & Mortgage (Exhibit 5
defendant) is TCT 26539, why is it that TCT 13574 of defendant Araneta
cancels TCT 6196 instead of TCT 26539. That was never explained. TCT
6196 was not even presented in Court. IDASHa
6)How come TCT 26538 of Jose Ma. Rato with an area of 593,606.90 was
cancelled by TCT 7784 with an area of only 390,282 sq.m.
7)How was defendant Araneta able to have TCT 7784 issued in its name,
when the registration of the document entitled Novation of Contract, Deed
of Sale & Mortgage (Exhibit M) was suspended/denied (Exhibit N) and no
title was received by the Register of Deeds of Pasig at the time the said
document was filed in the said Office on March 4, 1948 (Exhibit N and N-
1).
Under Sec. 55 of Land Registration Act (Act No. 496) now Sec. 53 of
Presidential Decree No. 1529, no new certificate of title shall be entered,
no memorandum shall be made upon any certificate of title by the register
of deeds, in pursuance of any deed or other voluntary instrument, unless
the owner's duplicate certificate is presented for such endorsement.
8)The sale by Jose Ma. Rato in favor of defendant Araneta is not reflected
on the Memorandum of Encumbrances of TCT 26538 (Exhibit 7-defendant)
meaning that TCT 26538 still exists and intact except for the
encumbrances annotated in the Memorandum of Encumbrances affecting
the said title (Exhibits 16, 16-A and 16-N David & Santos)
9)In the encumbrances annotated at the back of TCT 26539 (Exhibit 4-
defendant) there appears under entry No. 450 T-6196 Victoneta,
Incorporated covering parcel of land canceling said title (TCT 26539) and
TCT 6196 was issued (. . .) which could have referred to the Deed (sic) of
Sale and Mortgage of 8-23-47 (Exhibit 5-defendant) entered before Entry
5170 T-8692 Convenio Philippine Land Improvement Company, with Date
of Instrument: 1-10-29, and Date of Inscription: 9-21-29.
In TCT 26838 this Entry 5170 T-8692 Convenio Philippine Land
20
Improvement Company (Exhibit 16-J-1) appears, but the document,
Novation of Contract, Deed of Sale & Mortgage dated November 13, 1947
(Exhibit M) does not appear. IcaEDC
Entry marked Exhibit 16-J-1 on TCT 26538 shows only the extent of the
value of P42,000.00 invested by Jose Ma. Rato in the Philippine Land
Improvement Company. Said entry was also entered on TCT 26539.
The Court also wonders why it would seem that all the documents
presented by defendant Araneta are not in possession of said defendant,
for according to witness Zacarias Quintan, the real estate officer of the
said defendant Araneta since 1970, his knowledge of the land now in
possession of defendant Araneta was acquired by him from all its
documents marked in evidence which were obtained only lately when they
needed for presentation before this Court. [ 39 ] 40
The Special Division then proceeded to analyze these factual contentions, and ultimately concluded that
the Araneta claim to title was wholly valid. We adopt in full the following factual findings of the Special
Division, thus:
As for the proprietary claim of ARANETA, it maintains that it has
established by direct evidence that its titles were validly derived from OCT
No. 994 dated 3 May 1917. With regard to the imputed flaws, it
asseverates that these were unfounded and thus, labored to refute all of
them. ARANETA further expounded on the nullity of the Palma and Sayo
Orders which was the basis of DIMSON's titles.
The documentary exhibits it proffered traced its certificates of title to OCT
No. 994 registered on 3 May 1917. From the titles submitted, its
predecessor-in-interest was Jose Ma. Rato y Tuazon ["RATO"], one of the
co-heirs named in OCT No. 994, who was allotted the share of nine and
five hundred twelve one thousandths (9-512/1000) percent share of the
Maysilo Estate. [ 41 ] For this reason, to ascertain the legitimacy of the
derivative title of ARANETA, the origin and authenticity of the title of RATO
need to be reassessed.
Verily, attesting to RATO's share on the property, Entry No. 12343/O-994
of the Owner's Duplicate Copy of OCT No. 994, records the following:
"12343/O-994 Auto: Jose Rato y Tuason Queda
cancelado el presente seartificado en cuanto a una estension
superficial de 1,405,725.90 metro Cuadrados mas o menos
descrita en el Lote No. 25-A-3, an virtud del auto dictado por
el Juzgado de Primera Instancia de Riza, de fecha 28 de Julio
de 1924, y que en au lugar se had expedido el Certificados
de Titulo No. 8692, folio 492 del Tomo T-35 del Libro de
Certicadads de Transferencia. ASCTac
Date of Instrument Julio 28, 1924.
Date of Inscription Agosto 1, 1024 10:19 a.m.
SGD. GLICERIO OPINION, Register of deeds
Agosto 19, 1924" [ 42 ]
In accordance with the decree, RATO was issued on 1 August 1924, TCT
No. 8692 [ 43 ] which covers "Lote No. 25 A-3 del plano del subdivision,
parte del Lote No. 25-A, plano Psu-(not legible), "Hacienda de Maysilo",
situado en el Munisipio de Caloocan, Provincia del Rizal . . . ." [ 44 ] The
parcel of land covers an approximate area of "UN MILLION
CUATROCIENTOS CINCO MIL SETECIENTOS VEINTICINCO metros
cuadrados con NOVENTA decimetros cuadrados (1,405,725.90) mas o
menos". As reflected under Entry No. 14517. . . T-8692, [ 45 ] the parcel
of land covered under this certificate of title was subdivided into five (5)
lots under subdivision plan Psd-6599 as per Order of the court of First
Instance of Rizal. Consequently, TCT Nos. 21855, 21856, 21857, 21858
and 21859 were issued.
Focusing on TCT No. 21857 issued on 23 May 1932, this certificate of title
issued in RATO's name, [ 46 ] cancelled TCT No. 8692 [ 47 ] with respect
to the property it covers. On its face, TCT No. 21857, [ 48 ] was a
derivative of OCT No. 994 registered on 3 May 1917. It covers Lot No. 25
A-3-C of subdivision plan Psd-6589, being a portion of Lot No. 25-A-3,
G.L.R.O Record No. 4429. Thereafter, TCT No. 21857 was cancelled by
TCT No. 26538 [ 49 ] and TCT No. 26539 [ 50 ] which were both issued
in the name of Jose Ma. Rato y Tuazon on 17 September 1934.
With respect to TCT No. 26539, the certificate of title showed that it
covered a parcel of land designated as Section No. 2 of the subdivision
plan Psd-10114, being a portion of Lot 25-A-3-C having an approximate
area of 581,872 square meters. [ 51 ] Thereafter, TCT No. 26539 was
cancelled by TCT No. 6196 [ 52 ] whose registered owner appears to be a
certain Victoneta, Inc. This parcel of land has an area of 581,872 square
meters designated as section No. 2 of subdivision plan Psd-10114, being a
portion of Lot 25-A-3-C. CAcIES

As shown on its face, TCT No. 6196 issued on 18 October 1947 in the
name of Victoneta, Inc. and its mother title were traced from OCT No. 994
registered on 3 May 1917. Later, TCT No. 6196 was cancelled, and in lieu
thereof, TCT No. 13574 was issued in favor of Araneta Institute of
Agriculture on 20 May 1949. [ 53 ] It covers a parcel of land designated as
section No. 2 of subdivision plan Psd-10114, being a portion of Lot 25-A-3-
C. It has an aggregate area of 581,872 square meters.
On the other hand, appearing under Entry No. 16086/T-No. 13574 of TCT
No. 6196 is the following:
"Entry No. 16086/T-No. 13574 SALE in favor of the
ARANETA INSTITUTE OF AGRICULTURE, vendee: Conveying
the property described in this certificate of title which is
hereby cancelled and issuing in lieu thereof Transfer
Certificate of Title No. 13574, page 74, Book T-345 in the
name of the vendee. (Doc. No. 149, page 98, Book II, S. of
1949 of Notary Public for Manila, Hospicio B. Bias).
Date of Instrument May 18, 1949
Date of the Inscription May 30, 1949 at 11:00 a.m. [ 54 ]
TCT No. 26538 [ 55 ] in turn showed on its face that it covers a parcel of
land designated as Section 1 of subdivision plan Psd-10114 being a portion
of Lot 25-A-3-C having an area of 592,606.90 square meters. [ 56 ]
On 4 March 1948, TCT No. 26538 was cancelled by TCT No. 7784, which
was issued in favor of Araneta Institute of Agriculture. TCT No. 7784
21
covers four (4) parcels of land with an aggregate area of 390,282 square
meters. [ 57 ] It would appear from the records of CA-G.R. SP No. 34819
consolidated with CA-G.R. CV No. 41883 that TCT No. 7784 was eventually
cancelled by TCT No. 21343. [ 58 ] As per attachment of ARANETA in its
Answer dated 6 March 1980 filed in Civil Case No. 8050, a mere copy of
TCT No. 21343 showed that it covers a parcel of land designated as Lot 6-
B of the subdivision plan Psd-24962 being a portion of Lot 6, described as
plan Psd-21943, G.L.R.O. Record No. 4429 with an approximate area of
333,377 square meters. [ 59 ] However, for reasons unknown, a copy of
TCT No. 21343, whether original or certified true copy thereof, was not
submitted before this Court. aAEHCI
In summation, ARANETA had shown that RATO, as one of the co-owners
of the property covered by OCT NO. 994, was assigned Lot No. 25-A-3. His
evidence of ownership is reflected on TCT No. 8692 issued in his name.
RATO held title to these parcels of land even after its subdivision in the
1930's. Further subdividing the property, RATO was again issued TCT No.
21857, and later TCT Nos. 26538 and 26539, still covering Lot No. 25 A-3-
C. In all his certificates of title, including those that ultimately passed
ownership to ARANETA, the designation of the lot as either belonging to or
portions of Lot 25-A-3 was retained, thereby proving identity of the land.
More importantly, the documentary trail of land titles showed that all of
them were derived from OCT No. 994 registered on 3 May 1917. For
purposes of tracing ARANETA's titles to Oct No. 994, it would appear that
the evidence presented ultimately shows a direct link of TCT Nos. 7784
and 13574 to said mother title. Suffice it to state, the origin and legitimacy
of the proprietary claim of ARANETA had been well substantiated by the
evidence on record and on this note, said titles deserve validation.
Under the guidelines set, we shall now proceed to evaluate the imputed
flaws which had been the previous bases of the trial court in invalidating
ARANETA's titles.
One of the flaws observed on the titles of ARANETA's predecessor-in-
interest was that TCT No. 26538 and TCT No. 26539 in Rato's name refer
to Decree No. 4429 and Record No. 4429, as basis of their issuance. This
is being questioned inasmuch as Decree No. 4429 refers to a decree issued
by the CFI of Isabela while Record No. 4429 was issued for ordinary Land
Registration Case No. 31 March 1911 in CLR No. 5898 of Laguna.
Explaining this discrepancy, ARANETA insisted that the same was a mere
typographical error and did not have any effect on the validity of their title.
It further contended that the number "4429" was the case number of
Decree No. 36455 and was used interchangeably as the record number.
This Court finds that the incorrect entry with respect to the Decree and
Record Number appearing on the title of ARANETA's predecessor-in-
interest cannot, by itself, invalidate the titles of ARANETA's predecessors-
in-interest and ultimately, that of ARANETA. To the mind of this Court, the
incorrect entries alluded to would not have the effect of rendering the
previous titles void sans any strong showing of fraudulent or intentional
wrongdoing on the part of the person making such entries. Fraud is never
presumed but must be established by clear and convincing evidence. [ 60
] The strongest suspicion cannot sway judgment or overcome the
presumption of regularity. The sea of suspicion has no shore, and the
court that embarks upon it is without rudder or compass. [ 61 ] cDCHaS
The Supreme Court, in Encinas v. National Bookstore, Inc. [ 62 ]
acknowledged that certain defects on a certificate of title, specifically, the
interchanging of numbers, may occur and "it is certainly believable that
such variance in the copying of entries could be merely a typographical or
clerical error". In such cases, citing with approval the decision of the
appellate court, the technical description in the title should prevail over the
record number. [ 63 ]
Thus, what is of utmost importance is that the designation and the
technical description of the land, as stated on the face of the title, had not
been shown to be erroneous or otherwise inconsistent with the source of
titles. In ARANETA's case, all the titles pertaining to Lot No. 25 had been
verified to be an offshoot of Decree No. 36455 and are all located in
Tinajeros, Malabon. At any rate, despite the incorrect entries on the title,
the properties, covered by the subject certificates of title can still be
determined with sufficient certainty.
It was also opined that TCT No. 26538 and TCT No. 26539 in the name of
RATO had not been annotated on OCT No. 994 from which said titles had
supposedly originated. It should be stressed that what partially cancelled
OCT No. 994 with respect to this subject lot were not TCT Nos. 26538 and
26539 but TCT No. 8692 issued on 1 August 1924. In fact, TCT Nos.
26538 and 26539 are not even the immediate predecessors of OCT No.
994 but were mere derivatives of TCT No. 21857. Logically therefore,
these two certificates of title could not have been annotated on OCT No.
994, they not being the preceding titles.
In any case, a perusal of OCT No. 994 shows an entry, which pertains to
Jose Ma. Rato but, on account of the physical condition of the copy
submitted to this Court, the entry remains illegible for us to make a
definite conclusion. [ 64 ] On the other hand, Entry No. 12343/O-994
found on the Owner's Duplicate Copy of OCT No. 994 specifically recorded
the issuance of TCT No. 8692 over Lot No. 25-A-3. [ 65 ] CTDAaE
The other flaws noted on ARANETA's certificates of title pertained to its
failure to present TCT Nos. 21857, 6196 and 21343. As we have discussed,
ARANETA offered in evidence a certified microfilm copy of TCT No. 21857
and a certified true copy of TCT No. 6196 marked as Exhibits 5-A1A and
19-A1A, respectively. However, it failed to submit a copy of said TCT No.
21343. Be that as it may, we will not hasten to declare void TCT No. 7784
as a consequence of such omission, especially so since TCT No. 21343
appears to be a mere derivative of TCT No. 7784. Given that the validity of
TCT No. 7784 had been preponderantly proven in these proceedings, the
authenticity of said title must be sustained. Besides, ARANETA's failure to
submit TCT No. 21343 had never been put into issue in these proceedings.
With respect to the difference in the area of more than 200,0000 * square
meters between TCT No. 7784 and TCT No. 26538, we find that the trial
court failed to consider the several conveyances of portions of TCT No.
26538 before they finally passed on to ARANETA. Thus, on the
Memorandum of Encumbrance of TCT No. 26538, it is apparent that
portions of this piece of land had been sold to various individuals before
the same were transferred to ARANETA on 4 March 1948. Naturally, since
the subject land had been partially cancelled with respect to the portion
disposed of, it could not be expected that the area of TCT No. 26538 will
remain the same at the time of its transfer to ARANETA. Even assuming
that the entire area covered by TCT No. 26538 had been disposed of, this
fact alone, cannot lend * us to conclude that the conveyance was
irregular. An anomaly exists if the area covered under the derivative title
will be much more than its predecessor-in-interest. Evidently, this is not so
in the case before us.
22
The trial court, relying on Exhibit "N", further asserted that ARANETA
should not have been issued TCT No. 7784 considering that the
registration of the Novation of Contract, deed of Sale & Mortgage was
suspended/denied and no title was received by the Register of Deeds of
Pasig at the time the said document was filed in the said Office on March
4, 1948. A perusal of Exhibit "N" submitted before the trial court, shows
that the suspension or denial was merely conditional considering that the
person seeking registration had give days * within which to correct the
defects before final denial thereof. As we see it, the Notice merely
contained a warning regarding the denial of the registration of the
voluntary deed but, in no way, did it affect the vested rights of ARANETA
to be land. The fact that the title to the land was subsequently issued free
from any notation of the alluded defect creates a reasonable presumption
that ARANETA was in fact able to comply with the condition imposed. This
is especially true since the notice itself contained a note, "Just Completed",
written across the face of the letter.

Records also reveal the RTC's observation with regard to Araneta's failure
to disprove the result of the plotting made on the subject land (Exhibit K)
to the effect that TCT 26538 overlaps 1/2 portion of TCT 15159 and TCT
26539 also overlaps the other 1/2 portion of said TCT R-15169. The trial
court further noted that "TCT R-15169 (Jose Dimson) and TCT 26539
(Jose Rato) and TCT 21343 (Araneta) are overlapping each other within
Lot 25-A. That portion of TCT R-15169 (Jose Dimson) along bearing
distance points to 17 to 18 to 19 to 20 to 21 to 1 and 2 shaded in yellow
color in the Plan is not covered by TCT 21343 (Araneta)". [ 66 ] SCHTac
Scrutinizing Exhibit "K", it becomes apparent that the said evidence relied
upon was only a private survey conducted by Geodetic Engineer Reggie P.
Garcia which had not been duly approved by the Bureau of Lands and was
based only on photocopies of relevant land titles. [ 67 ] What is more, said
geodetic engineer also failed to adequately explain his observations,
approach and manner of plotting the relative positions of the lots. [ 68 ]
From all indications, the conclusions reached by said geodetic engineer
were anchored on unfounded generalizations.
Another defect cited on ARANETA's title was the absence of any entry on
the Memorandum of Encumbrances of TCT No. 26538 of the alleged sale
between RATO and ARANETA. As pointed out by ARANETA, the copy of
TCT No. 26538 submitted to the trial court contained entries only up to the
year 1947, thus, explaining the (1) lack of entry with regard to the
issuance of TCT No. 7784 in favor of ARANETA considering that the same
was issued a year later and; (2) entry pertaining to Convenio Philippine
Land Improvement Company which was entered way back on 21 August
1929.
Nonetheless, it still cannot be denied that Rato and ARANETA together
with Don Salvador Araneta, entered into a voluntary agreement with the
intention of transferring the ownership of the subject property. Moreover,
no conclusion should have been reached regarding the total cancellation of
TCT No. 26538 inasmuch as TCT No. 7784 cancelled the former certificate
of title to the extent only of Three Hundred Ninety Thousand Two Hundred
Eighty Two (390,282) square meters.
Notably also, with the evident intent to discredit and refute the title of
ARANETA, DIMSON submitted TCT Nos. 26538 [ 69 ] and 21857, [ 70 ]
which are both derivatives of OCT No. 994 registered on 3 May 1917 and
cover parcels of land located in Malabon, Rizal. However, these certificates
of title reflect different registered owners and designation of the land
covered.
Pertinently, Exhibit "M-Dimson" relating to TCT No. 26538, registered on
12 June 1952, points to one Angela Bautista de Alvarez as the registered
owner of a 240 square meter of land designated as Lot No. 19, Block 14 of
the subdivision plan Psd-5254 being a portion of Lot No. 7-A-1-A. This
certificate of title cancels TCT No. 14112/T-348 and refers to a certain TCT
No. 30473 on the inscriptions. EIAScH
Exhibit "N-Dimson", on the other hand, pertaining to TCT No. 21857 was
issued on 30 March 1951 to one Angela I. Tuason de Perez married to
Antonio Perez. This certificate of Title covers a parcel of land described as
Lot No. 21, Block 16 of the consolidation and subdivision plan Pcs-140,
G.L.R.O. Record No. 4429. It has an area of 436 square meters and
cancels TCT No. 21856.
Exhibit "Q-Dimson" [ 71 ] consisting of TCT No. 8692 covers two parcels
of land designated as Lot Nos. 1 and 2 of Block No. 44 of the consolidation
Subdivision Plan Pcs-188 with a total area of 3,372 square meters. It was
issued to Gregorio Araneta, Incorporated on 7 May 1948. This certificate of
title cancelled TCT No. 46118.
Comparing these titles to those of the ARANETA, it is apparent that no
identity of the land could be found. The Supreme Court, in the case of
Alonso v. Cebu City Country Club, Inc. [ 72 ] agreeing with the Court of
Appeals' dissertation in said case, ruled that there is nothing fraudulent for
a certificate of title to bear the same number as another title to another
land. On this score, the Supreme Court elucidated as follows:
"On the question that TCT No. RT-1310 (T-1151) bears the
same number as another title to another land, we agree with
the Court of Appeals that there is nothing fraudulent with the
fact that Cebu Country Club, Inc.'s reconstituted title bears
the same number as the title of another parcel of land. This
came about because under General Land Registration Office
(GLRO) Circular No. 17, dated February 19, 1947, and
Republic Act No. 26 and Circular No. 6, RD 3, dated August
5, 1946, which were in force at the time the title was
reconstituted on July 26, 1946, the titles issued before the
inauguration of the Philippine Republic were numbered
consecutively and the titles issued after the inauguration
were numbered also consecutively starting with No. 1, so
that eventually, the titles issued before the inauguration were
duplicated by titles issued after the inauguration of the
Philippine Republic . . . ." cCaSHA
Parenthetically, in their Motion for Partial Reconsideration of this Court's
Resolution dated 30 October 2008, DIMSON objected to the admissibility
of Exhibits 4-A1A to 7-A1A on the ground that ARANETA failed to submit
the original copies of these certificates of title and contended that the
"originals" contain different "contents" from their own Exhibits M, N and Q.
[ 73 ] The fact that the entries contained in ARANETA's pieces of evidence
are different from that of DIMSON's do not automatically make ARANETA's
exhibits inferior replications or a confirmation of their falsity. Interestingly,
the objection regarding the non-submission of the "original copy" had not
been raised by DIMSON in their Comments/Objections to Consolidated
Formal Offer of Evidence (Of Araneta Institute of Agriculture, Inc.). [ 74 ]
In any case, we find the objections unwarranted considering that certified
true copies or certified microfilm copies of Exhibits 4-A1A to 7-A1A had
23
been submitted by ARANETA in these proceedings.
Lastly, on the alleged non-registration of Philippine Land Improvement
Company at the time the special power of attorney was executed by Jose
Ma. Rato to represent him in the execution of the deed of conveyances,
the same only proves that Philippine Land Improvement Company was not
yet registered and this does not go as far as proving the existence or non-
existence of the company at which time it was executed. In effect, the
company was not precluded to enter into contracts and be bound by them
but it will do so at the risk of the adverse effects of non-registration under
the law.
Ultimately, the question of whether the aforesaid certificates of title
constitute as clouds on ARANETA's titles are not for this Court to rule upon
for purposes of the present remand. Needless to state, it is not for the
Heirs of Dimson to rely on the weakness of ARANETA's titles and profit
from it. Rather, they should have focused on the strength of their own
titles since it is not within our office to decide in whose hands the
contested lands should go, our task being merely to trace back the parties'
claims to OCT No. 994 dated 3 May 1917. 75
There is no question that the Araneta titles were derived from OCT No. 994 dated 3 May 1917,
particularly from the share of Jose Ma. Rato y Tuazon, one of the co-heirs named in OCT No. 994. The
Special Division correctly assessed, among others, the reference to Decree No. 4429 and Record No.
4429 in some of the antecedent titles of Araneta 76 as mere clerical errors that could not have
invalidated said titles, "4429" being the case number of Decree No. 36455, and the designation and the
technical description of the land on those titles not having been shown to be erroneous or variant with
the source title. The Special Division also correctly considered that the trial court had failed to take into
account the several conveyances of TCT No. 26538 before it was ultimately transferred to Araneta in
1948, which explain the difference in area between TCT No. 7784 and TCT No. 26538. The imputed
overlap of TCT No. 26538 and TCT No. 26539 with the titles held by Dimson was based on a private
survey which had not been duly approved by the Bureau of Lands. The alleged absence of any entry on
the Memorandum of Encumbrances of TCT No. 26538 of the sale of the property between Rato and
Araneta did not, according to the Special Division, discount the fact that Rato and Araneta entered into a
voluntary agreement with the intention of transferring the ownership of the subject property. Finally, the
Special Division noted that the titles derived from OCT No. 994, which Dimson had submitted as evidence
to discredit the Araneta claim, pertain to properties wholly different from those covered by the Araneta
titles.
There is no cause to dispute the factual findings and conclusions of the Special Division on the validity of
the Araneta titles, and we affirm the same. HEIcDT
B.
It appears that the claim to title of the Manotoks is somewhat more controversial. The Special Division
did not discount the fact that there could have been flaws in some of the intervening titles between the 3
May 1917 OCT No. 994 and the present titles of the Manotoks. However, the significant event was the
expropriation proceedings undertaken by the Republic of the Philippines sometime in 1947. At least some
of the titles in the name of the Manotoks were sourced from the titles issued to and subsequently
distributed by the Republic. The Special Division explained the milieu in full:
VALIDITY OF THE MANOTOK TITLES
The notation under Entry No. 6655/O-994, found on page 17 of OCT 994
of the Owner's Duplicate Copy, shows that Lot No. 26 had been a subject
of sale in favor of Alejandro Ruiz and Mariano P. Leuterio. [ 77 ] The
notation reads:
"Ap. 6655/O-994 Venta: Queda Cancelado el presente
Certificado en cuanto a una extension superficial de 3,052.93
Metros cuadrados y 16,512.50 metros Cuadrados y descrita
en el Lote No. 26 vendida a favor de Alejandro Ruis y
Mariano P. Leuterio, el primar casado con Diogracias
Quinones y el Segundo con Josefa Garcia y se be expedido el
Certificado de Titulo No. 4210, Pagina 163, Libro T-22.

Date of the Instrument Aug. 29, 1918
Date of Inscription Sept. 9, 1918 10:50 a.m.
(SGD.) L. GARDUNIO, Register of Deeds"
"Ap. 6665/O-994-Venta: Queda Cancelado el presente
Certificado en cuanto a una extension superficial de
871,982.00 metros cuadrados, descrita en el Lote No. 26,
vendida a favor de Alejandro Ruiz y Mariano P. Leuterio, el
primar casado con Deogracias Quinones y el Segundo con
Josefa Garcia y se be expedido el Certificado de Titulo No.
4211, Pagina 164, Libro T-No. 22.
Date of Instrument Aug. 21, 1918
Date of Inscription Sept. 9, 1918 10:50 a.m.
(SGD.) L. GARDUNIO, Register of Deeds"
As a result, TCT No. 4211 was cancelled by TCT No. 5261 which was
issued in the name of Francisco Gonzales. Inscribed on the "Memorandum
of the Incumbrances Affecting the Property Described in this Certificate"
was the sale executed in favor of Francisco Gonzales dated 3 March 1920.
Thus, on 6 April 1920, TCT No. 5261 was issued in the name of Francisco
Gonzales. [ 78 ]
On 22 August 1938, TCT No. 5261 was cancelled by TCT No. 35486 in the
names of Jose Gonzales y Narciso married to Maria P. Gutierrez, Consuelo
Susana Gonzales y Narciso married to Alfonso D. Prescilla; Juana Francisco
Gonzales y Narciso married to Fortunato de Leon; Maria Clara Gonzales y
Narciso married to Delfin Hilario; Francisco Felipe Gonzales y Narciso
married to Pilar Narciso, and Concepcion Andrea Gonzales y Narciso
married to Melquiades M. Virata, Jr. aEACcS
Appearing on the "Memorandum" of TCT No. 5261 is NOTA: Ap 2111
which reads as follows: [ 79 ]
"A/2111 Adjudicado el torreno descrito en este certificado
de titulo, a Rufina Narciso Vda. de Gonzales, a cuenta de la
participacion de osia esta en (not legible) los tienes de la
eseledad de genanciales. Habida entre la misma y el finado
Francisco J. Gonzales, per una orden del Hon. Fernando
Jugo, Juez del Juzgado de Primera Instancia de Manila Sala
II, dienada el 20 de Septiembre de 19 (not legible), en el
Expidiente de intestado del nombrado Francisco J. Gonzales,
No. 49034, se cancela el presente certificado de tituto y se
expide otre a hombre decha Rufina Narciso, con (not legible)
No. 35486, folio 86, Tomo T-168 del libro de transferencias,
archivando se la copia de dicha orden da que se ha heche
referencia en al Legajo T-No. 35486.
24
(SGD.) TEODORO GONZALES,
Registrado de Titulos." ICASEH
The property was later subdivided into seven lots in accordance with
subdivision plan Psd-21154. [ 80 ] Partitioning the lots among the co-
owners, TCT No. 35486 was eventually cancelled and in lieu thereof six (6)
certificates of titles were individually issued [ 81 ] to Francisco Gonzales's
six (6) children, specifically, TCT Nos. 1368-1373 while TCT No. 1374 was
issued in favor of all the children. [ 82 ]
As previously mentioned, the properties covered by TCT Nos. 1368-1374
were expropriated by the Republic of the Philippines and were eventually
subdivided and sold to various vendees. Eighteen (18) lots were obtained
by MRI from the years 1965 to 1974, while it acquired the lot covered by
TCT No. 165119 in 1988. On the other hand, MEC acquired from PhilVille
Development Housing Corporation Lot No. 19-B by virtue of Deed of
Exchange executed in its favor for which, TCT No. 232568 was issued on 9
May 1991.
The 20 certificates of titles were traced by the MANOTOKS, as follows:
1)TCT No. 7528 registered in the name of MRI covers Lot No.
2 of consolidation-subdivision plan (LRC) Pcs-1828 which has
an area of 4,988 square meters. MRI purchased this lot from
one Basilio Caina who was issued TCT No. 7526 which
cancelled TCT Nos. 36657-62 registered in the name of the
Republic of the Philippines. [ 83 ]
2)TCT No. 7762, covering Lot 1-C, was obtained by MRI from
one Narcisa Buenaventura. The Parcel of land has an
approximate area of 2,876 square meters. Buenaventura's
ownership was evidenced by TCT No. 7525, [ 84 ] deriving
the same from TCT No. 36657-63. [ 85 ]
3)TCT No. 8012 in the name of MRI covers Lot No. 12-1
having an area of 20,000 square meters. [ 86 ] This
certificate of title was traced from one Filemon Custodio who
held TCT No. 7792. Custodio was in turn a transferee of
Guillermo Rivera, the latter having been issued TCT No. 7760
by virtue of sale between him and then People's Homesite
and Housing Corporation ["PHHC"]. The latter title eventually
cancelled TCT No. 36557-63 of the Republic. [ 87 ] SADECI
4)TCT No. 9866 issued to MRI covers Lot No. 21 and has an
approximate area of 23,979 square meters. MRI's certificate
of title was derived from TCT No. 9854 registered in the
name of Filemon Custodio, a transferee of Jose Dionisio, who
was issued TCT No. 9853. Dionisio's title in turn cancelled the
Republic's TCT No. 36657-63. [ 88 ]
5)TCT No. 21107 issued to MRI covers Lot 22 with an
approximate area of 2,557 square meters. MRI acquired the
same by virtue of sale between him and Francisco Custodio,
holder of TCT No. 21040. Francisco Custodio was a
transferee of Lorenzo Caina, registered owner of TCT No.
21039 as evidenced by a Deed of Sale between Caina and
the PHHC, the latter's certificate of title canceling TCT No.
36557-63 of the Republic. [ 89 ]
6)TCT No. 21485 was issued to MRI by virtue of sale
between it and Francisco Custodio, registered owner of TCT
No. 21484. The certificate of title covers Lot 20 with an
approximate area of 25,276 square meters Custodio was in
turn a transferee of Lorenzo Caina, the latter being the
registered owner of TCT No. 21013 by reason of sale
between him and PHHC. [ 90 ] Under Entry No. 6277/T-
21485, it would appear that portions of the property covered
under TCT No. 21485 and TCT No. 232568 had been subject
of an expropriation proceedings to which the Manotok Estate
Corporation, et al. interposed no objections subject to the
payment of just compensation. [ 91 ]
7)TCT Nos. 26405 [ 92 ] and 26406, [ 93 ] both registered
in the name of MRI, cancelled TCT Nos. 9773 and 9774,
respectively. TCT Nos. 9773 and 9774 were registered in the
names of Romulo, Rosalina, Lucila, Felix and Emilia all
surnamed Jacinto, ["JACINTOS"], before the same were
transferred to MRI by reason of sale in favor of the latter.
The JACINTOS' certificates of title were in turn derived from
TCT Nos. 8014 and 8015 issued in the name of Filemon
Custodio [ 94 ] Both TCT Nos. 8014 and 8015 cancelled TCT
7792/T-39. However, for purposes of tracing TCT No.
7792/T-39 to the Republic's certificate of titles, this certificate
of title was not submitted in evidence. EcATDH
8)TCT No. 26407 [ 95 ] issued to MRI was traced back to the
title of Lourdes Mercado Cloribel who was the registered
owner of TCT No. 8404 by virtue of sale between the two,
thereby transferring ownership to MRI. On the fact of TCT
No. 8404, it would show that it cancelled TCT No. 8013/T41
but there is no showing in whose name TCT No. 8013 was
registered and what certificate of title it cancelled.
9)TCT No. 33904 [ 96 ] of MRI cancelled TCT No. 8017 of
Filemon Custodio by virtue of sale between the latter and
MRI. [ 97 ] We note that TCT No. 8017 cancelled TCT No.
7792/T-39 but there is no showing whether the same could
be traced back to the Republic's certificates of title.
10)TCT No. 34255, covering Lot No. 11-Bm, Psd-75797 with
an area of 11,000 square meters, reflects MRI as the
registered owner. This certificate of title cancels TCT No.
36557-63 of the Republic. [ 98 ]
11)TCT No. 254875 [ 99 ] bears MRI as the registered owner
of Lot 55-A with an area of approximately 1,910 square
meters. This certificate of title cancelled TCT No. 41956
which covers Lot 55, also registered in the name of MRI. It
would appear that MRI acquired the lot covered under TCT
No. 41956 from one Joaquin Caina who was the registered
owner of TCT No. 25715 being a vendee of PHHC. [ 100 ]
12)TCT No. 53268 of MRI covered Lot No. 15, [ 101 ] which
was purchased by MRI from one Maria V. Villacorta who held
TCT No. 53155. Villacorta in turn acquired the same land
from one Eufrocina Mackay whose TCT No. 7827 was
eventually cancelled by Villacorta's land title. [ 102 ] It would
appear that TCT No. 7827 cancelled TCT No. 7826/T-40 but
there is no trace to whom the latter title was registered and
25
what certificate of title it cancelled.
13)TCT No. 55897 shows MRI as the registered owner of Lot
3 of the consolidation-subdivision plan (LRC) Pcs-1828 of the
Maysilo Estate covering an area of more or less 20,531
square meters. This certificate of title cancelled TCT No.
53122 in the names of MRI (19,531 square meters) and one
Silvestre Domingo (1,000 square meters). TCT No. 53122 in
turn cancelled TCT No. 21347 registered in the names of
Jesus Hipona (19,531 square meters) and Silvestre Domingo
(1,000 square meters). Notably, TCT No. 21347 cancelled
TCT No. 21315/T-107 but there is no indication to whom TCT
No. 21315 was registered and what certificate of title it
cancelled. [ 103 ] cIECaS
14)TCT No. C-17272 reflects MRI as the registered owner of
Lot 6-C which has an approximate area of 27,850 square
meters. MRI's certificate of title cancelled TCT No. C-17234
registered in the names of MRI (27,750 square meters),
Roberto S. David (3,0000 * square meters) and Jose Madulid
(500 square meters). It would appear that TCT No. C-17234
cancelled TCT No. 53124 registered in the names of MRI,
Spouses Priscila and Antonio Sebastian and Jose Madulid. [
104 ] MRI also submitted in evidence a Deed of Partition
between itself, Roberto David and Madulid thereby
subdividing the property into Lots 6-A, 6-B and 6-C as per
subdivision plan (LRC) Psd-277091. [ 105 ] Again, we note
that TCT No. 53124 cancelled TCT No. 21350/T-107 but the
records are bereft of any indication what certificate of title it
cancelled and to whom the same was registered.

15)TCT No. C-35267, covering Lot 56-B of subdivision plan
(LRC) Psd-292683 with an approximate area of 9,707 square
meters, was a by-product of TCT No. 25146, also registered
in the name of MRI, after the same was subdivided into two
lots, namely, Lot Nos. 56-A and 56-B. TCT No. 25146
cancelled TCT No. 25145 registered in the name of Quirino
Labing-isa by virtue of sale in favor of MRI. In turn, TCT No.
21545 cancelled TCT Nos. (36557) 12836 to (36563) 12842.
[ 106 ]
16)TCT No. T-121428, registered in the name of MRI covers
Lot No. 5-C of subdivision plan (LRC) psd-315272 which has
an approximate area of 4,650 square meters. It was
previously registered in the names of MRI (4,650 square
meters), Ricardo Cruz (941 square meters) and Conchita
Umali (1,000 square meters) under TCT No. 53123 by order
of the Court of First Instance of Rizal, Caloocan City, Branch
XII and as per agreement of the parties in Civil Case No. C-
424. TCT No. 53123 in turn cancelled TCT No. 21346 whose
registered owners were Conchita Umali (1,000 square
meters), Ricardo Cruz (941 square meters) and Jesus Hipona
(4,650 square meters). [ 107 ] Like some of the other titles,
TCT No. 21346 cancelled TCT No. 21316 but there is no trace
of this latter certificate of title. aCcEHS
17)TCT No. 163902, registered in the name of MRI, covers
Lot No. 4-B-2 and has an area of more or less 6,354 square
meters and a by-product of TCT No. 9022, also in the name
of MRI, after the same was subdivided under subdivision plan
(LRC) Psd-334454. TCT No. 9022, in turn, cancelled TCT No.
8994/T-45 registered in the name of Filemon S. Custodio
whose ownership thereon was transferred to MRI by virtue of
a voluntary sale. [ 108 ] TCT No. 8894 cancelled TCT No.
8846/T-45 but this latter certificate of title was not submitted
in evidence for purposes of tracing back to the Republic's
title.
18)TCT No. 165119 [ 109 ] was issued to MRI by virtue of a
Deed of Sale between Spouses Francisca Labing-isa and Juan
Ignacio [SPOUSES IGNACIO] and MRI, as a result of which,
TCT No. C-36960 of the SPOUSES IGNACIO was cancelled. [
110 ] It would appear that TCT No. C-39690 cancelled TCT
No. 35266/T-173 but TCT No. 35266/T-173 was not
submitted in evidence.
19)TCT No. T-232568 of the Manotok Estate Corporation,
covering Lot No. 19-B of subdivision plan Psd-13011152 with
an area of 23,206 square meters, was derived from the
certificate of title held by PhilVille Development and Housing
Corporation under TCT No. 197357. MEC acquired the
subject parcel of land by virtue of Deed of Exchange between
it and PHILVILLE DATED 9 May 1991. [ 111 ] TCT No.
197357 cancelled TCT No. 195730/T-974 but there is no
trace what certificate of title the latter title cancelled.
By and large, all the certificates of title submitted by the MANOTOKS,
including their derivative titles, were all traced to OCT No. 994 registered
on 3 May 1917. Likewise, they declared all the lots covered by such titles
for taxation purposes. Without doubt, MRI had successfully traced back
some of their certificates of title to the valid OCT No. 994, they having
acquired the lots from some of the vendees of the PHHC after the same
were expropriated by the Republic from the Gonzalezes. TCcSDE
The fact that these lots were subjected to expropriation proceedings
sometime in 1947 under Commonwealth Act No. 539 for resale to tenants
is beyond question, as also enunciated by the Supreme Court in Republic
of the Philippines v. Jose Leon Gonzales, et al. To bolster this fact,
paragraph "r" of the Majority Report noted that the seven properties
covered by TCT Nos. 1368 to 1374 were expropriated by the Republic from
the Gonzalezes.
The fact that these lots were subjected to expropriation proceedings
sometime in 1947 under Commonwealth Act No. 539 for resale to tenants
is beyond question, as also enunciated by the Supreme Court in Republic
of the Philippines vs. Jose Leon Gonzales, et al. To bolster this fact,
paragraph "r" of the Majority Report noted that the seven properties
covered by TCT Nos. 1368 to 1374 were expropriated by the People's
Homesite and Housing Corporation which were later consolidated and
subdivided into 77 lots for resale to tenants. No sign of protest was ever
raised by CLT on this point. 112
The fact of expropriation is extremely significant, for titles acquired by the State by way of expropriation
are deemed cleansed of whatever previous flaws may have attended these titles. As Justice Vitug
explained in Republic v. Court of Appeals, 113 and then Associate Justice (now Chief Justice) Puno
reiterated in Reyes v. NHA: 114 "In an rem proceeding, condemnation acts upon the property. After
condemnation, the paramount title is in the public under a new and independent title; thus, by giving
notice to all claimants to a disputed title, condemnation proceedings provide a judicial process for
26
securing better title against all the world than may be obtained by voluntary conveyance". 115 This
doctrine was derived from the opinion of then Chief Judge (now U.S. Supreme Court Justice) Stephen
Breyer in Cadorette v. U.S., 116 which in turn cited the pronouncement of the U.S. Supreme Court in
U.S. v. Carmack 117 that "[b]y giving notice to all claimants to a disputed title, condemnation
proceedings provide a judicial process for securing better title against all the world than may be obtained
by voluntary conveyance". 118
In annulling the Manotok titles, focus was laid on the alleged defects of TCT No. 4211 issued in
September of 1918. However, TCT No. 4211 was issued decades before the property was expropriated.
Thus, any and all defects that may have attended that particular title would have been purged when the
property covered by it was subsequently acquired by the State through eminent domain. The Special
Division noted as much: DcTAIH
As it is, the validity of most of MRI's certificates of title should be upheld
because they were derived from the Republic's valid certificates of title. In
fact, some of the MANOTOKS' titles can be traced back to the
Government's titles as a result of the expropriation in 1947.
Relevantly, the titles of the Republic, as the predecessor-in-interest of the
MANOTOKS, are presumed valid by virtue of their acquisition resulting
from the exercise of its inherent power of eminent domain that need not
be granted even by the fundamental law. Thus, the alleged flaws
concerning the certificates of title issued previous to the exercise of the
State of its inherent power did not affect or render invalid the subsequent
transfers after the forced sale. Indeed, when land has been acquired for
public use in fee simple unconditionally, either by the exercise of eminent
domain or by purchase, the former owner retains no rights in the land, and
the public use may be abandoned, or the land may be devoted to a
different use, without any impairment of the estate or title acquired or any
reversion to the former owner. 119
The Special Division also took exception to the majority report of the Commissioners (Majority Report)
who had been tasked by the trial court to examine the validity of the Manotok titles. The Majority Report
had arrived at several conclusions with respect to the TCTs from which the Manotok titles were derived.
120 The Special Division, however, concluded that such report was in fact tainted by the fact that it was
determined "outside the scope of the issues framed and agreed upon by the parties". To wit:
In meeting the issue, the MANOTOKS disproved the "opinion" with regard
to the alleged defects of their titles inasmuch as the majority report
submitted before the trial court was made outside the scope of the tasks
which the trial court confined them to perform. The MANOTOKS also
argued that before this proceeding on remand, CLT failed to introduce
evidence of such flaws neither were the concerned geodetic engineers
presented as witnesses. Moreover, the MANOTOKS further maintained that
CLT failed to submit any factual or legal bases to prove the authenticity
and validity of the Palma and Sayo Orders. They insisted that the Palma
Order was a void one for being conditional and having resulted to the
issuance of "duplicate certificates of land title".
With respect to the imputed flaws on the MANOTOKS' titles which were
based on the Majority Report, we find that the bases of the alleged defects
proceeded from unreliable sources thus, tainting the veracity of the said
report.
The records of the case between CLT and the MANOTOKS reveal that the
parties approved the creation of a commission to resolve only these two
issues, to wit:
"xxx xxx xxx
These issues to be resolved by the 3 Commissioners are as
follows:
1)Whether or not the property covered by the
Transfer Certificates of Title of defendants
pertain to or involve Lot No. 26 of the Maysilo
Estate presently titled in the name of the
plaintiff; and
2)Whether or not the property covered by the
title of the plaintiff and the property covered by
the titles of the defendants overlap. [ 121 ]
ACTIcS
Scrutinizing the Majority Report upon which the trial court's conclusions
were based, it would appear that the findings therein were outside the
scope of the issues framed and agreed upon by the parties. Specifically,
the deductions with regard to the technical infirmities and defects of TCT
Nos. 4211, 4210, 5261 and 35486 do not involve the question of whether
or not the subject properties were identified as Lot No. 26 of the Maysilo
estate or whether there was overlapping of titles. Records bear out that
the MANOTOKS took exception to the procedure taken citing therein the
"ultra vires" acts of the two Commissioners.
In addition, the majority report focused on the alleged flaws and inherent
technical defects of TCT Nos. 4211, 5261 and 35486, ranging from the
language of the technical descriptions, absence of subdivision plan, lot
number and survey plan. Evidently, these defects go only as far as the
certificates of title issued prior to those of the Republic. Remarkably, no
specific flaw was found on the MANOTOKS' titles indicating any irregularity
on their issuance. In fact, the Commissioners who signed the majority
report even concluded that only TCT Nos. 4211, 4210, 5261, 35486, 1368
thru 1324 (sic) [ 122 ] were irregularly and questionably issued without
any reference to the MANOTOKS' certificates of title. [ 123 ] Otherwise
stated, the imputed flaws affect only those certificates of title issued prior
to those registered in the name of the Republic. No flaw had been
specifically identified or established in the proceedings below, which would
taint the titles held by the MANOTOKS in so far as the regularity of their
issuance is concerned. 124

At the same time, the Special Division was not prepared to uphold the validity of all of the Manotok titles.
It took issue with the particular titles which could not be retraced to the titles acquired by the Republic of
the Philippines by way of expropriation.
Although the MANOTOKS had traced their title from the vendees of PHHC,
there are, however, some certificates of title which could not be traced
back to the titles previously held by the Republic specifically, MRI's TCT
Nos. 26405 and 26406, 26407, 33904, 53268, 55897, C-17272, T-121428,
163903, 165119 and MEC's TCT No. T-232568. As to these certificates of
title, the MANOTOKS failed to make any specific reference to the preceding
certificates of title which they cancelled and to whose names they were
subsequently transferred and registered. Thus, we find no sufficient basis
to make a conclusion as to their origins. 125 TaSEHD
V.
The Special Division supplied the following precise and concise summary of its conclusions:
27
In prcis, the factual milieu of the present controversy and the evidence
on record clearly establish the failure of DIMSON and CLT to substantiate
their titles and overcome the onus of proving that said titles are derivatives
of OCT 994 registered on 3 May 1917, and not 19 April 1917, as what is
reflected in their titles. In contrast, the MANOTOKS and ARANETA, both of
which had consistently anchored their proprietary claims on OCT No. 994
registered on 3 May 1917, have, in this remand proceeding, been able to
support their claims of ownership over the respective portions of the
Maysilo Estate. Except in the case of the MANOTOKS which had failed to
substantiate the validity of some of their certificates of title, the
MANOTOKS and ARANETA presented evidence proving the identity, the
extent and the origin of their titles. HSIDTE
Answering the issues assigned by the Supreme Court relative to the
tenability of the respective imputed flaws in the titles of the MANOTOKS
and ARANETA and whether such flaws are sufficient to defeat said claims,
this Court finds that, as discussed above, such flaws are inconsequential
and ineffectual in invalidating the MANOTOKS and ARANETA titles.
Significantly, since the respective certificates of title of herein contending
parties are contradictory to each other and stand to refute the validity of
their opposing titles, it cannot be gainsaid that said certificates of title have
correspondingly been subjected to dispute on the basis of separate and
distinct imputed flaws. Still, the crucial difference between the imputed
flaws allegedly tainting said contending titles, DIMSON and CLT on one
hand, and the MANOTOKS and ARANETA, on the other, is that the imputed
flaws purportedly beleaguering the respective certificates of title of the
MANOTOKS and ARANETA relate to the mechanical and technical aspect of
the transcription of their titles and are therefore inconsequential to the
import and validity thereof. Said imputed flaws do not depart from the fact
that the predecessors-in-interest of the MANOTOKS and ARANETA had
been clothed with the right of ownership over the disputed portions of the
Maysilo Estate.
On the other hand, the flaws attending the titles of DIMSON and CLT
primarily stem from infirmities attending or otherwise affecting the very
crux of their claim of ownership. Having derived their titles from RIVERA,
whose title is questionable and dubious to the core, DIMSON and CLT
cannot rightly insist on the validity of their titles. Such flaws are hard to
overcome as they delve into the substance of their proprietary claims. As
stated, DIMSON and CLT miserably failed to overcome their onus and
instead opted to hap on the supposed flaws of the adverse parties. For
these reasons, the titles of DIMSON and CLT should be declared a nullity.
aSDCIE
xxx xxx xxx
From the foregoing evaluation and in conformity with the Supreme Court
2007 Resolution, this Court arrived at the following conclusions as to the
status of the original title and its subsequent conveyances:
1.As categorically declared by the Supreme Court, there is
only one OCT 994, the registration date of which had already
been decisively settled as 3 May 1917 and not 19 April 1917.
OCT 994 which reflects the date of 19 April 1917 as its
registration date is null and void.
2.In view thereof and in addition to other grounds we have
already discussed, the certificates of title of the deceased
Jose Dimson and his successor-in-interest, CLT, having been
traced back to OCT 994 dated 19 April 1917, are NULL and
VOID and thus vest no legal right or claim in favor of
DIMSON and CLT.
3.The 13 June 1966 Palma Order and the 18 October 1977
Sayo Order, on which DIMSON and CLT anchor the validity of
their respective titles, do not substantiate their proprietary
claims. While the existence of said Orders are admitted, the
legal import thereof nonetheless fails to confer a semblance
of legality on the titles of DIMSON and consequently, of CLT,
more so, a superior right to defeat the titles of the
MANOTOKS and ARANETA, respectively.
4.Portions of Lot No. 26 pertinent to this controversy,
particularly that being disputed by the MANOTOKS and CLT,
were expropriated by the Republic of the Philippines
sometime in 1947 under Commonwealth Act No. 539 for
resale to tenants. The MANOTOKS, thus as successor-in-
interest of the Republic, were able to establish that some of
their certificates of title had indeed originated or were
derived from said expropriated parcels of land. ADSTCI
5.The evidence on record confirm that the certificates of title
covering the land being claimed by ARANETA were derived
from OCT NO. 994 registered on 3 May 1917 thereby
ultimately showing a direct link of TCT Nos. 7784 and 13574
to said mother title. By reason of which, that is either
belonging to or portions of Lot 25-A-3 as previously owned
by RATO, had been well substantiated and proven to be
superior to that of DIMSON.
6.For reasons above-stated and in view of the established
rights of ownership of both the MANOTOKS and ARANETA
over the contested properties, we find that the imputed flaws
on their titles cannot defeat the valid claims of the
MANOTOKS and ARANETA over the disputed portions of the
Maysilo Estate. 126
Inasmuch as we agree with the factual findings and evaluation of the Special Division, we likewise adopt
the above conclusions. As we earlier stated, it was incumbent on the Heirs of Dimson and/or CLT to
establish their claim to title for reasons other than the fact that OCT No. 994 dated 19 April 1917 is
extant. They failed to do so. It should be noted that the instant cases arose from separate actions filed
by Jose Dimson and CLT seeking the recovery of possession and/or annulment of title against Araneta
and the Manotok Group. Thus, the burden of evidence was on Dimson and CLT to establish the strength
of their respective claims of ownership, and not merely to rely upon whatever weaknesses in the claims
of the Manotoks and Araneta for their causes of action to prosper. The well-settled legal principle in
actions for annulment or reconveyance of title is that a party seeking it should establish not merely by a
preponderance of evidence but by clear and convincing evidence that the land sought to be reconveyed
is his. 127 In an action to recover, the property must be identified, and the plaintiff must rely on the
strength of his title and not on the weakness of the defendant's claim. 128
We now proceed to tackle the recommendations submitted by the Special Division. They are as follows:
RECOMMENDATIONS
Apropos to said conclusions, this Court hereby respectfully makes the
following recommendations regarding the validity of the conflicting
proprietary claims as interposed by the herein contending parties:
28
1.To declare with finality that the certificates of title of DIMSON and CLT
including other derivative titles issued to their successors-in-interest, if
any, are NULL and VOID, thus invalidating their legal claims over the
subject parcels of land. SEHACI
2.To declare LEGAL and VALID the proprietary claims the MANOTOKS over
the parcels of land covered by the following certificates of title:
a)TCT No. 7528 registered in the name of MRI covers Lot No.
2 of consolidation-subdivision plan (LRC) Pcs-1828 which has
an area of 4,988 square meters.
b)TCT No. 7762 covering Lot 1-C, with an approximate area
of 2,287 square meters.
c)TCT No. 8012 covering Lot No. 12-1 having an area of
20,000 square meters.
d)TCT No. 9866 covering Lot No. 21 and has an approximate
area of 23,979 square meters.
e)TCT No. 21107 covering Lot 22 with an approximate area
of 2,557 square meters.
f)TCT No. 21485 covering Lot 20 with an approximate area of
25,276 square meters.
g)TCT No. 34255 covering Lot No. 11-Bm, Psd-75797 with an
area of 11,000 square meters.
h)TCT No. 254875 covering Lot 55-A with an area of
approximately 1,910 square meters.
i)TCT No. C-35267 covering Lot 56-B of subdivision plan
(LRC) Psd-292683 with an approximate area of 9,707 square
meters.
With regard to the following certificates of title, namely: ASaTCE
3.A.MANOTOK REALTY INC.
a)TCT No. 26405 covering Lot No. 12-E with an area of
1,0000 * square meters.
b)TCT No. 26406 covering Lot No. 12-F with an area of 1,000
square meters.
c)TCT No. 26407 covering Lot No. 12-B with an area of 1,000
square meters.
d)TCT No. 33904 covering Lot No. 12-H with an area of
1,802 square meters.
e)TCT No. 53268 covering Lot No. 15 purchased by MRI from
one Maria V. Villacorta with an approximate area of 3,163
square meters.
f)TCT No. 55897 covering Lot 3 of consolidation-subdivision
plan (LRC) Pcs-1828 of the Maysilo Estate covering an area
of more or less 20,531 square meters.
g)TCT No. C-17272 covering Lot 6-C which has an
approximate area of 27,850 square meters.
h)TCT No. T-121428 covering Lot No. 5-C of subdivision plan
(LRC) psd-315278, which has an approximate area of 4,650
square meters.

i)TCT No. 163902 covering Lot No. 4-B-2 with an area of
more or less 6,354 square meters allegedly a by-product of
TCT No. 9022, which in turn, cancelled TCT No. 8994/T-45
registered in the name of Filemon S. Custodio.
j)TCT No. 165119 which allegedly cancelled TCT No. C-36960
of the SPOUSES IGNACIO by virtue of a Deed of Sale
between said Spouses and MRI.
3.B.MANOTOK ESTATE CORPORATION
a)TCT No. T-232568 covering Lot No. 19-B of subdivision
plan Psd-13011152 with an area of 23,206 square meters.
The foregoing certificates of title (3.A and 3.B), failing to
make specific references to the particular certificates of title
which they cancelled and in whose name they were
registered, may be declared NULL and VOID, or in the
alternative, subject the same to further technical verification.
4.To declare LEGAL and VALID the title of ARANETA respecting parcels of
land covered by the following certificates of title:
a)TCT No. 13574 covering a parcel of land designated as
Section No. 2 of subdivision plan Psd-10114, being a portion
of Lot 25-A-3-C with an aggregate area of 581,872 square
meters; cHSTEA
b)TCT No. 7784 covering four (4) parcels of land with an
aggregate area of 390,383 square meters. 129
The first, second and fourth recommendations are well taken as they logically arise from the facts and
conclusions, as determined by the Special Division, which this Court adopts.
The third recommendation that eleven (11) of the titles held by the Manotoks be declared null and
void or subjected to further technical verification warrants some analysis.
The Court has verified that the titles mentioned in the third recommendation do not, as stated by the
Special Division, sufficiently indicate that they could be traced back to the titles acquired by the Republic
when it expropriated portions of the Maysilo Estate in the 1940s. On the other hand, the Manotok titles
that were affirmed by the Special Division are traceable to the titles of the Republic and thus have
benefited, as they should, from the cleansing effect the expropriation had on whatever flaws that
attached to the previous titles. However, although the Special Division did not concede the same benefit
to the other Manotok titles named in the third recommendation, at the same time it did not conclude that
such titles were false or fraudulently acquired. Absent such a finding, we are disinclined to take the
ultimate step of annulling those titles.
Said titles have as their origin what we have acknowledged to be a valid mother title OCT No. 994
29
dated 3 May 1917. This is in stark contrast with the titles of CLT, the oppositors to the Manotoks, which
all advert to an inexistent mother title. On their face, the Manotok titles do not reflect any error or fraud,
and certainly the Special Division do not point to any such flaw in these titles. Nothing on the face of the
titles gives cause for the Court to annul the same. DCSTAH
It is worth mentioning that the Special Division refused to adopt the Majority Report earlier rendered in
the case between the Manotoks and CLT, said report having exhaustively listed the perceived flaws in the
antecedent TCTs from which the Manotoks derived their claim. The Special Division concluded that such
findings had been reached by the Commissioners in excess of their original mandate and, thus, ultra
vires. Assuming that such flaws were extant, they existed on the titles and anteceded the expropriation
of the properties by the Government. As stated earlier, such expropriation would have cleansed the titles
of the prior flaws. But even if the Manotok titles enumerated in the third recommendation could not be
sourced from the titles acquired by the Republic through expropriation, still the rejection of the Majority
Report signifies that the flaws adverted to therein could not form the basis for the annulment of the titles
involved. Indeed, the Special Division's rejection of the Majority Report further diminishes any ground to
annul the Manotok titles referred to in the third recommendation.
Yet, the Court is cognizant that the inability to trace the Manotok titles specified in the third
recommendation to those titles acquired by the Government through expropriation puts such titles in
doubt somehow. In addition, the Court is aware that the ground utilized by the Special Division in
rejecting the Majority Report that the determinations were made outside the scope of the issues
framed and agreed upon by the parties does not categorically refute the technical findings made
therein. Those circumstances, while insufficient for now to annul the Manotoks' titles listed in the third
recommendation, should be sufficiently made public.
Hence, in lieu of annulling the Manotok titles per the Special Division's third recommendation, the Court
deems it sufficient to require the Registers of Deeds concerned to annotate this Resolution on said titles
so as to sufficiently notify the public of their unclear status, more particularly the inability of the
Manotoks to trace the titles without any gap back to OCT No. 994 issued on 3 May 1917. If there should
be any cause for the annulment of those titles from a proper party's end, then let the proper case be
instituted before the appropriate court. aETAHD
WHEREFORE, the Court hereby adopts the Report of the Special Division and issues the following reliefs:
1)The certificates of title of the DIMSONs and CLT including other
derivative titles issued to their successors-in-interest, if any,
are declared NULL and VOID, thus invalidating their legal
claims over the subject parcels of land;
2.The proprietary claims of the MANOTOKS over the parcels of land
covered by the following certificates of title are declared
LEGAL and VALID, to wit:
a)TCT No. 7528 registered in the name of MRI covers Lot No.
2 of consolidation-subdivision plan (LRC) Pcs-
1828 which has an area of 4,988 square
meters.
b)TCT No. 7762 covering Lot 1-C, with an approximate area
of 2,287 square meters.
c)TCT No. 8012 covering Lot No. 12-1 having an area of
20,000 square meters.
d)TCT No. 9866 covering Lot No. 21 and having an
approximate area of 23,979 square meters.
e)TCT No. 21107 covering Lot 22 with an approximate area
of 2,557 square meters.
f)TCT No. 21485 covering Lot 20 with an approximate area of
25,276 square meters.
g)TCT No. 34255 covering Lot No. 11-Bm, Psd-75797 with an
area of 11,000 square meters.
h)TCT No. 254875 covering Lot 55-A with an area of
approximately 1,910 square meters. AScHCD
i)TCT No. C-35267 covering Lot 56-B of subdivision plan
(LRC) Psd-292683 with an approximate area of
9,707 square meters.
3)The following certificates of titles in the name of ARANETA are hereby
declared LEGAL and VALID, to wit:
a)TCT No. 13574 covering a parcel of land designated as
Section No. 2 of subdivision plan Psd-10114,
being a portion of Lot 25-A-3-C with an
aggregate area of 581,872 square meters;
b)TCT No. 7784 covering four (4) parcels of land with an
aggregate area of 390,383 square meters.
4)On the following titles in the name of Manotok Realty, Inc. or Manotok
Estate Corporation, to wit:
a)TCT No. 26405 covering Lot No. 12-E with an area of 1,000
square meters;
b)TCT No. 26406 covering Lot No. 12-F with an area of 1,000
square meters;
c)TCT No. 26407 covering Lot No. 12-B with an area of 1,000
square meters;
d)TCT No. 33904 covering Lot No. 12-H with an area of
1,802 square meters;
e)TCT No. 53268 covering Lot No. 15 purchased by MRI from
one Maria V. Villacorta with an approximate
area of 3,163 square meters;
f)TCT No. 55897 covering Lot 3 of consolidation-subdivision
plan (LRC) Pcs-1828 of the Maysilo Estate
covering an area of more or less 20,531 square
meters; cCHITA
g)TCT No. C-17272 covering Lot 6-C which has an
approximate area of 27,850 square meters;
h)TCT No. T-121428 covering Lot No. 5-C of subdivision plan
(LRC) psd-315278, which has an approximate
area of 4,650 square meters;
i)TCT No. 163902 covering Lot No. 4-B-2 with an area of
more or less 6,354 square meters allegedly a
by-product of TCT No. 9022, which in turn,
cancelled TCT No. 8994/T-45 registered in the
30
name of Filemon S. Custodio;
j)TCT No. 165119 which allegedly cancelled TCT No. C-36960
of the SPOUSES IGNACIO by virtue of a Deed
of Sale between said spouses and MRI;
k)TCT No. T-232568 covering Lot No. 19-B of subdivision
plan Psd-13011152 with an area of 23,206
square meters.
the Registers of Deeds concerned are ordered to annotate that as determined in the foregoing
Resolution, the registered owners of the said titles "failed to make any specific reference to the
preceding certificates of title which they cancelled and to whose names they were subsequently
transferred and registered", thereby leading the Supreme Court "to find no sufficient basis to make a
conclusion as to their origins". 130
Costs against private respondents.
SO ORDERED.























f. Classification of Public Lands
i. Who classifies
ii. Law governing classification
1. C.A. No. 141 Public Land Act
iii. Classification of Lands
iv. Cases



G.R. No. 83609. October 26, 1989.
DIRECTOR OF LANDS, petitioner, vs. COURT OF APPEALS, IBARRA BISNAR and
AMELIA BISNAR, respondents.
D E C I S I O N
GRIO-AQUINO, J p:
Petitioner Director of Lands, through the Solicitor General, seeks a review of the decision dated May 27,
1988, of the Court of Appeals in CA-G.R. CV No. 66426, entitled "Ibarra Bisnar, et al. vs. Director of
Lands," affirming in toto the decision of the Court of First Instance of Capiz, granting the private
respondents' application for confirmation and registration of their title to two (2) parcels of land in LRC
Cad. Rec. 1256. cdll
In their joint application for registration of title to two (2) parcels of land filed on July 20, 1976, the
applicants Ibarra and Amelia Bisnar claimed to be the owners in fee simple of Lots 866 and 870 of the
Pilar Cadastre Plan AP-06-000869, respectively containing an area of 28 hectares (284,424 sq.m.) and 34
hectares (345,385 sq.m.) situated in barrio Gen. Hizon, Municipality of President Roxas, Province of Capiz
(p. 14, Rollo). The applicants alleged that they inherited those parcels of land (p. 41, Rollo) and they had
been paying the taxes thereon (p. 40, Rollo).
On December 16, 1976, the Director of Lands and the Director of the Bureau of Forest Development,
opposed the application on the grounds that:
"1.Neither the applicants nor their predecessors-in-interest possess
sufficient title to acquire ownership in fee simple of the land or lots applied
for, the same not having been acquired by any of the various types of title
issued by the Spanish Government, such as, (1) 'titulo real' or royal grant,
31
(2) the 'concession especial' or special grant, (3) the 'composicion con el
estado titulo' or adjustment title, (4) the 'titulo de compra' or title by
purchase, and (5) the 'informacion possessoria' or possessory information
under the Royal Decree of 13 February 1894, or any other recognized
mode of acquisition of title over realty under pertinent applicable laws.
"2.Neither the applicants nor their predecessors-in-interest have been in
open, continuous, exclusive and notorious possession and occupation of
the land in question for at least thirty (30) years immediately preceding
the filing of the application.
"3.The properties in question are a portion of the public domain belonging
to the Republic of the Philippines, not subject to private appropriation, (pp
17-19, Record on Appeal)." (pp. 14-15, Rollo.)
On February 24, 1977, the applicants filed an amended application, which was approved on March 14,
1977, and included the following allegation:
"Should the Land Registration Act invoked be not applicable to the case,
they hereby apply for the benefits of Chapter 8, Commonwealth Act 141,
as amended, as they and their predecessors-in-interest have been in
possession of the land as owners for more than fifty (50) years." (p. 16,
Rollo.)
After hearing, the trial court ordered the registration of the title of the lots in the names of the applicants,
herein private respondents. It found that applicants and their predecessors-in-interest have been in open,
public, continuous, peaceful and adverse possession of the subject parcels of land under bona fide claims
of ownership for more than eighty (80) years (not only 30) prior to the filing of the application for
registration, introduced improvements on the lands by planting coconuts, bamboos and other plants, and
converted a part of the land into productive fishponds (p. 68, Rollo).
On appeal, the Appellate Court affirmed the trial court's decision. It held that the classification of the lots
as timberland by the Director of Forestry cannot prevail in the absence of proof that the said lots are
indeed more valuable as forest land than as agricultural land, citing as authority the case of Ankron vs.
Government of the Philippine Islands (40 Phil. 10). In this petition, the government alleges that:
1.the classification or reclassification of public lands into alienable or
disposable agricultural land, mineral land or forest land is a prerogative of
the Executive Department of the government and not of the courts;
2.that possession of forest lands, no matter how long, cannot ripen into
private ownership; and
3.that an applicant for registration of title has the burden of proving that
he meets the requirements of Section 48 of Com. Act No. 141, as
amended. (p. 19, Rollo.)
The principal issue in this appeal is whether the lots in question may be registered under Section 48 (b)
of CA 141, as amended.
The petition is impressed with merit.
In the case of Bureau of Forestry vs. Court of Appeals, 153 SCRA 351, we ruled:
"As provided for under Section 6 of Commonwealth Act 141, which was
lifted from Act 2874, the classification or reclassification of public lands into
alienable or disposable, mineral or forest lands is now a prerogative of the
Executive Department of the government and not the courts. With these
rules, there should be no more room for doubt that it is not the court
which determines the classification of lands of the public domain into
agricultural, forest or mineral but the Executive Branch of the government,
through the Office of the President. Hence, it was grave error and/or
abuse of discretion for respondent court to ignore the uncontroverted facts
that (1) the disputed area is within a timberland block, and (2) as certified
to by the then Director of Forestry, the area is needed for forest
purposes." (pp. 21-22, Rollo.)
It bears emphasizing that a positive act of the government is needed to declassify land which is classified
as forest and to convert it into alienable or disposable land for agricultural or other purposes (Republic
vs. Animas, 56 SCRA 499). Unless and until the land classified as forest is released in an official
proclamation to that effect so that it may form part of the disposable agricultural lands of the public
domain, the rules on confirmation of imperfect title do not apply (Amunategui vs. Director of Forestry,
126 SCRA 69; Director of Lands vs. Court of Appeals, 129 SCRA 689; Director of Lands vs. Court of
Appeals, 133 SCRA 701; Republic vs. Court of Appeals, 148 SCRA 480; Vallarta vs. Intermediate Appellate
Court, 151 SCRA 679).
Thus, possession of forest lands, however long, cannot ripen into private ownership (Vano vs.
Government, 41 Phil. 161 [1920]; Adorable vs. Director of Forestry, 107 Phil. 401 [1960]). A parcel of
forest land is within the exclusive jurisdiction of the Bureau of Forestry and beyond the power and
jurisdiction of the cadastral court to register under the Torrens System (Republic vs. Court of Appeals, 89
SCRA 648; Republic vs. Vera, 120 SCRA 210 [1983]; Director of Lands vs. Court of Appeals, 129 SCRA
689 [1984]). Cdpr

Section 48 (b) of Commonwealth Act No. 141, as amended, applies exclusively to public agricultural land.
Forest lands or areas covered with forests are excluded (p. 26, Rollo). We reiterate our ruling in
Amunategiu that:
"In confirmation of imperfect title cases, the applicant shoulders the
burden of proving that he meets the requirements of Section 48,
Commonwealth Act No. 141, as amended by Republic Act 1942. He must
overcome the presumption that the land he is applying for is part of the
public domain but that he has an interest therein sufficient to warrant
registration in his name because of an imperfect title such as those derived
from old Spanish grants or that he has had continuous, open and notorious
possession and occupation of agricultural lands of the public domain under
a bona fide claim of acquisition of ownership for at least thirty (30) years
preceding the filing of his application." (Heirs of Amunategui vs. Director of
Forestry, 126 SCRA 69.)
WHEREFORE, the appealed decision is reversed and set aside. The application for registration in LRC
Cad. Rec. 1256 of the former Court of First Instance, is hereby dismissed without costs.
SO ORDERED.






32













G.R. No. 155450. August 6, 2008.
REPUBLIC OF THE PHILIPPINES represented by the Regional Executive Director,
Department of Environment and Natural Resources, Regional Office No. 2,
petitioner, vs. COURT OF APPEALS, HEIRS OF ANTONIO CARAG AND VICTORIA
TURINGAN, THE REGISTER OF DEEDS OF CAGAYAN, and the COURT OF FIRST
INSTANCE OF CAGAYAN, respondents.
D E C I S I O N
CARPIO, J p:
The Case
This is a petition for review 1 of the 21 May 2001 2 and 25 September 2002 3 Resolutions of the Court
of Appeals in CA-G.R. SP No. 47965. The 21 May 2001 Resolution dismissed petitioner Republic of the
Philippines' (petitioner) amended complaint for reversion, annulment of decree, cancellation and
declaration of nullity of titles. The 25 September 2002 Resolution denied petitioner's motion for
reconsideration.
The Facts
On 2 June 1930, the then Court of First Instance of Cagayan (trial court) issued Decree No. 381928 4 in
favor of spouses Antonio Carag and Victoria Turingan (spouses Carag), predecessors-in-interest of private
respondents Heirs of Antonio Carag and Victoria Turingan (private respondents), covering a parcel of
land identified as Lot No. 2472, Cad. 151, containing an area of 7,047,673 square meters (subject
property), situated in Tuguegarao, Cagayan. On 19 July 1938, pursuant to said Decree, the Register of
Deeds of Cagayan issued Original Certificate of Title No. 11585 5 (OCT No. 11585) in the name of
spouses Carag. DcICEa
On 2 July 1952, OCT No. 11585 was cancelled to discharge the encumbrance expressly stated in Decree
No. 381928. Two transfer certificates of title were issued: Transfer Certificate of Title No. T-1277, 6
issued in the name of the Province of Cagayan, covering Lot 2472-B consisting of 100,000 square meters
and Transfer Certificate of Title No. T-1278, 7 issued in the name of the private respondents, covering
Lot 2472-A consisting of 6,997,921 square meters.
On 19 May 1994, Bienvenida Taguiam Vda. De Dayag and others filed with the Regional Office No. 2 of
the Department of Environment and Natural Resources (DENR), Tuguegarao, Cagayan, a letter-petition
requesting the DENR to initiate the filing of an action for the annulment of Decree No. 381928 on the
ground that the trial court did not have jurisdiction to adjudicate a portion of the subject property which
was allegedly still classified as timber land at the time of the issuance of Decree No. 381928. SEHTAC
The Regional Executive Director of the DENR created an investigating team to conduct ground verification
and ocular inspection of the subject property.
The investigating team reported that:
A)The portion of Lot 2472 Cad-151 as shown in the Plan prepared for
spouses Carag, and covered under LC Project 3-L of Tuguegarao,
Cagayan, was found to be still within the timberland area at the time of
the issuance of the Decree and O.C.T. of the spouses Antonio Carag and
Victoria Turingan, and the same was only released as alienable and
disposable on February 22, 1982, as certified by USEC Jose G. Solis of the
NAMRIA on 27 May 1994.
B)Petitioner Bienvenida Taguiam Vda. De Dayag and others have
possessed and occupied by themselves and thru their predecessors-in-
interest the portion of Lot 2472 Cad-151, covered by LC Project 3-L of LC
Map 2999, since time immemorial. 8
Thus, the investigating team claimed that "a portion of Lot 2472 Cad-151" was "only released as
alienable and disposable on 22 February 1982".
In a Memorandum dated 9 September 1996, the Legal Division of the Land Management Bureau
recommended to the Director of Lands that an action for the cancellation of OCT No. 11585, as well as its
derivative titles, be filed with the proper court. The Director of Lands approved the recommendation.
On 10 June 1998, or 68 years after the issuance of Decree No. 381928, petitioner filed with the
Court of Appeals a complaint for annulment of judgment, cancellation and declaration of nullity of titles 9
on the ground that in 1930 the trial court had no jurisdiction to adjudicate a portion of the subject
property, which portion consists of 2,640,000 square meters (disputed portion). The disputed portion was
allegedly still classified as timber land at the time of issuance of Decree No. 381928 and, therefore, was
not alienable and disposable until 22 February 1982 when the disputed portion was classified as alienable
and disposable.
On 19 October 1998, private respondents filed a motion to dismiss. 10 Private respondents alleged that
petitioner failed to comply with Rule 47 of the Rules of Court because the real ground for the complaint
was mistake, not lack of jurisdiction, and that petitioner, as a party in the original proceedings, could
have availed of the ordinary remedies of new trial, appeal, petition for relief or other appropriate
remedies but failed to do so. Private respondents added that petitioner did not attach to the complaint a
certified true copy of the decision sought to be annulled. Private respondents also maintained that the
complaint was barred by the doctrines of res judicata and law of the case and by Section 38 of Act No.
496. 11 Private respondents also stated that not all the heirs of spouses Carag were brought before the
Court of Appeals for an effective resolution of the case. Finally, private respondents claimed that the real
party in interest was not petitioner but a certain Alfonso Bassig, who had an ax to grind against private
respondents. 12 DHEACI
On 3 March 1999, petitioner filed an amended complaint for reversion, annulment of decree, cancellation
33
and declaration of nullity of titles. 13
The Ruling of the Court of Appeals
On 21 May 2001, the Court of Appeals dismissed the complaint because of lack of jurisdiction over the
subject matter of the case. The Court of Appeals declared:
The rule is clear that such judgments, final orders and resolutions in civil
actions which this court may annul are those which the "ordinary remedies
of new trial, appeal, petition for relief or other appropriate remedies are no
longer available". The Amended Complaint contains no such allegations
which are jurisdictional neither can such circumstances be divined from its
allegations. Furthermore, such actions for Annulment may be based only
on two (2) grounds: extrinsic fraud and lack of jurisdiction. Neither ground
is alleged in the Amended Complaint which is for Reversion/Annulment of
Decree, Cancellation and Declaration of Nullity of Titles. It merely alleges
that around 2,640,000 square meters of timberland area within Lot 2472
Cad. 151, had been erroneously included in the title of the Spouses
Antonio Carag and Victoria Turingan under Decree No. 381928 and O.C.T.
No. 11585 issued on June 2, 1930 and July 19, 1938, respectively; that
hence, such adjudication and/or Decree and Title covering a timberland
area is null and void ab initio under the provisions of the 1935, 1973 and
1987 Constitutions.
Finally, it is clear that the issues raised in the Amended Complaint as well
as those in the Motion to dismiss are factual in nature and should be
threshed out in the proper trial court in accordance with Section 101 of the
Public Land Act. 14 (Citations omitted)
Petitioner filed a motion for reconsideration. In its 25 September 2002 Resolution,
the Court of Appeals denied the motion for reconsideration. IcSADC
Hence, this petition.
The Issues
Petitioner raises the following issues:
1.Whether the allegations of the complaint clearly stated that the ordinary
remedies of new trial, appeal, petition for relief and other
appropriate remedies are no longer available;
2.Whether the amended complaint clearly alleged the ground of lack of
jurisdiction;
3.Whether the Court of Appeals may try the factual issues raised in the
amended complaint and in the motion to dismiss;
4.Whether the then Court of First Instance of Cagayan had jurisdiction to
adjudicate a tract of timberland in favor of respondent
spouses Antonio Carag and Victoria Turingan;
5.Whether the fact that the Director of Lands was a party to the original
proceedings changed the nature of the land and granted
jurisdiction to the then Court of First Instance over the land;
SAHaTc
6.Whether the doctrine of res judicata applies in this case; and
7.Whether Section 38 of Act No. 496 is applicable in this case.
The Ruling of the Court
While the Court of Appeals erred in dismissing the complaint on procedural grounds, we will still deny the
petition because the complaint for annulment of decree has no merit.
Petitioner Complied with Rule 47 of the Rules of Court
First, the Court of Appeals ruled that petitioner failed to allege either of the grounds of extrinsic fraud or
lack of jurisdiction in the complaint for annulment of decree. 15
We find otherwise. In its complaint and amended complaint, petitioner stated:
11.In view of the fact that in 1930 or in 1938, only the Executive Branch of
the Government had the authority and power to declassify or reclassify
land of the public domain, the Court did not, therefore, have the
power and authority to adjudicate in favor of the spouses Antonio
Carag and Victoria Turingan the said tract of timberland, portion
of the Lot 2472 Cad-151, at the time of the issuance of the Decree
and the Original Certificate of Title of the said spouses; and such
adjudication and/or Decree and Title issued covering the timberland area is
null and void ab initio considering the provisions of the 1935, 1973 and
1987 Philippine constitution.
xxx xxx xxx
15.The issuance of Decree No. 381928 and O.C.T. No. 11585 in the name
of spouses Antonio Carag and Victoria Turingan, and all the derivative
titles thereto in the name of the Heirs and said spouses, specifically with
respect to the inclusion thereto of timberland area, by the then Court of
First Instance (now the Regional Trial Court), and the Register of Deeds of
Cagayan is patently illegal and erroneous for the reason that said Court
and/or the Register of Deeds of Cagayan did not have any
authority or jurisdiction to decree or adjudicate the said
timberland area of Lot 2472 Cad-151, consequently, the same are
null and void ab initio, and of no force and effect whatsoever. 16
(Emphasis supplied; citations omitted) CHDTEA

Petitioner clearly alleged in the complaint and amended complaint that it was seeking to annul Decree
No. 381928 on the ground of the trial court's lack of jurisdiction over the subject land, specifically over
the disputed portion, which petitioner maintained was classified as timber land and was not alienable and
disposable.
Second, the Court of Appeals also dismissed the complaint on the ground of petitioner's failure to allege
that the "ordinary remedies of new trial, appeal, petition for relief or other appropriate remedies are no
longer available".
In Ancheta v. Ancheta, 17 we ruled:
In a case where a petition for annulment of judgment or final order of the
RTC filed under Rule 47 of the Rules of Court is grounded on lack of
jurisdiction over the person of the defendant/respondent or over the
nature or subject of the action, the petitioner need not allege in the
petition that the ordinary remedy of new trial or reconsideration of the
final order or judgment or appeal therefrom are no longer available
through no fault of her own. This is so because a judgment rendered or
final order issued by the RTC without jurisdiction is null and void and may
be assailed any time either collaterally or in a direct action or by resisting
such judgment or final order in any action or proceeding whenever it is
34
invoked, unless barred by laches. 18
Since petitioner's complaint is grounded on lack of jurisdiction over the subject of the action, petitioner
need not allege that the ordinary remedies of new trial, appeal, petition for relief or other appropriate
remedies are no longer available through no fault of petitioner.
Third, the Court of Appeals ruled that the issues raised in petitioner's complaint were factual in nature
and should be threshed out in the proper trial court in accordance with Section 101 of the Public Land
Act. 19
Section 6, Rule 47 of the Rules of Court provides:
SEC. 6.Procedure. The procedure in ordinary civil cases shall be
observed. Should a trial be necessary, the reception of evidence may be
referred to a member of the court or a judge of a Regional Trial Court.
Therefore, the Court of Appeals may try the factual issues raised in the complaint for the complete
and proper determination of the case.
However, instead of remanding the complaint to the Court of Appeals for further proceedings, we shall
decide the case on the merits.
Complaint for Annulment of Decree Has No Merit
Petitioner contends that the trial court had no jurisdiction to adjudicate to spouses Carag the disputed
portion of the subject property. Petitioner claims that the disputed portion was still classified as timber
land, and thus not alienable and disposable, when Decree No. 381928 was issued in 1930. In effect,
petitioner admits that the adjacent 4,407,673 square meters of the subject property, outside of the
disputed portion, were alienable and disposable in 1930. Petitioner argues that in 1930 or in 1938, only
the Executive Branch of the Government, not the trial courts, had the power to declassify or reclassify
lands of the public domain. IEAHca
Lack of jurisdiction, as a ground for annulment of judgment, refers to either lack of jurisdiction over the
person of the defending party or over the subject matter of the claim. 20 Jurisdiction over the subject
matter is conferred by law and is determined by the statute in force at the time of the filing of the action.
21
Under the Spanish regime, all Crown lands were per se alienable. In Aldecoa v. Insular Government, 22
we ruled:
From the language of the foregoing provisions of law, it is deduced that,
with the exception of those comprised within the mineral and timber zone,
all lands owned by the State or by the sovereign nation are public
in character, and per se alienable and, provided they are not destined
to the use of the public in general or reserved by the Government in
accordance with law, they may be acquired by any private or juridical
person . . . 23 (Emphasis supplied)
Thus, unless specifically declared as mineral or forest zone, or reserved by the State for some
public purpose in accordance with law, all Crown lands were deemed alienable.
In this case, petitioner has not alleged that the disputed portion had been declared as mineral or forest
zone, or reserved for some public purpose in accordance with law, during the Spanish regime or
thereafter. The land classification maps 24 petitioner attached to the complaint also do not show that in
1930 the disputed portion was part of the forest zone or reserved for some public purpose. The
certification of the National Mapping and Resources Information Authority, dated 27 May 1994, contained
no statement that the disputed portion was declared and classified as timber land. 25 HcDSaT
The law prevailing when Decree No. 381928 was issued in 1930 was Act No. 2874, 26 which provides:
SECTION 6.The Governor-General, upon the recommendation of the
Secretary of Agriculture and Natural Resources, shall from time to time
classify the lands of the public domain into
(a)Alienable or disposable
(b)Timber and
(c)Mineral lands
and may at any time and in a like manner transfer such lands from one
class to another, for the purposes of their government and disposition.
Petitioner has not alleged that the Governor-General had declared the disputed portion of the
subject property timber or mineral land pursuant to Section 6 of Act No. 2874.
It is true that Section 8 of Act No. 2874 opens to disposition only those lands which have been declared
alienable or disposable. Section 8 provides:
SECTION 8.Only those lands shall be declared open to disposition or
concession which have been officially delimited and classified and, when
practicable, surveyed, and which have not been reserved for public or
quasi-public uses, not appropriated by the Government, nor in any
manner become private property, nor those on which a private
right authorized and recognized by this Act or any other valid law
may be claimed, or which, having been reserved or appropriated, have
ceased to be so. However, the Governor-General may, for reasons of
public interest, declare lands of the public domain open to disposition
before the same have had their boundaries established or been surveyed,
or may, for the same reasons, suspend their concession or disposition by
proclamation duly published or by Act of the Legislature. (Emphasis
supplied) IEHDAT
However, Section 8 provides that lands which are already private lands, as well as lands on which a
private claim may be made under any law, are not covered by the classification requirement in Section 8
for purposes of disposition. This exclusion in Section 8 recognizes that during the Spanish regime, Crown
lands were per se alienable unless falling under timber or mineral zones, or otherwise reserved for
some public purpose in accordance with law.
Clearly, with respect to lands excluded from the classification requirement in Section 8, trial courts had
jurisdiction to adjudicate these lands to private parties. Petitioner has not alleged that the disputed
portion had not become private property prior to the enactment of Act No. 2874. Neither has petitioner
alleged that the disputed portion was not land on which a private right may be claimed under any
existing law at that time.
In Republic of the Philippines v. Court of Appeals, 27 the Republic sought to annul the judgment of the
Court of First Instance (CFI) of Rizal, sitting as a land registration court, because when the application for
land registration was filed in 1927 the land was alleged to be unclassified forest land. The Republic also
alleged that the CFI of Rizal had no jurisdiction to determine whether the land applied for was forest or
agricultural land since the authority to classify lands was then vested in the Director of Lands as provided
in Act Nos. 926 28 and 2874. The Court ruled:
We are inclined to agree with the respondent that it is legally doubtful if
the authority of the Governor General to declare lands as alienable and
disposable would apply to lands that have become private property or
lands that have been impressed with a private right authorized and
recognized by Act 2874 or any valid law. By express declaration of Section
45 (b) of Act 2874 which is quoted above, those who have been in open,
continuous, exclusive and notorious possession and occupation of
agricultural lands of the public domain under a bona fide claim of
35
acquisition of ownership since July 26, 1894 may file an application with
the Court of First Instance of the province where the land is located for
confirmation of their claims and these applicants shall be conclusively
presumed to have performed all the conditions essential to a government
grant and shall be entitled to a certificate of title. When the land
registration court issued a decision for the issuance of a decree
which was the basis of an original certificate of title to the land,
the court had already made a determination that the land was
agricultural and that the applicant had proven that he was in
open and exclusive possession of the subject land for the
prescribed number of years. It was the land registration court
which had the jurisdiction to determine whether the land applied
for was agricultural, forest or timber taking into account the
proof or evidence in each particular case. (Emphasis supplied)
CSIHDA
As with this case, when the trial court issued the decision for the issuance of Decree No. 381928 in 1930,
the trial court had jurisdiction to determine whether the subject property, including the disputed portion,
applied for was agricultural, timber or mineral land. The trial court determined that the land was
agricultural and that spouses Carag proved that they were entitled to the decree and a certificate of title.
The government, which was a party in the original proceedings in the trial court as required by law, did
not appeal the decision of the trial court declaring the subject land as agricultural. Since the trial court
had jurisdiction over the subject matter of the action, its decision rendered in 1930, or 78 years ago, is
now final and beyond review.

The finality of the trial court's decision is further recognized in Section 1, Article XII of the 1935
Constitution which provides:
SECTION 1.All agricultural, timber, and mineral lands of the public domain,
waters, minerals, coal, petroleum, and other mineral oils, all forces of
potential energy, and other natural resources of the Philippines belong to
the State, and their disposition, exploitation, development, or utilization
shall be limited to citizens of the Philippines, or to corporations or
associations at least sixty per centum of the capital of which is owned by
such citizens, subject to any existing right, grant, lease, or
concession at the time of the inauguration of the Government
established under this Constitution. (Emphasis supplied)
Thus, even as the 1935 Constitution declared that all agricultural, timber and mineral lands of the public
domain belong to the State, it recognized that these lands were "subject to any existing right, grant,
lease or concession at the time of the inauguration of the Government established under this
Constitution". 29 When the Commonwealth Government was established under the 1935 Constitution,
spouses Carag had already an existing right to the subject land, including the disputed portion, pursuant
to Decree No. 381928 issued in 1930 by the trial court. IaAEHD
WHEREFORE, we DENY the petition. We DISMISS petitioner Republic of the Philippines' complaint for
reversion, annulment of decree, cancellation and declaration of nullity of titles for lack of merit.
SO ORDERED.

















G.R. No. 167707. October 8, 2008.
THE SECRETARY OF THE DEPARTMENT OF ENVIRONMENT AND NATURAL
RESOURCES, THE REGIONAL EXECUTIVE DIRECTOR, DENR-REGION VI,
REGIONAL TECHNICAL DIRECTOR FOR LANDS, LANDS MANAGEMENT BUREAU,
REGION VI PROVINCIAL ENVIRONMENT AND NATURAL RESOURCES OFFICER OF
KALIBO, AKLAN, REGISTER OF DEEDS, DIRECTOR OF LAND REGISTRATION
AUTHORITY, DEPARTMENT OF TOURISM SECRETARY, DIRECTOR OF PHILIPPINE
TOURISM AUTHORITY, petitioners, vs. MAYOR JOSE S. YAP, LIBERTAD TALAPIAN,
MILA Y. SUMNDAD, and ANICETO YAP, in their behalf and in behalf of all those
similarly situated, respondents.
G.R. No. 173775. October 8, 2008.
DR. ORLANDO SACAY and WILFREDO GELITO, joined by THE LANDOWNERS OF
BORACAY SIMILARLY SITUATED NAMED IN A LIST, ANNEX "A" OF THIS
PETITION, petitioners, vs. THE SECRETARY OF THE DEPARTMENT OF
ENVIRONMENT AND NATURAL RESOURCES, THE REGIONAL TECHNICAL
DIRECTOR FOR LANDS, LANDS MANAGEMENT BUREAU, REGION VI,
PROVINCIAL ENVIRONMENT AND NATURAL RESOURCES OFFICER, KALIBO,
AKLAN, respondents.
D E C I S I O N
REYES, R.T., J p:
36
At stake in these consolidated cases is the right of the present occupants of Boracay Island to secure
titles over their occupied lands.
There are two consolidated petitions. The first is G.R. No. 167707, a petition for review on certiorari of
the Decision 1 of the Court of Appeals (CA) affirming that 2 of the Regional Trial Court (RTC) in Kalibo,
Aklan, which granted the petition for declaratory relief filed by respondents-claimants Mayor Jose Yap, et
al. and ordered the survey of Boracay for titling purposes. The second is G.R. No. 173775, a petition for
prohibition, mandamus, and nullification of Proclamation No. 1064 3 issued by President Gloria
Macapagal-Arroyo classifying Boracay into reserved forest and agricultural land.
The Antecedents
G.R. No. 167707
Boracay Island in the Municipality of Malay, Aklan, with its powdery white sand beaches and warm
crystalline waters, is reputedly a premier Philippine tourist destination. The island is also home to 12,003
inhabitants 4 who live in the bone-shaped island's three barangays. 5
On April 14, 1976, the Department of Environment and Natural Resources (DENR) approved the National
Reservation Survey of Boracay Island, 6 which identified several lots as being occupied or claimed by
named persons. 7
On November 10, 1978, then President Ferdinand Marcos issued Proclamation No. 1801 8 declaring
Boracay Island, among other islands, caves and peninsulas in the Philippines, as tourist zones and
marine reserves under the administration of the Philippine Tourism Authority (PTA). President Marcos
later approved the issuance of PTA Circular 3-82 9 dated September 3, 1982, to implement
Proclamation No. 1801. CTHaSD
Claiming that Proclamation No. 1801 and PTA Circular No 3-82 precluded them from filing an application
for judicial confirmation of imperfect title or survey of land for titling purposes, respondents-claimants
Mayor Jose S. Yap, Jr., Libertad Talapian, Mila Y. Sumndad, and Aniceto Yap filed a petition for
declaratory relief with the RTC in Kalibo, Aklan.
In their petition, respondents-claimants alleged that Proclamation No. 1801 and PTA Circular No. 3-82
raised doubts on their right to secure titles over their occupied lands. They declared that they
themselves, or through their predecessors-in-interest, had been in open, continuous, exclusive, and
notorious possession and occupation in Boracay since June 12, 1945, or earlier since time immemorial.
They declared their lands for tax purposes and paid realty taxes on them. 10
Respondents-claimants posited that Proclamation No. 1801 and its implementing Circular did not place
Boracay beyond the commerce of man. Since the Island was classified as a tourist zone, it was
susceptible of private ownership. Under Section 48 (b) of Commonwealth Act (CA) No. 141, otherwise
known as the Public Land Act, they had the right to have the lots registered in their names through
judicial confirmation of imperfect titles.
The Republic, through the Office of the Solicitor General (OSG), opposed the petition for declaratory
relief. The OSG countered that Boracay Island was an unclassified land of the public domain. It formed
part of the mass of lands classified as "public forest", which was not available for disposition pursuant to
Section 3 (a) of Presidential Decree (PD) No. 705 or the Revised Forestry Code, 11 as amended.
The OSG maintained that respondents-claimants' reliance on PD No. 1801 and PTA Circular No. 3-82 was
misplaced. Their right to judicial confirmation of title was governed by CA No. 141 and PD No. 705. Since
Boracay Island had not been classified as alienable and disposable, whatever possession they had cannot
ripen into ownership. ASIETa
During pre-trial, respondents-claimants and the OSG stipulated on the following facts: (1) respondents-
claimants were presently in possession of parcels of land in Boracay Island; (2) these parcels of land
were planted with coconut trees and other natural growing trees; (3) the coconut trees had heights of
more or less twenty (20) meters and were planted more or less fifty (50) years ago; and (4)
respondents-claimants declared the land they were occupying for tax purposes. 12
The parties also agreed that the principal issue for resolution was purely legal: whether Proclamation No.
1801 posed any legal hindrance or impediment to the titling of the lands in Boracay. They decided to
forego with the trial and to submit the case for resolution upon submission of their respective
memoranda. 13
The RTC took judicial notice 14 that certain parcels of land in Boracay Island, more particularly Lots 1
and 30, Plan PSU-5344, were covered by Original Certificate of Title No. 19502 (RO 2222) in the name of
the Heirs of Ciriaco S. Tirol. These lots were involved in Civil Case Nos. 5222 and 5262 filed before the
RTC of Kalibo, Aklan. 15 The titles were issued on August 7, 1933. 16
RTC and CA Dispositions
On July 14, 1999, the RTC rendered a decision in favor of respondents-claimants, with a fallo reading:
WHEREFORE, in view of the foregoing, the Court declares that
Proclamation No. 1801 and PTA Circular No. 3-82 pose no legal obstacle to
the petitioners and those similarly situated to acquire title to their lands in
Boracay, in accordance with the applicable laws and in the manner
prescribed therein; and to have their lands surveyed and approved by
respondent Regional Technical Director of Lands as the approved survey
does not in itself constitute a title to the land. CITcSH
SO ORDERED. 17
The RTC upheld respondents-claimants' right to have their occupied lands titled in their name. It ruled
that neither Proclamation No. 1801 nor PTA Circular No. 3-82 mentioned that lands in Boracay were
inalienable or could not be the subject of disposition. 18 The Circular itself recognized private ownership
of lands. 19 The trial court cited Sections 87 20 and 53 21 of the Public Land Act as basis for
acknowledging private ownership of lands in Boracay and that only those forested areas in public lands
were declared as part of the forest reserve. 22
The OSG moved for reconsideration but its motion was denied. 23 The Republic then appealed to the CA.
On December 9, 2004, the appellate court affirmed in toto the RTC decision, disposing as follows:
cADEHI
WHEREFORE, in view of the foregoing premises, judgment is hereby
rendered by us DENYING the appeal filed in this case and AFFIRMING the
decision of the lower court. 24
The CA held that respondents-claimants could not be prejudiced by a declaration that the lands they
occupied since time immemorial were part of a forest reserve.
Again, the OSG sought reconsideration but it was similarly denied. 25 Hence, the present petition under
Rule 45.
G.R. No. 173775
On May 22, 2006, during the pendency of G.R. No. 167707, President Gloria Macapagal-Arroyo issued
Proclamation No. 1064 26 classifying Boracay Island into four hundred (400) hectares of reserved forest
land (protection purposes) and six hundred twenty-eight and 96/100 (628.96) hectares of agricultural
land (alienable and disposable). The Proclamation likewise provided for a fifteen-meter buffer zone on
each side of the centerline of roads and trails, reserved for right-of-way and which shall form part of the
area reserved for forest land protection purposes. ITECSH
On August 10, 2006, petitioners-claimants Dr. Orlando Sacay, 27 Wilfredo Gelito, 28 and other
landowners 29 in Boracay filed with this Court an original petition for prohibition, mandamus, and
nullification of Proclamation No. 1064. 30 They allege that the Proclamation infringed on their "prior
vested rights" over portions of Boracay. They have been in continued possession of their respective lots
in Boracay since time immemorial. They have also invested billions of pesos in developing their lands and
37
building internationally renowned first class resorts on their lots. 31
Petitioners-claimants contended that there is no need for a proclamation reclassifying Boracay into
agricultural land. Being classified as neither mineral nor timber land, the island is deemed agricultural
pursuant to the Philippine Bill of 1902 and Act No. 926, known as the first Public Land Act. 32 Thus, their
possession in the concept of owner for the required period entitled them to judicial confirmation of
imperfect title.
Opposing the petition, the OSG argued that petitioners-claimants do not have a vested right over their
occupied portions in the island. Boracay is an unclassified public forest land pursuant to Section 3 (a) of
PD No. 705. Being public forest, the claimed portions of the island are inalienable and cannot be the
subject of judicial confirmation of imperfect title. It is only the executive department, not the courts,
which has authority to reclassify lands of the public domain into alienable and disposable lands. There is
a need for a positive government act in order to release the lots for disposition. HEcaIC

On November 21, 2006, this Court ordered the consolidation of the two petitions as they principally
involve the same issues on the land classification of Boracay Island. 33
Issues
G.R. No. 167707
The OSG raises the lone issue of whether Proclamation No. 1801 and PTA Circular No. 3-82 pose any
legal obstacle for respondents, and all those similarly situated, to acquire title to their occupied lands in
Boracay Island. 34
G.R. No. 173775
Petitioners-claimants hoist five (5) issues, namely:
I.
AT THE TIME OF THE ESTABLISHED POSSESSION OF PETITIONERS IN
CONCEPT OF OWNER OVER THEIR RESPECTIVE AREAS IN BORACAY,
SINCE TIME IMMEMORIAL OR AT THE LATEST SINCE 30 YRS. PRIOR TO
THE FILING OF THE PETITION FOR DECLARATORY RELIEF ON NOV. 19,
1997, WERE THE AREAS OCCUPIED BY THEM PUBLIC AGRICULTURAL
LANDS AS DEFINED BY LAWS THEN ON JUDICIAL CONFIRMATION OF
IMPERFECT TITLES OR PUBLIC FOREST AS DEFINED BY SEC. 3a, PD 705?
HcTSDa
II.
HAVE PETITIONERS OCCUPANTS ACQUIRED PRIOR VESTED RIGHT OF
PRIVATE OWNERSHIP OVER THEIR OCCUPIED PORTIONS OF BORACAY
LAND, DESPITE THE FACT THAT THEY HAVE NOT APPLIED YET FOR
JUDICIAL CONFIRMATION OF IMPERFECT TITLE?
III.
IS THE EXECUTIVE DECLARATION OF THEIR AREAS AS ALIENABLE AND
DISPOSABLE UNDER SEC 6, CA 141 [AN] INDISPENSABLE PRE-REQUISITE
FOR PETITIONERS TO OBTAIN TITLE UNDER THE TORRENS SYSTEM?
IV.
IS THE ISSUANCE OF PROCLAMATION 1064 ON MAY 22, 2006, VIOLATIVE
OF THE PRIOR VESTED RIGHTS TO PRIVATE OWNERSHIP OF
PETITIONERS OVER THEIR LANDS IN BORACAY, PROTECTED BY THE DUE
PROCESS CLAUSE OF THE CONSTITUTION OR IS PROCLAMATION 1064
CONTRARY TO SEC. 8, CA 141, OR SEC. 4(a) OF RA 6657. IHCacT
V.
CAN RESPONDENTS BE COMPELLED BY MANDAMUS TO ALLOW THE
SURVEY AND TO APPROVE THE SURVEY PLANS FOR PURPOSES OF THE
APPLICATION FOR TITLING OF THE LANDS OF PETITIONERS IN
BORACAY? 35 (Underscoring supplied)
In capsule, the main issue is whether private claimants (respondents-claimants in G.R. No. 167707 and
petitioners-claimants in G.R. No. 173775) have a right to secure titles over their occupied portions in
Boracay. The twin petitions pertain to their right, if any, to judicial confirmation of imperfect title under
CA No. 141, as amended. They do not involve their right to secure title under other pertinent laws.
DCIEac
Our Ruling
Regalian Doctrine and power of the executive to reclassify lands of the public domain
Private claimants rely on three (3) laws and executive acts in their bid for judicial confirmation of
imperfect title, namely: (a) Philippine Bill of 1902 36 in relation to Act No. 926, later amended and/or
superseded by Act No. 2874 and CA No. 141; 37 (b) Proclamation No. 1801 38 issued by then President
Marcos; and (c) Proclamation No. 1064 39 issued by President Gloria Macapagal-Arroyo. We shall
proceed to determine their rights to apply for judicial confirmation of imperfect title under these laws and
executive acts.
But first, a peek at the Regalian principle and the power of the executive to reclassify lands of the public
domain.
The 1935 Constitution classified lands of the public domain into agricultural, forest or timber. 40
Meanwhile, the 1973 Constitution provided the following divisions: agricultural, industrial or commercial,
residential, resettlement, mineral, timber or forest and grazing lands, and such other classes as may be
provided by law, 41 giving the government great leeway for classification. 42 Then the 1987 Constitution
reverted to the 1935 Constitution classification with one addition: national parks. 43 Of these, only
agricultural lands may be alienated. 44 Prior to Proclamation No. 1064 of May 22, 2006, Boracay Island
had never been expressly and administratively classified under any of these grand divisions. Boracay
was an unclassified land of the public domain. cCTIaS
The Regalian Doctrine dictates that all lands of the public domain belong to the State, that the State is
the source of any asserted right to ownership of land and charged with the conservation of such
patrimony. 45 The doctrine has been consistently adopted under the 1935, 1973, and 1987
Constitutions. 46
All lands not otherwise appearing to be clearly within private ownership are presumed to belong to the
State. 47 Thus, all lands that have not been acquired from the government, either by purchase or by
grant, belong to the State as part of the inalienable public domain. 48 Necessarily, it is up to the State to
determine if lands of the public domain will be disposed of for private ownership. The government, as the
agent of the state, is possessed of the plenary power as the persona in law to determine who shall be the
favored recipients of public lands, as well as under what terms they may be granted such privilege, not
excluding the placing of obstacles in the way of their exercise of what otherwise would be ordinary acts
of ownership. 49
Our present land law traces its roots to the Regalian Doctrine. Upon the Spanish conquest of the
Philippines, ownership of all lands, territories and possessions in the Philippines passed to the Spanish
Crown. 50 The Regalian doctrine was first introduced in the Philippines through the Laws of the Indies
and the Royal Cedulas, which laid the foundation that "all lands that were not acquired from the
Government, either by purchase or by grant, belong to the public domain." 51
38
The Laws of the Indies was followed by the Ley Hipotecaria or the Mortgage Law of 1893. The Spanish
Mortgage Law provided for the systematic registration of titles and deeds as well as possessory claims.
52
The Royal Decree of 1894 or the Maura Law 53 partly amended the Spanish Mortgage Law and the Laws
of the Indies. It established possessory information as the method of legalizing possession of vacant
Crown land, under certain conditions which were set forth in said decree. 54 Under Section 393 of the
Maura Law, an informacion posesoria or possessory information title, 55 when duly inscribed in the
Registry of Property, is converted into a title of ownership only after the lapse of twenty (20) years of
uninterrupted possession which must be actual, public, and adverse, 56 from the date of its inscription.
57 However, possessory information title had to be perfected one year after the promulgation of the
Maura Law, or until April 17, 1895. Otherwise, the lands would revert to the State. 58
In sum, private ownership of land under the Spanish regime could only be founded on royal concessions
which took various forms, namely: (1) titulo real or royal grant; (2) concesion especial or special grant;
(3) composicion con el estado or adjustment title; (4) titulo de compra or title by purchase; and (5)
informacion posesoria or possessory information title. 59
The first law governing the disposition of public lands in the Philippines under American rule was
embodied in the Philippine Bill of 1902. 60 By this law, lands of the public domain in the Philippine
Islands were classified into three (3) grand divisions, to wit: agricultural, mineral, and timber or forest
lands. 61 The act provided for, among others, the disposal of mineral lands by means of absolute grant
(freehold system) and by lease (leasehold system). 62 It also provided the definition by exclusion of
"agricultural public lands". 63 Interpreting the meaning of "agricultural lands" under the Philippine Bill of
1902, the Court declared in Mapa v. Insular Government: 64 THADEI
. . . In other words, that the phrase "agricultural land" as used in Act
No. 926 means those public lands acquired from Spain which are
not timber or mineral lands. . . . 65 (Emphasis Ours)
On February 1, 1903, the Philippine Legislature passed Act No. 496, otherwise known as the Land
Registration Act. The act established a system of registration by which recorded title becomes absolute,
indefeasible, and imprescriptible. This is known as the Torrens system. 66
Concurrently, on October 7, 1903, the Philippine Commission passed Act No. 926, which was the first
Public Land Act. The Act introduced the homestead system and made provisions for judicial and
administrative confirmation of imperfect titles and for the sale or lease of public lands. It permitted
corporations regardless of the nationality of persons owning the controlling stock to lease or purchase
lands of the public domain. 67 Under the Act, open, continuous, exclusive, and notorious possession and
occupation of agricultural lands for the next ten (10) years preceding July 26, 1904 was sufficient for
judicial confirmation of imperfect title. 68
On November 29, 1919, Act No. 926 was superseded by Act No. 2874, otherwise known as the
second Public Land Act. This new, more comprehensive law limited the exploitation of agricultural lands
to Filipinos and Americans and citizens of other countries which gave Filipinos the same privileges. For
judicial confirmation of title, possession and occupation en concepto dueo since time immemorial, or
since July 26, 1894, was required. 69
After the passage of the 1935 Constitution, CA No. 141 amended Act No. 2874 on December 1, 1936.
To this day, CA No. 141, as amended, remains as the existing general law governing the classification
and disposition of lands of the public domain other than timber and mineral lands, 70 and privately
owned lands which reverted to the State. 71
Section 48 (b) of CA No. 141 retained the requirement under Act No. 2874 of possession and occupation
of lands of the public domain since time immemorial or since July 26, 1894. However, this provision was
superseded by Republic Act (RA) No. 1942, 72 which provided for a simple thirty-year prescriptive period
for judicial confirmation of imperfect title. The provision was last amended by PD No. 1073, 73 which
now provides for possession and occupation of the land applied for since June 12, 1945, or earlier.
74

The issuance of PD No. 892 75 on February 16, 1976 discontinued the use of Spanish titles as evidence
in land registration proceedings. 76 Under the decree, all holders of Spanish titles or grants should apply
for registration of their lands under Act No. 496 within six (6) months from the effectivity of the decree
on February 16, 1976. Thereafter, the recording of all unregistered lands 77 shall be governed by
Section 194 of the Revised Administrative Code, as amended by Act No. 3344. TAcSaC
On June 11, 1978, Act No. 496 was amended and updated by PD No. 1529, known as the Property
Registration Decree. It was enacted to codify the various laws relative to registration of property. 78 It
governs registration of lands under the Torrens system as well as unregistered lands, including chattel
mortgages. 79
A positive act declaring land as alienable and disposable is required. In keeping with the
presumption of State ownership, the Court has time and again emphasized that there must be a positive
act of the government, such as an official proclamation, 80 declassifying inalienable public land into
disposable land for agricultural or other purposes. 81 In fact, Section 8 of CA No. 141 limits alienable or
disposable lands only to those lands which have been "officially delimited and classified." 82
The burden of proof in overcoming the presumption of State ownership of the lands of the public domain
is on the person applying for registration (or claiming ownership), who must prove that the land subject
of the application is alienable or disposable. 83 To overcome this presumption, incontrovertible evidence
must be established that the land subject of the application (or claim) is alienable or disposable. 84
There must still be a positive act declaring land of the public domain as alienable and disposable. To
prove that the land subject of an application for registration is alienable, the applicant must establish the
existence of a positive act of the government such as a presidential proclamation or an executive order;
an administrative action; investigation reports of Bureau of Lands investigators; and a legislative act or a
statute. 85 The applicant may also secure a certification from the government that the land claimed to
have been possessed for the required number of years is alienable and disposable. 86 aITECA
In the case at bar, no such proclamation, executive order, administrative action, report, statute, or
certification was presented to the Court. The records are bereft of evidence showing that, prior to 2006,
the portions of Boracay occupied by private claimants were subject of a government proclamation that
the land is alienable and disposable. Absent such well-nigh incontrovertible evidence, the Court cannot
accept the submission that lands occupied by private claimants were already open to disposition before
2006. Matters of land classification or reclassification cannot be assumed. They call for proof. 87
Ankron and de Aldecoa did not make the whole of Boracay Island, or portions of it,
agricultural lands. Private claimants posit that Boracay was already an agricultural land pursuant to the
old cases Ankron v. Government of the Philippine Islands (1919) 88 and de Aldecoa v. The Insular
Government (1909). 89 These cases were decided under the provisions of the Philippine Bill of 1902 and
Act No. 926. There is a statement in these old cases that "in the absence of evidence to the contrary,
that in each case the lands are agricultural lands until the contrary is shown." 90
Private claimants' reliance on Ankron and de Aldecoa is misplaced. These cases did not have the effect of
converting the whole of Boracay Island or portions of it into agricultural lands. It should be stressed that
the Philippine Bill of 1902 and Act No. 926 merely provided the manner through which land registration
courts would classify lands of the public domain. Whether the land would be classified as timber, mineral,
or agricultural depended on proof presented in each case.
Ankron and De Aldecoa were decided at a time when the President of the Philippines had no power to
classify lands of the public domain into mineral, timber, and agricultural. At that time, the courts were
free to make corresponding classifications in justiciable cases, or were vested with implicit power to do
so, depending upon the preponderance of the evidence. 91 This was the Court's ruling in Heirs of the
Late Spouses Pedro S. Palanca and Soterranea Rafols Vda. de Palanca v. Republic, 92 in which it stated,
through Justice Adolfo Azcuna, viz.:
. . . Petitioners furthermore insist that a particular land need not be
formally released by an act of the Executive before it can be deemed open
39
to private ownership, citing the cases of Ramos v. Director of Lands and
Ankron v. Government of the Philippine Islands. HCDaAS
xxx xxx xxx
Petitioner's reliance upon Ramos v. Director of Lands and Ankron v. Government is misplaced. These
cases were decided under the Philippine Bill of 1902 and the first Public Land Act No. 926 enacted by the
Philippine Commission on October 7, 1926, under which there was no legal provision vesting in the Chief
Executive or President of the Philippines the power to classify lands of the public domain into mineral,
timber and agricultural so that the courts then were free to make corresponding classifications in
justiciable cases, or were vested with implicit power to do so, depending upon the preponderance of the
evidence. 93
To aid the courts in resolving land registration cases under Act No. 926, it was then necessary to devise a
presumption on land classification. Thus evolved the dictum in Ankron that "the courts have a right to
presume, in the absence of evidence to the contrary, that in each case the lands are agricultural lands
until the contrary is shown." 94
But We cannot unduly expand the presumption in Ankron and De Aldecoa to an argument that all lands
of the public domain had been automatically reclassified as disposable and alienable agricultural lands. By
no stretch of imagination did the presumption convert all lands of the public domain into agricultural
lands.
If We accept the position of private claimants, the Philippine Bill of 1902 and Act No. 926 would have
automatically made all lands in the Philippines, except those already classified as timber or mineral land,
alienable and disposable lands. That would take these lands out of State ownership and worse, would be
utterly inconsistent with and totally repugnant to the long-entrenched Regalian doctrine. aESIDH
The presumption in Ankron and De Aldecoa attaches only to land registration cases brought under the
provisions of Act No. 926, or more specifically those cases dealing with judicial and administrative
confirmation of imperfect titles. The presumption applies to an applicant for judicial or administrative
conformation of imperfect title under Act No. 926. It certainly cannot apply to landowners, such as
private claimants or their predecessors-in-interest, who failed to avail themselves of the benefits of Act
No. 926. As to them, their land remained unclassified and, by virtue of the Regalian doctrine, continued
to be owned by the State.
In any case, the assumption in Ankron and De Aldecoa was not absolute. Land classification was, in the
end, dependent on proof. If there was proof that the land was better suited for non-agricultural uses, the
courts could adjudge it as a mineral or timber land despite the presumption. In Ankron, this Court stated:
In the case of Jocson vs. Director of Forestry (supra), the Attorney-General
admitted in effect that whether the particular land in question belongs to
one class or another is a question of fact. The mere fact that a tract of
land has trees upon it or has mineral within it is not of itself sufficient to
declare that one is forestry land and the other, mineral land. There must
be some proof of the extent and present or future value of the forestry
and of the minerals. While, as we have just said, many definitions have
been given for "agriculture", "forestry", and "mineral" lands, and that in
each case it is a question of fact, we think it is safe to say that in order to
be forestry or mineral land the proof must show that it is more valuable for
the forestry or the mineral which it contains than it is for agricultural
purposes. (Sec. 7, Act No. 1148.) It is not sufficient to show that there
exists some trees upon the land or that it bears some mineral. Land may
be classified as forestry or mineral today, and, by reason of the exhaustion
of the timber or mineral, be classified as agricultural land tomorrow. And
vice-versa, by reason of the rapid growth of timber or the discovery of
valuable minerals, lands classified as agricultural today may be differently
classified tomorrow. Each case must be decided upon the proof in
that particular case, having regard for its present or future value
for one or the other purposes. We believe, however, considering the
fact that it is a matter of public knowledge that a majority of the lands in
the Philippine Islands are agricultural lands that the courts have a right to
presume, in the absence of evidence to the contrary, that in each case the
lands are agricultural lands until the contrary is shown. Whatever the
land involved in a particular land registration case is forestry or
mineral land must, therefore, be a matter of proof. Its superior
value for one purpose or the other is a question of fact to be
settled by the proof in each particular case. The fact that the land is
a manglar [mangrove swamp] is not sufficient for the courts to decide
whether it is agricultural, forestry, or mineral land. It may perchance
belong to one or the other of said classes of land. The Government, in the
first instance, under the provisions of Act No. 1148, may, by reservation,
decide for itself what portions of public land shall be considered forestry
land, unless private interests have intervened before such reservation is
made. In the latter case, whether the land is agricultural, forestry, or
mineral, is a question of proof. Until private interests have intervened, the
Government, by virtue of the terms of said Act (No. 1148), may decide for
itself what portions of the "public domain" shall be set aside and reserved
as forestry or mineral land. (Ramos vs. Director of Lands, 39 Phil. 175;
Jocson vs. Director of Forestry, supra) 95 (Emphasis ours) ACSaHc

Since 1919, courts were no longer free to determine the classification of lands from the facts of each
case, except those that have already became private lands. 96 Act No. 2874, promulgated in 1919 and
reproduced in Section 6 of CA No. 141, gave the Executive Department, through the President, the
exclusive prerogative to classify or reclassify public lands into alienable or disposable, mineral or forest.
96-a Since then, courts no longer had the authority, whether express or implied, to determine the
classification of lands of the public domain. 97
Here, private claimants, unlike the Heirs of Ciriaco Tirol who were issued their title in 1933, 98 did not
present a justiciable case for determination by the land registration court of the property's land
classification. Simply put, there was no opportunity for the courts then to resolve if the land the Boracay
occupants are now claiming were agricultural lands. When Act No. 926 was supplanted by Act No. 2874
in 1919, without an application for judicial confirmation having been filed by private claimants or their
predecessors-in-interest, the courts were no longer authorized to determine the property's land
classification. Hence, private claimants cannot bank on Act No. 926.
We note that the RTC decision 99 in G.R. No. 167707 mentioned Krivenko v. Register of Deeds of Manila,
100 which was decided in 1947 when CA No. 141, vesting the Executive with the sole power to classify
lands of the public domain was already in effect. Krivenko cited the old cases Mapa v. Insular
Government, 101 De Aldecoa v. The Insular Government, 102 and Ankron v. Government of the
Philippine Islands. 103
Krivenko, however, is not controlling here because it involved a totally different issue. The pertinent issue
in Krivenko was whether residential lots were included in the general classification of agricultural lands;
and if so, whether an alien could acquire a residential lot. This Court ruled that as an alien, Krivenko was
prohibited by the 1935 Constitution 104 from acquiring agricultural land, which included residential lots.
Here, the issue is whether unclassified lands of the public domain are automatically deemed agricultural.
ASIETa
Notably, the definition of "agricultural public lands" mentioned in Krivenko relied on the old cases decided
prior to the enactment of Act No. 2874, including Ankron and De Aldecoa. 105 As We have already
stated, those cases cannot apply here, since they were decided when the Executive did not have the
authority to classify lands as agricultural, timber, or mineral.
Private claimants' continued possession under Act No. 926 does not create a presumption
that the land is alienable. Private claimants also contend that their continued possession of portions of
Boracay Island for the requisite period of ten (10) years under Act No. 926 106 ipso facto converted the
island into private ownership. Hence, they may apply for a title in their name. EHSADc
40
A similar argument was squarely rejected by the Court in Collado v. Court of Appeals. 107 Collado, citing
the separate opinion of now Chief Justice Reynato S. Puno in Cruz v. Secretary of Environment and
Natural Resources, 107-a ruled:
"Act No. 926, the first Public Land Act, was passed in
pursuance of the provisions of the Philippine Bill of 1902. The
law governed the disposition of lands of the public domain. It
prescribed rules and regulations for the homesteading,
selling and leasing of portions of the public domain of the
Philippine Islands, and prescribed the terms and conditions to
enable persons to perfect their titles to public lands in the
Islands. It also provided for the "issuance of patents to
certain native settlers upon public lands", for the
establishment of town sites and sale of lots therein, for the
completion of imperfect titles, and for the cancellation or
confirmation of Spanish concessions and grants in the
Islands". In short, the Public Land Act operated on the
assumption that title to public lands in the Philippine Islands
remained in the government; and that the government's title
to public land sprung from the Treaty of Paris and other
subsequent treaties between Spain and the United States.
The term "public land" referred to all lands of the public
domain whose title still remained in the government and are
thrown open to private appropriation and settlement, and
excluded the patrimonial property of the government and the
friar lands."
Thus, it is plain error for petitioners to argue that under the
Philippine Bill of 1902 and Public Land Act No. 926, mere
possession by private individuals of lands creates the legal
presumption that the lands are alienable and disposable. 108
(Emphasis Ours)
Except for lands already covered by existing titles, Boracay was an unclassified land of the
public domain prior to Proclamation No. 1064. Such unclassified lands are considered public
forest under PD No. 705. The DENR 109 and the National Mapping and Resource Information
Authority 110 certify that Boracay Island is an unclassified land of the public domain. SEHTIc
PD No. 705 issued by President Marcos categorized all unclassified lands of the public domain as public
forest. Section 3 (a) of PD No. 705 defines a public forest as "a mass of lands of the public domain which
has not been the subject of the present system of classification for the determination of which lands are
needed for forest purpose and which are not". Applying PD No. 705, all unclassified lands, including those
in Boracay Island, are ipso facto considered public forests. PD No. 705, however, respects titles already
existing prior to its effectivity.
The Court notes that the classification of Boracay as a forest land under PD No. 705 may seem to be out
of touch with the present realities in the island. Boracay, no doubt, has been partly stripped of its forest
cover to pave the way for commercial developments. As a premier tourist destination for local and foreign
tourists, Boracay appears more of a commercial island resort, rather than a forest land.
Nevertheless, that the occupants of Boracay have built multi-million peso beach resorts on the island;
111 that the island has already been stripped of its forest cover; or that the implementation of
Proclamation No. 1064 will destroy the island's tourism industry, do not negate its character as public
forest. AaIDCS
Forests, in the context of both the Public Land Act and the Constitution 112 classifying lands of the
public domain into "agricultural, forest or timber, mineral lands, and national parks", do not necessarily
refer to large tracts of wooded land or expanses covered by dense growths of trees and underbrushes.
113 The discussion in Heirs of Amunategui v. Director of Forestry 114 is particularly instructive:
A forested area classified as forest land of the public domain does not lose
such classification simply because loggers or settlers may have stripped it
of its forest cover. Parcels of land classified as forest land may actually be
covered with grass or planted to crops by kaingin cultivators or other
farmers. "Forest lands" do not have to be on mountains or in out of the
way places. Swampy areas covered by mangrove trees, nipa palms, and
other trees growing in brackish or sea water may also be classified as
forest land. The classification is descriptive of its legal nature or
status and does not have to be descriptive of what the land
actually looks like. Unless and until the land classified as "forest" is
released in an official proclamation to that effect so that it may form part
of the disposable agricultural lands of the public domain, the rules on
confirmation of imperfect title do not apply. 115 (Emphasis supplied)
There is a big difference between "forest" as defined in a dictionary and "forest or timber land" as a
classification of lands of the public domain as appearing in our statutes. One is descriptive of what
appears on the land while the other is a legal status, a classification for legal purposes. 116 At any rate,
the Court is tasked to determine the legal status of Boracay Island, and not look into its physical layout.
Hence, even if its forest cover has been replaced by beach resorts, restaurants and other commercial
establishments, it has not been automatically converted from public forest to alienable agricultural land.
AHDacC
Private claimants cannot rely on Proclamation No. 1801 as basis for judicial confirmation of
imperfect title. The proclamation did not convert Boracay into an agricultural land. However,
private claimants argue that Proclamation No. 1801 issued by then President Marcos in 1978 entitles
them to judicial confirmation of imperfect title. The Proclamation classified Boracay, among other islands,
as a tourist zone. Private claimants assert that, as a tourist spot, the island is susceptible of private
ownership.
Proclamation No. 1801 or PTA Circular No. 3-82 did not convert the whole of Boracay into an agricultural
land. There is nothing in the law or the Circular which made Boracay Island an agricultural land. The
reference in Circular No. 3-82 to "private lands" 117 and "areas declared as alienable and disposable"
118 does not by itself classify the entire island as agricultural. Notably, Circular No. 3-82 makes
reference not only to private lands and areas but also to public forested lands. Rule VIII, Section 3
provides:
No trees in forested private lands may be cut without prior authority from
the PTA. All forested areas in public lands are declared forest
reserves. (Emphasis supplied) AHDacC
Clearly, the reference in the Circular to both private and public lands merely recognizes that the island
can be classified by the Executive department pursuant to its powers under CA No. 141. In fact, Section
5 of the Circular recognizes the then Bureau of Forest Development's authority to declare areas in the
island as alienable and disposable when it provides:

Subsistence farming, in areas declared as alienable and disposable by the
Bureau of Forest Development.
Therefore, Proclamation No. 1801 cannot be deemed the positive act needed to classify Boracay Island
as alienable and disposable land. If President Marcos intended to classify the island as alienable and
disposable or forest, or both, he would have identified the specific limits of each, as President Arroyo did
in Proclamation No. 1064. This was not done in Proclamation No. 1801. HEISca
The Whereas clauses of Proclamation No. 1801 also explain the rationale behind the declaration of
Boracay Island, together with other islands, caves and peninsulas in the Philippines, as a tourist zone and
marine reserve to be administered by the PTA to ensure the concentrated efforts of the public and
private sectors in the development of the areas' tourism potential with due regard for ecological balance
41
in the marine environment. Simply put, the proclamation is aimed at administering the islands for
tourism and ecological purposes. It does not address the areas' alienability. 119
More importantly, Proclamation No. 1801 covers not only Boracay Island, but sixty-four (64) other
islands, coves, and peninsulas in the Philippines, such as Fortune and Verde Islands in Batangas, Port
Galera in Oriental Mindoro, Panglao and Balicasag Islands in Bohol, Coron Island, Puerto Princesa and
surrounding areas in Palawan, Camiguin Island in Cagayan de Oro, and Misamis Oriental, to name a few.
If the designation of Boracay Island as tourist zone makes it alienable and disposable by virtue of
Proclamation No. 1801, all the other areas mentioned would likewise be declared wide open for private
disposition. That could not have been, and is clearly beyond, the intent of the proclamation.
It was Proclamation No. 1064 of 2006 which positively declared part of Boracay as alienable
and opened the same to private ownership. Sections 6 and 7 of CA No. 141 120 provide that it is
only the President, upon the recommendation of the proper department head, who has the authority to
classify the lands of the public domain into alienable or disposable, timber and mineral lands. 121
In issuing Proclamation No. 1064, President Gloria Macapagal-Arroyo merely exercised the authority
granted to her to classify lands of the public domain, presumably subject to existing vested rights.
Classification of public lands is the exclusive prerogative of the Executive Department, through the Office
of the President. Courts have no authority to do so. 122 Absent such classification, the land remains
unclassified until released and rendered open to disposition. 123
Proclamation No. 1064 classifies Boracay into 400 hectares of reserved forest land and 628.96 hectares
of agricultural land. The Proclamation likewise provides for a 15-meter buffer zone on each side of the
center line of roads and trails, which are reserved for right of way and which shall form part of the area
reserved for forest land protection purposes. HCSEIT
Contrary to private claimants' argument, there was nothing invalid or irregular, much less
unconstitutional, about the classification of Boracay Island made by the President through Proclamation
No. 1064. It was within her authority to make such classification, subject to existing vested rights.
Proclamation No. 1064 does not violate the Comprehensive Agrarian Reform Law. Private
claimants further assert that Proclamation No. 1064 violates the provision of the Comprehensive Agrarian
Reform Law (CARL) or RA No. 6657 barring conversion of public forests into agricultural lands. They
claim that since Boracay is a public forest under PD No. 705, President Arroyo can no longer convert it
into an agricultural land without running afoul of Section 4 (a) of RA No. 6657, thus:
SEC. 4.Scope. The Comprehensive Agrarian Reform Law of 1988 shall
cover, regardless of tenurial arrangement and commodity produced, all
public and private agricultural lands as provided in Proclamation No. 131
and Executive Order No. 229, including other lands of the public domain
suitable for agriculture. aEHASI
More specifically, the following lands are covered by the Comprehensive
Agrarian Reform Program:
(a)All alienable and disposable lands of the public domain
devoted to or suitable for agriculture. No
reclassification of forest or mineral lands to
agricultural lands shall be undertaken after the
approval of this Act until Congress, taking into
account ecological, developmental and equity
considerations, shall have determined by law,
the specific limits of the public domain.
That Boracay Island was classified as a public forest under PD No. 705 did not bar the Executive from
later converting it into agricultural land. Boracay Island still remained an unclassified land of the public
domain despite PD No. 705.
In Heirs of the Late Spouses Pedro S. Palanca and Soterranea Rafols v. Republic, 124 the Court stated
that unclassified lands are public forests.
While it is true that the land classification map does not
categorically state that the islands are public forests, the fact that
they were unclassified lands leads to the same result. In the
absence of the classification as mineral or timber land, the land remains
unclassified land until released and rendered open to disposition. 125
(Emphasis supplied)
Moreover, the prohibition under the CARL applies only to a "reclassification" of land. If the land had never
been previously classified, as in the case of Boracay, there can be no prohibited reclassification under the
agrarian law. We agree with the opinion of the Department of Justice 126 on this point:
Indeed, the key word to the correct application of the prohibition in
Section 4 (a) is the word "reclassification". Where there has been no
previous classification of public forest [referring, we repeat, to the mass of
the public domain which has not been the subject of the present system of
classification for purposes of determining which are needed for forest
purposes and which are not] into permanent forest or forest reserves or
some other forest uses under the Revised Forestry Code, there can be no
"reclassification of forest lands" to speak of within the meaning of Section
4(a). DcCIAa
Thus, obviously, the prohibition in Section 4(a) of the CARL against the
reclassification of forest lands to agricultural lands without a prior law
delimiting the limits of the public domain, does not, and cannot, apply to
those lands of the public domain, denominated as "public forest" under the
Revised Forestry Code, which have not been previously determined, or
classified, as needed for forest purposes in accordance with the provisions
of the Revised Forestry Code. 127
Private claimants are not entitled to apply for judicial confirmation of imperfect title under
CA No. 141. Neither do they have vested rights over the occupied lands under the said law.
There are two requisites for judicial confirmation of imperfect or incomplete title under CA No. 141,
namely: (1) open, continuous, exclusive, and notorious possession and occupation of the subject land by
himself or through his predecessors-in-interest under a bona fide claim of ownership since time
immemorial or from June 12, 1945; and (2) the classification of the land as alienable and disposable land
of the public domain. 128
As discussed, the Philippine Bill of 1902, Act No. 926, and Proclamation No. 1801 did not convert portions
of Boracay Island into an agricultural land. The island remained an unclassified land of the public domain
and, applying the Regalian doctrine, is considered State property.
Private claimants' bid for judicial confirmation of imperfect title, relying on the Philippine Bill of 1902, Act
No. 926, and Proclamation No. 1801, must fail because of the absence of the second element of alienable
and disposable land. Their entitlement to a government grant under our present Public Land Act
presupposes that the land possessed and applied for is already alienable and disposable. This is clear
from the wording of the law itself. 129 Where the land is not alienable and disposable, possession of the
land, no matter how long, cannot confer ownership or possessory rights. 130
Neither may private claimants apply for judicial confirmation of imperfect title under Proclamation No.
1064, with respect to those lands which were classified as agricultural lands. Private claimants failed to
prove the first element of open, continuous, exclusive, and notorious possession of their lands in Boracay
since June 12, 1945.
We cannot sustain the CA and RTC conclusion in the petition for declaratory relief that private claimants
complied with the requisite period of possession.
42
The tax declarations in the name of private claimants are insufficient to prove the first element of
possession. We note that the earliest of the tax declarations in the name of private claimants were issued
in 1993. Being of recent dates, the tax declarations are not sufficient to convince this Court that the
period of possession and occupation commenced on June 12, 1945. IEAHca
Private claimants insist that they have a vested right in Boracay, having been in possession of the island
for a long time. They have invested millions of pesos in developing the island into a tourist spot. They say
their continued possession and investments give them a vested right which cannot be unilaterally
rescinded by Proclamation No. 1064.
The continued possession and considerable investment of private claimants do not automatically give
them a vested right in Boracay. Nor do these give them a right to apply for a title to the land they are
presently occupying. This Court is constitutionally bound to decide cases based on the evidence
presented and the laws applicable. As the law and jurisprudence stand, private claimants are ineligible to
apply for a judicial confirmation of title over their occupied portions in Boracay even with their continued
possession and considerable investment in the island.

One Last Note
The Court is aware that millions of pesos have been invested for the development of Boracay Island,
making it a by-word in the local and international tourism industry. The Court also notes that for a
number of years, thousands of people have called the island their home. While the Court commiserates
with private claimants' plight, We are bound to apply the law strictly and judiciously. This is the law and it
should prevail. Ito ang batas at ito ang dapat umiral. HScCEa
All is not lost, however, for private claimants. While they may not be eligible to apply for judicial
confirmation of imperfect title under Section 48 (b) of CA No. 141, as amended, this does not denote
their automatic ouster from the residential, commercial, and other areas they possess now classified as
agricultural. Neither will this mean the loss of their substantial investments on their occupied alienable
lands. Lack of title does not necessarily mean lack of right to possess.
For one thing, those with lawful possession may claim good faith as builders of improvements. They can
take steps to preserve or protect their possession. For another, they may look into other modes of
applying for original registration of title, such as by homestead 131 or sales patent, 132 subject to the
conditions imposed by law.
More realistically, Congress may enact a law to entitle private claimants to acquire title to their occupied
lots or to exempt them from certain requirements under the present land laws. There is one such bill
133 now pending in the House of Representatives. Whether that bill or a similar bill will become a law is
for Congress to decide.
In issuing Proclamation No. 1064, the government has taken the step necessary to open up the island to
private ownership. This gesture may not be sufficient to appease some sectors which view the
classification of the island partially into a forest reserve as absurd. That the island is no longer overrun by
trees, however, does not becloud the vision to protect its remaining forest cover and to strike a healthy
balance between progress and ecology. Ecological conservation is as important as economic progress.
EacHCD
To be sure, forest lands are fundamental to our nation's survival. Their promotion and protection are not
just fancy rhetoric for politicians and activists. These are needs that become more urgent as destruction
of our environment gets prevalent and difficult to control. As aptly observed by Justice Conrado Sanchez
in 1968 in Director of Forestry v. Munoz: 134
The view this Court takes of the cases at bar is but in adherence to public
policy that should be followed with respect to forest lands. Many have
written much, and many more have spoken, and quite often, about the
pressing need for forest preservation, conservation, protection,
development and reforestation. Not without justification. For, forests
constitute a vital segment of any country's natural resources. It is of
common knowledge by now that absence of the necessary green cover on
our lands produces a number of adverse or ill effects of serious
proportions. Without the trees, watersheds dry up; rivers and lakes which
they supply are emptied of their contents. The fish disappear. Denuded
areas become dust bowls. As waterfalls cease to function, so will
hydroelectric plants. With the rains, the fertile topsoil is washed away;
geological erosion results. With erosion come the dreaded floods that
wreak havoc and destruction to property crops, livestock, houses, and
highways not to mention precious human lives. Indeed, the foregoing
observations should be written down in a lumberman's decalogue. 135
WHEREFORE, judgment is rendered as follows:
1.The petition for certiorari in G.R. No. 167707 is GRANTED and the Court of Appeals Decision in CA-G.R.
CV No. 71118 REVERSED AND SET ASIDE.
2.The petition for certiorari in G.R. No. 173775 is DISMISSED for lack of merit.
SO ORDERED.



















43





g. Non-registrable properties

G.R. No. L-27873. November 29, 1983.
HEIRS OF JOSE AMUNATEGUI, petitioners, vs. DIRECTOR OF FORESTRY, respondent.
G.R. No. L-30035. November 29, 1983.
ROQUE BORRE and ENCARNACION DELFIN, petitioners, vs. ANGEL ALPASAN,
HEIRS OF MELQUIADES BORRE, EMETERIO BEREBER and HEIRS OF JOSE
AMUNATEGUI and THE CAPIZ COURT OF FIRST INSTANCE, respondents.
D E C I S I O N
GUTIERREZ, JR., J p:
The two petitions for review on certiorari before us question the decision of the Court of Appeals which
declared the disputed property as forest land, not subject to titling in favor of private persons.
These two petitions have their genesis in an application for confirmation of imperfect title and its
registration filed with the Court of First Instance of Capiz. The parcel of land sought to be registered is
known as Lot No. 885 of the Cadastral Survey of Pilar, Capiz, and has an area of 645,703 square meters.
LexLib
Roque Borre, petitioner in G.R. No, L-30035, and Melquiades Borre, filed the application for registration.
In due time, the heirs of Jose Amunategui, petitioners in G.R. No. L-27873 filed an opposition to the
application of Roque and Melquiades Borre. At the same time, they prayed that the title to a portion of
Lot No. 885 of Pilar Cadastre containing 527,747 square meters be confirmed and registered in the
names of said Heirs of Jose Amunategui.
The Director of Forestry, through the Provincial Fiscal of Capiz, also filed an opposition to the application
for registration of title claiming that the land was mangrove swamp which was still classified as forest
land and part of the public domain.
Another oppositor, Emeterio Bereber filed his opposition insofar as a portion of Lot No. 885 containing
117,956 square meters was concerned and prayed that title to said portion be confirmed and registered
in his name.
During the progress of the trial, applicant-petitioner Roque Borre sold whatever rights and interests he
may have on Lot No. 885 to Angel Alpasan. The latter also filed an opposition, claiming that he is entitled
to have said lot registered in his name.
After trial, the Court of First Instance of Capiz adjudicated 117,956 square meters to Emeterio Bereber
and the rest of the land containing 527,747 square meters was adjudicated in the proportion of 5/6 share
to Angel Alpasan and 1/6 share to Melquiades Borre.
Only the Heirs of Jose Amunategui and the Director of Forestry filed their respective appeals with the
Court of Appeals, The case was docketed as CA-G.R. No. 34190-R.
In its decision, the Court of Appeals held:
". . . the conclusion so far must have to be that as to the private litigants
that have been shown to have a better right over Lot 885 are, as to the
northeastern portion of a little less than 117,956 square meters, it was
Emeterio Bereber and as to the rest of 527,747 square meters, it was the
heirs of Jose Amunategui; but the last question that must have to be
considered is whether after all, the title that these two (2) private litigants
have shown did not amount to a registerable one in view of the opposition
and evidence of the Director of Forestry; . . .
". . . turning back the clock thirty (30) years from 1955 when the
application was filed which would place it at 1925, the fact must have to
be accepted that during that period, the land was a classified forest land
so much so that timber licenses had to be issued to certain licensee before
1926 and after that; that even Jose Amunategui himself took the trouble
to ask for a license to cut timber within the area; and this can only mean
that the Bureau of Forestry had stood and maintained its ground that it
was a forest land as indeed the testimonial evidence referred to above
persuasively indicates, and the only time when the property was converted
into a fishpond was sometime after 1950; or a bare five (5) years before
the filing of the application; but only after there had been a previous
warning by the District Forester that that could not be done because it was
classified as a public forest; so that having these in mind and remembering
that even under Republic Act 1942 which came into effect in 1957, two (2)
years after this case had already been filed in the lower Court, in order for
applicant to be able to demonstrate a registerable title he must have
shown.
"'open, continuous, exclusive and notorious possession and
occupation of agricultural lands of the public domain under a
bona fide claim of acquisition of ownership for at least thirty
(30) years, preceding the filing of the application;'
the foregoing details cannot but justify the conclusion that not one of the
applicants or oppositors had shown that during the required period of
thirty (30) years prescribed by Republic Act 1942 in order for him to have
shown a registerable title for the entire period of thirty (30) years before
filing of the application, he had been in
"'open, continuous, exclusive and notorious possession and
occupation of agricultural lands of the public domain',
it is evident that the Bureau of Forestry had insisted on its claim all
throughout that period of thirty (30) years and even before and applicants
and their predecessors had made implicit recognition of that; the result
must be to deny all these applications; this Court stating that it had felt
impelled notwithstanding, just the same to resolve the conflicting positions
of the private litigants among themselves as to who of them had
demonstrated a better right to possess because this Court foresees that
this litigation will go all the way to the Supreme Court and it is always
better that the findings be as complete as possible to enable the Highest
Court to pass final judgment;
44
"IN VIEW WHEREOF, the decision must have to be as it is hereby
reversed; the application as well as all the oppositions with the exception
of that of the Director of Forestry which is hereby sustained are dismissed;
no more pronouncement as to costs."
A petition for review on certiorari was filed by the Heirs of Jose Amunategui contending that the disputed
lot had been in the possession of private persons for over thirty years and therefore in accordance with
Republic Act No. 1942, said lot could still be the subject of registration and confirmation of title in the
name of a private person in accordance with Act No. 496 known as the Land Registration Act. On the
other hand, another petition for review on certiorari was filed by Roque Borre and Encarnacion Delfin,
contending that the trial court committed grave abuse of discretion in dismissing their complaint against
the Heirs of Jose Amunategui. The Borre complaint was for the annulment of the deed of absolute sale of
Lot No. 885 executed by them in favor of the Heirs of Amunategui. The complaint was dismissed on the
basis of the Court of Appeals' decision that the disputed lot is part of the public domain. The petitioners
also question the jurisdiction of the Court of Appeals in passing upon the relative rights of the parties
over the disputed lot when its final decision after all is to declare said lot a part of the public domain
classified as forest land. LLpr
The need for resolving the questions raised by Roque Borre and Encarnacion Delfin in their petition
depends on the issue raised by the Heirs of Jose Amunategui, that is, whether or not Lot No. 885 is
public forest land, not capable of registration in the names of the private applicants.

The Heirs of Jose Amunategui maintain that Lot No. 885 cannot be classified as forest land because it is
not thickly forested but is a "mangrove swamp". Although conceding that a "mangrove swamp" is
included in the classification of forest land in accordance with Section 1820 of the Revised Administrative
Code, the petitioners argue that no big trees classified in Section 1821 of said Code as first, second and
third groups are found on the land in question. Furthermore, they contend that Lot 885, even if it is a
mangrove swamp, is still subject to land registration proceedings because the property had been in
actual possession of private persons for many years, and therefore, said land was already "private land"
better adapted and more valuable for agricultural than for forest purposes and not required by the public
interests to be kept under forest classification.
The petition is without merit.
A forested area classified as forest land of the public domain does not lose such classification simply
because loggers or settlers may have stripped it of its forest cover. Parcels of land classified as forest
land may actually be covered with grass or planted to crops by kaingin cultivators or other farmers.
"Forest lands" do not have to be on mountains or in out of the way places. Swampy areas covered by
mangrove trees, nipa palms, and other trees growing in brackish or sea water may also be classified as
forest land. The classification is descriptive of its legal nature or status and does not have to be
descriptive of what the land actually looks like. Unless and until the land classified as "forest" is released
in an official proclamation to that effect so that it may form part of the disposable agricultural lands of
the public domain, the rules on confirmation of imperfect title do not apply.
This Court ruled in the leading case of Director of Forestry v. Muoz (23 SCRA 1184) that possession of
forest lands, no matter how long, cannot ripen into private ownership. And in Republic v. Animas (56
SCRA 499), we granted the petition on the ground that the area covered by the patent and title was not
disposable public land, it being a part of the forest zone and any patent and title to said area is void ab
initio. It bears emphasizing that a positive act of Government is needed to declassify land which is
classified as forest and to convert it into alienable or disposable land for agricultural or other purposes.
The findings of the Court of Appeals are particularly well-grounded in the instant petition.
The fact that no trees enumerated in Section 1821 of the Revised Administrative Code are found in Lot
No. 885 does not divest such land of its being classified as forest land, much less as land of the public
domain. The appellate court found that in 1912, the land must have been a virgin forest as stated by
Emeterio Bereber's witness Deogracias Gavacao, and that as late as 1926, it must have been a thickly
forested area as testified by Jaime Bertolde. The opposition of the Director of Forestry was strengthened
by the appellate court's finding that timber licenses had to be issued to certain licensees and even Jose
Amunategui himself took the trouble to ask for a license to cut timber within the area. It was only
sometime in 1950 that the property was converted into fishpond but only after a previous warning from
the District Forester that the same could not be done because it was classified as "public forest." LibLex
In confirmation of imperfect title cases, the applicant shoulders the burden of proving that he meets the
requirements of Section 48, Commonwealth Act No. 141, as amended by Republic Act No. 1942. He must
overcome the presumption that the land he is applying for is part of the public domain but that he has an
interest therein sufficient to warrant registration in his name because of an imperfect title such as those
derived from old Spanish grants or that he has had continuous, open, and notorious possession and
occupation of agricultural lands of the public domain under a bona fide claim of acquisition of ownership
for at least thirty (30) years preceding the filing of his application.
The decision of the appellate court is not based merely on the presumptions implicit in Commonwealth
Act No. 141 as amended. The records show that Lot No. 88S never ceased to be classified as forest land
of the public domain.
In Republic v. Gonong (118 SCRA 729) we ruled:
"As held in Oh Cho v. Director of Lands, 75 Phil. 890, all lands that were
not acquired from the Government, either by purchase or by grant, belong
to the public domain. An exception to the rule would be any land that
should have been in the possession of an occupant and of his predecessors
in-interests since time immemorial, for such possession would justify the
presumption that the land had never been part of the public domain or
that it had been a private property even before the Spanish conquest."
In the instant petitions, the exception in the Oh Cho case does not apply. The evidence is clear that Lot
No. 885 had always been public land classified as forest.
Similarly, in Republic v. Vera (120 SCRA 210), we ruled:
". . . The possession of public land however long the period thereof may
have extended, never confers title thereto upon the possessor because the
statute of limitations with regard to public land does not operate against
the State, unless the occupant can prove possession and occupation of the
same under claim of ownership for the required number of years to
constitute a grant from the State. (Director of Lands v. Reyes, 68 SCRA
177, 195)."
We, therefore, affirm the finding that the disputed property Lot No. 885 is part of the public domain,
classified as public forest land. There is no need for us to pass upon the other issues raised by petitioners
Roque Borre and Encarnacion Delfin, as such issues are rendered moot by this finding. Cdpr
WHEREFORE, the petitions in G. R. No. L-30035 and G. R. No. L-27873 are DISMISSED for lack of merit.
Costs against the petitioners.
SO ORDERED.





45















G.R. No. L-39473. April 30, 1979.
REPUBLIC OF THE PHILIPPINES, petitioner, vs. HON. COURT OF APPEALS and
ISABEL LASTIMADO, respondents.
D E C I S I O N
MELENCIO-HERRERA, J p:
This is a Petition for Review (Appeal) by Certiorari filed by the Republic of the Philippines from the
Decision of the Court of Appeals promulgated on September 30, 1974 in CA-G.R. No. Sp-01504 denying
the State's Petition for Certiorari and Mandamus.
Briefly, the facts of the case are as follows:
Private respondent, Isabel Lastimado, filed on September 11, 1967, in the Court of First Instance of
Bataan, Branch I, a Petition for the reopening of cadastral proceedings over a portion of Lot No. 626 of
the Mariveles Cadastre, consisting of 971.0569 hectares, pursuant to Republic Act No. 931, as amended
by Republic Act No. 2061, docketed as Cad Case No. 19, LRC Cad. Rec. No. 1097. In the absence of any
opposition, whether from the Government or from private individuals, private respondent was allowed to
present her evidence ex-parte. On October 14, 1967, the trial Court rendered a Decision granting the
Petition and adjudicating the land in favor of private respondent. The trial Court issued an order for the
issuance of a decree of registration on November 20, 1967, and on November 21, 1967, the Land
Registration Commission issued Decree No. N-117573 in favor of private respondent. Eventually, Original
Certificate of Title No. N-144 was also issued in her favor. Private respondent thereafter subdivided the
land into ten lots, and the corresponding titles. Transfer Certificates of Title Nos. 18905 to 18914
inclusive, were issued by the Register of Deeds. LibLex
On June 3, 1968, or within one year from the entry of the decree of registration, petitioner filed a Petition
for Review pursuant to Sec. 38, Act No. 496, on the ground of fraud alleging that during the period of
alleged adverse possession by private respondent, said parcel of land was part of the U.S. Military
Reservation in Bataan, which was formally turned over to the Republic of the Philippines only on
December 22, 1965, and that the same is inside the public forest of Mariveles, Bataan and, therefore, not
subject to disposition or acquisition under the Public Land Law. Respondent field an Opposition thereto,
which was considered by the trial Court, as a Motion to Dismiss, and on December 20, 1968, said Court
(Judge Tito V. Tizon, presiding) issued an Order dismissing the Petition for Review mainly on the ground
that the Solicitor General had failed to file opposition to the original Petition for reopening of the
cadastral proceedings and was, therefore, estopped from questioning the decree of registration ordered
issued therein. On January 28, 1969, petitioner moved for reconsideration, which was denied by the trial
Court in its Order dated May 20, 1969, for lack of merit.
Petitioner seasonably filed a Notice of Appeal and a Record on Appeal, which was objected to by private
respondent. On July 15, 1972, or three years later, * the trial Court (Judge Abraham P. Vera, presiding)
refused to give due course to the appeal. Petitioner filed a Motion for Reconsideration but the trial Court
denied it in its Order of October 14, 1972 on the ground that the proper remedy of petitioner was a
Certiorari petition, not an ordinary appeal and that the Order sought to be appealed from had long
become final and executory as petitioner's Motion for Reconsideration was pro-forma and did not
suspend the running of the reglementary period of appeal.
On November 9, 1972, petitioner filed a Petition for Certiorari and Mandamus with the Court of Appeals
claiming that the trial Court gravely abused its discretion, amounting to lack of jurisdiction when, without
the benefit of hearing, it summarily dismissed the Petition for Review; and since said Petition raised
certain issues of fact which cannot be decided except in a trial on the merits, the dismissal of the Petition
on the basis of private respondent's Opposition, considered as a Motion to Dismiss, constituted a denial
of due process of law. Petitioner then prayed that the Order of the trial Court, dated December 20, 1968
dismissing the Petition for Review, be declared null and void, and that said trial Court be directed to give
due course to the Petition for Review; or, in the alternative, to give due course to petitioner's appeal.
On September 30, 1974, the Court of Appeals upheld the trial Court's dismissal of the Petition for Review
stating:
". . . We cannot find any allegation in the petition for review which shows
that private respondent had committed fraud against petitioner. Its
representations and officials were duly notified of private respondent's
petition for reopening and registration of title in her name. In said petition,
the technical descriptions of the portion of Lot No. 626 of the Mariveles
(Bataan) Cadastre, subject-matter of the petition were expressly stated,
the boundaries, specifically delineated. The alleged ground that the land
forms part of a forest land exists at the time petitioner was duly notified of
said petition. Failure to file opposition is in effect, an admission that the
petition is actually not part of a forest land. Indubitably, therefore, no
justifiable reason exists for the annulment of the Order, dated December
20, 1968 (Annex D-Petition) of the lower court dismissing herein
petitioner's petition for review of the decree issued in favor of private
respondent Lastimado." 1
The Court of Appeals then disposed as follows:
"WHEREFORE, finding that the respondent Judge has not committed any
grave abuse of discretion amounting to lack of jurisdiction in the issuance
of an Order, dated December 20, 1968 (Annex D-Petition) dismissing
herein petitioner's petition for review, the present petition for review is
hereby denied.
46
The issuance of the writ of mandamus as prayed for in the petition is no
longer necessary as this Court, in the exercise of its appellate jurisdiction
and authority to supervise orderly administration of justice, has already
resolved on the merits the question whether or not the dismissal of the
petition for review had been done with grave abuse of discretion
amounting to lack of jurisdiction." 2
From this Decision, petitioner filed the present Petition for Review (Appeal) by Certiorari assigning the
following errors to the Court of Appeals and to the trial Court:

1.The Lower Court as well us the Court of Appeals erred in finding that
there can he possession, even for the purpose of claiming title, of land
which at the time of possession is subject to a military reservation.
2.The Lower Court as well as the Court of Appeals erred in finding that
such land which is subject to a government reservation, may appropriately
be the subject of cadastral proceedings, and hence, also of a petition to
reopen cadastral proceedings.
3.The Lower Court as well as the Court of Appeals erred in finding that a
parcel of land which is part of the public forest is susceptible of occupation
and registration in favor of private individual.
4.The Lower Court as well as the Court of Appeals erred in not finding that
the Republic of the Philippines is not estopped from questioning the decree
of registration and the title issued pursuant thereto in favor of respondent
Lastimado over the parcel of land in question.
5.The Lower Court erred in dismissing the petition for review of the
Republic of the Philippines.
6.The Court of Appeals erred in denying Petitioner's petition for certiorari
and mandamus.
Section 38 of the Land Registration Act (Act 496) provides:
"Section 38. Decree of registration, and remedies after entry of decree.
If the court after hearing finds that the applicant or adverse claimant has
title as stated in his application or adverse claim and proper for
registration, a decree of confirmation and registration shall be entered.
Every decree of registration shall bind the land, and quiet title thereto,
subject only to the exceptions stated in the following section. It shall be
conclusive upon and against all persons, including the Insular Government
and all the branches thereof, whether mentioned by name in the
application, notice of citation, or included in the general description 'To all
whom it may concern'. Such decree shall not be opened by reason of the
absence, infancy, or other disability of any person affected thereby, nor by
any proceeding in any court for reversing judgments or decrees; subject,
however, to the right of any person deprived of land or of any estate or
interest therein by decree of registration obtained by fraud to file in the
competent Court of First Instance a petition for review within one year
after entry of the decree provided no innocent purchaser for value has
acquired an interest. . . . ." 3
The essential elements for the allowance of the reopening or review of a decree are: a) that the
petitioner has a real and dominical right; b) that he has been deprived thereof; c) through fraud; d) that
the petition is filed within one year from the issuance of the decree; and e) that the property has not as
yet been transferred to an innocent purchaser. 4
However, for fraud to justify the review of a decree, it must be extrinsic or collateral and the facts upon
which it is based have not been controverted or resolved in the case where the judgment sought to be
annulled was rendered. 5 The following ruling spells out the difference between extrinsic and intrinsic
fraud: LLpr
"Extrinsic or collateral fraud, as distinguished from intrinsic fraud connotes
any fraudulent scheme executed by a prevailing litigant "outside the trial of
a case against the defeated party, or his agents, attorneys or witnesses,
whereby said defeated party is prevented from presenting fully and fairly
his side of the case." But intrinsic fraud takes the form of "acts of a party
in a litigation during the trial, such as the use of forged instruments or
perjured testimony, which did not affect the present action of the case, but
did prevent a fair and just determination of the case." 6
The fraud is one that affects and goes into the jurisdiction of the Court. 7
In its Petition for Review filed before the trial Court, petitioner alleged that fraud was committed by
private respondent when she misrepresented that she and her predecessors-in-interest had been in
possession of the land publicly, peacefully, exclusively and adversely against the whole world as owner
for more than forty years when, in fact, the subject land was inside the former U.S. Military Reservation,
which was formally turned over to the Republic of the Philippines only on December 22, 1965, and that
she likewise contended that her rights, as derived from the original and primitive occupants of the land in
question, are capable of judicial confirmation under existing laws, when the truth is, said parcel of land is
within the public forest of Mariveles, Bataan, and is not subject to disposition or acquisition by private
persons under the Public Land Law.
The trial Court ruled, and was upheld by the Court of Appeals, that no fraud was committed by private
respondent, which deprived petitioner of its day in Court as there was no showing that she was aware of
the facts alleged by the Government, so that she could not have suppressed them with intent to deceive.
The trial Court also noted that petitioner had failed to file an opposition to the reopening of the cadastral
proceedings despite notices sent not only to the Solicitor General as required by Republic Act No. 931,
but to the Bureau of Lands and the Bureau of Forestry as well. It then concluded that "the remedy
granted by section 38 of the Land Registration Act is designed to give relief to victims of fraud, not to
those who are victims of their own neglect, inaction or carelessness, especially when no attempt is ever
made to excuse or justify the neglect." With the foregoing as the essential basis, the trial Court dismissed
the Petition for Review.
We find reversible error. Although there was an agreement by the parties to submit for resolution the
Opposition to the Petition for Review, which was treated as a motion to dismiss, the trial Court, in the
exercise of sound judicial discretion, should not have dismissed the Petition outright but should have
afforded petitioner an opportunity to present evidence in support of the facts alleged to constitute actual
and extrinsic fraud committed by private respondent. Thus, in the case of Republic vs. Sioson, et al., 8 it
was held that "the action of the lower Court in denying the petition for review of a decree of registration
filed within one year from entry of the decree, without hearing the evidence in support of the allegation
and claim that actual and extrinsic fraud upon which the petition is predicated, is held to be in error,
because the lower Court should have afforded the petitioner an opportunity to prove it."
If the allegation of petitioner that the land in question was inside the military reservation at the time it
was claimed is true, then, it cannot be the object of any cadastral proceeding nor can it be the object of
reopening under Republic Act No. 931. 9 Similarly, if the land in question, indeed, forms part of the
public forest, then, possession thereof, however long, cannot convert it into private property as it is
within the exclusive jurisdiction of the Bureau of Forestry and beyond the power and jurisdiction of the
Cadastral Court to register under the Torrens System. 10
Even assuming that the government agencies can be faulted for inaction and neglect (although the
Solicitor General claims that it received no notice), yet, the same cannot operate to bar action by the
State as it cannot be estopped by the mistake or error of its officials or agents. 11 Further, we cannot
47
lose sight of the cardinal consideration that "the State as persona in law is the juridical entity, which is
the source of any asserted right to ownership in land" under basic Constitutional precepts, and that it is
moreover charged with the conservation of such patrimony. 12
WHEREFORE, the Decision of the Court of Appeals dated September 30, 1974, dismissing the Petition for
Certiorari and Mandamus filed before it, as well as the Order of the Court of First Instance of Bataan
(Branch I) dated December 20, 1968, dismissing the Petition for Review, are hereby set aside and the
records of this case hereby remanded to the latter Court for further proceedings to enable petitioner to
present evidence in support of its Petition for Review. LLjur
No pronouncement as to costs.
SO ORDERED.















G.R. No. 92013. July 25, 1990.
SALVADOR H. LAUREL, petitioner, vs. RAMON GARCIA, as head of the Asset Privatization
Trust, RAUL MANGLAPUS, as Secretary of Foreign Affairs, and CATALINO MACARAIG, as
Executive Secretary, respondents.
G.R. No. 92047. July 25, 1990.
DIONISIO S. OJEDA, petitioner, vs. EXECUTIVE SECRETARY MACARAIG, JR., ASSETS
PRIVATIZATION TRUST CHAIRMAN RAMON T. GARCIA, AMBASSADOR RAMON DEL
ROSARIO, et al., as members of the PRINCIPAL AND BIDDING COMMITTEES ON THE
UTILIZATION/DISPOSITION OF PHILIPPINE GOVERNMENT PROPERTIES IN JAPAN,
respondents.
Arturo M. Tolentino for petitioner in 92013.
D E C I S I O N
GUTIERREZ, JR., J p:
These are two petitions for prohibition seeking to enjoin respondents, their representatives and agents
from proceeding with the bidding for the sale of the 3,179 square meters of land at 306 Ropponggi, 5-
Chome Minato-ku, Tokyo, Japan scheduled on February 21, 1990. We granted the prayer for a temporary
restraining order effective February 20, 1990. One of the petitioners (in G.R. No. 92047) likewise prayer
for a writ of mandamus to compel the respondents to fully disclose to the public the basis of their
decision to push through with the sale of the Roppongi property inspite of strong public opposition and to
explain the proceedings which effectively prevent the participation of Filipino citizens and entities in the
bidding process.
The oral arguments in G.R. No. 92013, Laurel v. Garcia, et al. were heard by the Court on March 13,
1990. After G.R. No. 92047, Ojeda v. Secretary Macaraig, et al. was filed, the respondents were required
to file a comment by the Court's resolution dated February 22, 1990. The two petitions were consolidated
on March 27, 1990 when the memoranda of the parties in the Laurel case were deliberated upon.
The Court could not act on these cases immediately because the respondents filed a motion for an
extension of thirty (30) days to file comment in G.R. No. 92047, followed by a second motion for an
extension of another thirty (30) days which we granted on May 8, 1990, a third motion for extension of
time granted on May 24, 1990 and a fourth motion for extension of time which we granted on June 5,
1990 but calling the attention of the respondents to the length of time the petitions have been pending.
After the comment was filed, the petitioner in G.R. No. 92047 asked for thirty (30) days to file a reply.
We noted his motion and resolved to decide the two (2) cases. LexLib
I
The subject property in this case is one of the four (4) properties in Japan acquired by the Philippine
government under the Reparations Agreement entered into with Japan on May 9, 1956, the other lots
being:
(1)The Nampeidai Property at 11-24 Nampeidai-machi, Shibuya-ku, Tokyo
which has an area of approximately 2,489.96 square meters,
and is at present the site of the Philippine Embassy
Chancery;
(2)The Kobe Commercial Property at 63 Naniwa-cho, Kobe, with an area of
around 764.72 square meters and categorized as a
commercial lot now being used as a warehouse and parking
lot for the consulate staff; and
(3)The Kobe Residential Property at 1-980-2 Obanoyamacho, Shinohara,
Nada-ku, Kobe, a residential lot which is now vacant.
The properties and the capital goods and services procured from the Japanese government for national
development projects are part of the indemnification to the Filipino people for their losses in life and
property and their suffering during World War II.
The Reparations Agreement provides that reparations valued at $550 million would be payable in twenty
(20) years in accordance with annual schedules of procurements to be fixed by the Philippine and
Japanese governments (Article 2, Reparations Agreement). Rep. Act. No. 1789, the Reparations Law,
prescribes the national policy on procurement and utilization of reparations and development loans. The
procurements are divided into those for use by the government sector and those for private parties in
48
projects as the then National Economic Council shall determine. Those intended for the private sector
shall be made available by sale to Filipino citizens or to one hundred (100%) percent Filipino-owned
entities in national development projects.
The Roppongi property was acquired from the Japanese government under the Second Year Schedule
and listed under the heading "Government Sector", through Reparations Contract No. 300 dated June 27,
1958. The Roponggi property consists of the land and building "for the Chancery of the Philippine
Embassy" (Annex M-D to Memorandum for Petitioner, p. 503). As intended, it became the site of the
Philippine Embassy until the latter was transferred to Nampeidai on July 22, 1976 when the Roppongi
building needed major repairs. Due to the failure of our government to provide necessary funds, the
Roppongi property has remained undeveloped since that time.
A proposal was presented to President Corazon C. Aquino by former Philippine Ambassador to Japan,
Carlos J. Valdez, to make the property the subject of a lease agreement with a Japanese firm Kajima
Corporation which shall construct two (2) buildings in Roppongi and one (1) building in Nampeidai and
renovate the present Philippine Chancery in Nampeidai. The consideration of the construction would be
the lease to the foreign corporation of one (1) of the buildings to be constructed in Roppongi and the two
(2) buildings in Nampeidai. The other building in Roppongi shall then be used as the Philippine Embassy
Chancery. At the end of the lease period, all the three leased buildings shall be occupied and used by the
Philippine government. No change of ownership or title shall occur. (See Annex "B" to Reply to Comment)
The Philippine government retains the title all throughout the lease period and thereafter. However, the
government has not acted favorably on this proposal which is pending approval and ratification between
the parties. Indeed, on August 11, 1986, President Aquino created a committee to study the
disposition/utilization of Philippine government properties in Tokyo and Kobe, Japan through
Administrative Order No. 3, followed by Administrative Orders Numbered 3-A, B, C and D.
On July 25, 1987, the President issued Executive Order No. 296 entitling non-Filipino citizens or entities
to avail of reparations' capital goods and services in the event of sale, lease or disposition. The four
properties in Japan including the Roppongi were specifically mentioned in the first "Whereas" clause.
Amidst opposition by various sectors, the Executive branch of the government has been pushing, with
great vigor, its decision to sell the reparations properties starting with the Roppongi lot. The property has
twice been set for bidding at a minimum floor price at $225 million. The first bidding was a failure since
only one bidder qualified. The second one, after postponements, has not yet materialized. The last
scheduled bidding on February 21, 1990 was restrained by his Court. Later, the rules on bidding were
changed such that the $225 million floor price became merely a suggested floor price. cdrep
The Court finds that each of the herein petitions raises distinct issues. The petitioner in G.R. No. 92013
objects to the alienation of the Roppongi property to anyone while the petitioner in G.R. No. 92047 adds
as a principal objection the alleged unjustified bias of the Philippine government in favor of selling the
property to non-Filipino citizens and entities. These petitions have been consolidated and are resolved at
the same time for the objective is the same to stop the sale of the Roppongi property.
The petitioner in G.R. No. 92013 raises the following issues:
(1)Can the Roppongi property and others of its kind be alienated by the
Philippine Government?; and
(2)Does the Chief Executive, her officers and agents, have the authority
and jurisdiction, to sell the Roppongi property?
Petitioner Dionisio Ojeda in G.R. NO. 92047, apart from questioning the authority of the government to
alienate the Roppongi property assails the constitutionality of Executive Order No. 296 in making the
property available for the sale to non-Filipino citizens and entities. He also questions the bidding
procedures of the Committee on the Utilization or Disposition of Philippine Government Properties in
Japan for being discriminatory against Filipino citizens and Filipino-owned entities by denying them the
right to be informed about the bidding requirements.
II
In G.R. No. 92013, petitioner Laurel asserts that the Roppongi property and the related lots were
acquired as part of the reparations from the Japanese government for diplomatic and consular use by the
Philippine government. Vice-President Laurel states that the Roppongi property is classified as one of
public dominion, and not of private ownership under Article 420 of the Civil Code (See infra).
The petitioner submits that the Roppongi property comes under "property intended for public service" in
paragraph 2 of the above provision. He states that being one of public dominion, no ownership by any
one can attach to it, not even by the State. The Roppongi and related properties were acquired for "sites
for chancery, diplomatic, and consular quarters, buildings and other improvements" (Second Year
Reparations Schedule). The petitioner states that they continue to be intended for a necessary service.
They are held by the State in anticipation of an opportune use. (Citing 3 Manresa 65-66). Hence, it
cannot be appropriated, is outside the commerce of man, or to put it in more simple terms, it cannot be
alienated nor be the subject matter of contracts (Citing Municipality of Cavite v. Rojas, 30 Phil. 20
[1915]). Noting the non-use of the Roppongi property at the moment, the petitioner avers that the same
remains property of public dominion so long as the government has not used it for other purposes nor
adopted any measure constituting a removal of its original purpose or use.
The respondents, for their part, refute the petitioner's contention by saying that the subject property is
not governed by our Civil Code but by the laws of Japan where the property is located. They rely upon
the rule of lex situs which is used in determining the applicable law regarding the acquisition, transfer
and devolution of the title to a property. They also invoke Opinion No. 21, Series of 1988, dated January
27, 1988 of the Secretary of Justice which used the lex situs in explaining the inapplicability of Philippine
law regarding a property situated in Japan.

The respondents add that even assuming for the sake of argument that the Civil Code is applicable, the
Roppongi property has ceased to become property of public dominion. It has become patrimonial
property because it has not been used for public service or for diplomatic purposes for over thirteen (13)
years now (Citing Article 422, Civil Code) and because the intention by the Executive Department and the
Congress to convert it to private use has been manifested by overt acts, such as, among others; (1) the
transfer of the Philippine Embassy to Nampeidai; (2) the issuance of administrative orders for the
possibility of alienating the four government properties in Japan; (3) the issuance of Executive Order No.
296; (4) the enactment by the Congress of Rep. Act No. 6657 [the Comprehensive Agrarian Reform Law]
on June 10, 1988 which contains a provision stating that funds may be taken from the sale of Philippine
properties in foreign countries; (5) the holding of the public bidding of the Roppongi property but which
failed; (6) the deferment by the Senate in Resolution No. 55 of the bidding to a future date; thus an
acknowledgment by the Senate of the government's intention to remove the Roppongi property from the
public service purpose; and (7) the resolution of this Court dismissing the petition in Ojeda v. Bidding
Committee, et al., G.R. No. 87478 which sought to enjoin the second bidding of the Roppongi property
scheduled on March 30, 1989.
III
In G.R. No. 94047, petitioner Ojeda once more asks this Court to rule on the constitutionality of
Executive Order No. 296. He had earlier filed a petition in G.R. No. 87478 which the Court dismissed on
August 1, 1989. He now avers that the executive order contravenes the constitutional mandate to
conserve and develop the national patrimony stated in the Preamble of the 1987 Constitution. It also
allegedly violates:
(1)The reservation of the ownership and acquisition of alienable lands of
the public domain to Filipino citizens. (Sections 2 and 3,
Article XII, Constitution; Section 22 and 23 of Commonwealth
Act 141).
(2)The preference for Filipino citizens in the grant of rights, privileges and
concessions covering the national economy and patrimony
(Section 10, Article VI, Constitution);
(3)The protection given to Filipino enterprises against unfair competition
49
and trade practices;
(4)The guarantee of the right of the people to information on all matters of
public concern (Section 7, Article III, Constitution);
(5)The prohibition against the sale to non-Filipino citizens or entities not
wholly owned by Filipino citizens of capital goods received by
the Philippines under the Reparations Act (Sections 2 and 12
of Rep. Act No. 1789); and
(6)The declaration of the state policy of full public disclosure of all
transactions involving public interest (Sections 28, Article II,
Constitution).
Petitioner Ojeda warns that the use of public funds in the execution of an unconstitutional executive
order is a misapplication of public funds. He states that since the details of the bidding for the Roppongi
property were never publicly disclosed until February 15, 1990 (or a few days before the scheduled
bidding), the bidding guidelines are available only in Tokyo, and the accomplishment of requirements and
the selection of qualified bidders should be done in Tokyo, interested Filipino citizens or entities owned by
them did not have the chance to comply with Purchase Offer Requirements on the Roppongi. Worse, the
Roppongi shall be sold for a minimum price of $225 million from which price capital gains tax under
Japanese law of about 50 to 70% of the floor price would still be deducted. cdll
IV
The petitioners and respondents in both cases do not dispute the fact that the Roppongi site and the
three related properties were acquired through reparations agreements, that these were assigned to the
government sector and that the Roppongi property itself was specifically designated under the
Reparations Agreement to house the Philippine Embassy.
The nature of the Roppongi lot as property for public service is expressly spelled out. It is dictated by the
terms of the Reparations Agreement and the corresponding contract of procurement which bind both the
Philippine government and the Japanese government.
There can be no doubt that it is of public dominion unless it is convincingly shown that the property has
become patrimonial. This, the respondents have failed to do.
As property of public dominion, the Roppongi lot is outside the commerce of man. It cannot be alienated.
Its ownership is a special collective ownership for general use and enjoyment, an application to the
satisfaction of collective needs, and resides in the social group. The purpose is not to serve the State as a
juridical person, but the citizens; it is intended for the common and public welfare and cannot be the
object of appropriation. (Taken from 3 Manresa, 66-69; cited in Tolentino, Commentaries on the Civil
Code of the Philippines, 1963 Edition, Vol. II, p. 26).
The applicable provisions of the Civil Code are:
"ART. 419.Property is either of public dominion or of private ownership.
"ART. 420.The following things are property of public dominion:
"(1)Those intended for public use, such as roads, canals, rivers, torrents,
ports and bridges constructed by the State, banks, shores, roadsteads, and
others of similar character;
(2)Those which belong to the State, without being for public use, and are
intended for some public service or for the development of the national
wealth.
"ART. 421.All other property of the State, which is not of the character
stated in the preceding article, is patrimonial property."
The Roppongi property is correctly classified under paragraph 2 of Article 420 of the Civil Code as
property belonging to the State and intended for some public service.
Has the intention of the government regarding the use of the property been changed because the lot has
been idle for some years? Has it become patrimonial?
The fact that the Roppongi site has not been used for a long time for actual Embassy service does not
automatically convert it to patrimonial property. Any such conversion happens only if the property is
withdrawn from public use (Cebu Oxygen and Acetylene Co. v. Bercilles, 66 SCRA 481 [1975]). A
property continues to be part of the public domain, not available for private appropriation or ownership
"until there is a formal declaration on the part of the government to withdraw it from being such (Ignacio
v. Director of Lands, 108 Phil. 335 [1960]).
The respondents enumerate various pronouncements by concerned public officials insinuating a change
of intention. We emphasize, however, that an abandonment of the intention to use the Roppongi
property for public service and to make it patrimonial property under Article 422 of the Civil Code must
be definite. Abandonment cannot be inferred from the non-use alone specially if the non-use was
attributable not to the government's own deliberate and indubitable will but to a lack of financial support
to repair and improve the property (See Heirs of Felino Santiago v. Lazarao, 166 SCRA 368 [1988]).
Abandonment must be a certain and positive act based on correct legal premises. LexLib
A mere transfer of the Philippine Embassy to Nampeidai in 1976 is not relinquishment of the Roppongi
property's original purpose. Even the failure by the government to repair the building in Roppongi is not
abandonment since as earlier stated, there simply was a shortage of government funds. The recent
Administrative Orders authorizing a study of the status and conditions of government properties in Japan
were merely directives for investigation but did not in any way signify a clear intention to dispose of the
properties.
Executive Order No. 296, though its title declares an "authority to sell", does not have a provision in this
text expressly authorizing the sale of the four properties procured from Japan for the government sector.
The executive order does not declare that the properties lost their public character. It merely intends to
make the properties available to foreigners and not to Filipinos alone in case of a sale, lease or other
disposition. It merely eliminates the restriction under Rep. Act. 1789 that reparations goods may be sold
only to Filipino citizens and one hundred (100%) percent Filipino-owned entities. The text of Executive
Order No. 296 provides:
"Section 1.The provisions of Republic Act No. 1789, as amended, and of
other laws to the contrary notwithstanding, the abovementioned properties
can be made available for sale, lease or any other manner of disposition to
non-Filipino citizens or to entities owned by non-Filipino citizens."
Executive Order No. 296 is based on the wrong premise or assumption that the Roppongi and the three
other properties were earlier converted into alienable real properties. As earlier stated, Rep. Act No. 1789
differentiates the procurements for the government sector and the private sector (Sections 2 and 12,
Rep. Act No. 1789). Only the private sector properties can be sold to end-users who must be Filipinos or
entities owned by Filipinos. It is this nationality provision which was amended by Executive Order No.
296.
Section 63 (c) of Rep. Act No. 6657 (the CARP Law) which provides as one of the sources of funds for its
implementation, the proceeds of the disposition of the properties of the Government in foreign countries,
did not withdraw the Roppongi property from being classified as one of public dominion when it mentions
Philippine properties abroad. Section 63 (c) refers to properties which are alienable and not to those
reserved for public use or service. Rep Act No. 6657, therefore, does not authorize the Executive
Department to sell the Roppongi property. It merely enumerates possible sources of future funding to
augment (as and when needed) the Agrarian Reform Fund created under Executive Order No. 299.
Obviously any property outside of the commerce of man cannot be tapped as a source of funds.

50
The respondents try to get around the public dominion character of the Roppongi property by insisting
that Japanese law and not our Civil Code should apply.
It is exceedingly strange why our top government officials, of all people, should be the ones to insist that
in the sale of extremely valuable government property, Japanese law and not Philippine law should
prevail. The Japanese law its coverage and effects, when enacted, and exceptions to its provisions
is not presented to the Court. It is simply asserted that the lex loci rei sitae or Japanese law should apply
without stating what that law provides. It is assumed on faith that Japanese law would allow the sale.
We see no reason why a conflict of law rule should apply when no conflict of law situation exists. A
conflict of law situation arises only when: (1) There is a dispute over the title or ownership of an
immovable, such that the capacity to take and transfer immovables, the formalities of conveyance, the
essential validity and effect of the transfer, or the interpretation and effect of a conveyance, are to be
determined (See Salonga, Private International Law, 1981 ed., pp. 377-383); and (2) A foreign law on
land ownership and its conveyance is asserted to conflict with a domestic law on the same matters.
Hence, the need to determine which law should apply.
In the instant case, none of the above elements exists.
The issues are not concerned with validity of ownership or title. There is no question that the property
belongs to the Philippines. The issue is the authority of the respondent officials to validly dispose of
property belonging to the State. And the validity of the procedures adopted to effect its sale. This is
governed by Philippine Law. The rule of lex situs does not apply.
The assertion that the opinion of the Secretary of Justice sheds light on the relevance of the lex situs rule
is misplaced. The opinion does not tackle the alienability of the real properties procured through
reparations nor the existence in what body of the authority to sell them. In discussing who are capable of
acquiring the lots, the Secretary merely explains that it is the foreign law which should determine who
can acquire the properties so that the constitutional limitation on acquisition of lands of the public domain
to Filipino citizens and entities wholly owned by Filipinos is inapplicable. We see no point in belaboring
whether or not this opinion is correct. Why should we discuss who can acquire the Roppongi lot when
there is no showing that it can be sold?
The subsequent approval on October 4, 1988 by President Aquino of the recommendation by the
investigating committee to sell the Roppongi property was premature or, at the very least, conditioned on
a valid change in the public character of the Roppongi property. Moreover, the approval does not have
the force and effect of law since the President already lost her legislative powers. The Congress had
already convened for more than a year.
Assuming for the sale of argument, however, that the Roppongi property is no longer of public dominion,
there is another obstacle to its sale by the respondents.
There is no law authorizing its conveyance.
Section 79 (f) of the Revised Administrative Code of 1917 provides:
"Section 79 (f).Conveyances and contracts to which the Government is a
party. In cases in which the Government of the Republic of the
Philippines is a party to any deed or other instrument conveying the title to
real estate or to any other property the value of which is in excess of one
hundred thousand pesos, the respective Department Secretary shall
prepare the necessary papers which, together with the proper
recommendations, shall be submitted to the Congress of the Philippines for
approval by the same. Such deed, instrument, or contract shall be
executed and signed by the President of the Philippines on behalf of the
Government of the Philippines unless the Government of the Philippines
unless the authority therefor be expressly vested by law in another
officer." (Emphasis supplied)
The requirement has been retained in Section 48, Book I of the Administrative Code of 1987 (Executive
Order No. 292).
"SEC. 48.Official Authorized to Convey Real Property. Whenever real
property of the Government is authorized by law to be conveyed, the deed
of conveyance shall be executed in behalf of the government by the
following:
"(1)For property belonging to and titled in the name of the Republic of the
Philippines, by the President, unless the authority therefor is expressly
vested by law in another officer.
"(2)For property belonging to the Republic of the Philippines but titled in
the name of any political subdivision or of any corporate agency or
instrumentality, by the executive head of the agency or instrumentality."
(Emphasis supplied).
It is not for the President to convey valuable real property of the government on his or her own sole will.
Any such conveyance must be authorized and approved by a law enacted by the Congress. It requires
executive and legislative concurrence.
Resolution No. 55 of the Senate dated June 8, 1989, asking for the deferment of the sale of the Roppongi
property does not withdraw the property from public domain much less authorize its sale. It is a mere
resolution; it is not a formal declaration abandoning the public character of the Roppongi property. In
fact, the Senate Committee on Foreign Relations is conducting hearings on Senate Resolution No. 734
which raises serious policy considerations and calls for a fact-finding investigation of the circumstances
behind the decision to sell the Philippine government properties in Japan. LexLib
The resolution of this Court in Ojeda v. Bidding Committee, et al., supra, did not pass upon the
constitutionality of Executive Order No. 296. Contrary to respondents' assertion, we did not uphold the
authority of the President to sell the Roppongi property. The Court stated that the constitutionality of the
executive order was not the real issue and that resolving the constitutional question was "neither
necessary nor finally determinative of the case." The Court noted that "[W]hat petitioner ultimately
questions is the use of the proceeds of the disposition of the Roppongi property." In emphasizing that
"the decision of the Executive to dispose of the Roppongi property to finance the CARP . . . cannot be
questioned" in view of Section 63 (c) of Rep. Act. No. 6657, the Court did not acknowledge the fact that
the property became alienable nor did it indicate that the President was authorized to dispose of the
Roppongi property. The resolution should be read to mean that in case the Roppongi property is re-
classified to be patrimonial and alienable by authority of law, the proceeds of a sale may be used for
national economic development projects including the CARP.
Moreover, the sale in 1989 did not materialize. The petitions before us question the proposed 1990 sale
of the Roppongi property. We are resolving the issues raised in these petitions, not the issues raised in
1989.
Having declared a need for a law or formal declaration to withdraw the Roppongi property from public
domain to make it alienable and a need for legislative authority to allow the sale of the property, we see
no compelling reason to tackle the constitutional issue raised by petitioner Ojeda.
The Court does not ordinarily pass upon constitutional questions unless these questions are properly
raised in appropriate cases and their resolution is necessary for the determination of the case (People v.
Vera, 65 Phil. 56 [1937]). The Court will not pass upon a constitutional question although property
presented by the record if the case can be disposed of on some other ground such as the application of a
statute or general law (Siler v. Louisville and Nashville R. Co., 213 U.S. 175, [1909], Railroad Commission
v. Pullman Co., 312 U.S. 496 [1941]).
The petitioner in G.R. No. 92013 states why the Roppongi property should not be sold:
The Roppongi property is not just like any piece of property. It was given
51
to the Filipino people in reparation for the lives and blood of Filipinos who
died and suffered during the Japanese military occupation, for the
suffering of widows and orphans who lost their loved ones and kindred, for
the homes and other properties lost by countless Filipinos during the war.
The Tokyo properties are a monument to the bravery and sacrifice of the
Filipino people in the face of an invader; like the monuments of Rizal,
Quezon, and other Filipino heroes, we do not expect economic or financial
benefits from them. But who would think of selling these monuments?
Filipino honor and national dignity dictate that we keep our properties in
Japan as memorials to the countless Filipinos who died and suffered. Even
if we should become paupers we should not think of selling them. For it
would be as if we sold the lives and blood and tears of our countrymen."
(Rollo-G.R. No. 92013, p. 147).
The petitioner in G.R. No. 92047 also states:
"Roppongi is no ordinary property. It is one ceded by the Japanese
government in atonement for its past belligerence, for the valiant sacrifice
of life and limb and for deaths, physical dislocation and economic
devastation the whole Filipino people endured in World War II.
"It is for what it stands for, and for what it could never bring back to life,
that its significance today remains undimmed, inspite of the lapse of 45
years since the war ended, inspite of the passage of 32 years since the
property passed on to the Philippine government.
"Roppongi is a reminder that cannot should not be dissipated. . . ."
(Rollo-92047, p. 9)
It is indeed true that the Roppongi property is valuable not so much because of the inflated prices
fetched by real property in Tokyo but more so because of its symbolic value to all Filipinos veterans
and civilians alike. Whether or not the Roppongi and related properties will eventually be sold is a policy
determination where both the President and congress must concur. Considering the properties'
importance and value, the laws on conversion and disposition of property of public dominion must be
faithfully followed.

WHEREFORE, IN VIEW OF THE FOREGOING, the petitions are GRANTED. A writ of prohibition is issued
enjoining the respondents from proceeding with the sale of the Roppongi property in Tokyo, Japan. The
February 20, 1990 Temporary Restraining Order is made PERMANENT.
SO ORDERED.



















iv. Mineral Resources
[G.R. No. 127882. December 1, 2004.]
LA BUGAL-B'LAAN TRIBAL ASSOCIATION, INC., Represented by
its Chairman F'LONG MIGUEL M. LUMAYONG; WIGBERTO E.
TAADA; PONCIANO BENNAGEN; JAIME TADEO; RENATO R.
CONSTANTINO JR.; F'LONG AGUSTIN M. DABIE; ROBERTO P.
AMLOY; RAQIM L. DABIE; SIMEON H. DOLOJO; IMELDA M.
GANDON; LENY B. GUSANAN; MARCELO L. GUSANAN; QUINTOL A.
LABUAYAN; LOMINGGES D. LAWAY; BENITA P. TACUAYAN;
Minors JOLY L. BUGOY, Represented by His Father UNDERO D.
BUGOY and ROGER M. DADING; Represented by His Father
ANTONIO L. DADING; ROMY M. LAGARO, Represented by His
Father TOTING A. LAGARO; MIKENY JONG B. LUMAYONG,
Represented by His Father MIGUEL M. LUMAYONG; RENE T.
MIGUEL, Represented by His Mother EDITHA T. MIGUEL;
ALDEMAR L. SAL, Represented by His Father DANNY M. SAL;
DAISY RECARSE, Represented by Her Mother LYDIA S. SANTOS;
EDWARD M. EMUY; ALAN P. MAMPARAIR; MARIO L. MANGCAL;
ALDEN S. TUSAN; AMPARO S. YAP; VIRGILIO CULAR; MARVIC
M.V.F. LEONEN; JULIA REGINA CULAR, GIAN CARLO CULAR,
VIRGILIO CULAR JR., Represented by Their Father VIRGILIO
CULAR; PAUL ANTONIO P. VILLAMOR, Represented by His
Parents JOSE VILLAMOR and ELIZABETH PUA-VILLAMOR; ANA
GININA R. TALJA, Represented by Her Father MARIO JOSE B.
TALJA; SHARMAINE R. CUNANAN, Represented by Her Father
ALFREDO M. CUNANAN; ANTONIO JOSE A. VITUG III,
Represented by His Mother ANNALIZA A. VITUG, LEAN D.
NARVADEZ, Represented by His Father MANUEL E. NARVADEZ JR.;
ROSERIO MARALAG LINGATING, Represented by Her Father RIO
OLIMPIO A. LINGATING; MARIO JOSE B. TALJA; DAVID E. DE
VERA; MARIA MILAGROS L. SAN JOSE; Sr. SUSAN O. BOLANIO,
OND; LOLITA G. DEMONTEVERDE; BENJIE L. NEQUINTO; 1 ROSE
LILIA S. ROMANO; ROBERTO S. VERZOLA; EDUARDO AURELIO C. REYES;
LEAN LOUEL A. PERIA, Represented by His Father ELPIDIO V. PERIA; 2
GREEN FORUM PHILIPPINES; GREEN FORUM WESTERN VISAYAS (GF-
52
WV); ENVIRONMENTAL LEGAL ASSISTANCE CENTER (ELAC); KAISAHAN
TUNGO SA KAUNLARAN NG KANAYUNAN AT REPORMANG PANSAKAHAN
(KAISAHAN); 3 PARTNERSHIP FOR AGRARIAN REFORM and RURAL
DEVELOPMENT SERVICES, INC. (PARRDS); PHILIPPINE PARTNERSHIP
FOR THE DEVELOPMENT OF HUMAN RESOURCES IN THE RURAL AREAS,
INC. (PHILDHRRA); WOMEN'S LEGAL BUREAU (WLB); CENTER FOR
ALTERNATIVE DEVELOPMENT INITIATIVES, INC. (CADI); UPLAND
DEVELOPMENT INSTITUTE (UDI); KINAIYAHAN FOUNDATION, INC.;
SENTRO NG ALTERNATIBONG LINGAP PANLIGAL (SALIGAN); and LEGAL
RIGHTS AND NATURAL RESOURCES CENTER, INC. (LRC), petitioners, vs.
VICTOR O. RAMOS, Secretary, Department of Environment and
Natural Resources (DENR); HORACIO RAMOS, Director, Mines and
Geosciences Bureau (MGB-DENR); RUBEN TORRES, Executive
Secretary; and WMC (PHILIPPINES), INC., 4 respondents.
R E S O L U T I O N
PANGANIBAN, J p:
All mineral resources are owned by the State. Their exploration, development and utilization (EDU) must
always be subject to the full control and supervision of the State. More specifically, given the inadequacy
of Filipino capital and technology in large-scale EDU activities, the State may secure the help of foreign
companies in all relevant matters especially financial and technical assistance provided that, at all
times, the State maintains its right of full control. The foreign assistor or contractor assumes all financial,
technical and entrepreneurial risks in the EDU activities; hence, it may be given reasonable management,
operational, marketing, audit and other prerogatives to protect its investments and to enable the
business to succeed.
Full control is not anathematic to day-to-day management by the contractor, provided that the State
retains the power to direct overall strategy; and to set aside, reverse or modify plans and actions of the
contractor. The idea of full control is similar to that which is exercised by the board of directors of a
private corporation: the performance of managerial, operational, financial, marketing and other functions
may be delegated to subordinate officers or given to contractual entities, but the board retains full
residual control of the business.
Who or what organ of government actually exercises this power of control on behalf of the State? The
Constitution is crystal clear: the President. Indeed, the Chief Executive is the official constitutionally
mandated to "enter into agreements with foreign owned corporations." On the other hand, Congress may
review the action of the President once it is notified of "every contract entered into in accordance with
this [constitutional] provision within thirty days from its execution." In contrast to this express mandate
of the President and Congress in the EDU of natural resources, Article XII of the Constitution is silent on
the role of the judiciary. However, should the President and/or Congress gravely abuse their discretion in
this regard, the courts may in a proper case exercise their residual duty under Article VIII. Clearly
then, the judiciary should not inordinately interfere in the exercise of this presidential power of control
over the EDU of our natural resources.
The Constitution should be read in broad, life-giving strokes. It should not be used to strangulate
economic growth or to serve narrow, parochial interests. Rather, it should be construed to grant the
President and Congress sufficient discretion and reasonable leeway to enable them to attract foreign
investments and expertise, as well as to secure for our people and our posterity the blessings of
prosperity and peace.
On the basis of this control standard, this Court upholds the constitutionality of the Philippine Mining Law,
its Implementing Rules and Regulations insofar as they relate to financial and technical agreements
as well as the subject Financial and Technical Assistance Agreement (FTAA). 5
Background
The Petition for Prohibition and Mandamus before the Court challenges the constitutionality of (1)
Republic Act No. [RA] 7942 (The Philippine Mining Act of 1995); (2) its Implementing Rules and
Regulations (DENR Administrative Order No. [DAO] 96-40); and (3) the FTAA dated March 30, 1995, 6
executed by the government with Western Mining Corporation (Philippines), Inc. (WMCP). 7
On January 27, 2004, the Court en banc promulgated its Decision 8 granting the Petition and declaring
the unconstitutionality of certain provisions of RA 7942, DAO 96-40, as well as of the entire FTAA
executed between the government and WMCP, mainly on the finding that FTAAs are service contracts
prohibited by the 1987 Constitution.
The Decision struck down the subject FTAA for being similar to service contracts, 9 which, though
permitted under the 1973 Constitution, 10 were subsequently denounced for being antithetical to the
principle of sovereignty over our natural resources, because they allowed foreign control over the
exploitation of our natural resources, to the prejudice of the Filipino nation.
The Decision quoted several legal scholars and authors who had criticized service contracts for, inter alia,
vesting in the foreign contractor exclusive management and control of the enterprise, including operation
of the field in the event petroleum was discovered; control of production, expansion and development;
nearly unfettered control over the disposition and sale of the products discovered/extracted; effective
ownership of the natural resource at the point of extraction; and beneficial ownership of our economic
resources. According to the Decision, the 1987 Constitution (Section 2 of Article XII) effectively banned
such service contracts.
Subsequently, respondents filed separate Motions for Reconsideration. In a Resolution dated March 9,
2004, the Court required petitioners to comment thereon. In the Resolution of June 8, 2004, it set the
case for Oral Argument on June 29, 2004.
After hearing the opposing sides, the Court required the parties to submit their respective Memoranda in
amplification of their arguments. In a Resolution issued later the same day, June 29, 2004, the Court
noted, inter alia, the Manifestation and Motion (in lieu of comment) filed by the Office of the Solicitor
General (OSG) on behalf of public respondents. The OSG said that it was not interposing any objection to
the Motion for Intervention filed by the Chamber of Mines of the Philippines, Inc. (CMP) and was in fact
joining and adopting the latter's Motion for Reconsideration.
Memoranda were accordingly filed by the intervenor as well as by petitioners, public respondents, and
private respondent, dwelling at length on the three issues discussed below. Later, WMCP submitted its
Reply Memorandum, while the OSG in obedience to an Order of this Court filed a Compliance
submitting copies of more FTAAs entered into by the government.
Three Issues Identified by the Court
During the Oral Argument, the Court identified the three issues to be resolved in the present controversy,
as follows:
1.Has the case been rendered moot by the sale of WMC shares in WMCP to Sagittarius (60 percent of
Sagittarius' equity is owned by Filipinos and/or Filipino-owned corporations while 40 percent is owned by
Indophil Resources NL, an Australian company) and by the subsequent transfer and registration of the
FTAA from WMCP to Sagittarius?
2.Assuming that the case has been rendered moot, would it still be proper to resolve the constitutionality
of the assailed provisions of the Mining Law, DAO 96-40 and the WMCP FTAA?
3.What is the proper interpretation of the phrase Agreements Involving Either Technical or Financial
Assistance contained in paragraph 4 of Section 2 of Article XII of the Constitution?
53
Should the Motion for Reconsideration Be Granted?
Respondents' and intervenor's Motions for Reconsideration should be granted, for the reasons discussed
below. The foregoing three issues identified by the Court shall now be taken up seriatim.
First Issue: Mootness
In declaring unconstitutional certain provisions of RA 7942, DAO 96-40, and the WMCP FTAA, the
majority Decision agreed with petitioners' contention that the subject FTAA had been executed in
violation of Section 2 of Article XII of the 1987 Constitution. According to petitioners, the FTAAs entered
into by the government with foreign-owned corporations are limited by the fourth paragraph of the said
provision to agreements involving only technical or financial assistance for large-scale exploration,
development and utilization of minerals, petroleum and other mineral oils. Furthermore, the foreign
contractor is allegedly permitted by the FTAA in question to fully manage and control the mining
operations and, therefore, to acquire "beneficial ownership" of our mineral resources.

The Decision merely shrugged off the Manifestation by WMPC informing the Court (1) that on January
23, 2001, WMC had sold all its shares in WMCP to Sagittarius Mines, Inc., 60 percent of whose equity
was held by Filipinos; and (2) that the assailed FTAA had likewise been transferred from WMCP to
Sagittarius. 11 The ponencia declared that the instant case had not been rendered moot by the transfer
and registration of the FTAA to a Filipino-owned corporation, and that the validity of the said transfer
remained in dispute and awaited final judicial determination. 12 Patently therefore, the Decision is
anchored on the assumption that WMCP had remained a foreign corporation.
The crux of this issue of mootness is the fact that WMCP, at the time it entered into the FTAA, happened
to be wholly owned by WMC Resources International Pty., Ltd. (WMC), which in turn was a wholly owned
subsidiary of Western Mining Corporation Holdings Ltd., a publicly listed major Australian mining and
exploration company.
The nullity of the FTAA was obviously premised upon the contractor being a foreign corporation. Had the
FTAA been originally issued to a Filipino-owned corporation, there would have been no constitutionality
issue to speak of. Upon the other hand, the conveyance of the WMCP FTAA to a Filipino corporation can
be likened to the sale of land to a foreigner who subsequently acquires Filipino citizenship, or who later
resells the same land to a Filipino citizen. The conveyance would be validated, as the property in question
would no longer be owned by a disqualified vendee.
And, inasmuch as the FTAA is to be implemented now by a Filipino corporation, it is no longer possible
for the Court to declare it unconstitutional. The case pending in the Court of Appeals is a dispute
between two Filipino companies (Sagittarius and Lepanto), both claiming the right to purchase the
foreign shares in WMCP. So, regardless of which side eventually wins, the FTAA would still be in the
hands of a qualified Filipino company. Considering that there is no longer any justiciable controversy, the
plea to nullify the Mining Law has become a virtual petition for declaratory relief, over which this Court
has no original jurisdiction. DCcTHa
In their Final Memorandum, however, petitioners argue that the case has not become moot, considering
the invalidity of the alleged sale of the shares in WMCP from WMC to Sagittarius, and of the transfer of
the FTAA from WMCP to Sagittarius, resulting in the change of contractor in the FTAA in question. And
even assuming that the said transfers were valid, there still exists an actual case predicated on the
invalidity of RA 7942 and its Implementing Rules and Regulations (DAO 96-40). Presently, we shall
discuss petitioners' objections to the transfer of both the shares and the FTAA. We shall take up the
alleged invalidity of RA 7942 and DAO 96-40 later on in the discussion of the third issue.
No Transgression of the Constitutionby the Transfer of the WMCP Shares
Petitioners claim, first, that the alleged invalidity of the transfer of the WMCP shares to Sagittarius
violates the fourth paragraph of Section 2 of Article XII of the Constitution; second, that it is contrary to
the provisions of the WMCP FTAA itself; and third, that the sale of the shares is suspect and should
therefore be the subject of a case in which its validity may properly be litigated.
On the first ground, petitioners assert that paragraph 4 of Section 2 of Article XII permits the government
to enter into FTAAs only with foreign-owned corporations. Petitioners insist that the first paragraph of
this constitutional provision limits the participation of Filipino corporations in the exploration,
development and utilization of natural resources to only three species of contracts production
sharing, co-production and joint venture to the exclusion of all other arrangements or variations
thereof, and the WMCP FTAA may therefore not be validly assumed and implemented by Sagittarius. In
short, petitioners claim that a Filipino corporation is not allowed by the Constitution to enter into an FTAA
with the government.
However, a textual analysis of the first paragraph of Section 2 of Article XII does not support petitioners'
argument. The pertinent part of the said provision states: "Sec. 2. . . . The exploration, development and
utilization of natural resources shall be under the full control and supervision of the State. The State may
directly undertake such activities, or it may enter into co-production, joint venture, or production-sharing
agreements with Filipino citizens, or corporations or associations at least sixty per centum of whose
capital is owned by such citizens. . . ." Nowhere in the provision is there any express limitation or
restriction insofar as arrangements other than the three aforementioned contractual schemes are
concerned.
Neither can one reasonably discern any implied stricture to that effect. Besides, there is no basis to
believe that the framers of the Constitution, a majority of whom were obviously concerned with
furthering the development and utilization of the country's natural resources, could have wanted to
restrict Filipino participation in that area. This point is clear, especially in the light of the overarching
constitutional principle of giving preference and priority to Filipinos and Filipino corporations in the
development of our natural resources.
Besides, even assuming (purely for argument's sake) that a constitutional limitation barring Filipino
corporations from holding and implementing an FTAA actually exists, nevertheless, such provision would
apply only to the transfer of the FTAA to Sagittarius, but definitely not to the sale of WMC's equity stake
in WMCP to Sagittarius. Otherwise, an unreasonable curtailment of property rights without due process of
law would ensue. Petitioners' argument must therefore fail.
FTAA Not IntendedSolely for Foreign Corporation
Equally barren of merit is the second ground cited by petitioners that the FTAA was intended to apply
solely to a foreign corporation, as can allegedly be seen from the provisions therein. They manage to cite
only one WMCP FTAA provision that can be regarded as clearly intended to apply only to a foreign
contractor: Section 12, which provides for international commercial arbitration under the auspices of the
International Chamber of Commerce, after local remedies are exhausted. This provision, however, does
not necessarily imply that the WMCP FTAA cannot be transferred to and assumed by a Filipino
corporation like Sagittarius, in which event the said provision should simply be disregarded as a
superfluity.
No Need for a SeparateLitigation of the Sale of Shares
Petitioners claim as third ground the "suspicious" sale of shares from WMC to Sagittarius; hence, the
need to litigate it in a separate case. Section 40 of RA 7942 (the Mining Law) allegedly requires the
President's prior approval of a transfer.
A re-reading of the said provision, however, leads to a different conclusion. "Sec. 40.
Assignment/Transfer A financial or technical assistance agreement may be assigned or transferred, in
whole or in part, to a qualified person subject to the prior approval of the President: Provided, That the
President shall notify Congress of every financial or technical assistance agreement assigned or converted
in accordance with this provision within thirty (30) days from the date of the approval thereof ."
Section 40 expressly applies to the assignment or transfer of the FTAA, not to the sale and transfer of
shares of stock in WMCP. Moreover, when the transferee of an FTAA is another foreign corporation, there
is a logical application of the requirement of prior approval by the President of the Republic and
notification to Congress in the event of assignment or transfer of an FTAA. In this situation, such
approval and notification are appropriate safeguards, considering that the new contractor is the subject
of a foreign government.
54
On the other hand, when the transferee of the FTAA happens to be a Filipino corporation, the need for
such safeguard is not critical; hence, the lack of prior approval and notification may not be deemed fatal
as to render the transfer invalid. Besides, it is not as if approval by the President is entirely absent in this
instance. As pointed out by private respondent in its Memorandum, 13 the issue of approval is the
subject of one of the cases brought by Lepanto against Sagittarius in GR No. 162331. That case involved
the review of the Decision of the Court of Appeals dated November 21, 2003 in CA-GR SP No. 74161,
which affirmed the DENR Order dated December 31, 2001 and the Decision of the Office of the President
dated July 23, 2002, both approving the assignment of the WMCP FTAA to Sagittarius.
Petitioners also question the sale price and the financial capacity of the transferee. According to the Deed
of Absolute Sale dated January 23, 2001, executed between WMC and Sagittarius, the price of the WMCP
shares was fixed at US$9,875,000, equivalent to P553 million at an exchange rate of 56:1. Sagittarius
had an authorized capital stock of P250 million and a paid up capital of P60 million. Therefore, at the
time of approval of the sale by the DENR, the debt-to-equity ratio of the transferee was over 9:1
hardly ideal for an FTAA contractor, according to petitioners.
However, private respondents counter that the Deed of Sale specifically provides that the payment of the
purchase price would take place only after Sagittarius' commencement of commercial production from
mining operations, if at all. Consequently, under the circumstances, we believe it would not be
reasonable to conclude, as petitioners did, that the transferee's high debt-to-equity ratio per se
necessarily carried negative implications for the enterprise; and it would certainly be improper to
invalidate the sale on that basis, as petitioners propose.

FTAA Not Void, Thus Transferrable
To bolster further their claim that the case is not moot, petitioners insist that the FTAA is void and, hence
cannot be transferred; and that its transfer does not operate to cure the constitutional infirmity that is
inherent in it; neither will a change in the circumstances of one of the parties serve to ratify the void
contract.
While the discussion in their Final Memorandum was skimpy, petitioners in their Comment (on the MR)
did ratiocinate that this Court had declared the FTAA to be void because, at the time it was executed with
WMCP, the latter was a fully foreign-owned corporation, in which the former vested full control and
management with respect to the exploration, development and utilization of mineral resources, contrary
to the provisions of paragraph 4 of Section 2 of Article XII of the Constitution. And since the FTAA was
per se void, no valid right could be transferred; neither could it be ratified, so petitioners conclude.
Petitioners have assumed as fact that which has yet to be established. First and foremost, the Decision of
this Court declaring the FTAA void has not yet become final. That was precisely the reason the Court still
heard Oral Argument in this case. Second, the FTAA does not vest in the foreign corporation full control
and supervision over the exploration, development and utilization of mineral resources, to the exclusion
of the government. This point will be dealt with in greater detail below; but for now, suffice it to say that
a perusal of the FTAA provisions will prove that the government has effective overall direction and control
of the mining operations, including marketing and product pricing, and that the contractor's work
programs and budgets are subject to its review and approval or disapproval.
As will be detailed later on, the government does not have to micro-manage the mining operations and
dip its hands into the day-to-day management of the enterprise in order to be considered as having
overall control and direction. Besides, for practical and pragmatic reasons, there is a need for
government agencies to delegate certain aspects of the management work to the contractor. Thus the
basis for declaring the FTAA void still has to be revisited, reexamined and reconsidered.
Petitioners sniff at the citation of Chavez v. Public Estates Authority, 14 and Halili v. CA, 15 claiming that
the doctrines in these cases are wholly inapplicable to the instant case.
Chavez clearly teaches: "Thus, the Court has ruled consistently that where a Filipino citizen sells land to
an alien who later sells the land to a Filipino, the invalidity of the first transfer is corrected by the
subsequent sale to a citizen. Similarly, where the alien who buys the land subsequently acquires
Philippine citizenship, the sale is validated since the purpose of the constitutional ban to limit land
ownership to Filipinos has been achieved. In short, the law disregards the constitutional disqualification
of the buyer to hold land if the land is subsequently transferred to a qualified party, or the buyer himself
becomes a qualified party." 16
In their Comment, petitioners contend that in Chavez and Halili, the object of the transfer (the land) was
not what was assailed for alleged unconstitutionality. Rather, it was the transaction that was assailed;
hence subsequent compliance with constitutional provisions would cure its infirmity. In contrast, in the
instant case it is the FTAA itself, the object of the transfer, that is being assailed as invalid and
unconstitutional. So, petitioners claim that the subsequent transfer of a void FTAA to a Filipino
corporation would not cure the defect. DAETcC
Petitioners are confusing themselves. The present Petition has been filed, precisely because the grantee
of the FTAA was a wholly owned subsidiary of a foreign corporation. It cannot be gainsaid that anyone
would have asserted that the same FTAA was void if it had at the outset been issued to a Filipino
corporation. The FTAA, therefore, is not per se defective or unconstitutional. It was questioned only
because it had been issued to an allegedly non-qualified, foreign-owned corporation.
We believe that this case is clearly analogous to Halili, in which the land acquired by a non-Filipino was
re-conveyed to a qualified vendee and the original transaction was thereby cured. Paraphrasing Halili, the
same rationale applies to the instant case: assuming arguendo the invalidity of its prior grant to a foreign
corporation, the disputed FTAA being now held by a Filipino corporation can no longer be
assailed; the objective of the constitutional provision to keep the exploration, development and
utilization of our natural resources in Filipino hands has been served.
More accurately speaking, the present situation is one degree better than that obtaining in Halili, in which
the original sale to a non-Filipino was clearly and indisputably violative of the constitutional prohibition
and thus void ab initio. In the present case, the issuance/grant of the subject FTAA to the then foreign-
owned WMCP was not illegal, void or unconstitutional at the time. The matter had to be brought to court,
precisely for adjudication as to whether the FTAA and the Mining Law had indeed violated the
Constitution. Since, up to this point, the decision of this Court declaring the FTAA void has yet to become
final, to all intents and purposes, the FTAA must be deemed valid and constitutional. 17
At bottom, we find completely outlandish petitioners' contention that an FTAA could be entered into by
the government only with a foreign corporation, never with a Filipino enterprise. Indeed, the nationalistic
provisions of the Constitution are all anchored on the protection of Filipino interests. How petitioners can
now argue that foreigners have the exclusive right to FTAAs totally overturns the entire basis of the
Petition preference for the Filipino in the exploration, development and utilization of our natural
resources. It does not take deep knowledge of law and logic to understand that what the Constitution
grants to foreigners should be equally available to Filipinos.
Second Issue: Whether the Court Can Still Decide the Case, Even Assuming It Is Moot
All the protagonists are in agreement that the Court has jurisdiction to decide this controversy, even
assuming it to be moot.
Petitioners stress the following points. First, while a case becomes moot and academic when "there is no
more actual controversy between the parties or no useful purpose can be served in passing upon the
merits," 18 what is at issue in the instant case is not only the validity of the WMCP FTAA, but also the
constitutionality of RA 7942 and its Implementing Rules and Regulations. Second, the acts of private
respondent cannot operate to cure the law of its alleged unconstitutionality or to divest this Court of its
jurisdiction to decide. Third, the Constitution imposes upon the Supreme Court the duty to declare invalid
any law that offends the Constitution.
Petitioners also argue that no amendatory laws have been passed to make the Mining Act of 1995
conform to constitutional strictures (assuming that, at present, it does not); that public respondents will
continue to implement and enforce the statute until this Court rules otherwise; and that the said law
continues to be the source of legal authority in accepting, processing and approving numerous
applications for mining rights.
55
Indeed, it appears that as of June 30, 2002, some 43 FTAA applications had been filed with the Mines
and Geosciences Bureau (MGB), with an aggregate area of 2,064,908.65 hectares spread over Luzon,
the Visayas and Mindanao 19 applied for. It may be a bit far-fetched to assert, as petitioners do, that
each and every FTAA that was entered into under the provisions of the Mining Act "invites potential
litigation" for as long as the constitutional issues are not resolved with finality. Nevertheless, we must
concede that there exists the distinct possibility that one or more of the future FTAAs will be the subject
of yet another suit grounded on constitutional issues.
But of equal if not greater significance is the cloud of uncertainty hanging over the mining industry,
which is even now scaring away foreign investments. Attesting to this climate of anxiety is the fact that
the Chamber of Mines of the Philippines saw the urgent need to intervene in the case and to present its
position during the Oral Argument; and that Secretary General Romulo Neri of the National Economic
Development Authority (NEDA) requested this Court to allow him to speak, during that Oral Argument, on
the economic consequences of the Decision of January 27, 2004. 20
We are convinced. We now agree that the Court must recognize the exceptional character of the
situation and the paramount public interest involved, as well as the necessity for a ruling to put an end to
the uncertainties plaguing the mining industry and the affected communities as a result of doubts cast
upon the constitutionality and validity of the Mining Act, the subject FTAA and future FTAAs, and the
need to avert a multiplicity of suits. Paraphrasing Gonzales v. Commission on Elections, 21 it is evident
that strong reasons of public policy demand that the constitutionality issue be resolved now. 22
In further support of the immediate resolution of the constitutionality issue, public respondents cite Acop
v. Guingona, 23 to the effect that the courts will decide question otherwise moot and academic if
it is "capable of repetition, yet evading review." 24 Public respondents ask the Court to avoid a situation
in which the constitutionality issue may again arise with respect to another FTAA, the resolution of which
may not be achieved until after it has become too late for our mining industry to grow out of its infancy.
They also recall Salonga v. Cruz-Pao 25 in which this Court declared that "(t)he Court also has the
duty to formulate guiding and controlling constitutional principles, precepts, doctrines or rules. It has the
symbolic function of educating the bench and bar on the extent of protection given by constitutional
guarantees. . . ."

The mootness of the case in relation to the WMCP FTAA led the undersigned ponente to state in his
dissent to the Decision that there was no more justiciable controversy and the plea to nullify the Mining
Law has become a virtual petition for declaratory relief. 26 The entry of the Chamber of Mines of the
Philippines, Inc., however, has put into focus the seriousness of the allegations of unconstitutionality of
RA 7942 and DAO 96-40 which converts the case to one for prohibition 27 in the enforcement of the said
law and regulations.
Indeed, this CMP entry brings to fore that the real issue in this case is whether paragraph 4 of Section 2
of Article XII of the Constitution is contravened by RA 7942 and DAO 96-40, not whether it was violated
by specific acts implementing RA 7942 and DAO 96-40. "[W]hen an act of the legislative department is
seriously alleged to have infringed the Constitution, settling the controversy becomes the duty of this
Court. By the mere enactment of the questioned law or the approval of the challenged action, the dispute
is said to have ripened into a judicial controversy even without any other overt act." 28 This ruling can
be traced from Taada v. Angara, 29 in which the Court said:
"In seeking to nullify an act of the Philippine Senate on the ground that it
contravenes the Constitution, the petition no doubt raises a justiciable
controversy. Where an action of the legislative branch is seriously alleged
to have infringed the Constitution, it becomes not only the right but in fact
the duty of the judiciary to settle the dispute.
xxx xxx xxx
"As this Court has repeatedly and firmly emphasized in many cases, it will
not shirk, digress from or abandon its sacred duty and authority to uphold
the Constitution in matters that involve grave abuse of discretion brought
before it in appropriate cases, committed by any officer, agency,
instrumentality or department of the government." 30
Additionally, the entry of CMP into this case has also effectively forestalled any possible objections arising
from the standing or legal interest of the original parties.
For all the foregoing reasons, we believe that the Court should proceed to a resolution of the
constitutional issues in this case.
Third Issue: The Proper Interpretation of the Constitutional Phrase"Agreements Involving Either
Technical or Financial Assistance"
The constitutional provision at the nucleus of the controversy is paragraph 4 of Section 2 of Article XII of
the 1987 Constitution. In order to appreciate its context, Section 2 is reproduced in full:
"Sec. 2.All lands of the public domain, waters, minerals, coal, petroleum,
and other mineral oils, all forces of potential energy, fisheries, forests or
timber, wildlife, flora and fauna, and other natural resources are owned by
the State. With the exception of agricultural lands, all other natural
resources shall not be alienated. The exploration, development and
utilization of natural resources shall be under the full control and
supervision of the State. The State may directly undertake such activities,
or it may enter into co-production, joint venture or production-sharing
agreements with Filipino citizens or corporations or associations at least
sixty per centum of whose capital is owned by such citizens. Such
agreements may be for a period not exceeding twenty-five years,
renewable for not more than twenty-five years, and under such terms and
conditions as may be provided by law. In cases of water rights for
irrigation, water supply, fisheries, or industrial uses other than the
development of water power, beneficial use may be the measure and limit
of the grant.
"The State shall protect the nation's marine wealth in its archipelagic
waters, territorial sea, and exclusive economic zone, and reserve its use
and enjoyment exclusively to Filipino citizens.
"The Congress may, by law, allow small-scale utilization of natural
resources by Filipino citizens, as well as cooperative fish farming, with
priority to subsistence fishermen and fish-workers in rivers, lakes, bays
and lagoons.
"The President may enter into agreements with foreign-owned
corporations involving either technical or financial assistance for large-scale
exploration, development, and utilization of minerals, petroleum, and other
mineral oils according to the general terms and conditions provided by law,
based on real contributions to the economic growth and general welfare of
the country. In such agreements, the State shall promote the development
and use of local scientific and technical resources. IAEcCa
"The President shall notify the Congress of every contract entered into in
accordance with this provision, within thirty days from its execution." 31
No Restriction of Meaning bya Verba Legis Interpretation
To interpret the foregoing provision, petitioners adamantly assert that the language of the Constitution
should prevail; that the primary method of interpreting it is to seek the ordinary meaning of the words
used in its provisions. They rely on rulings of this Court, such as the following:
"The fundamental principle in constitutional construction however is that
56
the primary source from which to ascertain constitutional intent or purpose
is the language of the provision itself. The presumption is that the words in
which the constitutional provisions are couched express the objective
sought to be attained. In other words, verba legis prevails. Only when the
meaning of the words used is unclear and equivocal should resort be made
to extraneous aids of construction and interpretation, such as the
proceedings of the Constitutional Commission or Convention to shed light
on and ascertain the true intent or purpose of the provision being
construed." 32
Very recently, in Francisco v. The House of Representatives, 33 this Court indeed had the occasion to
reiterate the well-settled principles of constitutional construction:
"First, verba legis, that is, wherever possible, the words used in the
Constitution must be given their ordinary meaning except where technical
terms are employed. . . .
xxx xxx xxx
"Second, where there is ambiguity, ratio legis est anima. The words of the
Constitution should be interpreted in accordance with the intent of its
framers. . . .
xxx xxx xxx
"Finally, ut magis valeat quam pereat. The Constitution is to be interpreted
as a whole." 34
For ease of reference and in consonance with verba legis, we reconstruct and stratify the aforequoted
Section 2 as follows:
1.All natural resources are owned by the State. Except for agricultural
lands, natural resources cannot be alienated by the State.
2.The exploration, development and utilization (EDU) of natural resources
shall be under the full control and supervision of the State.
3.The State may undertake these EDU activities through either of the
following:
(a)By itself directly and solely
(b)By (i) co-production; (ii) joint venture; or (iii) production
sharing agreements with Filipino citizens or
corporations, at least 60 percent of the capital
of which is owned by such citizens
4.Small-scale utilization of natural resources may be allowed by law in
favor of Filipino citizens.
5.For large-scale EDU of minerals, petroleum and other mineral oils, the
President may enter into "agreements with foreign-owned corporations
involving either technical or financial assistance according to the general
terms and conditions provided by law. . . ."
Note that in all the three foregoing mining activities exploration, development and utilization the
State may undertake such EDU activities by itself or in tandem with Filipinos or Filipino corporations,
except in two instances: first, in small-scale utilization of natural resources, which Filipinos may be
allowed by law to undertake; and second, in large-scale EDU of minerals, petroleum and mineral oils,
which may be undertaken by the State via "agreements with foreign-owned corporations involving either
technical or financial assistance" as provided by law.
Petitioners claim that the phrase "agreements . . . involving either technical or financial assistance"
simply means technical assistance or financial assistance agreements, nothing more and nothing else.
They insist that there is no ambiguity in the phrase, and that a plain reading of paragraph 4 quoted
above leads to the inescapable conclusion that what a foreign-owned corporation may enter into with the
government is merely an agreement for either financial or technical assistance only, for the large-scale
exploration, development and utilization of minerals, petroleum and other mineral oils; such a limitation,
they argue, excludes foreign management and operation of a mining enterprise. 35
This restrictive interpretation, petitioners believe, is in line with the general policy enunciated by the
Constitution reserving to Filipino citizens and corporations the use and enjoyment of the country's natural
resources. They maintain that this Court's Decision 36 of January 27, 2004 correctly declared the WMCP
FTAA, along with pertinent provisions of RA 7942, void for allowing a foreign contractor to have direct
and exclusive management of a mining enterprise. Allowing such a privilege not only runs counter to the
"full control and supervision" that the State is constitutionally mandated to exercise over the exploration,
development and utilization of the country's natural resources; doing so also vests in the foreign
company "beneficial ownership" of our mineral resources. It will be recalled that the Decision of January
27, 2004 zeroed in on "management or other forms of assistance" or other activities associated with the
"service contracts" of the martial law regime, since "the management or operation of mining activities by
foreign contractors, which is the primary feature of service contracts, was precisely the evil that the
drafters of the 1987 Constitution sought to eradicate."

On the other hand, the intervenor 37 and public respondents argue that the FTAA allowed by paragraph
4 is not merely an agreement for supplying limited and specific financial or technical services to the State.
Rather, such FTAA is a comprehensive agreement for the foreign-owned corporation's integrated
exploration, development and utilization of mineral, petroleum or other mineral oils on a large-scale
basis. The agreement, therefore, authorizes the foreign contractor's rendition of a whole range of
integrated and comprehensive services, ranging from the discovery to the development, utilization and
production of minerals or petroleum products.
We do not see how applying a strictly literal or verba legis interpretation of paragraph 4 could inexorably
lead to the conclusions arrived at in the ponencia. First, the drafters' choice of words their use of the
phrase agreements . . . involving either technical or financial assistance does not indicate the intent
to exclude other modes of assistance. The drafters opted to use involving when they could have simply
said agreements for financial or technical assistance, if that was their intention to begin with. In this
case, the limitation would be very clear and no further debate would ensue.
In contrast, the use of the word "involving" signifies the possibility of the inclusion of other forms of
assistance or activities having to do with, otherwise related to or compatible with financial or technical
assistance. The word "involving" as used in this context has three connotations that can be differentiated
thus: one, the sense of "concerning," "having to do with," or "affecting"; two, "entailing," "requiring,"
"implying" or "necessitating"; and three, "including," "containing" or "comprising." 38
Plainly, none of the three connotations convey a sense of exclusivity. Moreover, the word "involving,"
when understood in the sense of "including," as in including technical or financial assistance, necessarily
implies that there are activities other than those that are being included. In other words, if an agreement
includes technical or financial assistance, there is apart from such assistance something else already
in and covered or may be covered, by the said agreement.
In short, it allows for the possibility that matters, other than those explicitly mentioned, could be made
part of the agreement. Thus, we are now led to the conclusion that the use of the word "involving"
implies that these agreements with foreign corporations are not limited to mere financial or technical
assistance. The difference in sense becomes very apparent when we juxtapose "agreements for technical
or financial assistance" against "agreements including technical or financial assistance." This much is
unalterably clear in a verba legis approach.
57
Second, if the real intention of the drafters was to confine foreign corporations to financial or technical
assistance and nothing more, their language would have certainly been so unmistakably restrictive and
stringent as to leave no doubt in anyone's mind about their true intent. For example, they would have
used the sentence foreign corporations are absolutely prohibited from involvement in the management or
operation of mining or similar ventures or words of similar import. A search for such stringent wording
yields negative results. Thus, we come to the inevitable conclusion that there was a conscious and
deliberate decision to avoid the use of restrictive wording that bespeaks an intent not to use the
expression "agreements . . . involving either technical or financial assistance" in an exclusionary and
limiting manner.
Deletion of "Service Contracts" toAvoid Pitfalls of Previous Constitutions, Not to Ban Service
Contracts Per Se
Third, we do not see how a verba legis approach leads to the conclusion that "the management or
operation of mining activities by foreign contractors, which is the primary feature of service contracts,
was precisely the evil that the drafters of the 1987 Constitution sought to eradicate." Nowhere in the
above-quoted Section can be discerned the objective to keep out of foreign hands the management or
operation of mining activities or the plan to eradicate service contracts as these were understood in the
1973 Constitution. Still, petitioners maintain that the deletion or omission from the 1987 Constitution of
the term "service contracts" found in the 1973 Constitution sufficiently proves the drafters' intent to
exclude foreigners from the management of the affected enterprises.
To our mind, however, such intent cannot be definitively and conclusively established from the mere
failure to carry the same expression or term over to the new Constitution, absent a more specific, explicit
and unequivocal statement to that effect. What petitioners seek (a complete ban on foreign participation
in the management of mining operations, as previously allowed by the earlier Constitutions) is nothing
short of bringing about a momentous sea change in the economic and developmental policies; and the
fundamentally capitalist, free-enterprise philosophy of our government. We cannot imagine such a radical
shift being undertaken by our government, to the great prejudice of the mining sector in particular and
our economy in general, merely on the basis of the omission of the terms service contract from or the
failure to carry them over to the new Constitution. There has to be a much more definite and even
unarguable basis for such a drastic reversal of policies.
Fourth, a literal and restrictive interpretation of paragraph 4, such as that proposed by petitioners, suffers
from certain internal logical inconsistencies that generate ambiguities in the understanding of the
provision. As the intervenor pointed out, there has never been any constitutional or statutory provision
that reserved to Filipino citizens or corporations, at least 60 percent of which is Filipino-owned, the
rendition of financial or technical assistance to companies engaged in mining or the development of any
other natural resource. The taking out of foreign-currency or peso-denominated loans or any other kind
of financial assistance, as well as the rendition of technical assistance whether to the State or to any
other entity in the Philippines has never been restricted in favor of Filipino citizens or corporations
having a certain minimum percentage of Filipino equity. Such a restriction would certainly be
preposterous and unnecessary. As a matter of fact, financial, and even technical assistance, regardless of
the nationality of its source, would be welcomed in the mining industry anytime with open arms, on
account of the dearth of local capital and the need to continually update technological know-how and
improve technical skills.
There was therefore no need for a constitutional provision specifically allowing foreign-owned
corporations to render financial or technical assistance, whether in respect of mining or some other
resource development or commercial activity in the Philippines. The last point needs to be emphasized: if
merely financial or technical assistance agreements are allowed, there would be no need to limit them to
large-scale mining operations, as there would be far greater need for them in the smaller-scale mining
activities (and even in non-mining areas). Obviously, the provision in question was intended to refer to
agreements other than those for mere financial or technical assistance.
In like manner, there would be no need to require the President of the Republic to report to Congress, if
only financial or technical assistance agreements are involved. Such agreements are in the nature of
foreign loans that pursuant to Section 20 of Article VII 39 of the 1987 Constitution the President
may contract or guarantee, merely with the prior concurrence of the Monetary Board. In turn, the Board
is required to report to Congress within thirty days from the end of every quarter of the calendar year,
not thirty days after the agreement is entered into.
And if paragraph 4 permits only agreements for loans and other forms of financial, or technical
assistance, what is the point of requiring that they be based on real contributions to the economic growth
and general welfare of the country? For instance, how is one to measure and assess the "real
contributions" to the "economic growth" and "general welfare" of the country that may ensue from a
foreign-currency loan agreement or a technical-assistance agreement for, say, the refurbishing of an
existing power generating plant for a mining operation somewhere in Mindanao? Such a criterion would
make more sense when applied to a major business investment in a principal sector of the industry.
The conclusion is clear and inescapable a verba legis construction shows that paragraph 4 is not to
be understood as one limited only to foreign loans (or other forms of financial support) and to technical
assistance. There is definitely more to it than that. These are provisions permitting participation by
foreign companies; requiring the President's report to Congress; and using, as yardstick, contributions
based on economic growth and general welfare. These were neither accidentally inserted into the
Constitution nor carelessly cobbled together by the drafters in lip service to shallow nationalism. The
provisions patently have significance and usefulness in a context that allows agreements with foreign
companies to include more than mere financial or technical assistance.
Fifth, it is argued that Section 2 of Article XII authorizes nothing more than a rendition of specific and
limited financial service or technical assistance by a foreign company. This argument begs the question
"To whom or for whom would it be rendered"? or Who is being assisted? If the answer is "The State,"
then it necessarily implies that the State itself is the one directly and solely undertaking the large-scale
exploration, development and utilization of a mineral resource, so it follows that the State must itself bear
the liability and cost of repaying the financing sourced from the foreign lender and/or of paying
compensation to the foreign entity rendering technical assistance.

However, it is of common knowledge, and of judicial notice as well, that the government is and has for
many many years been financially strapped, to the point that even the most essential services have
suffered serious curtailments education and health care, for instance, not to mention judicial services
have had to make do with inadequate budgetary allocations. Thus, government has had to resort to
build-operate-transfer and similar arrangements with the private sector, in order to get vital infrastructure
projects built without any governmental outlay. TCaADS
The very recent brouhaha over the gargantuan "fiscal crisis" or "budget deficit" merely confirms what the
ordinary citizen has suspected all along. After the reality check, one will have to admit the implausibility
of a direct undertaking by the State itself of large-scale exploration, development and utilization
of minerals, petroleum and other mineral oils. Such an undertaking entails not only humongous capital
requirements, but also the attendant risk of never finding and developing economically viable quantities
of minerals, petroleum and other mineral oils. 40
It is equally difficult to imagine that such a provision restricting foreign companies to the rendition of only
financial or technical assistance to the government was deliberately crafted by the drafters of the
Constitution, who were all well aware of the capital-intensive and technology-oriented nature of large-
scale mineral or petroleum extraction and the country's deficiency in precisely those areas. 41 To say so
would be tantamount to asserting that the provision was purposely designed to ladle the large-scale
development and utilization of mineral, petroleum and related resources with impossible conditions, and
to remain forever and permanently "reserved" for future generations of Filipinos.
A More Reasonable Lookat the Charter's Plain Language
Sixth, we shall now look closer at the plain language of the Charter and examining the logical inferences.
The drafters chose to emphasize and highlight agreements . . . involving either technical or financial
assistance in relation to foreign corporations' participation in large-scale EDU. The inclusion of this clause
on "technical or financial assistance" recognizes the fact that foreign business entities and multinational
corporations are the ones with the resources and know-how to provide technical and/or financial
assistance of the magnitude and type required for large-scale exploration, development and utilization of
these resources.
58
The drafters whose ranks included many academicians, economists, businessmen, lawyers, politicians
and government officials were not unfamiliar with the practices of foreign corporations and
multinationals.
Neither were they so naive as to believe that these entities would provide "assistance" without
conditionalities or some quid pro quo. Definitely, as business persons well know and as a matter of
judicial notice, this matter is not just a question of signing a promissory note or executing a technology
transfer agreement. Foreign corporations usually require that they be given a say in the management, for
instance, of day-to-day operations of the joint venture. They would demand the appointment of their
own men as, for example, operations managers, technical experts, quality control heads, internal auditors
or comptrollers. Furthermore, they would probably require seats on the Board of Directors all these
to ensure the success of the enterprise and the repayment of the loans and other financial assistance and
to make certain that the funding and the technology they supply would not go to waste. Ultimately, they
would also want to protect their business reputation and bottom lines. 42
In short, the drafters will have to be credited with enough pragmatism and savvy to know that these
foreign entities will not enter into such "agreements involving assistance" without requiring arrangements
for the protection of their investments, gains and benefits.
Thus, by specifying such "agreements involving assistance," the drafters necessarily gave implied assent
to everything that these agreements necessarily entailed; or that could reasonably be deemed necessary
to make them tenable and effective, including management authority with respect to the day-to-day
operations of the enterprise and measures for the protection of the interests of the foreign corporation,
PROVIDED THAT Philippine sovereignty over natural resources and full control over the enterprise
undertaking the EDU activities remain firmly in the State.
Petitioners' Theory Deflated by theAbsence of Closing-Out Rules or Guidelines
Seventh and final point regarding the plain-language approach, one of the practical difficulties that
results from it is the fact that there is nothing by way of transitory provisions that would serve to confirm
the theory that the omission of the term "service contract" from the 1987 Constitution signaled the
demise of service contracts.
The framers knew at the time they were deliberating that there were various service contracts extant and
in force and effect, including those in the petroleum industry. Many of these service contracts were long-
term (25 years) and had several more years to run. If they had meant to ban service contracts
altogether, they would have had to provide for the termination or pretermination of the existing
contracts. Accordingly, they would have supplied the specifics and the when and how of effecting the
extinguishment of these existing contracts (or at least the mechanics for determining them); and of
putting in place the means to address the just claims of the contractors for compensation for their
investments, lost opportunities, and so on, if not for the recovery thereof .
If the framers had intended to put an end to service contracts, they would have at least left specific
instructions to Congress to deal with these closing-out issues, perhaps by way of general guidelines and a
timeline within which to carry them out. The following are some extant examples of such transitory
guidelines set forth in Article XVIII of our Constitution:
"Section 23.Advertising entities affected by paragraph (2), Section 11 of
Article XVI of this Constitution shall have five years from its ratification to
comply on a graduated and proportionate basis with the minimum Filipino
ownership requirement therein.
xxx xxx xxx
"Section 25.After the expiration in 1991 of the Agreement between the
Republic of the Philippines and the United States of America concerning
military bases, foreign military bases, troops, or facilities shall not be
allowed in the Philippines except under a treaty duly concurred in by the
Senate and, when the Congress so requires, ratified by a majority of the
votes cast by the people in a national referendum held for that purpose,
and recognized as a treaty by the other contracting State.
"Section 26.The authority to issue sequestration or freeze orders under
Proclamation No. 3 dated March 25, 1986 in relation to the recovery of ill-
gotten wealth shall remain operative for not more than eighteen months
after the ratification of this Constitution. However, in the national interest,
as certified by the President, the Congress may extend such period.
A sequestration or freeze order shall be issued only upon showing of a
prima facie case. The order and the list of the sequestered or frozen
properties shall forthwith be registered with the proper court. For orders
issued before the ratification of this Constitution, the corresponding judicial
action or proceeding shall be filed within six months from its ratification.
For those issued after such ratification, the judicial action or proceeding
shall be commenced within six months from the issuance thereof.
The sequestration or freeze order is deemed automatically lifted if no
judicial action or proceeding is commenced as herein provided." 43
It is inconceivable that the drafters of the Constitution would leave such an important matter an
expression of sovereignty as it were indefinitely hanging in the air in a formless and ineffective state.
Indeed, the complete absence of even a general framework only serves to further deflate petitioners'
theory, like a child's balloon losing its air.
Under the circumstances, the logical inconsistencies resulting from petitioners' literal and purely verba
legis approach to paragraph 4 of Section 2 of Article XII compel a resort to other aids to interpretation.
Petitioners' Posture Also Negatedby Ratio Legis Et Anima
Thus, in order to resolve the inconsistencies, incongruities and ambiguities encountered and to supply the
deficiencies of the plain-language approach, there is a need for recourse to the proceedings of the 1986
Constitutional Commission. There is a need for ratio legis et anima.
Service Contracts Not "Deconstitutionalized"
Pertinent portions of the deliberations of the members of the Constitutional Commission (ConCom)
conclusively show that they discussed agreements involving either technical or financial assistance in the
same breadth as service contracts and used the terms interchangeably. The following exchange between
Commissioner Jamir (sponsor of the provision) and Commissioner Suarez irrefutably proves that the
"agreements involving technical or financial assistance" were none other than service contracts.
THE PRESIDENT.Commissioner Jamir is recognized. We are still on Section
3.
MR. JAMIR.Yes, Madam President. With respect to the second paragraph
of Section 3, my amendment by substitution reads: THE
PRESIDENT MAY ENTER INTO AGREEMENTS WITH
FOREIGN-OWNED CORPORATIONS INVOLVING EITHER
TECHNICAL OR FINANCIAL ASSISTANCE FOR LARGE-SCALE
EXPLORATION, DEVELOPMENT AND UTILIZATION OF
NATURAL RESOURCES ACCORDING TO THE TERMS AND
CONDITIONS PROVIDED BY LAW.
MR. VILLEGAS.The Committee accepts the amendment. Commissioner
Suarez will give the background.

MR. JAMIR.Thank you.
59
THE PRESIDENT.Commissioner Suarez is recognized.
MR. SUAREZ.Thank you, Madam President.
Will Commissioner Jamir answer a few clarificatory questions?
MR. JAMIR.Yes, Madam President.
MR. SUAREZ.This particular portion of the section has reference to what
was popularly known before as service contracts, among
other things, is that correct?
MR. JAMIR.Yes, Madam President.
MR. SUAREZ.As it is formulated, the President may enter into service
contracts but subject to the guidelines that may be
promulgated by Congress?
MR. JAMIR.That is correct.
MR. SUAREZ.Therefore, that aspect of negotiation and consummation will
fall on the President, not upon Congress?
MR. JAMIR.That is also correct, Madam President.
MR. SUAREZ.Except that all of these contracts, service or otherwise, must
be made strictly in accordance with guidelines prescribed by
Congress?
MR. JAMIR.That is also correct.
MR. SUAREZ.And the Gentleman is thinking in terms of a law that
uniformly covers situations of the same nature?
MR. JAMIR.That is 100 percent correct.
MR. SUAREZ.I thank the Commissioner.
MR. JAMIR.Thank you very much. 44
The following exchange leaves no doubt that the commissioners knew exactly what they were dealing
with: service contracts.
THE PRESIDENT.Commissioner Gascon is recognized.
MR. GASCON.Commissioner Jamir had proposed an amendment with
regard to special service contracts which was accepted by
the Committee. Since the Committee has accepted it, I would
like to ask some questions.
THE PRESIDENT.Commissioner Gascon may proceed.
MR. GASCON.As it is proposed now, such service contracts will be entered
into by the President with the guidelines of a general law on
service contract to be enacted by Congress. Is that correct?
MR. VILLEGAS.The Commissioner is right, Madam President.
MR. GASCON.According to the original proposal, if the President were to
enter into a particular agreement, he would need the
concurrence of Congress. Now that it has been changed by
the proposal of Commissioner Jamir in that Congress will set
the general law to which the President shall comply, the
President will, therefore, not need the concurrence of
Congress every time he enters into service contracts. Is that
correct?
MR. VILLEGAS.That is right.
MR. GASCON.The proposed amendment of Commissioner Jamir is in
indirect contrast to my proposed amendment, so I would like
to object and present my proposed amendment to the body.
xxx xxx xxx
MR. GASCON.Yes, it will be up to the body.
I feel that the general law to be set by Congress as regard service contract
agreements which the President will enter into might be too
general or since we do not know the content yet of such a
law, it might be that certain agreements will be detrimental
to the interest of the Filipinos. This is in direct contrast to my
proposal which provides that there be effective constraints in
the implementation of service contracts.
So instead of a general law to be passed by Congress to serve as a
guideline to the President when entering into service contract
agreements, I propose that every service contract entered
into by the President would need the concurrence of
Congress, so as to assure the Filipinos of their interests with
regard to the issue in Section 3 on all lands of the public
domain. My alternative amendment, which we will discuss
later, reads: THAT THE PRESIDENT SHALL ENTER INTO
SUCH AGREEMENTS ONLY WITH THE CONCURRENCE OF
TWO-THIRDS VOTE OF ALL THE MEMBERS OF CONGRESS
SITTING SEPARATELY.
xxx xxx xxx
MR. BENGZON.The reason we made that shift is that we realized the
original proposal could breed corruption. By the way, this is
not just confined to service contracts but also to financial
assistance. If we are going to make every single contract
subject to the concurrence of Congress which, according
to the Commissioner's amendment is the concurrence of two-
thirds of Congress voting separately then (1) there is a
very great chance that each contract will be different from
another; and (2) there is a great temptation that it would
breed corruption because of the great lobbying that is going
to happen. And we do not want to subject our legislature to
that.
Now, to answer the Commissioner's apprehension, by "general law," we do
not mean statements of motherhood. Congress can build all
the restrictions that it wishes into that general law so that
every contract entered into by the President under that
specific area will have to be uniform. The President has no
choice but to follow all the guidelines that will be provided by
law.
60
MR. GASCON.But my basic problem is that we do not know as of yet the
contents of such a general law as to how much constraints
there will be in it. And to my mind, although the Committee's
contention that the regular concurrence from Congress would
subject Congress to extensive lobbying, I think that is a risk
we will have to take since Congress is a body of
representatives of the people whose membership will be
changing regularly as there will be changing circumstances
every time certain agreements are made. It would be best
then to keep in tab and attuned to the interest of the Filipino
people, whenever the President enters into any agreement
with regard to such an important matter as technical or
financial assistance for large-scale exploration, development
and utilization of natural resources or service contracts, the
people's elected representatives should be on top of it.
xxx xxx xxx
MR. OPLE.Madam President, we do not need to suspend the session. If
Commissioner Gascon needs a few minutes, I can fill up the
remaining time while he completes his proposed amendment.
I just wanted to ask Commissioner Jamir whether he would
entertain a minor amendment to his amendment, and it
reads as follows: THE PRESIDENT SHALL SUBSEQUENTLY
NOTIFY CONGRESS OF EVERY SERVICE CONTRACT
ENTERED INTO IN ACCORDANCE WITH THE GENERAL LAW.
I think the reason is, if I may state it briefly, as
Commissioner Bengzon said, Congress can always change
the general law later on to conform to new perceptions of
standards that should be built into service contracts. But the
only way Congress can do this is if there were a notification
requirement from the Office of the President that such
service contracts had been entered into, subject then to the
scrutiny of the Members of Congress. This pertains to a
situation where the service contracts are already entered
into, and all that this amendment seeks is the reporting
requirement from the Office of the President. Will
Commissioner Jamir entertain that?
MR. JAMIR.I will gladly do so, if it is still within my power.
MR. VILLEGAS.Yes, the Committee accepts the amendment.
xxx xxx xxx
SR. TAN.Madam President, may I ask a question?
THE PRESIDENT.Commissioner Tan is recognized.
SR. TAN.Am I correct in thinking that the only difference between these
future service contracts and the past service contracts under
Mr. Marcos is the general law to be enacted by the
legislature and the notification of Congress by the President?
That is the only difference, is it not?
MR. VILLEGAS.That is right.
SR. TAN.So those are the safeguards.
MR. VILLEGAS.Yes. There was no law at all governing service contracts
before. TAECSD
SR. TAN.Thank you, Madam President. 45
More Than Mere Financial and Technical AssistanceEntailed by the Agreements
The clear words of Commissioner Jose N. Nolledo quoted below explicitly and eloquently demonstrate
that the drafters knew that the agreements with foreign corporations were going to entail not mere
technical or financial assistance but, rather, foreign investment in and management of an enterprise
involved in large-scale exploration, development and utilization of minerals, petroleum, and other mineral
oils.
THE PRESIDENT.Commissioner Nolledo is recognized.
MR. NOLLEDO.Madam President, I have the permission of the Acting Floor
Leader to speak for only two minutes in favor of the
amendment of Commissioner Gascon.
THE PRESIDENT.Commissioner Nolledo may proceed.
MR. NOLLEDO.With due respect to the members of the Committee and
Commissioner Jamir, I am in favor of the objection of
Commissioner Gascon.
Madam President, I was one of those who refused to sign the 1973
Constitution, and one of the reasons is that there were many
provisions in the Transitory Provisions therein that favored
aliens. I was shocked when I read a provision authorizing
service contracts while we, in this Constitutional Commission,
provided for Filipino control of the economy. We are,
therefore, providing for exceptional instances where aliens
may circumvent Filipino control of our economy. And one
way of circumventing the rule in favor of Filipino control of
the economy is to recognize service contracts.
As far as I am concerned, if I should have my own way, I am for the
complete deletion of this provision. However, we are
presenting a compromise in the sense that we are requiring a
two-thirds vote of all the Members of Congress as a
safeguard. I think we should not mistrust the future Members
of Congress by saying that the purpose of this provision is to
avoid corruption. We cannot claim that they are less patriotic
than we are. I think the Members of this Commission should
know that entering into service contracts is an exception to
the rule on protection of natural resources for the interest of
the nation, and therefore, being an exception it should be
subject, whenever possible, to stringent rules. It seems to
me that we are liberalizing the rules in favor of aliens.
I say these things with a heavy heart, Madam President. I do not claim to
be a nationalist, but I love my country. Although we need
investments, we must adopt safeguards that are truly
reflective of the sentiments of the people and not mere
cosmetic safeguards as they now appear in the Jamir
amendment. (Applause)

Thank you, Madam President. 46
61
Another excerpt, featuring then Commissioner (now Chief Justice) Hilario G. Davide Jr., indicates the
limitations of the scope of such service contracts they are valid only in regard to minerals, petroleum
and other mineral oils, not to all natural resources.
THE PRESIDENT.Commissioner Davide is recognized.
MR. DAVIDE.Thank you, Madam President. This is an amendment to the
Jamir amendment and also to the Ople amendment. I
propose to delete "NATURAL RESOURCES" and substitute it
with the following: MINERALS, PETROLEUM AND OTHER
MINERAL OILS. On the Ople amendment, I propose to add:
THE NOTIFICATION TO CONGRESS SHALL BE WITHIN
THIRTY DAYS FROM THE EXECUTION OF THE SERVICE
CONTRACT.
THE PRESIDENT.What does the Committee say with respect to the first
amendment in lieu of "NATURAL RESOURCES"?
MR. VILLEGAS.Could Commissioner Davide explain that?
MR. DAVIDE.Madam President, with the use of "NATURAL RESOURCES"
here, it would necessarily include all lands of the public
domain, our marine resources, forests, parks and so on. So
we would like to limit the scope of these service contracts to
those areas really where these may be needed, the
exploitation, development and exploration of minerals,
petroleum and other mineral oils. And so, we believe that we
should really, if we want to grant service contracts at all, limit
the same to only those particular areas where Filipino capital
may not be sufficient, and not to all natural resources.
MR. SUAREZ.Just a point of clarification again, Madam President. When
the Commissioner made those enumerations and
specifications, I suppose he deliberately did not include
"agricultural land"?
MR. DAVIDE.That is precisely the reason we have to enumerate what
these resources are into which service contracts may enter.
So, beyond the reach of any service contract will be lands of
the public domain, timberlands, forests, marine resources,
fauna and flora, wildlife and national parks. 47
After the Jamir amendment was voted upon and approved by a vote of 21 to 10 with 2 abstentions,
Commissioner Davide made the following statement, which is very relevant to our quest:
THE PRESIDENT.Commissioner Davide is recognized.
MR. DAVIDE.I am very glad that Commissioner Padilla emphasized
minerals, petroleum and mineral oils. The Commission has
just approved the possible foreign entry into the
development, exploration and utilization of these minerals,
petroleum and other mineral oils by virtue of the Jamir
amendment. I voted in favor of the Jamir amendment
because it will eventually give way to vesting in exclusively
Filipino citizens and corporations wholly owned by Filipino
citizens the right to utilize the other natural resources. This
means that as a matter of policy, natural resources should be
utilized and exploited only by Filipino citizens or corporations
wholly owned by such citizens. But by virtue of the Jamir
amendment, since we feel that Filipino capital may not be
enough for the development and utilization of minerals,
petroleum and other mineral oils, the President can enter
into service contracts with foreign corporations precisely for
the development and utilization of such resources. And so,
there is nothing to fear that we will stagnate in the
development of minerals, petroleum and mineral oils because
we now allow service contracts. . .? 48
The foregoing are mere fragments of the framers' lengthy discussions of the provision dealing with
agreements . . . involving either technical or financial assistance, which ultimately became paragraph 4 of
Section 2 of Article XII of the Constitution. Beyond any doubt, the members of the ConCom were actually
debating about the martial-law-era service contracts for which they were crafting appropriate safeguards.
In the voting that led to the approval of Article XII by the ConCom, the explanations given by
Commissioners Gascon, Garcia and Tadeo indicated that they had voted to reject this provision on
account of their objections to the "constitutionalization" of the "service contract" concept.
Mr. Gascon said, "I felt that if we would constitutionalize any provision on service contracts, this should
always be with the concurrence of Congress and not guided only by a general law to be promulgated by
Congress." 49 Mr. Garcia explained, "Service contracts are given constitutional legitimization in Sec. 3,
even when they have been proven to be inimical to the interests of the nation, providing, as they do, the
legal loophole for the exploitation of our natural resources for the benefit of foreign interests." 50
Likewise, Mr. Tadeo cited inter alia the fact that service contracts continued to subsist, enabling foreign
interests to benefit from our natural resources. 51 It was hardly likely that these gentlemen would have
objected so strenuously, had the provision called for mere technical or financial assistance and nothing
more.
The deliberations of the ConCom and some commissioners' explanation of their votes leave no room for
doubt that the service contract concept precisely underpinned the commissioners' understanding of the
"agreements involving either technical or financial assistance."
Summation of theConcom Deliberations
At this point, we sum up the matters established, based on a careful reading of the ConCom
deliberations, as follows:
In their deliberations on what was to become paragraph 4, the framers
used the term service contracts in referring to agreements . .
. involving either technical or financial assistance.
They spoke of service contracts as the concept was understood in the
1973 Constitution.
It was obvious from their discussions that they were not about to ban
or eradicate service contracts.
Instead, they were plainly crafting provisions to put in place safeguards
that would eliminate or minimize the abuses prevalent during
the martial law regime. In brief, they were going to permit
service contracts with foreign corporations as contractors,
but with safety measures to prevent abuses, as an exception
to the general norm established in the first paragraph of
Section 2 of Article XII. This provision reserves or limits to
Filipino citizens and corporations at least 60 percent of
which is owned by such citizens the exploration,
development and utilization of natural resources.
This provision was prompted by the perceived insufficiency of Filipino
capital and the felt need for foreign investments in the EDU
62
of minerals and petroleum resources.
The framers for the most part debated about the sort of safeguards
that would be considered adequate and reasonable. But
some of them, having more "radical" leanings, wanted to ban
service contracts altogether; for them, the provision would
permit aliens to exploit and benefit from the nation's natural
resources, which they felt should be reserved only for
Filipinos.
In the explanation of their votes, the individual commissioners were
heard by the entire body. They sounded off their individual
opinions, openly enunciated their philosophies, and
supported or attacked the provisions with fervor. Everyone's
viewpoint was heard.
In the final voting, the Article on the National Economy and Patrimony
including paragraph 4 allowing service contracts with
foreign corporations as an exception to the general norm in
paragraph 1 of Section 2 of the same article was
resoundingly approved by a vote of 32 to 7, with 2
abstentions.
Agreements Involving Technical or Financial Assistance AreService Contracts With Safeguards
From the foregoing, we are impelled to conclude that the phrase agreements involving either technical or
financial assistance, referred to in paragraph 4, are in fact service contracts. But unlike those of the 1973
variety, the new ones are between foreign corporations acting as contractors on the one hand; and on
the other, the government as principal or "owner" of the works. In the new service contracts, the foreign
contractors provide capital, technology and technical know-how, and managerial expertise in the creation
and operation of large-scale mining/extractive enterprises; and the government, through its agencies
(DENR, MGB), actively exercises control and supervision over the entire operation.
Such service contracts may be entered into only with respect to minerals, petroleum and other mineral
oils. The grant thereof is subject to several safeguards, among which are these requirements:
(1)The service contract shall be crafted in accordance with a general law
that will set standard or uniform terms, conditions and requirements,
presumably to attain a certain uniformity in provisions and avoid the
possible insertion of terms disadvantageous to the country.
(2)The President shall be the signatory for the government because,
supposedly before an agreement is presented to the President for
signature, it will have been vetted several times over at different levels to
ensure that it conforms to law and can withstand public scrutiny.
(3)Within thirty days of the executed agreement, the President shall report
it to Congress to give that branch of government an opportunity to look
over the agreement and interpose timely objections, if any.
Use of the Record of theConCom to Ascertain Intent
At this juncture, we shall address, rather than gloss over, the use of the "framers' intent" approach, and
the criticism hurled by petitioners who quote a ruling of this Court:
"While it is permissible in this jurisdiction to consult the debates and
proceedings of the constitutional convention in order to arrive at the
reason and purpose of the resulting Constitution, resort thereto may be
had only when other guides fail as said proceedings are powerless to vary
the terms of the Constitution when the meaning is clear. Debates in the
constitutional convention 'are of value as showing the views of the
individual members, and as indicating the reason for their votes, but they
give us no light as to the views of the large majority who did not talk,
much less the mass of our fellow citizens whose votes at the polls gave
that instrument the force of fundamental law. We think it safer to construe
the constitution from what appears upon its face.' The proper
interpretation therefore depends more on how it was understood by the
people adopting it than in the framers' understanding thereof ." 52

The notion that the deliberations reflect only the views of those members who spoke out and not the
views of the majority who remained silent should be clarified. We must never forget that those who
spoke out were heard by those who remained silent and did not react. If the latter were silent because
they happened not to be present at the time, they are presumed to have read the minutes and kept
abreast of the deliberations. By remaining silent, they are deemed to have signified their assent to and/or
conformity with at least some of the views propounded or their lack of objections thereto. It was
incumbent upon them, as representatives of the entire Filipino people, to follow the deliberations closely
and to speak their minds on the matter if they did not see eye to eye with the proponents of the draft
provisions.
In any event, each and every one of the commissioners had the opportunity to speak out and to vote on
the matter. Moreover, the individual explanations of votes are on record, and they show where each
delegate stood on the issues. In sum, we cannot completely denigrate the value or usefulness of the
record of the ConCom, simply because certain members chose not to speak out.
It is contented that the deliberations therein did not necessarily reflect the thinking of the voting
population that participated in the referendum and ratified the Constitution. Verily, whether we like it or
not, it is a bit too much to assume that every one of those who voted to ratify the proposed Charter did
so only after carefully reading and mulling over it, provision by provision.
Likewise, it appears rather extravagant to assume that every one of those who did in fact bother to read
the draft Charter actually understood the import of its provisions, much less analyzed it vis- -vis the
previous Constitutions. We believe that in reality, a good percentage of those who voted in favor of it did
so more out of faith and trust. For them, it was the product of the hard work and careful deliberation of a
group of intelligent, dedicated and trustworthy men and women of integrity and conviction, whose love of
country and fidelity to duty could not be questioned.
In short, a large proportion of the voters voted "yes" because the drafters, or a majority of them,
endorsed the proposed Constitution. What this fact translates to is the inescapable conclusion that many
of the voters in the referendum did not form their own isolated judgment about the draft Charter, much
less about particular provisions therein. They only relied or fell back and acted upon the favorable
endorsement or recommendation of the framers as a group. In other words, by voting yes, they may be
deemed to have signified their voluntary adoption of the understanding and interpretation of the
delegates with respect to the proposed Charter and its particular provisions. "If it's good enough for
them, it's good enough for me;" or, in many instances, "If it's good enough for President Cory Aquino, it's
good enough for me."
And even for those who voted based on their own individual assessment of the proposed Charter, there
is no evidence available to indicate that their assessment or understanding of its provisions was in fact
different from that of the drafters. This unwritten assumption seems to be petitioners' as well. For all we
know, this segment of voters must have read and understood the provisions of the Constitution in the
same way the framers had, an assumption that would account for the favorable votes.
Fundamentally speaking, in the process of rewriting the Charter, the members of the ConCom as a group
were supposed to represent the entire Filipino people. Thus, we cannot but regard their views as being
very much indicative of the thinking of the people with respect to the matters deliberated upon and to
the Charter as a whole.
63
It is therefore reasonable and unavoidable to make the following conclusion, based on the above
arguments. As written by the framers and ratified and adopted by the people, the Constitution allows the
continued use of service contracts with foreign corporations as contractors who would invest in and
operate and manage extractive enterprises, subject to the full control and supervision of the State
sans the abuses of the past regime. The purpose is clear: to develop and utilize our mineral, petroleum
and other resources on a large scale for the immediate and tangible benefit of the Filipino people.
In view of the foregoing discussion, we should reverse the Decision of January 27, 2004, and in fact now
hold a view different from that of the Decision, which had these findings: (a) paragraph 4 of Section 2 of
Article XII limits foreign involvement in the local mining industry to agreements strictly for either financial
or technical assistance only; (b) the same paragraph precludes agreements that grant to foreign
corporations the management of local mining operations, as such agreements are purportedly in the
nature of service contracts as these were understood under the 1973 Constitution; (c) these service
contracts were supposedly "de-constitutionalized" and proscribed by the omission of the term service
contracts from the 1987 Constitution; (d) since the WMCP FTAA contains provisions permitting the
foreign contractor to manage the concern, the said FTAA is invalid for being a prohibited service contract;
and (e) provisions of RA 7942 and DAO 96-40, which likewise grant managerial authority to the foreign
contractor, are also invalid and unconstitutional.
Ultimate Test: State's "Control" Determinative of Constitutionality
But we are not yet at the end of our quest. Far from it. It seems that we are confronted with a possible
collision of constitutional provisions. On the one hand, paragraph 1 of Section 2 of Article XII explicitly
mandates the State to exercise "full control and supervision" over the exploration, development and
utilization of natural resources. On the other hand, paragraph 4 permits safeguarded service contracts
with foreign contractors. Normally, pursuant thereto, the contractors exercise management prerogatives
over the mining operations and the enterprise as a whole. There is thus a legitimate ground to be
concerned that either the State's full control and supervision may rule out any exercise of management
authority by the foreign contractor; or, the other way around, allowing the foreign contractor full
management prerogatives may ultimately negate the State's full control and supervision.
Ut Magis Valeat Quam Pereat
Under the third principle of constitutional construction laid down in Francisco ut magis valeat quam
pereat every part of the Constitution is to be given effect, and the Constitution is to be read and
understood as a harmonious whole. Thus, "full control and supervision" by the State must be understood
as one that does not preclude the legitimate exercise of management prerogatives by the foreign
contractor. Before any further discussion, we must stress the primacy and supremacy of the principle of
sovereignty and State control and supervision over all aspects of exploration, development and utilization
of the country's natural resources, as mandated in the first paragraph of Section 2 of Article XII.
But in the next breadth we have to point out that "full control and supervision" cannot be taken literally
to mean that the State controls and supervises everything involved, down to the minutest details, and
makes all decisions required in the mining operations. This strained concept of control and supervision
over the mining enterprise would render impossible the legitimate exercise by the contractors of a
reasonable degree of management prerogative and authority necessary and indispensable to their proper
functioning.
For one thing, such an interpretation would discourage foreign entry into large-scale exploration,
development and utilization activities; and result in the unmitigated stagnation of this sector, to the
detriment of our nation's development. This scenario renders paragraph 4 inoperative and useless. And
as respondents have correctly pointed out, the government does not have to micro-manage the mining
operations and dip its hands into the day-to-day affairs of the enterprise in order for it to be considered
as having full control and supervision.
The concept of control 53 adopted in Section 2 of Article XII must be taken to mean less than dictatorial,
all-encompassing control; but nevertheless sufficient to give the State the power to direct, restrain,
regulate and govern the affairs of the extractive enterprises. Control by the State may be on a macro
level, through the establishment of policies, guidelines, regulations, industry standards and similar
measures that would enable the government to control the conduct of affairs in various enterprises and
restrain activities deemed not desirable or beneficial.
The end in view is ensuring that these enterprises contribute to the economic development and general
welfare of the country, conserve the environment, and uplift the well-being of the affected local
communities. Such a concept of control would be compatible with permitting the foreign contractor
sufficient and reasonable management authority over the enterprise it invested in, in order to ensure that
it is operating efficiently and profitably, to protect its investments and to enable it to succeed. TCHEDA
The question to be answered, then, is whether RA 7942 and its Implementing Rules enable the
government to exercise that degree of control sufficient to direct and regulate the conduct of affairs of
individual enterprises and restrain undesirable activities.
On the resolution of these questions will depend the validity and constitutionality of certain provisions of
the Philippine Mining Act of 1995 (RA 7942) and its Implementing Rules and Regulations (DAO 96-40), as
well as the WMCP FTAA.
Indeed, petitioners charge 54 that RA 7942, as well as its Implementing Rules and Regulations, makes it
possible for FTAA contracts to cede full control and management of mining enterprises over to fully
foreign-owned corporations, with the result that the State is allegedly reduced to a passive regulator
dependent on submitted plans and reports, with weak review and audit powers. The State does not
supposedly act as the owner of the natural resources for and on behalf of the Filipino people; it
practically has little effective say in the decisions made by the enterprise. Petitioners then conclude that
the law, the implementing regulations, and the WMCP FTAA cede "beneficial ownership" of the mineral
resources to the foreign contractor.

A careful scrutiny of the provisions of RA 7942 and its Implementing Rules belies petitioners' claims.
Paraphrasing the Constitution, Section 4 of the statute clearly affirms the State's control thus:
"Sec. 4.Ownership of Mineral Resources. Mineral resources are owned
by the State and the exploration, development, utilization and processing
thereof shall be under its full control and supervision. The State may
directly undertake such activities or it may enter into mineral agreements
with contractors.
"The State shall recognize and protect the rights of the indigenous cultural
communities to their ancestral lands as provided for by the Constitution."
The aforequoted provision is substantively reiterated in Section 2 of DAO 96-40 as follows:
"Sec. 2.Declaration of Policy. All mineral resources in public and private
lands within the territory and exclusive economic zone of the Republic of
the Philippines are owned by the State. It shall be the responsibility of the
State to promote their rational exploration, development, utilization and
conservation through the combined efforts of the Government and private
sector in order to enhance national growth in a way that effectively
safeguards the environment and protects the rights of affected
communities."
Sufficient Control Over MiningOperations Vested in the Stateby RA 7942 and DAO 96-40
RA 7942 provides for the State's control and supervision over mining operations. The following provisions
thereof establish the mechanism of inspection and visitorial rights over mining operations and institute
reportorial requirements in this manner:
1.Sec. 8 which provides for the DENR's power of over-all supervision and
periodic review for "the conservation, management,
development and proper use of the State's mineral
resources";
64
2.Sec. 9 which authorizes the Mines and Geosciences Bureau (MGB) under
the DENR to exercise "direct charge in the administration and
disposition of mineral resources", and empowers the MGB to
"monitor the compliance by the contractor of the terms and
conditions of the mineral agreements", "confiscate surety and
performance bonds", and deputize whenever necessary any
member or unit of the Phil. National Police, barangay, duly
registered non-governmental organization (NGO) or any
qualified person to police mining activities;
3.Sec. 66 which vests in the Regional Director "exclusive jurisdiction over
safety inspections of all installations, whether surface or
underground", utilized in mining operations.
4.Sec. 35, which incorporates into all FTAAs the following terms, conditions
and warranties:
"(g)Mining operations shall be conducted in accordance with
the provisions of the Act and its IRR.
"(h)Work programs and minimum expenditures
commitments.
xxx xxx xxx
"(k)Requiring proponent to effectively use appropriate anti-
pollution technology and facilities to protect the
environment and restore or rehabilitate mined-
out areas.
"(l)The contractors shall furnish the Government records of
geologic, accounting and other relevant data
for its mining operation, and that books of
accounts and records shall be open for
inspection by the government. . . .
"(m)Requiring the proponent to dispose of the minerals at
the highest price and more advantageous terms
and conditions.
"(n) . . .
"(o)Such other terms and conditions consistent with the
Constitution and with this Act as the Secretary
may deem to be for the best interest of the
State and the welfare of the Filipino people."
The foregoing provisions of Section 35 of RA 7942 are also reflected and implemented in Section 56 (g),
(h), (l), (m) and (n) of the Implementing Rules, DAO 96-40.
Moreover, RA 7942 and DAO 96-40 also provide various stipulations confirming the government's control
over mining enterprises:
The contractor is to relinquish to the government those portions of the
contract area not needed for mining operations and not
covered by any declaration of mining feasibility (Section 35-e,
RA 7942; Section 60, DAO 96-40).
The contractor must comply with the provisions pertaining to mine
safety, health and environmental protection (Chapter XI, RA
7942; Chapters XV and XVI, DAO 96-40).
For violation of any of its terms and conditions, government may
cancel an FTAA. (Chapter XVII, RA 7942; Chapter XXIV, DAO
96-40).
An FTAA contractor is obliged to open its books of accounts and
records for inspection by the government (Section 56-m,
DAO 96-40).
An FTAA contractor has to dispose of the minerals and by-products at
the highest market price and register with the MGB a copy of
the sales agreement (Section 56-n, DAO 96-40).
MGB is mandated to monitor the contractor's compliance with the
terms and conditions of the FTAA; and to deputize, when
necessary, any member or unit of the Philippine National
Police, the barangay or a DENR-accredited non-governmental
organization to police mining activities (Section 7-d and -f,
DAO 96-40).
An FTAA cannot be transferred or assigned without prior approval by
the President (Section 40, RA 7942; Section 66, DAO 96-40).
A mining project under an FTAA cannot proceed to the
construction/development/utilization stage, unless its
Declaration of Mining Project Feasibility has been approved
by government (Section 24, RA 7942).
The Declaration of Mining Project Feasibility filed by the contractor
cannot be approved without submission of the following
documents:
1.Approved mining project feasibility study (Section 53-d,
DAO 96-40)
2.Approved three-year work program (Section 53-a-4, DAO
96-40)
3.Environmental compliance certificate (Section 70, RA 7942)
4.Approved environmental protection and enhancement
program (Section 69, RA 7942)
5.Approval by the Sangguniang
Panlalawigan/Bayan/Barangay (Section 70, RA
7942; Section 27, RA 7160)
6.Free and prior informed consent by the indigenous peoples
concerned, including payment of royalties
through a Memorandum of Agreement (Section
16, RA 7942; Section 59, RA 8371)
The FTAA contractor is obliged to assist in the development of its
mining community, promotion of the general welfare of its
inhabitants, and development of science and mining
technology (Section 57, RA 7942).
The FTAA contractor is obliged to submit reports (on quarterly, semi-
65
annual or annual basis as the case may be; per Section 270,
DAO 96-40), pertaining to the following:
1.Exploration
2.Drilling
3.Mineral resources and reserves
4.Energy consumption
5.Production
6.Sales and marketing
7.Employment
8.Payment of taxes, royalties, fees and other Government
Shares
9.Mine safety, health and environment
10.Land use
11.Social development
12.Explosives consumption
An FTAA pertaining to areas within government reservations cannot be
granted without a written clearance from the government
agencies concerned (Section 19, RA 7942; Section 54, DAO
96-40).
An FTAA contractor is required to post a financial guarantee bond in
favor of the government in an amount equivalent to its
expenditures obligations for any particular year. This
requirement is apart from the representations and warranties
of the contractor that it has access to all the financing,
managerial and technical expertise and technology necessary
to carry out the objectives of the FTAA (Section 35-b, -e, and
-f, RA 7942).
Other reports to be submitted by the contractor, as required under
DAO 96-40, are as follows: an environmental report on the
rehabilitation of the mined-out area and/or mine
waste/tailing covered area, and anti-pollution measures
undertaken (Section 35-a-2); annual reports of the mining
operations and records of geologic accounting (Section 56-
m); annual progress reports and final report of exploration
activities (Section 56-2).
Other programs required to be submitted by the contractor, pursuant
to DAO 96-40, are the following: a safety and health program
(Section 144); an environmental work program (Section
168); an annual environmental protection and enhancement
program (Section 171).
The foregoing gamut of requirements, regulations, restrictions and limitations imposed upon the FTAA
contractor by the statute and regulations easily overturn petitioners' contention. The setup under RA
7942 and DAO 96-40 hardly relegates the State to the role of a "passive regulator" dependent on
submitted plans and reports. On the contrary, the government agencies concerned are empowered to
approve or disapprove hence, to influence, direct and change the various work programs and the
corresponding minimum expenditure commitments for each of the exploration, development and
utilization phases of the mining enterprise.
Once these plans and reports are approved, the contractor is bound to comply with its commitments
therein. Figures for mineral production and sales are regularly monitored and subjected to government
review, in order to ensure that the products and by-products are disposed of at the best prices possible;
even copies of sales agreements have to be submitted to and registered with MGB. And the contractor is
mandated to open its books of accounts and records for scrutiny, so as to enable the State to determine
if the government share has been fully paid.
The State may likewise compel the contractor's compliance with mandatory requirements on mine safety,
health and environmental protection, and the use of anti-pollution technology and facilities. Moreover,
the contractor is also obligated to assist in the development of the mining community and to pay royalties
to the indigenous peoples concerned.
Cancellation of the FTAA may be the penalty for violation of any of its terms and conditions and/or
noncompliance with statutes or regulations. This general, all-around, multipurpose sanction is no trifling
matter, especially to a contractor who may have yet to recover the tens or hundreds of millions of dollars
sunk into a mining project.
Overall, considering the provisions of the statute and the regulations just discussed, we believe that the
State definitely possesses the means by which it can have the ultimate word in the operation of the
enterprise, set directions and objectives, and detect deviations and noncompliance by the contractor;
likewise, it has the capability to enforce compliance and to impose sanctions, should the occasion
therefor arise.

In other words, the FTAA contractor is not free to do whatever it pleases and get away with it; on the
contrary, it will have to follow the government line if it wants to stay in the enterprise. Ineluctably then,
RA 7942 and DAO 96-40 vest in the government more than a sufficient degree of control and supervision
over the conduct of mining operations.
Section 3(aq) of RA 7942Not Unconstitutional
An objection has been expressed that Section 3(aq) 55 of RA 7942 which allows a foreign contractor
to apply for and hold an exploration permit is unconstitutional. The reasoning is that Section 2 of
Article XII of the Constitution does not allow foreign-owned corporations to undertake mining operations
directly. They may act only as contractors of the State under an FTAA; and the State, as the party
directly undertaking exploitation of its natural resources, must hold through the government all
exploration permits and similar authorizations. Hence, Section 3(aq), in permitting foreign-owned
corporations to hold exploration permits, is unconstitutional. ESCacI
The objection, however, is not well-founded. While the Constitution mandates the State to exercise full
control and supervision over the exploitation of mineral resources, nowhere does it require the
government to hold all exploration permits and similar authorizations. In fact, there is no prohibition at all
against foreign or local corporations or contractors holding exploration permits. The reason is not hard to
see.
Pursuant to Section 20 of RA 7942, an exploration permit merely grants to a qualified person the right to
conduct exploration for all minerals in specified areas. Such a permit does not amount to an authorization
to extract and carry off the mineral resources that may be discovered. This phase involves nothing but
expenditures for exploring the contract area and locating the mineral bodies. As no extraction is involved,
there are no revenues or incomes to speak of. In short, the exploration permit is an authorization for the
grantee to spend its own funds on exploration programs that are pre-approved by the government,
without any right to recover anything should no minerals in commercial quantities be discovered. The
State risks nothing and loses nothing by granting these permits to local or foreign firms; in fact, it stands
66
to gain in the form of data generated by the exploration activities.
Pursuant to Section 24 of RA 7942, an exploration permit grantee who determines the commercial
viability of a mining area may, within the term of the permit, file with the MGB a declaration of mining
project feasibility accompanied by a work program for development. The approval of the mining project
feasibility and compliance with other requirements of RA 7942 vests in the grantee the exclusive right to
an MPSA or an other mineral agreement, or to an FTAA.
Thus, the permit grantee may apply for an MPSA, a joint venture agreement, a co-production agreement,
or an FTAA over the permit area, and the application shall be approved if the permit grantee meets the
necessary qualifications and the terms and conditions of any such agreement. Therefore, the contractor
will be in a position to extract minerals and earn revenues only when the MPSA or another mineral
agreement, or an FTAA, is granted. At that point, the contractor's rights and obligations will be covered
by an FTAA or a mineral agreement.
But prior to the issuance of such FTAA or mineral agreement, the exploration permit grantee (or
prospective contractor) cannot yet be deemed to have entered into any contract or agreement with the
State, and the grantee would definitely need to have some document or instrument as evidence of its
right to conduct exploration works within the specified area. This need is met by the exploration permit
issued pursuant to Sections 3(aq), 20 and 23 of RA 7942.
In brief, the exploration permit serves a practical and legitimate purpose in that it protects the interests
and preserves the rights of the exploration permit grantee (the would-be contractor) foreign or local
during the period of time that it is spending heavily on exploration works, without yet being able to
earn revenues to recoup any of its investments and expenditures. Minus this permit and the protection it
affords, the exploration works and expenditures may end up benefiting only claim-jumpers. Such a
possibility tends to discourage investors and contractors. Thus, Section 3(aq) of RA 7942 may not be
deemed unconstitutional.
The Terms of the WMCP FTAAA Deference to State Control
A perusal of the WMCP FTAA also reveals a slew of stipulations providing for State control and
supervision:
1.The contractor is obligated to account for the value of production and
sale of minerals (Clause 1.4).
2.The contractor's work program, activities and budgets must be approved
by/on behalf of the State (Clause 2.1).
3.The DENR secretary has the power to extend the exploration period
(Clause 3.2-a).
4.Approval by the State is necessary for incorporating lands into the FTAA
contract area (Clause 4.3-c).
5.The Bureau of Forest Development is vested with discretion in regard to
approving the inclusion of forest reserves as part of the FTAA
contract area (Clause 4.5).
6.The contractor is obliged to relinquish periodically parts of the contract
area not needed for exploration and development (Clause
4.6).
7.A Declaration of Mining Feasibility must be submitted for approval by the
State (Clause 4.6-b).
8.The contractor is obligated to report to the State its exploration activities
(Clause 4.9).
9.The contractor is required to obtain State approval of its work programs
for the succeeding two-year periods, containing the proposed
work activities and expenditures budget related to
exploration (Clause 5.1).
10.The contractor is required to obtain State approval for its proposed
expenditures for exploration activities (Clause 5.2).
11.The contractor is required to submit an annual report on geological,
geophysical, geochemical and other information relating to its
explorations within the FTAA area (Clause 5.3-a).
12,The contractor is to submit within six months after expiration of
exploration period a final report on all its findings in the
contract area (Clause 5.3-b).
13.The contractor, after conducting feasibility studies, shall submit a
declaration of mining feasibility, along with a description of
the area to be developed and mined, a description of the
proposed mining operations and the technology to be
employed, and a proposed work program for the
development phase, for approval by the DENR secretary
(Clause 5.4).
14.The contractor is obliged to complete the development of the mine,
including construction of the production facilities, within the
period stated in the approved work program (Clause 6.1).
15.The contractor is obligated to submit for approval of the DENR
secretary a work program covering each period of three fiscal
years (Clause 6.2).
16.The contractor is to submit reports to the DENR secretary on the
production, ore reserves, work accomplished and work in
progress, profile of its work force and management staff, and
other technical information (Clause 6.3).
17.Any expansions, modifications, improvements and replacements of
mining facilities shall be subject to the approval of the
secretary (Clause 6.4).
18.The State has control with respect to the amount of funds that the
contractor may borrow within the Philippines (Clause 7.2).
19.The State has supervisory power with respect to technical, financial and
marketing issues (Clause 10.1-a).
20.The contractor is required to ensure 60 percent Filipino equity in the
contractor, within ten years of recovering specified
expenditures, unless not so required by subsequent
legislation (Clause 10.1).
21.The State has the right to terminate the FTAA for the contractor's
unremedied substantial breach thereof (Clause 13.2);
22.The State's approval is needed for any assignment of the FTAA by the
contractor to an entity other than an affiliate (Clause 14.1).
We should elaborate a little on the work programs and budgets, and what they mean with respect to the
67
State's ability to exercise full control and effective supervision over the enterprise. For instance,
throughout the initial five-year exploration and feasibility phase of the project, the contractor is mandated
by Clause 5.1 of the WMCP FTAA to submit a series of work programs (copy furnished the director of
MGB) to the DENR secretary for approval. The programs will detail the contractor's proposed exploration
activities and budget covering each subsequent period of two fiscal years.
In other words, the concerned government officials will be informed beforehand of the proposed
exploration activities and expenditures of the contractor for each succeeding two-year period, with the
right to approve/disapprove them or require changes or adjustments therein if deemed necessary.
Likewise, under Clause 5.2(a), the amount that the contractor was supposed to spend for exploration
activities during the first contract year of the exploration period was fixed at not less than P24 million;
and then for the succeeding years, the amount shall be as agreed between the DENR secretary and the
contractor prior to the commencement of each subsequent fiscal year. If no such agreement is arrived
upon, the previous year's expenditure commitment shall apply.
This provision alone grants the government through the DENR secretary a very big say in the exploration
phase of the project. This fact is not something to be taken lightly, considering that the government has
absolutely no contribution to the exploration expenditures or work activities and yet is given veto power
over such a critical aspect of the project. We cannot but construe as very significant such a degree of
control over the project and, resultantly, over the mining enterprise itself.
Following its exploration activities or feasibility studies, if the contractor believes that any part of the
contract area is likely to contain an economic mineral resource, it shall submit to the DENR secretary a
declaration of mining feasibility (per Clause 5.4 of the FTAA), together with a technical description of the
area delineated for development and production, a description of the proposed mining operations
including the technology to be used, a work program for development, an environmental impact
statement, and a description of the contributions to the economic and general welfare of the country to
be generated by the mining operations (pursuant to Clause 5.5).

The work program for development is subject to the approval of the DENR secretary. Upon its approval,
the contractor must comply with it and complete the development of the mine, including the construction
of production facilities and installation of machinery and equipment, within the period provided in the
approved work program for development (per Clause 6.1).
Thus, notably, the development phase of the project is likewise subject to the control and supervision of
the government. It cannot be emphasized enough that the proper and timely construction and
deployment of the production facilities and the development of the mine are of pivotal significance to the
success of the mining venture. Any missteps here will potentially be very costly to remedy. Hence, the
submission of the work program for development to the DENR secretary for approval is particularly
noteworthy, considering that so many millions of dollars worth of investments courtesy of the
contractor are made to depend on the State's consideration and action.
Throughout the operating period, the contractor is required to submit to the DENR secretary for approval,
copy furnished the director of MGB, work programs covering each period of three fiscal years (per Clause
6.2). During the same period (per Clause 6.3), the contractor is mandated to submit various quarterly
and annual reports to the DENR secretary, copy furnished the director of MGB, on the tonnages of
production in terms of ores and concentrates, with corresponding grades, values and destinations;
reports of sales; total ore reserves, total tonnage of ores, work accomplished and work in progress
(installations and facilities related to mining operations), investments made or committed, and so on and
so forth.
Under Section VIII, during the period of mining operations, the contractor is also required to submit to
the DENR secretary (copy furnished the director of MGB) the work program and corresponding budget for
the contract area, describing the mining operations that are proposed to be carried out during the period
covered. The secretary is, of course, entitled to grant or deny approval of any work program or budget
and/or propose revisions thereto. Once the program/budget has been approved, the contractor shall
comply therewith.
In sum, the above provisions of the WMCP FTAA taken together, far from constituting a surrender of
control and a grant of beneficial ownership of mineral resources to the contractor in question, bestow
upon the State more than adequate control and supervision over the activities of the contractor and the
enterprise.
No Surrender of Control Under the WMCP FTAA
Petitioners, however, take aim at Clause 8.2, 8.3, and 8.5 of the WMCP FTAA which, they say, amount to
a relinquishment of control by the State, since it "cannot truly impose its own discretion" in respect of the
submitted work programs.
"8.2.The Secretary shall be deemed to have approved any Work
Programme or Budget or variation thereof submitted by the
Contractor unless within sixty (60) days after submission by
the Contractor the Secretary gives notice declining such
approval or proposing a revision of certain features and
specifying its reasons therefor ('the Rejection Notice').
8.3.If the Secretary gives a Rejection Notice, the Parties shall promptly
meet and endeavor to agree on amendments to the Work
Programme or Budget. If the Secretary and the Contractor
fail to agree on the proposed revision within 30 days from
delivery of the Rejection Notice then the Work Programme or
Budget or variation thereof proposed by the Contractor shall
be deemed approved, so as not to unnecessarily delay the
performance of the Agreement.
8.4. . . .
8.5.So far as is practicable, the Contractor shall comply with any approved
Work Programme and Budget. It is recognized by the
Secretary and the Contractor that the details of any Work
Programmes or Budgets may require changes in the light of
changing circumstances. The Contractor may make such
changes without approval of the Secretary provided they do
not change the general objective of any Work Programme,
nor entail a downward variance of more than twenty per
centum (20 percent) of the relevant Budget. All other
variations to an approved Work Programme or Budget shall
be submitted for approval of the Secretary." AcSEHT
From the provisions quoted above, petitioners generalize by asserting that the government does not
participate in making critical decisions regarding the operations of the mining firm. Furthermore, while
the State can require the submission of work programs and budgets, the decision of the contractor will
still prevail, if the parties have a difference of opinion with regard to matters affecting operations and
management.
We hold, however, that the foregoing provisions do not manifest a relinquishment of control. For
instance, Clause 8.2 merely provides a mechanism for preventing the business or mining operations from
grinding to a complete halt as a result of possibly over-long and unjustified delays in the government's
handling, processing and approval of submitted work programs and budgets. Anyway, the provision does
give the DENR secretary more than sufficient time (60 days) to react to submitted work programs and
budgets. It cannot be supposed that proper grounds for objecting thereto, if any exist, cannot be
discovered within a period of two months.
On the other hand, Clause 8.3 seeks to provide a temporary, stop-gap solution in the event a
disagreement over the submitted work program or budget arises between the State and the contractor
and results in a stalemate or impasse, in order that there will be no unreasonably long delays in the
68
performance of the works.
These temporary or stop-gap solutions are not necessarily evil or wrong. Neither does it follow that the
government will inexorably be aggrieved if and when these temporary remedies come into play. First,
avoidance of long delays in these situations will undoubtedly redound to the benefit of the State as well
as the contractor. Second, who is to say that the work program or budget proposed by the contractor
and deemed approved under Clause 8.3 would not be the better or more reasonable or more effective
alternative? The contractor, being the "insider," as it were, may be said to be in a better position than
the State an outsider looking in to determine what work program or budget would be
appropriate, more effective, or more suitable under the circumstances.
All things considered, we take exception to the characterization of the DENR secretary as a subservient
nonentity whom the contractor can overrule at will, on account of Clause 8.3. And neither is it true that
under the same clause, the DENR secretary has no authority whatsoever to disapprove the work
program. As Respondent WMCP reasoned in its Reply-Memorandum, the State despite Clause 8.3
still has control over the contract area and it may, as sovereign authority, prohibit work thereon until the
dispute is resolved. And ultimately, the State may terminate the agreement, pursuant to Clause 13.2 of
the same FTAA, citing substantial breach thereof. Hence, it clearly retains full and effective control of the
exploitation of the mineral resources.
On the other hand, Clause 8.5 is merely an acknowledgment of the parties' need for flexibility, given that
no one can accurately forecast under all circumstances, or predict how situations may change. Hence,
while approved work programs and budgets are to be followed and complied with as far as practicable,
there may be instances in which changes will have to be effected, and effected rapidly, since events may
take shape and unfold with suddenness and urgency. Thus, Clause 8.5 allows the contractor to move
ahead and make changes without the express or implicit approval of the DENR secretary. Such changes
are, however, subject to certain conditions that will serve to limit or restrict the variance and prevent the
contractor from straying very far from what has been approved.
Clause 8.5 provides the contractor a certain amount of flexibility to meet unexpected situations, while still
guaranteeing that the approved work programs and budgets are not abandoned altogether. Clause 8.5
does not constitute proof that the State has relinquished control. And ultimately, should there be
disagreement with the actions taken by the contractor in this instance as well as under Clause 8.3
discussed above, the DENR secretary may resort to cancellation/termination of the FTAA as the ultimate
sanction.
Discretion to Select Contract Area Not an Abdication of Control
Next, petitioners complain that the contractor has full discretion to select and the government has no
say whatsoever as to the parts of the contract area to be relinquished pursuant to Clause 4.6 of the
WMCP FTAA. 56 This clause, however, does not constitute abdication of control. Rather, it is a mere
acknowledgment of the fact that the contractor will have determined, after appropriate exploration
works, which portions of the contract area do not contain minerals in commercial quantities sufficient to
justify developing the same and ought therefore to be relinquished. The State cannot just substitute its
judgment for that of the contractor and dictate upon the latter which areas to give up.
Moreover, we can be certain that the contractor's self-interest will propel proper and efficient
relinquishment. According to private respondent, 57 a mining company tries to relinquish as much non-
mineral areas as soon as possible, because the annual occupation fees paid to the government are based
on the total hectarage of the contract area, net of the areas relinquished. Thus, the larger the remaining
area, the heftier the amount of occupation fees to be paid by the contractor. Accordingly, relinquishment
is not an issue, given that the contractor will not want to pay the annual occupation fees on the non-
mineral parts of its contract area. Neither will it want to relinquish promising sites, which other
contractors may subsequently pick up.

Government Not a Subcontractor
Petitioners further maintain that the contractor can compel the government to exercise its power of
eminent domain to acquire surface areas within the contract area for the contractor's use. Clause 10.2
(e) of the WMCP FTAA provides that the government agrees that the contractor shall "(e) have the right
to require the Government at the Contractor's own cost, to purchase or acquire surface areas for and on
behalf of the Contractor at such price and terms as may be acceptable to the contractor. At the
termination of this Agreement such areas shall be sold by public auction or tender and the Contractor
shall be entitled to reimbursement of the costs of acquisition and maintenance, adjusted for inflation,
from the proceeds of sale." HETDAC
According to petitioners, "government becomes a subcontractor to the contractor" and may, on account
of this provision, be compelled "to make use of its power of eminent domain, not for public purposes but
on behalf of a private party, i.e., the contractor." Moreover, the power of the courts to determine the
amount corresponding to the constitutional requirement of just compensation has allegedly also been
contracted away by the government, on account of the latter's commitment that the acquisition shall be
at such terms as may be acceptable to the contractor.
However, private respondent has proffered a logical explanation for the provision. 58 Section 10.2(e)
contemplates a situation applicable to foreign-owned corporations. WMCP, at the time of the execution of
the FTAA, was a foreign-owned corporation and therefore not qualified to own land. As contractor, it has
at some future date to construct the infrastructure the mine processing plant, the camp site, the
tailings dam, and other infrastructure needed for the large-scale mining operations. It will then have
to identify and pinpoint, within the FTAA contract area, the particular surface areas with favorable
topography deemed ideal for such infrastructure and will need to acquire the surface rights. The State
owns the mineral deposits in the earth, and is also qualified to own land.
Section 10.2(e) sets forth the mechanism whereby the foreign-owned contractor, disqualified to own
land, identifies to the government the specific surface areas within the FTAA contract area to be acquired
for the mine infrastructure. The government then acquires ownership of the surface land areas on behalf
of the contractor, in order to enable the latter to proceed to fully implement the FTAA.
The contractor, of course, shoulders the purchase price of the land. Hence, the provision allows it, after
termination of the FTAA, to be reimbursed from proceeds of the sale of the surface areas, which the
government will dispose of through public bidding. It should be noted that this provision will not be
applicable to Sagittarius as the present FTAA contractor, since it is a Filipino corporation qualified to own
and hold land. As such, it may therefore freely negotiate with the surface rights owners and acquire the
surface property in its own right.
Clearly, petitioners have needlessly jumped to unwarranted conclusions, without being aware of the
rationale for the said provision. That provision does not call for the exercise of the power of eminent
domain and determination of just compensation is not an issue as much as it calls for a qualified
party to acquire the surface rights on behalf of a foreign-owned contractor.
Rather than having the foreign contractor act through a dummy corporation, having the State do the
purchasing is a better alternative. This will at least cause the government to be aware of such
transaction/s and foster transparency in the contractor's dealings with the local property owners. The
government, then, will not act as a subcontractor of the contractor; rather, it will facilitate the transaction
and enable the parties to avoid a technical violation of the Anti-Dummy Law.
Absence of ProvisionRequiring Sale at PostedPrices Not Problematic
The supposed absence of any provision in the WMCP FTAA directly and explicitly requiring the contractor
to sell the mineral products at posted or market prices is not a problem. Apart from Clause 1.4 of the
FTAA obligating the contractor to account for the total value of mineral production and the sale of
minerals, we can also look to Section 35 of RA 7942, which incorporates into all FTAAs certain terms,
conditions and warranties, including the following:
"(l)The contractors shall furnish the Government records of geologic,
accounting and other relevant data for its mining operation,
and that books of accounts and records shall be open for
inspection by the government. . . .
69
(m)Requiring the proponent to dispose of the minerals at the highest price
and more advantageous terms and conditions."
For that matter, Section 56(n) of DAO 99-56 specifically obligates an FTAA contractor to dispose of the
minerals and by-products at the highest market price and to register with the MGB a copy of the sales
agreement. After all, the provisions of prevailing statutes as well as rules and regulations are deemed
written into contracts.
Contractor's Right to MortgageNot Objectionable Per Se
Petitioners also question the absolute right of the contractor under Clause 10.2 (l) to mortgage and
encumber not only its rights and interests in the FTAA and the infrastructure and improvements
introduced, but also the mineral products extracted. Private respondents do not touch on this matter, but
we believe that this provision may have to do with the conditions imposed by the creditor-banks of the
then foreign contractor WMCP to secure the lendings made or to be made to the latter. Ordinarily, banks
lend not only on the security of mortgages on fixed assets, but also on encumbrances of goods produced
that can easily be sold and converted into cash that can be applied to the repayment of loans. Banks
even lend on the security of accounts receivable that are collectible within 90 days. 59
It is not uncommon to find that a debtor corporation has executed deeds of assignment "by way of
security" over the production for the next twelve months and/or the proceeds of the sale thereof or
the corresponding accounts receivable, if sold on terms in favor of its creditor-banks. Such deeds
may include authorizing the creditors to sell the products themselves and to collect the sales proceeds
and/or the accounts receivable.
Seen in this context, Clause 10.2(l) is not something out of the ordinary or objectionable. In any case, as
will be explained below, even if it is allowed to mortgage or encumber the mineral end-products
themselves, the contractor is not freed of its obligation to pay the government its basic and additional
shares in the net mining revenue, which is the essential thing to consider.
In brief, the alarumraised over the contractor's right to mortgage the minerals is simply unwarranted.
Just the same, the contractor must account for the value of mineral production and the sales proceeds
therefrom. Likewise, under the WMCP FTAA, the government remains entitled to its sixty percent share in
the net mining revenues of the contractor. The latter's right to mortgage the minerals does not negate
the State's right to receive its share of net mining revenues.
Shareholders Freeto Sell Their Stocks
Petitioners likewise criticize Clause 10.2(k), which gives the contractor authority "to change its equity
structure at any time." This provision may seem somewhat unusual, but considering that WMCP then was
100 percent foreign-owned, any change would mean that such percentage would either stay unaltered or
be decreased in favor of Filipino ownership. Moreover, the foreign-held shares may change hands freely.
Such eventuality is as it should be.
We believe it is not necessary for government to attempt to limit or restrict the freedom of the
shareholders in the contractor to freely transfer, dispose of or encumber their shareholdings, consonant
with the unfettered exercise of their business judgment and discretion. Rather, what is critical is that,
regardless of the identity, nationality and percentage ownership of the various shareholders of the
contractor and regardless of whether these shareholders decide to take the company public, float
bonds and other fixed-income instruments, or allow the creditor-banks to take an equity position in the
company the foreign-owned contractor is always in a position to render the services required under
the FTAA, under the direction and control of the government.
Contractor's Right to AskFor Amendment Not Absolute
With respect to Clauses 10.4(e) and (i), petitioners complain that these provisions bind government to
allow amendments to the FTAA if required by banks and other financial institutions as part of the
conditions for new lendings. However, we do not find anything wrong with Clause 10.4(e), which only
states that "if the Contractor seeks to obtain financing contemplated herein from banks or other financial
institutions, (the Government shall) cooperate with the Contractor in such efforts provided that such
financing arrangements will in no event reduce the Contractor's obligations or the Government's rights
hereunder." The colatilla obviously safeguards the State's interests; if breached, it will give the
government cause to object to the proposed amendments.
On the other hand, Clause 10.4(i) provides that "the Government shall favourably consider any request
from [the] Contractor for amendments of this Agreement which are necessary in order for the Contractor
to successfully obtain the financing." Petitioners see in this provision a complete renunciation of control.
We disagree.
The proviso does not say that the government shall grant any request for amendment. Clause 10.4(i)
only obliges the State to favorably consider any such request, which is not at all unreasonable, as it is not
equivalent to saying that the government must automatically consent to it. This provision should be read
together with the rest of the FTAA provisions instituting government control and supervision over the
mining enterprise. The clause should not be given an interpretation that enables the contractor to wiggle
out of the restrictions imposed upon it by merely suggesting that certain amendments are requested by
the lenders.

Rather, it is up to the contractor to prove to the government that the requested changes to the FTAA are
indispensable, as they enable the contractor to obtain the needed financing; that without such contract
changes, the funders would absolutely refuse to extend the loan; that there are no other sources of
financing available to the contractor (a very unlikely scenario); and that without the needed financing,
the execution of the work programs will not proceed. But the bottom line is, in the exercise of its power
of control, the government has the final say on whether to approve or disapprove such requested
amendments to the FTAA. In short, approval thereof is not mandatory on the part of the government.
In fine, the foregoing evaluation and analysis of the aforementioned FTAA provisions sufficiently
overturns petitioners' litany of objections to and criticisms of the State's alleged lack of control.
Financial Benefits Not Surrendered to the Contractor
One of the main reasons certain provisions of RA 7942 were struck down was the finding mentioned in
the Decision that beneficial ownership of the mineral resources had been conveyed to the contractor.
This finding was based on the underlying assumption, common to the said provisions, that the foreign
contractor manages the mineral resources in the same way that foreign contractors in service contracts
used to. "By allowing foreign contractors to manage or operate all the aspects of the mining operation,
the above-cited provisions of R.A. No. 7942 have in effect conveyed beneficial ownership over the
nation's mineral resources to these contractors, leaving the State with nothing but bare title thereto." 60
As the WMCP FTAA contained similar provisions deemed by the ponente to be abhorrent to the
Constitution, the Decision struck down the Contract as well. TSDHCc
Beneficial ownership has been defined as ownership recognized by law and capable of being enforced in
the courts at the suit of the beneficial owner. 61 Black's Law Dictionary indicates that the term is used in
two senses: first, to indicate the interest of a beneficiary in trust property (also called "equitable
ownership"); and second, to refer to the power of a corporate shareholder to buy or sell the shares,
though the shareholder is not registered in the corporation's books as the owner. 62 Usually, beneficial
ownership is distinguished from naked ownership, which is the enjoyment of all the benefits and
privileges of ownership, as against possession of the bare title to property.
An assiduous examination of the WMCP FTAA uncovers no indication that it confers upon WMCP
ownership, beneficial or otherwise, of the mining property it is to develop, the minerals to be produced,
or the proceeds of their sale, which can be legally asserted and enforced as against the State.
As public respondents correctly point out, any interest the contractor may have in the proceeds of the
mining operation is merely the equivalent of the consideration the government has undertaken to pay for
its services. All lawful contracts require such mutual prestations, and the WMCP FTAA is no different. The
contractor commits to perform certain services for the government in respect of the mining operation,
and in turn it is to be compensated out of the net mining revenues generated from the sale of mineral
products. What would be objectionable is a contractual provision that unduly benefits the contractor far
70
in excess of the service rendered or value delivered, if any, in exchange therefor.
A careful perusal of the statute itself and its implementing rules reveals that neither RA 7942 nor DAO
99-56 can be said to convey beneficial ownership of any mineral resource or product to any foreign FTAA
contractor.
Equitable Sharingof Financial Benefits
On the contrary, DAO 99-56, entitled "Guidelines Establishing the Fiscal Regime of Financial or Technical
Assistance Agreements" aims to ensure an equitable sharing of the benefits derived from mineral
resources. These benefits are to be equitably shared among the government (national and local), the
FTAA contractor, and the affected communities. The purpose is to ensure sustainable mineral resources
development; and a fair, equitable, competitive and stable investment regime for the large-scale
exploration, development and commercial utilization of minerals. The general framework or concept
followed in crafting the fiscal regime of the FTAA is based on the principle that the government expects
real contributions to the economic growth and general welfare of the country, while the contractor
expects a reasonable return on its investments in the project. 63
Specifically, under the fiscal regime, the government's expectation is, inter alia, the receipt of its share
from the taxes and fees normally paid by a mining enterprise. On the other hand, the FTAA contractor is
granted by the government certain fiscal and non-fiscal incentives 64 to help support the former's cash
flow during the most critical phase (cost recovery) and to make the Philippines competitive with other
mineral-producing countries. After the contractor has recovered its initial investment, it will pay all the
normal taxes and fees comprising the basic share of the government, plus an additional share for the
government based on the options and formulae set forth in DAO 99-56.
The said DAO spells out the financial benefits the government will receive from an FTAA, referred to as
"the Government Share," composed of a basic government share and an additional government share.
The basic government share is comprised of all direct taxes, fees and royalties, as well as other payments
made by the contractor during the term of the FTAA. These are amounts paid directly to (i) the national
government (through the Bureau of Internal Revenue, Bureau of Customs, Mines & Geosciences Bureau
and other national government agencies imposing taxes or fees), (ii) the local government units where
the mining activity is conducted, and (iii) persons and communities directly affected by the mining
project. The major taxes and other payments constituting the basic government share are enumerated
below: 65
Payments to the National Government:
Excise tax on minerals 2 percent of the gross output of mining
operations
Contractor' income tax maximum of 32 percent of taxable income
for corporations
Customs duties and fees on imported capital equipment the rate is
set by the Tariff and Customs Code (37 percent for
chemicals; 310 percent for explosives; 315 percent for
mechanical and electrical equipment; and 310 percent for
vehicles, aircraft and vessels
VAT on imported equipment, goods and services 10 percent of
value
Royalties due the government on minerals extracted from mineral
reservations, if applicable 5 percent of the actual market
value of the minerals produced
Documentary stamp tax the rate depends on the type of
transaction
Capital gains tax on traded stocks 5 to 10 percent of the value of
the shares
Withholding tax on interest payments on foreign loans 15 percent
of the amount of interest
Withholding tax on dividend payments to foreign stockholders 15
percent of the dividend
Wharfage and port fees
Licensing fees (for example, radio permit, firearms permit, professional
fees)
Other national taxes and fees.
Payments to Local Governments:
Local business tax a maximum of 2 percent of gross sales or
receipts (the rate varies among local government units)
Real property tax 2 percent of the fair market value of the
property, based on an assessment level set by the local
government
Special education levy 1 percent of the basis used for the real
property tax
Occupation fees PhP50 per hectare per year; PhP100 per hectare
per year if located in a mineral reservation
Community tax maximum of PhP10,500 per year
All other local government taxes, fees and imposts as of the effective
date of the FTAA the rate and the type depend on the
local government
Other Payments:
Royalty to indigenous cultural communities, if any 1 percent of
gross output from mining operations
Special allowance payment to claim owners and surface rights
holders
Apart from the basic share, an additional government share is also collected from the FTAA contractor in
accordance with the second paragraph of Section 81 of RA 7942, which provides that the government
share shall be comprised of, among other things, certain taxes, duties and fees. The subject proviso
reads:
"The Government share in a financial or technical assistance agreement
shall consist of, among other things, the contractor's corporate income tax,
excise tax, special allowance, withholding tax due from the contractor's
foreign stockholders arising from dividend or interest payments to the said
foreign stockholder in case of a foreign national, and all such other taxes,
duties and fees as provided for under existing laws." (Emphasis supplied.)
The government, through the DENR and the MGB, has interpreted the insertion of the phrase among
other things as signifying that the government is entitled to an "additional government share" to be paid
71
by the contractor apart from the "basic share," in order to attain a fifty-fifty sharing of net benefits from
mining.
The additional government share is computed by using one of three options or schemes presented in
DAO 99-56: (1) a fifty-fifty sharing in the cumulative present value of cash flows; (2) the share based on
excess profits; and (3) the sharing based on the cumulative net mining revenue. The particular formula
to be applied will be selected by the contractor, with a written notice to the government prior to the
commencement of the development and construction phase of the mining project. 66
Proceeds from the government shares arising from an FTAA contract are distributed to and received by
the different levels of government in the following proportions:

National Government50 percent
Provincial Government10 percent
Municipal Government20 percent
Affected Barangays20 percent
The portion of revenues remaining after the deduction of the basic and additional government shares is
what goes to the contractor.
Government's Share in anFTAA Not Consisting Solelyof Taxes, Duties and Fees
In connection with the foregoing discussion on the basic and additional government shares, it is pertinent
at this juncture to mention the criticism leveled at the second paragraph of Section 81 of RA 7942,
quoted earlier. The said proviso has been denounced, because, allegedly, the State's share in FTAAs with
foreign contractors has been limited to taxes, fees and duties only; in effect, the State has been deprived
of a share in the after-tax income of the enterprise. In the face of this allegation, one has to consider
that the law does not define the term among other things; and the Office of the Solicitor General, in its
Motion for Reconsideration appears to have erroneously claimed that the phrase refers to indirect taxes.
The law provides no definition of the term among other things, for the reason that Congress deliberately
avoided setting unnecessary limitations as to what may constitute compensation to the State for the
exploitation and use of mineral resources. But the inclusion of that phrase clearly and unmistakably
reveals the legislative intent to have the State collect more than just the usual taxes, duties and fees.
Certainly, there is nothing in that phrase or in the second paragraph of Section 81 that would
suggest that such phrase should be interpreted as referring only to taxes, duties, fees and the like.
Precisely for that reason, to fulfill the legislative intent behind the inclusion of the phrase among other
things in the second paragraph of Section 81, 67 the DENR structured and formulated in DAO 99-56 the
said additional government share. Such a share was to consist not of taxes, but of a share in the
earnings or cash flows of the mining enterprise. The additional government share was to be paid by the
contractor on top of the basic share, so as to achieve a fifty-fifty sharing between the government
and the contractor of net benefits from mining. In the Ramos-DeVera paper, the explanation of the
three options or formulas 68 presented in DAO 99-56 for the computation of the additional
government share serves to debunk the claim that the government's take from an FTAA consists
solely of taxes, fees and duties.
Unfortunately, the Office of the Solicitor General although in possession of the relevant data
failed to fully replicate or echo the pertinent elucidation in the Ramos-DeVera paper regarding the three
schemes or options for computing the additional government share presented in DAO 99-56. Had due
case been taken by the OSG, the Court would have been duly apprised of the real nature and particulars
of the additional share. aCHcIE
But, perhaps, on account of the esoteric discussion in the Ramos-DeVera paper, and the even more
abstruse mathematical jargon employed in DAO 99-56, the OSG omitted any mention of the three
options. Instead, the OSG skipped to a side discussion of the effect of indirect taxes, which had nothing
at all to do with the additional government share, to begin with. Unfortunately, this move created the
wrong impression, pointed out in Justice Antonio T. Carpio's Opinion, that the OSG had taken the position
that the additional government share consisted of indirect taxes.
In any event, what is quite evident is the fact that the additional government share, as formulated, has
nothing to do with taxes direct or indirect or with duties, fees or charges. To repeat, it is over
and above the basic government share composed of taxes and duties. Simply put, the additional share
may be (a) an amount that will result in a 50-50 sharing of the cumulative present value of the cash
flows 69 of the enterprise; (b) an amount equivalent to 25 percent of the additional or excess profits of
the enterprise, reckoned against a benchmark return on investments; or (c) an amount that will result in
a fifty-fifty sharing of the cumulative net mining revenue from the end of the recovery period up to the
taxable year in question. The contractor is required to select one of the three options or formulae for
computing the additional share, an option it will apply to all of its mining operations.
As used above, "net mining revenue" is defined as the gross output from mining operations for a
calendar year, less deductible expenses (inclusive of taxes, duties and fees). Such revenue would roughly
be equivalent to "taxable income" or income before income tax. Definitely, as compared with, say,
calculating the additional government share on the basis of net income (after income tax), the net mining
revenue is a better and much more reasonable basis for such computation, as it gives a truer picture of
the profitability of the company.
To demonstrate that the three options or formulations will operate as intended, Messrs. Ramos and de
Vera also performed some quantifications of the government share via a financial modeling of each of the
three options discussed above. They found that the government would get the highest share from the
option that is based on the net mining revenue, as compared with the other two options, considering
only the basic and the additional shares; and that, even though production rate decreases, the
government share will actually increase when the net mining revenue and the additional profit-based
options are used.
Furthermore, it should be noted that the three options or formulae do not yet take into account the
indirect taxes 70 and other financial contributions 71 of mining projects. These indirect taxes and other
contributions are real and actual benefits enjoyed by the Filipino people and/or government. Now, if
some of the quantifiable items are taken into account in the computations, the financial modeling would
show that the total government share increases to 60 percent or higher in one instance, as much as
77 percent and even 89 percent of the net present value of total benefits from the project. As noted
in the Ramos-DeVera paper, these results are not at all shabby, considering that the contractor puts in all
the capital requirements and assumes all the risks, without the government having to contribute or risk
anything. TSADaI
Despite the foregoing explanation, Justice Carpio still insisted during the Court's deliberations that the
phrase among other things refers only to taxes, duties and fees. We are bewildered by his position. On
the one hand, he condemns the Mining Law for allegedly limiting the government's benefits only to taxes,
duties and fees; and on the other, he refuses to allow the State to benefit from the correct and proper
interpretation of the DENR/MGB. To remove all doubts then, we hold that the State's share is not limited
to taxes, duties and fees only and that the DENR/MGB interpretation of the phrase among other things is
correct. Definitely, this DENR/MGB interpretation is not only legally sound, but also greatly advantageous
to the government.
One last point on the subject. The legislature acted judiciously in not defining the terms among other
things and, instead, leaving it to the agencies concerned to devise and develop the various modes of
arriving at a reasonable and fair amount for the additional government share. As can be seen from DAO
99-56, the agencies concerned did an admirable job of conceiving and developing not just one formula,
but three different formulae for arriving at the additional government share. Each of these options is
quite fair and reasonable; and, as Messrs. Ramos and De Vera stated, other alternatives or schemes for a
possible improvement of the fiscal regime for FTAAs are also being studied by the government.
Besides, not locking into a fixed definition of the term among other things will ultimately be more
beneficial to the government, as it will have that innate flexibility to adjust to and cope with rapidly
changing circumstances, particularly those in the international markets. Such flexibility is especially
72
significant for the government in terms of helping our mining enterprises remain competitive in world
markets despite challenging and shifting economic scenarios.
In conclusion, we stress that we do not share the view that in FTAAs with foreign contractors under RA
7942, the government's share is limited to taxes, fees and duties. Consequently, we find the attacks on
the second paragraph of Section 81 of RA 7942 totally unwarranted.
Collections Not Made Uncertainby the Third Paragraph of Section 81
The third or last paragraph of Section 81 72 provides that the government share in FTAAs shall be
collected when the contractor shall have recovered its pre-operating expenses and exploration and
development expenditures. The objection has been advanced that, on account of the proviso, the
collection of the State's share is not even certain, as there is no time limit in RA 7942 for this grace
period or recovery period.
We believe that Congress did not set any time limit for the grace period, preferring to leave it to the
concerned agencies, which are, on account of their technical expertise and training, in a better position
to determine the appropriate durations for such recovery periods. After all, these recovery periods are
determined, to a great extent, by technical and technological factors peculiar to the mining industry.
Besides, with developments and advances in technology and in the geosciences, we cannot discount the
possibility of shorter recovery periods. At any rate, the concerned agencies have not been remiss in this
area. The 1995 and 1996 Implementing Rules and Regulations of RA 7942 specify that the period of
recovery, reckoned from the date of commercial operation, shall be for a period not exceeding five years,
or until the date of actual recovery, whichever comes earlier.

Approval of Pre-OperatingExpenses Required by RA 7942
Still, RA 7942 is criticized for allegedly not requiring government approval of pre-operating, exploration
and development expenses of the foreign contractors, who are in effect given unfettered discretion to
determine the amounts of such expenses. Supposedly, nothing prevents the contractors from recording
such expenses in amounts equal to the mining revenues anticipated for the first 10 or 15 years of
commercial production, with the result that the share of the State will be zero for the first 10 or 15 years.
Moreover, under the circumstances, the government would be unable to say when it would start to
receive its share under the FTAA.
We believe that the argument is based on incorrect information as well as speculation. Obviously, certain
crucial provisions in the Mining Law were overlooked. Section 23, dealing with the rights and obligations
of the exploration permit grantee, states: "The permittee shall undertake exploration work on the area as
specified by its permit based on an approved work program." The next proviso reads: "Any expenditure
in excess of the yearly budget of the approved work program may be carried forward and credited to the
succeeding years covering the duration of the permit. . . ." (emphasis supplied)
Clearly, even at the stage of application for an exploration permit, the applicant is required to submit
for approval by the government a proposed work program for exploration, containing a yearly budget
of proposed expenditures. The State has the opportunity to pass upon (and approve or reject) such
proposed expenditures, with the foreknowledge that if approved these will subsequently be
recorded as pre-operating expenses that the contractor will have to recoup over the grace period. That is
not all.
Under Section 24, an exploration permit holder who determines the commercial viability of a project
covering a mining area may, within the term of the permit, file with the Mines and Geosciences Bureau a
declaration of mining project feasibility. This declaration is to be accompanied by a work program for
development for the Bureau's approval, the necessary prelude for entering into an FTAA, a mineral
production sharing agreement (MPSA), or some other mineral agreement. At this stage, too, the
government obviously has the opportunity to approve or reject the proposed work program and
budgeted expenditures for development works on the project. Such expenditures will ultimately become
the pre-operating and development costs that will have to be recovered by the contractor.
Naturally, with the submission of approved work programs and budgets for the exploration and the
development/construction phases, the government will be able to scrutinize and approve or reject such
expenditures. It will be well-informed as to the amounts of pre-operating and other expenses that the
contractor may legitimately recover and the approximate period of time needed to effect such a recovery.
There is therefore no way the contractor can just randomly post any amount of pre-operating expenses
and expect to recover the same.
The aforecited provisions on approved work programs and budgets have counterparts in Section 35,
which deals with the terms and conditions exclusively applicable to FTAAs. The said provision requires
certain terms and conditions to be incorporated into FTAAs; among them, "a firm commitment . . . of an
amount corresponding to the expenditure obligation that will be invested in the contract area" and
"representations and warranties . . . to timely deploy these [financing, managerial and technical expertise
and technological] resources under its supervision pursuant to the periodic work programs and related
budgets . . .," as well as "work programs and minimum expenditures commitments." (emphasis supplied)
Unarguably, given the provisions of Section 35, the State has every opportunity to pass upon the
proposed expenditures under an FTAA and approve or reject them. It has access to all the information it
may need in order to determine in advance the amounts of pre-operating and developmental expenses
that will have to be recovered by the contractor and the amount of time needed for such recovery.
In summary, we cannot agree that the third or last paragraph of Section 81 of RA 7942 is in any manner
unconstitutional.
No Deprivation of Beneficial Rights
It is also claimed that aside from the second and the third paragraphs of Section 81 (discussed above),
Sections 80, 84 and 112 of RA 7942 also operate to deprive the State of beneficial rights of ownership
over mineral resources; and give them away for free to private business enterprises (including foreign
owned corporations). Likewise, the said provisions have been construed as constituting, together with
Section 81, an ingenious attempt to resurrect the old and discredited system of "license, concession or
lease."
Specifically, Section 80 is condemned for limiting the State's share in a mineral production-sharing
agreement (MPSA) to just the excise tax on the mineral product. Under Section 151(A) of the Tax Code,
such tax is only 2 percent of the market value of the gross output of the minerals. The colatilla in Section
84, the portion considered offensive to the Constitution, reiterates the same limitation made in Section
80. 73
It should be pointed out that Section 80 and the colatilla in Section 84 pertain only to MPSAs and have no
application to FTAAs. These particular statutory provisions do not come within the issues that were
defined and delineated by this Court during the Oral Argument particularly the third issue, which
pertained exclusively to FTAAs. Neither did the parties argue upon them in their pleadings. Hence, this
Court cannot make any pronouncement in this case regarding the constitutionality of Sections 80 and 84
without violating the fundamental rules of due process. Indeed, the two provisos will have to await
another case specifically placing them in issue.
On the other hand, Section 112 74 is disparaged for allegedly reverting FTAAs and all mineral
agreements to the old and discredited "license, concession or lease" system. This Section states in
relevant part that "the provisions of Chapter XIV [which includes Sections 80 to 82] on government share
in mineral production-sharing agreement . . . shall immediately govern and apply to a mining lessee or
contractor." (emphasis supplied) This provision is construed as signifying that the 2 percent excise tax
which, pursuant to Section 80, comprises the government share in MPSAs shall now also constitute the
government share in FTAAs as well as in co-production agreements and joint venture agreements
to the exclusion of revenues of any other nature or from any other source.
Apart from the fact that Section 112 likewise does not come within the issues delineated by this Court
during the Oral Argument, and was never touched upon by the parties in their pleadings, it must also be
noted that the criticism hurled against this Section is rooted in unwarranted conclusions made without
considering other relevant provisions in the statute. Whether Section 112 may properly apply to co-
production or joint venture agreements, the fact of the matter is that it cannot be made to apply to
73
FTAAs.
First, Section 112 does not specifically mention or refer to FTAAs; the only reason it is being applied to
them at all is the fact that it happens to use the word "contractor." Hence, it is a bit of a stretch to insist
that it covers FTAAs as well. Second, mineral agreements, of which there are three types MPSAs, co-
production agreements, and joint venture agreements are covered by Chapter V of RA 7942. On the
other hand, FTAAs are covered by and in fact are the subject of Chapter VI, an entirely different chapter
altogether. The law obviously intends to treat them as a breed apart from mineral agreements, since
Section 35 (found in Chapter VI) creates a long list of specific terms, conditions, commitments,
representations and warranties which have not been made applicable to mineral agreements to
be incorporated into FTAAs.
Third, under Section 39, the FTAA contractor is given the option to "downgrade" to convert the FTAA
into a mineral agreement at any time during the term if the economic viability of the contract area is
inadequate to sustain large-scale mining operations. Thus, there is no reason to think that the law
through Section 112 intends to exact from FTAA contractors merely the same government share (a 2
percent excise tax) that it apparently demands from contractors under the three forms of mineral
agreements. In brief, Section 112 does not apply to FTAAs. EAHcCT
Notwithstanding the foregoing explanation, Justices Carpio and Morales maintain that the Court must rule
now on the constitutionality of Sections 80, 84 and 112, allegedly because the WMCP FTAA contains a
provision which grants the contractor unbridled and "automatic" authority to convert the FTAA into an
MPSA; and should such conversion happen, the State would be prejudiced since its share would be
limited to the 2 percent excise tax. Justice Carpio adds that there are five MPSAs already signed just
awaiting the judgment of this Court on respondents' and intervenor's Motions for Reconsideration. We
hold however that, at this point, this argument is based on pure speculation. The Court cannot rule on
mere surmises and hypothetical assumptions, without firm factual anchor. We repeat: basic due process
requires that we hear the parties who have a real legal interest in the MPSAs (i.e. the parties who
executed them) before these MPSAs can be reviewed, or worse, struck down by the Court. Anything less
than that requirement would be arbitrary and capricious.
In any event, the conversion of the present FTAA into an MPSA is problematic. First, the contractor must
comply with the law, particularly Section 39 of RA 7942; inter alia, it must convincingly show that the
"economic viability of the contract is found to be inadequate to justify large-scale mining operations;"
second, it must contend with the President's exercise of the power of State control over the EDU of
natural resources; and third, it will have to risk a possible declaration of the unconstitutionality (in a
proper case) of Sections 80, 84 and 112.

The first requirement is not as simple as it looks. Section 39 contemplates a situation in which an FTAA
has already been executed and entered into, and is presumably being implemented, when the contractor
"discovers" that the mineral ore reserves in the contract area are not sufficient to justify large-scale
mining, and thus the contractor requests the conversion of the FTAA into an MPSA. The contractor in
effect needs to explain why, despite its exploration activities, including the conduct of various geologic
and other scientific tests and procedures in the contract area, it was unable to determine correctly the
mineral ore reserves and the economic viability of the area. The contractor must explain why, after
conducting such exploration activities, it decided to file a declaration of mining feasibility, and to apply for
an FTAA, thereby leading the State to believe that the area could sustain large-scale mining. The
contractor must justify fully why its earlier findings, based on scientific procedures, tests and data, turned
out to be wrong, or were way off . It must likewise prove that its new findings, also based on scientific
tests and procedures, are correct. Right away, this puts the contractor's technical capabilities and
expertise into serious doubt. We wonder if anyone would relish being in this situation. The State could
even question and challenge the contractor's qualification and competence to continue the activity under
an MPSA.
All in all, while there may be cogent grounds to assail the aforecited Sections, this Court on
considerations of due process cannot rule upon them here. Anyway, if later on these Sections are
declared unconstitutional, such declaration will not affect the other portions since they are clearly
separable from the rest.
Our Mineral Resources Not Given Away for Free by RA 7942
Nevertheless, if only to disabuse our minds, we should address the contention that our mineral resources
are effectively given away for free by the law (RA 7942) in general and by Sections 80, 81, 84 and 112 in
particular.
Foreign contractors do not just waltz into town one day and leave the next, taking away mineral
resources without paying anything. In order to get at the minerals, they have to invest huge sums of
money (tens or hundreds of millions of dollars) in exploration works first. If the exploration proves
unsuccessful, all the cash spent thereon will not be returned to the foreign investors; rather, those funds
will have been infused into the local economy, to remain there permanently. The benefits therefrom
cannot be simply ignored. And assuming that the foreign contractors are successful in finding ore bodies
that are viable for commercial exploitation, they do not just pluck out the minerals and cart them off.
They have first to build camp sites and roadways; dig mine shafts and connecting tunnels; prepare tailing
ponds, storage areas and vehicle depots; install their machinery and equipment, generator sets, pumps,
water tanks and sewer systems, and so on.
In short, they need to expend a great deal more of their funds for facilities, equipment and supplies, fuel,
salaries of local labor and technical staff, and other operating expenses. In the meantime, they also have
to pay taxes, 75 duties, fees, and royalties. All told, the exploration, pre-feasibility, feasibility,
development and construction phases together add up to as many as eleven years. 76 The contractors
have to continually shell out funds for the duration of over a decade, before they can commence
commercial production from which they would eventually derive revenues. All that money translates into
a lot of "pump-priming" for the local economy.
Granted that the contractors are allowed subsequently to recover their pre-operating expenses, still, that
eventuality will happen only after they shall have first put out the cash and fueled the economy.
Moreover, in the process of recouping their investments and costs, the foreign contractors do not actually
pull out the money from the economy. Rather, they recover or recoup their investments out of actual
commercial production by not paying a portion of the basic government share corresponding to national
taxes, along with the additional government share, for a period of not more than five years 77 counted
from the commencement of commercial production.
It must be noted that there can be no recovery without commencing actual commercial production. In
the meantime that the contractors are recouping costs, they need to continue operating; in order to do
so, they have to disburse money to meet their various needs. In short, money is continually infused into
the economy.
The foregoing discussion should serve to rid us of the mistaken belief that, since the foreign contractors
are allowed to recover their investments and costs, the end result is that they practically get the minerals
for free, which leaves the Filipino people none the better for it.
All Businesses Entitledto Cost Recovery
Let it be put on record that not only foreign contractors, but all businessmen and all business entities in
general, have to recoup their investments and costs. That is one of the first things a student learns in
business school. Regardless of its nationality, and whether or not a business entity has a five-years cost
recovery period, it will must have to recoup its investments, one way or another. This is just
common business sense. Recovery of investments is absolutely indispensable for business survival; and
business survival ensures soundness of the economy, which is critical and contributory to the general
welfare of the people. Even government corporations must recoup their investments in order to survive
and continue in operation. And, as the preceding discussion has shown, there is no business that gets
ahead or earns profits without any cost to it.
It must also be stressed that, though the State owns vast mineral wealth, such wealth is not readily
accessible or transformable into usable and negotiable currency without the intervention of the credible
mining companies. Those untapped mineral resources, hidden beneath tons of earth and rock, may as
well not be there for all the good they do us right now. They have first to be extracted and converted
into marketable form, and the country needs the foreign contractor's funds, technology and know-how
for that.
74
After about eleven years of pre-operation and another five years for cost recovery, the foreign
contractors will have just broken even. Is it likely that they would at that point stop their operations and
leave? Certainly not. They have yet to make profits. Thus, for the remainder of the contract term, they
must strive to maintain profitability. During this period, they pay the whole of the basic government
share and the additional government share which, taken together with indirect taxes and other
contributions, amount to approximately 60 percent or more of the entire financial benefits generated by
the mining venture.
In sum, we can hardly talk about foreign contractors taking our mineral resources for free. It takes a lot
of hard cash to even begin to do what they do. And what they do in this country ultimately benefits the
local economy, grows businesses, generates employment, and creates infrastructure, as discussed above.
Hence, we definitely disagree with the sweeping claim that no FTAA under Section 81 will ever make any
real contribution to the growth of the economy or to the general welfare of the country. This is not a plea
for foreign contractors. Rather, this is a question of focusing the judicial spotlight squarely on all the
pertinent facts as they bear upon the issue at hand, in order to avoid leaping precipitately to ill-conceived
conclusions not solidly grounded upon fact.
Repatriation of After-Tax Income
Another objection points to the alleged failure of the Mining Law to ensure real contributions to the
economic growth and general welfare of the country, as mandated by Section 2 of Article XII of the
Constitution. Pursuant to Section 81 of the law, the entire after-tax income arising from the exploitation
of mineral resources owned by the State supposedly belongs to the foreign contractors, which will
naturally repatriate the said after-tax income to their home countries, thereby resulting in no real
contribution to the economic growth of this country. Clearly, this contention is premised on erroneous
assumptions.
First, as already discussed in detail hereinabove, the concerned agencies have correctly interpreted the
second paragraph of Section 81 of RA 7942 to mean that the government is entitled to an additional
share, to be computed based on any one of the following factors: net mining revenues, the present value
of the cash flows, or excess profits reckoned against a benchmark rate of return on investments. So it is
not correct to say that all of the after-tax income will accrue to the foreign FTAA contractor, as the
government effectively receives a significant portion thereof .
Second, the foreign contractors can hardly "repatriate the entire after-tax income to their home
countries." Even a bit of knowledge of corporate finance will show that it will be impossible to maintain a
business as a "going concern" if the entire "net profit" earned in any particular year will be taken out and
repatriated. The "net income" figure reflected in the bottom line is a mere accounting figure not
necessarily corresponding to cash in the bank, or other quick assets. In order to produce and set aside
cash in an amount equivalent to the bottom line figure, one may need to sell off assets or immediately
collect receivables or liquidate short-term investments; but doing so may very likely disrupt normal
business operations.
In terms of cash flows, the funds corresponding to the net income as of a particular point in time are
actually in use in the normal course of business operations. Pulling out such net income disrupts the cash
flows and cash position of the enterprise and, depending on the amount being taken out, could seriously
cripple or endanger the normal operations and financial health of the business enterprise. In short, no
sane business person, concerned with maintaining the mining enterprise as a going concern and keeping
a foothold in its market, can afford to repatriate the entire after-tax income to the home country.

The State's Receipt of SixtyPercent of an FTAA Contractor's After-Tax Income Not Mandatory
We now come to the next objection which runs this way: In FTAAs with a foreign contractor, the State
must receive at least 60 percent of the after-tax income from the exploitation of its mineral resources.
This share is the equivalent of the constitutional requirement that at least 60 percent of the capital, and
hence 60 percent of the income, of mining companies should remain in Filipino hands.
First, we fail to see how we can properly conclude that the Constitution mandates the State to extract at
least 60 percent of the after-tax income from a mining company run by a foreign contractor. The
argument is that the Charter requires the State's partner in a co-production agreement, joint venture
agreement or MPSA to be a Filipino corporation (at least 60 percent owned by Filipino citizens).
We question the logic of this reasoning, premised on a supposedly parallel or analogous situation. We
are, after all, dealing with an essentially different equation, one that involves different elements. The
Charter did not intend to fix an iron-clad rule on the 60 percent share, applicable to all situations at all
times and in all circumstances. If ever such was the intention of the framers, they would have spelt it out
in black and white. Verba legis will serve to dispel unwarranted and untenable conclusions. DHSACT
Second, if we would bother to do the math, we might better appreciate the impact (and reasonableness)
of what we are demanding of the foreign contractor. Let us use a simplified illustration. Let us base it on
gross revenues of, say, P500. After deducting operating expenses, but prior to income tax, suppose a
mining firm makes a taxable income of P100. A corporate income tax of 32 percent results in P32 of
taxable income going to the government, leaving the mining firm with P68. Government then takes 60
percent thereof, equivalent to P40.80, leaving only P27.20 for the mining firm.
At this point the government has pocketed P32.00 plus P40.80, or a total of P72.80 for every P100 of
taxable income, leaving the mining firm with only P27.20. But that is not all. The government has also
taken 2 percent excise tax "off the top," equivalent to another P10. Under the minimum 60 percent
proposal, the government nets around P82.80 (not counting other taxes, duties, fees and charges) from
a taxable income of P100 (assuming gross revenues of P500, for purposes of illustration). On the other
hand, the foreign contractor, which provided all the capital, equipment and labor, and took all the
entrepreneurial risks receives P27.20. One cannot but wonder whether such a distribution is even
remotely equitable and reasonable, considering the nature of the mining business. The amount of P82.80
out of P100.00 is really a lot it does not matter that we call part of it excise tax or income tax, and
another portion thereof income from exploitation of mineral resources. Some might think it wonderful to
be able to take the lion's share of the benefits. But we have to ask ourselves if we are really serious in
attracting the investments that are the indispensable and key element in generating the monetary
benefits of which we wish to take the lion's share. Fairness is a credo not only in law, but also in
business.
Third, the 60 percent rule in the petroleum industry cannot be insisted upon at all times in the mining
business. The reason happens to be the fact that in petroleum operations, the bulk of expenditures is in
exploration, but once the contractor has found and tapped into the deposit, subsequent investments and
expenditures are relatively minimal. The crude (or gas) keeps gushing out, and the work entailed is just a
matter of piping, transporting and storing. Not so in mineral mining. The ore body does not pop out on
its own. Even after it has been located, the contractor must continually invest in machineries and expend
funds to dig and build tunnels in order to access and extract the minerals from underneath hundreds of
tons of earth and rock.
As already stated, the numerous intrinsic differences involved in their respective operations and
requirements, cost structures and investment needs render it highly inappropriate to use petroleum
operations FTAAs as benchmarks for mining FTAAs. Verily, we cannot just ignore the realities of the
distinctly different situations and stubbornly insist on the "minimum 60 percent."
The Mining and the Oil IndustriesDifferent From Each Other
To stress, there is no independent showing that the taking of at least a 60 percent share in the after-tax
income of a mining company operated by a foreign contractor is fair and reasonable under most if not all
circumstances. The fact that some petroleum companies like Shell acceded to such percentage of sharing
does not ipso facto mean that it is per se reasonable and applicable to non-petroleum situations (that is,
mining companies) as well. We can take judicial notice of the fact that there are, after all, numerous
intrinsic differences involved in their respective operations and equipment or technological requirements,
costs structures and capital investment needs, and product pricing and markets.
There is no showing, for instance, that mining companies can readily cope with a 60 percent government
share in the same way petroleum companies apparently can. What we have is a suggestion to enforce
the 60 percent quota on the basis of a disjointed analogy. The only factor common to the two disparate
situations is the extraction of natural resources.
75
Indeed, we should take note of the fact that Congress made a distinction between mining firms and
petroleum companies. In Republic Act No. 7729 "An Act Reducing the Excise Tax Rates on Metallic
and Non-Metallic Minerals and Quarry Resources, Amending for the Purpose Section 151(a) of the
National Internal Revenue Code, as amended" the lawmakers fixed the excise tax rate on metallic
and non-metallic minerals at two percent of the actual market value of the annual gross output at the
time of removal. However, in the case of petroleum, the lawmakers set the excise tax rate for the first
taxable sale at fifteen percent of the fair international market price thereof.
There must have been a very sound reason that impelled Congress to impose two very dissimilar excise
tax rate. We cannot assume, without proof, that our honorable legislators acted arbitrarily, capriciously
and whimsically in this instance. We cannot just ignore the reality of two distinctly different situations and
stubbornly insist on going "minimum 60 percent."
To repeat, the mere fact that gas and oil exploration contracts grant the State 60 percent of the net
revenues does not necessarily imply that mining contracts should likewise yield a minimum of 60 percent
for the State. Jumping to that erroneous conclusion is like comparing apples with oranges. The
exploration, development and utilization of gas and oil are simply different from those of mineral
resources.
To stress again, the main risk in gas and oil is in the exploration. But once oil in commercial quantities is
struck and the wells are put in place, the risk is relatively over and black gold simply flows out
continuously with comparatively less need for fresh investments and technology.
On the other hand, even if minerals are found in viable quantities, there is still need for continuous fresh
capital and expertise to dig the mineral ores from the mines. Just because deposits of mineral ores are
found in one area is no guarantee that an equal amount can be found in the adjacent areas. There are
simply continuing risks and need for more capital, expertise and industry all the time.
Note, however, that the indirect benefits apart from the cash revenues are much more in the
mineral industry. As mines are explored and extracted, vast employment is created, roads and other
infrastructure are built, and other multiplier effects arise. On the other hand, once oil wells start
producing, there is less need for employment. Roads and other public works need not be constructed
continuously. In fine, there is no basis for saying that government revenues from the oil industry and
from the mineral industries are to be identical all the time.
Fourth, to our mind, the proffered "minimum 60 percent" suggestion tends to limit the flexibility and tie
the hands of government, ultimately hampering the country's competitiveness in the international
market, to the detriment of the Filipino people. This "you-have-to-give-us-60-percent-of-after-tax-
income-or-we- don't-do-business-with-you" approach is quite perilous. True, this situation may not seem
too unpalatable to the foreign contractor during good years, when international market prices are up and
the mining firm manages to keep its costs in check. However, under unfavorable economic and business
conditions, with costs spiraling skywards and minerals prices plummeting, a mining firm may consider
itself lucky to make just minimal profits.
The inflexible, carved-in-granite demand for a 60 percent government share may spell the end of the
mining venture, scare away potential investors, and thereby further worsen the already dismal economic
scenario. Moreover, such an unbending or unyielding policy prevents the government from responding
appropriately to changing economic conditions and shifting market forces. This inflexibility further renders
our country less attractive as an investment option compared with other countries.
And fifth, for this Court to decree imperiously that the government's share should be not less than 60
percent of the after-tax income of FTAA contractors at all times is nothing short of dictating upon the
government. The result, ironically, is that the State ends up losing control. To avoid compromising the
State's full control and supervision over the exploitation of mineral resources, this Court must back off
from insisting upon a "minimum 60 percent" rule. It is sufficient that the State has the power and means,
should it so decide, to get a 60 percent share (or more) in the contractor's net mining revenues or after-
tax income, or whatever other basis the government may decide to use in reckoning its share. It is not
necessary for it to do so in every case, regardless of circumstances.

In fact, the government must be trusted, must be accorded the liberty and the utmost flexibility to deal,
negotiate and transact with contractors and third parties as it sees fit; and upon terms that it ascertains
to be most favorable or most acceptable under the circumstances, even if it means agreeing to less than
60 percent. Nothing must prevent the State from agreeing to a share less than that, should it be deemed
fit; otherwise the State will be deprived of full control over mineral exploitation that the Charter has
vested in it.
To stress again, there is simply no constitutional or legal provision fixing the minimum share of the
government in an FTAA at 60 percent of the net profit. For this Court to decree such minimum is to wade
into judicial legislation, and thereby inordinately impinge on the control power of the State. Let it be
clear: the Court is not against the grant of more benefits to the State; in fact, the more the better. If
during the FTAA negotiations, the President can secure 60 percent, 78 or even 90 percent, then all the
better for our people. But, if under the peculiar circumstances of a specific contract, the President could
secure only 50 percent or 55 percent, so be it. Needless to say, the President will have to report (and be
responsible for) the specific FTAA to Congress, and eventually to the people.
Finally, if it should later be found that the share agreed to is grossly disadvantageous to the government,
the officials responsible for entering into such a contract on its behalf will have to answer to the courts
for their malfeasance. And the contract provision voided. But this Court would abuse its own authority
should it force the government's hand to adopt the 60 percent demand of some of our esteemed
colleagues.
Capital and Expertise Provided, Yet All Risks Assumed by Contractor
Here, we will repeat what has not been emphasized and appreciated enough: the fact that the contractor
in an FTAA provides all the needed capital, technical and managerial expertise, and technology required
to undertake the project.
In regard to the WMCP FTAA, the then foreign-owned WMCP as contractor committed, at the very outset,
to make capital investments of up to US$50 million in that single mining project. WMCP claims to have
already poured in well over P800 million into the country as of February 1998, with more in the pipeline.
These resources, valued in the tens or hundreds of millions of dollars, are invested in a mining project
that provides no assurance whatsoever that any part of the investment will be ultimately recouped.
At the same time, the contractor must comply with legally imposed environmental standards and the
social obligations, for which it also commits to make significant expenditures of funds. Throughout, the
contractor assumes all the risks 79 of the business; as mentioned earlier. These risks are indeed very
high, considering that the rate of success in exploration is extremely low. The probability of finding any
mineral or petroleum in commercially viable quantities is estimated to be about 1:1,000 only. On that slim
chance rides the contractor's hope of recouping investments and generating profits. And when the
contractor has recouped its initial investments in the project, the government share increases to sixty
percent of net benefits without the State ever being in peril of incurring costs, expenses and losses.
And even in the worst possible scenario an absence of commercial quantities of minerals to justify
development the contractor would already have spent several million pesos for exploration works,
before arriving at the point in which it can make that determination and decide to cut its losses. In fact,
during the first year alone of the exploration period, the contractor was already committed to spend not
less than P24 million. The FTAA therefore clearly ensures benefits for the local economy, courtesy of the
contractor.
All in all, this setup cannot be regarded as disadvantageous to the State or the Filipino people; it certainly
cannot be said to convey beneficial ownership of our mineral resources to foreign contractors.
Deductions Allowed by theWMCP FTAA Reasonable
Petitioners question whether the State's weak control might render the sharing arrangements ineffective.
They cite the so-called "suspicious" deductions allowed by the WMCP FTAA in arriving at the net mining
revenue, which is the basis for computing the government share. The WMCP FTAA, for instance, allows
76
expenditures for "development within and outside the Contract Area relating to the Mining Operations,"
80 "consulting fees incurred both inside and outside the Philippines for work related directly to the
Mining Operations," 81 and "the establishment and administration of field offices including administrative
overheads incurred within and outside the Philippines which are properly allocatable to the Mining
Operations and reasonably related to the performance of the Contractor's obligations and exercise of its
rights under this Agreement." 82
It is quite well known, however, that mining companies do perform some marketing activities abroad in
respect of selling their mineral products and by-products. Hence, it would not be improper to allow the
deduction of reasonable consulting fees incurred abroad, as well as administrative expenses and
overheads related to marketing offices also located abroad provided that these deductions are
directly related or properly allocatable to the mining operations and reasonably related to the
performance of the contractor's obligations and exercise of its rights. In any event, more facts are
needed. Until we see how these provisions actually operate, mere "suspicions" will not suffice to propel
this Court into taking action. AaSHED
Section 7.9 of the WMCP FTAAInvalid and Disadvantageous
Having defended the WMCP FTAA, we shall now turn to two defective provisos. Let us start with Section
7.9 of the WMCP FTAA. While Section 7.7 gives the government a 60 percent share in the net mining
revenues of WMCP from the commencement of commercial production, Section 7.9 deprives the
government of part or all of the said 60 percent. Under the latter provision, should WMCP's foreign
shareholders who originally owned 100 percent of the equity sell 60 percent or more of its
outstanding capital stock to a Filipino citizen or corporation, the State loses its right to receive its 60
percent share in net mining revenues under Section 7.7.
Section 7.9 provides:
The percentage of Net Mining Revenues payable to the Government
pursuant to Clause 7.7 shall be reduced by 1 percent of Net Mining
Revenues for every 1 percent ownership interest in the Contractor (i.e.,
WMCP) held by a Qualified Entity. 83
Evidently, what Section 7.7 grants to the State is taken away in the next breadth by Section 7.9 without
any offsetting compensation to the State. Thus, in reality, the State has no vested right to receive any
income from the FTAA for the exploitation of its mineral resources. Worse, it would seem that what is
given to the State in Section 7.7 is by mere tolerance of WMCP's foreign stockholders, who can at any
time cut off the government's entire 60 percent share. They can do so by simply selling 60 percent of
WMCP's outstanding capital stock to a Philippine citizen or corporation. Moreover, the proceeds of such
sale will of course accrue to the foreign stockholders of WMCP, not to the State.
The sale of 60 percent of WMCP's outstanding equity to a corporation that is 60 percent Filipino-owned
and 40 percent foreign-owned will still trigger the operation of Section 7.9. Effectively, the State will lose
its right to receive all 60 percent of the net mining revenues of WMCP; and foreign stockholders will own
beneficially up to 64 percent of WMCP, consisting of the remaining 40 percent foreign equity therein, plus
the 24 percent pro-rata share in the buyer-corporation. 84
In fact, the January 23, 2001 sale by WMCP's foreign stockholder of the entire outstanding equity in
WMCP to Sagittarius Mines, Inc. a domestic corporation at least 60 percent Filipino owned may
be deemed to have automatically triggered the operation of Section 7.9, without need of further action
by any party, and removed the State's right to receive the 60 percent share in net mining revenues.
At bottom, Section 7.9 has the effect of depriving the State of its 60 percent share in the net mining
revenues of WMCP without any offset or compensation whatsoever. It is possible that the inclusion of the
offending provision was initially prompted by the desire to provide some form of incentive for the
principal foreign stockholder in WMCP to eventually reduce its equity position and ultimately divest in
favor of Filipino citizens and corporations. However, as finally structured, Section 7.9 has the deleterious
effect of depriving government of the entire 60 percent share in WMCP's net mining revenues, without
any form of compensation whatsoever. Such an outcome is completely unacceptable.
The whole point of developing the nation's natural resources is to benefit the Filipino people, future
generations included. And the State as sovereign and custodian of the nation's natural wealth is
mandated to protect, conserve, preserve and develop that part of the national patrimony for their
benefit. Hence, the Charter lays great emphasis on "real contributions to the economic growth and
general welfare of the country" 85 as essential guiding principles to be kept in mind when negotiating
the terms and conditions of FTAAs.
Earlier, we held (1) that the State must be accorded the liberty and the utmost flexibility to deal,
negotiate and transact with contractors and third parties as it sees fit, and upon terms that it ascertains
to be most favorable or most acceptable under the circumstances, even if that should mean agreeing to
less than 60 percent; (2) that it is not necessary for the State to extract a 60 percent share in every case
and regardless of circumstances; and (3) that should the State be prevented from agreeing to a share
less than 60 percent as it deems fit, it will be deprived of the full control over mineral exploitation that
the Charter has vested in it.

That full control is obviously not an end in itself; it exists and subsists precisely because of the need to
serve and protect the national interest. In this instance, national interest finds particular application in the
protection of the national patrimony and the development and exploitation of the country's mineral
resources for the benefit of the Filipino people and the enhancement of economic growth and the general
welfare of the country. Undoubtedly, such full control can be misused and abused, as we now witness.
Section 7.9 of the WMCP FTAA effectively gives away the State's share of net mining revenues (provided
for in Section 7.7) without anything in exchange. Moreover, this outcome constitutes unjust enrichment
on the part of the local and foreign stockholders of WMCP. By their mere divestment of up to 60 percent
equity in WMCP in favor of Filipino citizens and/or corporations, the local and foreign stockholders get a
windfall. Their share in the net mining revenues of WMCP is automatically increased, without their having
to pay the government anything for it. In short, the provision in question is without a doubt grossly
disadvantageous to the government, detrimental to the interests of the Filipino people, and violative of
public policy.
Moreover, it has been reiterated in numerous decisions 86 that the parties to a contract may establish
any agreements, terms and conditions that they deem convenient; but these should not be contrary to
law, morals, good customs, public order or public policy. 87 Being precisely violative of anti-graft
provisions and contrary to public policy, Section 7.9 must therefore be stricken off as invalid.
Whether the government officials concerned acceded to that provision by sheer mistake or with full
awareness of the ill consequences, is of no moment. It is hornbook doctrine that the principle of estoppel
does not operate against the government for the act of its agents, 88 and that it is never estopped by
any mistake or error on their part. 89 It is therefore possible and proper to rectify the situation at this
time. Moreover, we may also say that the FTAA in question does not involve mere contractual rights;
being impressed as it is with public interest, the contractual provisions and stipulations must yield to the
common good and the national interest.
Since the offending provision is very much separable 90 from Section 7.7 and the rest of the FTAA, the
deletion of Section 7.9 can be done without affecting or requiring the invalidation of the WMCP FTAA
itself. Such a deletion will preserve for the government its due share of the benefits. This way, the
mandates of the Constitution are complied with and the interests of the government fully protected, while
the business operations of the contractor are not needlessly disrupted.
Section 7.8(e) of the WMCP FTAAAlso Invalid and Disadvantageous
Section 7.8(e) of the WMCP FTAA is likewise invalid. It provides thus:
"7.8The Government Share shall be deemed to include all of the following
sums:
"(a)all Government taxes, fees, levies, costs, imposts, duties
and royalties including excise tax, corporate
77
income tax, customs duty, sales tax, value
added tax, occupation and regulatory fees,
Government controlled price stabilization
schemes, any other form of Government
backed schemes, any tax on dividend payments
by the Contractor or its Affiliates in respect of
revenues from the Mining Operations and any
tax on interest on domestic and foreign loans or
other financial arrangements or
accommodations, including loans extended to
the Contractor by its stockholders;
"(b)any payments to local and regional government,
including taxes, fees, levies, costs, imposts,
duties, royalties, occupation and regulatory fees
and infrastructure contributions;
"(c)any payments to landowners, surface rights holders,
occupiers, indigenous people or Claim-owners;
"(d)costs and expenses of fulfilling the Contractor's
obligations to contribute to national
development in accordance with Clause 10.1(i)
(1) and 10.1(i) (2);
"(e)an amount equivalent to whatever benefits that may be
extended in the future by the Government to
the Contractor or to financial or technical
assistance agreement contractors in general;
"(f)all of the foregoing items which have not previously been
offset against the Government Share in an
earlier Fiscal Year, adjusted for inflation."
(emphasis supplied)
Section 7.8(e) is out of place in the FTAA. It makes no sense why, for instance, money spent by the
government for the benefit of the contractor in building roads leading to the mine site should still be
deductible from the State's share in net mining revenues. Allowing this deduction results in benefiting the
contractor twice over. It constitutes unjust enrichment on the part of the contractor at the expense of the
government, since the latter is effectively being made to pay twice for the same item. 91 For being
grossly disadvantageous and prejudicial to the government and contrary to public policy, Section 7.8(e) is
undoubtedly invalid and must be declared to be without effect. Fortunately, this provision can also easily
be stricken off without affecting the rest of the FTAA.
Nothing Left Over After Deductions?
In connection with Section 7.8, an objection has been raised:
Specified in Section 7.8 are numerous items of deduction from the State's 60 percent share. After taking
these into account, will the State ever receive anything for its ownership of the mineral resources?
We are confident that under normal circumstances, the answer will be yes. If we examine the various
items of "deduction" listed in Section 7.8 of the WMCP FTAA, we will find that they correspond closely to
the components or elements of the basic government share established in DAO 99-56, as discussed in
the earlier part of this Opinion.
Likewise, the balance of the government's 60 percent share after netting out the items of deduction
listed in Section 7.8 corresponds closely to the additional government share provided for in DAO 99-
56 which, we once again stress, has nothing at all to do with indirect taxes. The Ramos-DeVera paper 92
concisely presents the fiscal contribution of an FTAA under DAO 99-56 in this equation:
Receipts from an FTAA = basic gov't share + add'l gov't share
Transposed into a similar equation, the fiscal payments system from the WMCP FTAA assumes the
following formulation:
Government's 60 percent share in net mining revenues of WMCP = items
listed in Sec. 7.8 of the FTAA + balance of Gov't share, payable 4 months
from the end of the fiscal year
It should become apparent that the fiscal arrangement under the WMCP FTAA is very similar to that
under DAO 99-56, with the "balance of government share payable 4 months from end of fiscal year"
being the equivalent of the additional government share computed in accordance with the "net-mining-
revenue-based option" under DAO 99-56, as discussed above. As we have emphasized earlier, we find
each of the three options for computing the additional government share as presented in DAO 99-56
to be sound and reasonable.
We therefore conclude that there is nothing inherently wrong in the fiscal regime of the WMCP FTAA, and
certainly nothing to warrant the invalidation of the FTAA in its entirety.
Section 3.3 of the WMCPFTAA Constitutional
Section 3.3 of the WMCP FTAA is assailed for violating supposed constitutional restrictions on the term of
FTAAs. The provision in question reads:
"3.3This Agreement shall be renewed by the Government for a further
period of twenty-five (25) years under the same terms and
conditions provided that the Contractor lodges a request for
renewal with the Government not less than sixty (60) days
prior to the expiry of the initial term of this Agreement and
provided that the Contractor is not in breach of any of the
requirements of this Agreement." cCAaHD
Allegedly, the above provision runs afoul of Section 2 of Article XII of the 1987 Constitution, which states:
"Sec. 2.All lands of the public domain, waters, minerals, coal, petroleum,
and other mineral oils, all forces of potential energy, fisheries, forests or
timber, wildlife, flora and fauna, and other natural resources are owned by
the State. With the exception of agricultural lands, all other natural
resources shall not be alienated. The exploration, development and
utilization of natural resources shall be under the full control and
supervision of the State. The State may directly undertake such activities,
or it may enter into co-production, joint venture or production-sharing
agreements with Filipino citizens or corporations or associations at least
sixty per centum of whose capital is owned by such citizens. Such
agreements may be for a period not exceeding twenty-five years,
renewable for not more than twenty-five years, and under such terms and
conditions as may be provided by law. In cases of water rights for
irrigation, water supply, fisheries, or industrial uses other than the
development of water power, beneficial use may be the measure and limit
of the grant.
"The State shall protect the nation's marine wealth in its archipelagic
waters, territorial sea, and exclusive economic zone, and reserve its use
and enjoyment exclusively to Filipino citizens.
"The Congress may, by law, allow small-scale utilization of natural
resources by Filipino citizens, as well as cooperative fish farming, with
priority to subsistence fishermen and fish-workers in rivers, lakes, bays
and lagoons.
78
"The President may enter into agreements with foreign-owned
corporations involving either technical or financial assistance for large-scale
exploration, development, and utilization of minerals, petroleum, and other
mineral oils according to the general terms and conditions provided by law,
based on real contributions to the economic growth and general welfare of
the country. In such agreements, the State shall promote the development
and use of local scientific and technical resources.

"The President shall notify the Congress of every contract entered into in
accordance with this provision, within thirty days from its execution." 93
We hold that the term limitation of twenty five years does not apply to FTAAs. The reason is that the
above provision is found within paragraph 1 of Section 2 of Article XII, which refers to mineral
agreements co-production agreements, joint venture agreements and mineral production-sharing
agreements which the government may enter into with Filipino citizens and corporations, at least 60
percent owned by Filipino citizens. The word "such" clearly refers to these three mineral agreements
CPAs, JVAs and MPSAs, not to FTAAs.
Specifically, FTAAs are covered by paragraphs 4 and 5 of Section 2 of Article XII of the Constitution. It
will be noted that there are no term limitations provided for in the said paragraphs dealing with FTAAs.
This shows that FTAAs are sui generis, in a class of their own. This omission was obviously a deliberate
move on the part of the framers. They probably realized that FTAAs would be different in many ways
from MPSAs, JVAs and CPAs. The reason the framers did not fix term limitations applicable to FTAAs is
that they preferred to leave the matter to the discretion of the legislature and/or the agencies involved in
implementing the laws pertaining to FTAAs, in order to give the latter enough flexibility and elbow room
to meet changing circumstances.
Note also that, as previously stated, the exploratory phrases of an FTAA lasts up to eleven years.
Thereafter, a few more years would be gobbled up in start-up operations. It may take fifteen years
before an FTAA contractor can start earning profits. And thus, the period of 25 years may really be short
for an FTAA. Consider too that in this kind of agreement, the contractor assumes all entrepreneurial risks.
If no commercial quantities of minerals are found, the contractor bears all financial losses. To
compensate for this long gestation period and extra business risks, it would not be totally unreasonable
to allow it to continue EDU activities for another twenty five years.
In any event, the complaint is that, in essence, Section 3.3 gives the contractor the power to compel the
government to renew the WMCP FTAA for another 25 years and deprives the State of any say on
whether to renew the contract.
While we agree that Section 3.3 could have been worded so as to prevent it from favoring the contractor,
this provision does not violate any constitutional limits, since the said term limitation does not apply at all
to FTAAs. Neither can the provision be deemed in any manner to be illegal, as no law is being violated
thereby. It is certainly not illegal for the government to waive its option to refuse the renewal of a
commercial contract.
Verily, the government did not have to agree to Section 3.3. It could have said "No" to the stipulation,
but it did not. It appears that, in the process of negotiations, the other contracting party was able to
convince the government to agree to the renewal terms. Under the circumstances, it does not seem
proper for this Court to intervene and step in to undo what might have perhaps been a possible
miscalculation on the part of the State. If government believes that it is or will be aggrieved by the
effects of Section 3.3, the remedy is the renegotiation of the provision in order to provide the State the
option to not renew the FTAA.
Financial Benefits for Foreigners Not Forbidden by the Constitution
Before leaving this subject matter, we find it necessary for us to rid ourselves of the false belief that the
Constitution somehow forbids foreign-owned corporations from deriving financial benefits from the
development of our natural or mineral resources.
The Constitution has never prohibited foreign corporations from acquiring and enjoying "beneficial
interest" in the development of Philippine natural resources. The State itself need not directly undertake
exploration, development, and utilization activities. Alternatively, the Constitution authorizes the
government to enter into joint venture agreements (JVAs), co-production agreements (CPAs) and mineral
production sharing agreements (MPSAs) with contractors who are Filipino citizens or corporations that are
at least 60 percent Filipino-owned. They may do the actual "dirty work," the mining operations.
In the case of a 60 percent Filipino-owned corporation, the 40 percent individual and/or corporate non-
Filipino stakeholders obviously participate in the beneficial interest derived from the development and
utilization of our natural resources. They may receive by way of dividends, up to 40 percent of the
contractor's earnings from the mining project. Likewise, they may have a say in the decisions of the
board of directors, since they are entitled to representation therein to the extent of their equity
participation, which the Constitution permits to be up to 40 percent of the contractor's equity. Hence, the
non-Filipino stakeholders may in that manner also participate in the management of the contractor's
natural resource development work. All of this is permitted by our Constitution, for any natural resource,
and without limitation even in regard to the magnitude of the mining project or operations (see
paragraph 1 of Section 2 of Article XII).
It is clear, then, that there is nothing inherently wrong with or constitutionally objectionable about the
idea of foreign individuals and entities having or enjoying "beneficial interest" in and participating in
the management of operations relative to the exploration, development and utilization of our natural
resources.
FTAA More Advantageous Than Other Schemes Like CPA, JVA and MPSA
A final point on the subject of beneficial interest. We believe the FTAA is a more advantageous
proposition for the government as compared with other agreements permitted by the Constitution. In a
CPA that the government enters into with one or more contractors, the government shall provide inputs
to the mining operations other than the mineral resource itself . 94
In a JVA, a JV company is organized by the government and the contractor, with both parties having
equity shares (investments); and the contractor is granted the exclusive right to conduct mining
operations and to extract minerals found in the area. 95 On the other hand, in an MPSA, the government
grants the contractor the exclusive right to conduct mining operations within the contract area and
shares in the gross output; and the contractor provides the necessary financing, technology,
management and manpower.
The point being made here is that, in two of the three types of agreements under consideration, the
government has to ante up some risk capital for the enterprise. In other words, government funds (public
moneys) are withdrawn from other possible uses, put to work in the venture and placed at risk in case
the venture fails. This notwithstanding, management and control of the operations of the enterprise are
in all three arrangements in the hands of the contractor, with the government being mainly a
silent partner. The three types of agreement mentioned above apply to any natural resource, without
limitation and regardless of the size or magnitude of the project or operations.
In contrast to the foregoing arrangements, and pursuant to paragraph 4 of Section 2 of Article XII, the
FTAA is limited to large-scale projects and only for minerals, petroleum and other mineral oils. Here, the
Constitution removes the 40 percent cap on foreign ownership and allows the foreign corporation to own
up to 100 percent of the equity. Filipino capital may not be sufficient on account of the size of the
project, so the foreign entity may have to ante up all the risk capital.
Correlatively, the foreign stakeholder bears up to 100 percent of the risk of loss if the project fails. In
respect of the particular FTAA granted to it, WMCP (then 100 percent foreign owned) was responsible, as
contractor, for providing the entire equity, including all the inputs for the project. It was to bear 100
percent of the risk of loss if the project failed, but its maximum potential "beneficial interest" consisted
only of 40 percent of the net beneficial interest, because the other 60 percent is the share of the
government, which will never be exposed to any risk of loss whatsoever.
In consonance with the degree of risk assumed, the FTAA vested in WMCP the day-to-day management
of the mining operations. Still such management is subject to the overall control and supervision of the
79
State in terms of regular reporting, approvals of work programs and budgets, and so on.
So, one needs to consider in relative terms, the costs of inputs for, degree of risk attendant to, and
benefits derived or to be derived from a CPA, a JVA or an MPSA vis- -vis those pertaining to an FTAA. It
may not be realistically asserted that the foreign grantee of an FTAA is being unduly favored or benefited
as compared with a foreign stakeholder in a corporation holding a CPA, a JVA or an MPSA. Seen the
other way around, the government is definitely better off with an FTAA than a CPA, a JVA or an MPSA.
Developmental Policyon the Mining Industry
During the Oral Argument and in their Final Memorandum, petitioners repeatedly urged the Court to
consider whether mining as an industry and economic activity deserved to be accorded priority,
preference and government support as against, say, agriculture and other activities in which Filipinos and
the Philippines may have an "economic advantage." For instance, a recent US study 96 reportedly
examined the economic performance of all local US counties that were dependent on mining and 20
percent of whose labor earnings between 1970 and 2000 came from mining enterprises. AaDSEC
The study covering 100 US counties in 25 states dependent on mining showed that per capita
income grew about 30 percent less in mining-dependent communities in the 1980s and 25 percent less
for the entire period 1980 to 2000; the level of per capita income was also lower. Therefore, given the
slower rate of growth, the gap between these and other local counties increased.

Petitioners invite attention to the OXFAM America Report's warning to developing nations that mining
brings with it serious economic problems, including increased regional inequality, unemployment and
poverty. They also cite the final report 97 of the Extractive Industries Review project commissioned by
the World Bank (the WB-EIR Report), which warns of environmental degradation, social disruption,
conflict, and uneven sharing of benefits with local communities that bear the negative social and
environmental impact. The Report suggests that countries need to decide on the best way to exploit their
natural resources, in order to maximize the value added from the development of their resources and
ensure that they are on the path to sustainable development once the resources run out.
Whatever priority or preference may be given to mining vis- -vis other economic or non-economic
activities is a question of policy that the President and Congress will have to address; it is not for this
Court to decide. This Court declares what the Constitution and the laws say, interprets only when
necessary, and refrains from delving into matters of policy.
Suffice it to say that the State control accorded by the Constitution over mining activities assures a
proper balancing of interests. More pointedly, such control will enable the President to demand the best
mining practices and the use of the best available technologies to protect the environment and to
rehabilitate mined-out areas. Indeed, under the Mining Law, the government can ensure the protection
of the environment during and after mining. It can likewise provide for the mechanisms to protect the
rights of indigenous communities, and thereby mold a more socially-responsive, culturally-sensitive and
sustainable mining industry.
Early on during the launching of the Presidential Mineral Industry Environmental Awards on February 6,
1997, then President Fidel V. Ramos captured the essence of balanced and sustainable mining in these
words:
"Long term, high profit mining translates into higher revenues for
government, more decent jobs for the population, more raw materials to
feed the engines of downstream and allied industries, and improved
chances of human resource and countryside development by creating self-
reliant communities away from urban centers.
xxx xxx xxx
"Against a fragile and finite environment, it is sustainability that holds the
key. In sustainable mining, we take a middle ground where both
production and protection goals are balanced, and where parties-in-
interest come to terms."
Neither has the present leadership been remiss in addressing the concerns of sustainable mining
operations. Recently, on January 16, 2004 and April 20, 2004, President Gloria Macapagal Arroyo issued
Executive Orders Nos. 270 and 270-A, respectively, "to promote responsible mineral resources
exploration, development and utilization, in order to enhance economic growth, in a manner that adheres
to the principles of sustainable development and with due regard for justice and equity, sensitivity to the
culture of the Filipino people and respect for Philippine sovereignty." 98
Refutation Of Dissents
The Court will now take up a number of other specific points raised in the dissents of Justices Carpio and
Morales.
1.Justice Morales introduced us to Hugh Morgan, former president and chief executive officer of Western
Mining Corporation (WMC) and former president of the Australian Mining Industry Council, who
spearheaded the vociferous opposition to the filing by aboriginal peoples of native title claims against
mining companies in Australia in the aftermath of the landmark Mabo decision by the Australian High
Court. According to sources quoted by our esteemed colleague, Morgan was also a racist and a bigot. In
the course of protesting Mabo, Morgan allegedly uttered derogatory remarks belittling the aboriginal
culture and race.
An unwritten caveat of this introduction is that this Court should be careful not to permit the entry of the
likes of Hugh Morgan and his hordes of alleged racist-bigots at WMC. With all due respect, such scare
tactics should have no place in the discussion of this case. We are deliberating on the constitutionality of
RA 7942, DAO 96-40 and the FTAA originally granted to WMCP, which had been transferred to Sagittarius
Mining, a Filipino corporation. We are not discussing the apparition of white Anglo-Saxon racists/bigots
massing at our gates.
2.On the proper interpretation of the phrase agreements involving either technical or financial assistance,
Justice Morales points out that at times we "conveniently omitted" the use of the disjunctive either . . .
or, which according to her denotes restriction; hence the phrase must be deemed to connote restriction
and limitation.
But, as Justice Carpio himself pointed out during the Oral Argument, the disjunctive phrase either
technical or financial assistance would, strictly speaking, literally mean that a foreign contractor may
provide only one or the other, but not both. And if both technical and financial assistance were required
for a project, the State would have to deal with at least two different foreign contractors one for
financial and the other for technical assistance. And following on that, a foreign contractor, though very
much qualified to provide both kinds of assistance, would nevertheless be prohibited from providing one
kind as soon as it shall have agreed to provide the other.
But if the Court should follow this restrictive and literal construction, can we really find two (or more)
contractors who are willing to participate in one single project one to provide the "financial
assistance" only and the other the "technical assistance" exclusively; it would be excellent if these two or
more contractors happen to be willing and are able to cooperate and work closely together on the same
project (even if they are otherwise competitors). And it would be superb if no conflicts would arise
between or among them in the entire course of the contract. But what are the chances things will turn
out this way in the real world? To think that the framers deliberately imposed this kind of restriction is to
say that they were either exceedingly optimistic, or incredibly naive. This begs the question What
laudable objective or purpose could possibly be served by such strict and restrictive literal interpretation?
3.Citing Oposa v. Factoran Jr., Justice Morales claims that a service contract is not a contract or property
right which merits protection by the due process clause of the Constitution, but merely a license or
privilege which may be validly revoked, rescinded or withdrawn by executive action whenever dictated by
public interest or public welfare.
Oposa cites Tan v. Director of Forestry and Ysmael v. Deputy Executive Secretary as authority. The latter
cases dealt specifically with timber licenses only. Oposa allegedly reiterated that a license is merely a
80
permit or privilege to do what otherwise would be unlawful, and is not a contract between the authority,
federal, state or municipal, granting it and the person to whom it is granted; neither is it property or a
property right, nor does it create a vested right; nor is it taxation. Thus this Court held that the granting
of license does not create irrevocable rights, neither is it property or property rights.
Should Oposa be deemed applicable to the case at bar, on the argument that natural resources are also
involved in this situation? We do not think so. A grantee of a timber license, permit or license agreement
gets to cut the timber already growing on the surface; it need not dig up tons of earth to get at the logs.
In a logging concession, the investment of the licensee is not as substantial as the investment of a large-
scale mining contractor. If a timber license were revoked, the licensee packs up its gear and moves to a
new area applied for, and starts over; what it leaves behind are mainly the trails leading to the logging
site.
In contrast, the mining contractor will have sunk a great deal of money (tens of millions of dollars) into
the ground, so to speak, for exploration activities, for development of the mine site and infrastructure,
and for the actual excavation and extraction of minerals, including the extensive tunneling work to reach
the ore body. The cancellation of the mining contract will utterly deprive the contractor of its investments
(i.e., prevent recovery of investments), most of which cannot be pulled out.
To say that an FTAA is just like a mere timber license or permit and does not involve contract or property
rights which merit protection by the due process clause of the Constitution, and may therefore be
revoked or cancelled in the blink of an eye, is to adopt a well-nigh confiscatory stance; at the very least,
it is downright dismissive of the property rights of businesspersons and corporate entities that have
investments in the mining industry, whose investments, operations and expenditures do contribute to the
general welfare of the people, the coffers of government, and the strength of the economy. Such a
pronouncement will surely discourage investments (local and foreign) which are critically needed to fuel
the engine of economic growth and move this country out of the rut of poverty. In sum, Oposa is not
applicable.
4.Justice Morales adverts to the supposedly "clear intention" of the framers of the Constitution to reserve
our natural resources exclusively for the Filipino people. She then quoted from the records of the
ConCom deliberations a passage in which then Commissioner Davide explained his vote, arguing in the
process that aliens ought not be allowed to participate in the enjoyment of our natural resources. One
passage does not suffice to capture the tenor or substance of the entire extensive deliberations of the
commissioners, or to reveal the clear intention of the framers as a group. A re-reading of the entire
deliberations (quoted here earlier) is necessary if we are to understand the true intent of the framers.

5.Since 1935, the Filipino people, through their Constitution, have decided that the retardation or delay in
the exploration, development or utilization of the nation's natural resources is merely secondary to the
protection and preservation of their ownership of the natural resources, so says Justice Morales, citing
Aruego. If it is true that the framers of the 1987 Constitution did not care much about alleviating the
retardation or delay in the development and utilization of our natural resources, why did they bother to
write paragraph 4 at all? Were they merely paying lip service to large-scale exploration, development and
utilization? They could have just completely ignored the subject matter and left it to be dealt with
through a future constitutional amendment. But we have to harmonize every part of the Constitution and
to interpret each provision in a manner that would give life and meaning to it and to the rest of the
provisions. It is obvious that a literal interpretation of paragraph 4 will render it utterly inutile and
inoperative.
6.According to Justice Morales, the deliberations of the Constitutional Commission do not support our
contention that the framers, by specifying such agreements involving financial or technical assistance,
necessarily gave implied assent to everything that these agreements implicitly entailed, or that could
reasonably be deemed necessary to make them tenable and effective, including management authority in
the day-to-day operations. As proof thereof, she quotes one single passage from the ConCom
deliberations, consisting of an exchange among Commissioners Tingson, Garcia and Monsod.
However, the quoted exchange does not serve to contradict our argument; it even bolsters it. Comm.
Christian Monsod was quoted as saying: ". . . I think we have to make a distinction that it is not really
realistic to say that we will borrow on our own terms. Maybe we can say that we inherited unjust loans,
and we would like to repay these on terms that are not prejudicial to our own growth. But the general
statement that we should only borrow on our own terms is a bit unrealistic." Comm. Monsod is one who
knew whereof he spoke.
7.Justice Morales also declares that the optimal time for the conversion of an FTAA into an MPSA is after
completion of the exploration phase and just before undertaking the development and construction
phase, on account of the fact that the requirement for a minimum investment of $50 million is applicable
only during the development, construction and utilization phase, but not during the exploration phase,
when the foreign contractor need merely comply with minimum ground expenditures. Thus by
converting, the foreign contractor maximizes its profits by avoiding its obligation to make the minimum
investment of $50 million.
This argument forgets that the foreign contractor is in the game precisely to make money. In order to
come anywhere near profitability, the contractor must first extract and sell the mineral ore. In order to do
that, it must also develop and construct the mining facilities, set up its machineries and equipment and
dig the tunnels to get to the deposit. The contractor is thus compelled to expend funds in order to make
profits. If it decides to cut back on investments and expenditures, it will necessarily sacrifice the pace of
development and utilization; it will necessarily sacrifice the amount of profits it can make from the mining
operations. In fact, at certain less-than-optimal levels of operation, the stream of revenues generated
may not even be enough to cover variable expenses, let alone overhead expenses; this is a dismal
situation anyone would want to avoid. In order to make money, one has to spend money. This truism
applies to the mining industry as well.
8.Mortgaging the minerals to secure a foreign FTAA contractor's obligations is anomalous, according to
Justice Morales since the contractor was from the beginning obliged to provide all financing needed for
the mining operations. However, the mortgaging of minerals by the contractor does not necessarily
signify that the contractor is unable to provide all financing required for the project, or that it does not
have the financial capability to undertake large-scale operations. Mortgaging of mineral products, just like
the assignment (by way of security) of manufactured goods and goods in inventory, and the assignment
of receivables, is an ordinary requirement of banks, even in the case of clients with more than sufficient
financial resources. And nowadays, even the richest and best managed corporations make use of bank
credit facilities it does not necessarily signify that they do not have the financial resources or are
unable to provide the financing on their own; it is just a manner of maximizing the use of their funds.
9.Does the contractor in reality acquire the surface rights "for free," by virtue of the fact that it is entitled
to reimbursement for the costs of acquisition and maintenance, adjusted for inflation? We think not. The
"reimbursement" is possible only at the end of the term of the contract, when the surface rights will no
longer be needed, and the land previously acquired will have to be disposed of, in which case the
contractor gets reimbursement from the sales proceeds. The contractor has to pay out the acquisition
price for the land. That money will belong to the seller of the land. Only if and when the land is finally
sold off will the contractor get any reimbursement. In other words, the contractor will have been cash-
out for the entire duration of the term of the contract 25 or 50 years, depending. If we calculate the
cost of money at say 12 percent per annum, that is the cost or opportunity loss to the contractor, in
addition to the amount of the acquisition price. 12 percent per annum for 50 years is 600 percent; this,
without any compounding yet. The cost of money is therefore at least 600 percent of the original
acquisition cost; it is in addition to the acquisition cost. "For free?" Not by a long shot. CAcEaS
10.The contractor will acquire and hold up to 5,000 hectares? We doubt it. The acquisition by the State
of land for the contractor is just to enable the contractor to establish its mine site, build its facilities,
establish a tailings pond, set up its machinery and equipment, and dig mine shafts and tunnels, etc. It is
impossible that the surface requirement will aggregate 5,000 hectares. Much of the operations will
consist of the tunneling and digging underground, which will not require possessing or using any land
surface. 5,000 hectares is way too much for the needs of a mining operator. It simply will not spend its
cash to acquire property that it will not need; the cash may be better employed for the actual mining
operations, to yield a profit.
11.Justice Carpio claims that the phrase among other things (found in the second paragraph of Section
81 of the Mining Act) is being incorrectly treated as a delegation of legislative power to the DENR
secretary to issue DAO 99-56 and prescribe the formulae therein on the State's share from mining
81
operations. He adds that the phrase among other things was not intended as a delegation of legislative
power to the DENR secretary, much less could it be deemed a valid delegation of legislative power, since
there is nothing in the second paragraph of Section 81 which can be said to grant any delegated
legislative power to the DENR secretary. And even if there were, such delegation would be void, for lack
of any standards by which the delegated power shall be exercised.
While there is nothing in the second paragraph of Section 81 which can directly be construed as a
delegation of legislative power to the DENR secretary, it does not mean that DAO 99-56 is invalid per se,
or that the secretary acted without any authority or jurisdiction in issuing DAO 99-56. As we stated earlier
in our Prologue, "Who or what organ of government actually exercises this power of control on behalf of
the State? The Constitution is crystal clear: the President. Indeed, the Chief Executive is the official
constitutionally mandated to 'enter into agreements with foreign owned corporations.' On the other hand,
Congress may review the action of the President once it is notified of 'every contract entered into in
accordance with this [constitutional] provision within thirty days from its execution.'" It is the President
who is constitutionally mandated to enter into FTAAs with foreign corporations, and in doing so, it is
within the President's prerogative to specify certain terms and conditions of the FTAAs, for example, the
fiscal regime of FTAAs i.e., the sharing of the net mining revenues between the contractor and the
State.
Being the President's alter ego with respect to the control and supervision of the mining industry, the
DENR secretary, acting for the President, is necessarily clothed with the requisite authority and power to
draw up guidelines delineating certain terms and conditions, and specifying therein the terms of sharing
of benefits from mining, to be applicable to FTAAs in general. It is important to remember that DAO 99-
56 has been in existence for almost six years, and has not been amended or revoked by the President.
The issuance of DAO 99-56 did not involve the exercise of delegated legislative power. The legislature did
not delegate the power to determine the nature, extent and composition of the items that would come
under the phrase among other things. The legislature's power pertains to the imposition of taxes, duties
and fees. This power was not delegated to the DENR secretary. But the power to negotiate and enter
into FTAAs was withheld from Congress, and reserved for the President. In determining the sharing of
mining benefits, i.e., in specifying what the phrase among other things include, the President (through
the secretary acting in his/her behalf) was not determining the amount or rate of taxes, duties and fees,
but rather the amount of INCOME to be derived from minerals to be extracted and sold, income which
belongs to the State as owner of the mineral resources. We may say that, in the second paragraph of
Section 81, the legislature in a sense intruded partially into the President's sphere of authority when the
former provided that

"The Government share in financial or technical assistance agreement shall
consist of, among other things, the contractor's corporate income tax,
excise tax, special allowance, withholding tax due from the contractor's
foreign stockholders arising from dividend or interest payments to the said
foreign stockholder in case of a foreign national and all such other taxes,
duties and fees as provided for under existing laws." (Italics supplied)
But it did not usurp the President's authority since the provision merely included the enumerated items as
part of the government share, without foreclosing or in any way preventing (as in fact Congress could
not validly prevent) the President from determining what constitutes the State's compensation derived
from FTAAs. In this case, the President in effect directed the inclusion or addition of "other things," viz.,
INCOME for the owner of the resources, in the government's share, while adopting the items enumerated
by Congress as part of the government share also.
12.Justice Carpio's insistence on applying the ejusdem generis rule of statutory construction to the
phrase among other things is therefore useless, and must fall by the wayside. There is no point trying to
construe that phrase in relation to the enumeration of taxes, duties and fees found in paragraph 2 of
Section 81, precisely because "the constitutional power to prescribe the sharing of mining income
between the State and mining companies," to quote Justice Carpio pursuant to an FTAA is constitutionally
lodged with the President, not with Congress. It thus makes no sense to persist in giving the phrase
among other things a restricted meaning referring only to taxes, duties and fees.
13.Strangely, Justice Carpio claims that the DENR secretary can change the formulae in DAO 99-56 any
time even without the approval of the President, and the secretary is the sole authority to determine the
amount of consideration that the State shall receive in an FTAA, because Section 5 of the DAO states that
". . . any amendment of an FTAA other than the provision on fiscal regime shall require the negotiation
with the Negotiation Panel and the recommendation of the Secretary for approval of the President . . .".
Allegedly, because of that provision, if an amendment in the FTAA involves non-fiscal matters, the
amendment requires approval of the President, but if the amendment involves a change in the fiscal
regime, the DENR secretary has the final authority, and approval of the President may be dispensed with;
hence the secretary is more powerful than the President.
We believe there is some distortion resulting from the quoted provision being taken out of context.
Section 5 of DAO 99-56 reads as follows:
"Section 5.Status of Existing FTAAs. All FTAAs approved prior to the
effectivity of this Administrative Order shall remain valid and be recognized
by the Government: Provided, That should a Contractor desire to amend
its FTAA, it shall do so by filing a Letter of Intent (LOI) to the Secretary
thru the Director. Provided, further, That if the Contractor desires to
amend the fiscal regime of its FTAA, it may do so by seeking for the
amendment of its FTAA's whole fiscal regime by adopting the fiscal regime
provided hereof : Provided, finally, That any amendment of an FTAA other
than the provision on fiscal regime shall require the negotiation with the
Negotiating Panel and the recommendation of the Secretary for approval
of the President of the Republic of the Philippines." (emphasis supplied)
It looks like another case of misapprehension. The proviso being objected to by Justice Carpio is actually
preceded by a phrase that requires a contractor desiring to amend the fiscal regime of its FTAA, to
amend the same by adopting the fiscal regime prescribed in DAO 99-56 i.e., solely in that manner,
and in no other. Obviously, since DAO 99-56 was issued by the secretary under the authority and with
the presumed approval of the President, the amendment of an FTAA by merely adopting the fiscal regime
prescribed in said DAO 99-56 (and nothing more) need not have the express clearance of the President
anymore. It is as if the same had been pre-approved. We cannot fathom the complaint that that makes
the secretary more powerful than the President, or that the former is trying to hide things from the
President or Congress.
14.Based on the first sentence of Section 5 of DAO 99-56, which states "[A]ll FTAAs approved prior to the
effectivity of this Administrative Order shall remain valid and be recognized by the Government", Justice
Carpio concludes that said Administrative Order allegedly exempts FTAAs approved prior to its effectivity
like the WMCP FTAA from having to pay the State any share from their mining income, apart
from taxes, duties and fees.
We disagree. What we see in black and white is the statement that the FTAAs approved before the DAO
came into effect are to continue to be valid and will be recognized by the State. Nothing is said about
their fiscal regimes. Certainly, there is no basis to claim that the contractors under said FTAAs were being
exempted from paying the government a share in their mining incomes.
For the record, the WMCP FTAA is NOT and has never been exempt from paying the government share.
The WMCP FTAA has its own fiscal regime Section 7.7 which gives the government a 60 percent
share in the net mining revenues of WMCP from the commencement of commercial production.
For that very reason, we have never said that DAO 99-56 is the basis for claiming that the WMCP FTAA
has a consideration. Hence, we find quite out of place Justice Carpio's statement that ironically, DAO 99-
56, the very authority cited to support the claim that the WMCP FTAA has a consideration, does not apply
to the WMCP FTAA. By its own express terms, DAO 99-56 does not apply to FTAAs executed before the
issuance of DAO 99-56, like the WMCP FTAA. The majority's position has allegedly no leg to stand on
since even DAO 99-56, assuming it is valid, cannot save the WMCP FTAA from want of consideration.
Even assuming arguendo that DAO 99-56 does not apply to the WMCP FTAA, nevertheless, the WMCP
FTAA has its own fiscal regime, found in Section 7.7 thereof. Hence, there is no such thing as "want of
consideration" here.
82
Still more startling is this claim: The majority supposedly agrees that the provisions of the WMCP FTAA,
which grant a sham consideration to the State, are void. Since the majority agrees that the WMCP FTAA
has a sham consideration, the WMCP FTAA thus lacks the third element of a valid contract. The Decision
should declare the WMCP FTAA void for want of consideration unless it treats the contract as an MPSA
under Section 80. Indeed the only recourse of WMCP to save the validity of its contract is to convert it
into an MPSA.
To clarify, we said that Sections 7.9 and 7.8(e) of the WMCP FTAA are provisions grossly
disadvantageous to government and detrimental to the interests of the Filipino people, as well as
violative of public policy, and must therefore be stricken off as invalid. Since the offending provisions are
very much separable from Section 7.7 and the rest of the FTAA, the deletion of Sections 7.9 and 7.8(e)
can be done without affecting or requiring the invalidation of the WMCP FTAA itself, and such deletion
will preserve for government its due share of the 60 percent benefits. Therefore, the WMCP FTAA is NOT
bereft of a valid consideration (assuming for the nonce that indeed this is the "consideration" of the
FTAA).
Summation
To conclude, a summary of the key points discussed above is now in order.
The Meaning of "Agreements InvolvingEither Technical or Financial Assistance"
Applying familiar principles of constitutional construction to the phrase agreements involving either
technical or financial assistance, the framers' choice of words does not indicate the intent to exclude
other modes of assistance, but rather implies that there are other things being included or possibly being
made part of the agreement, apart from financial or technical assistance. The drafters avoided the use of
restrictive and stringent phraseology; a verba legis scrutiny of Section 2 of Article XII of the Constitution
discloses not even a hint of a desire to prohibit foreign involvement in the management or operation of
mining activities, or to eradicate service contracts. Such moves would necessarily imply an underlying
drastic shift in fundamental economic and developmental policies of the State. That change requires a
much more definite and irrefutable basis than mere omission of the words "service contract" from the
new Constitution.
Furthermore, a literal and restrictive interpretation of this paragraph leads to logical inconsistencies. A
constitutional provision specifically allowing foreign-owned corporations to render financial or technical
assistance in respect of mining or any other commercial activity was clearly unnecessary; the provision
was meant to refer to more than mere financial or technical assistance.
Also, if paragraph 4 permits only agreements for financial or technical assistance, there would be no
point in requiring that they be "based on real contributions to the economic growth and general welfare
of the country." And considering that there were various long-term service contracts still in force and
effect at the time the new Charter was being drafted, the absence of any transitory provisions to govern
the termination and closing-out of the then existing service contracts strongly militates against the theory
that the mere omission of "service contracts" signaled their prohibition by the new Constitution.
Resort to the deliberations of the Constitutional Commission is therefore unavoidable, and a careful
scrutiny thereof conclusively shows that the ConCom members discussed agreements involving either
technical or financial assistance in the same sense as service contracts, and used the terms
interchangeably. The drafters in fact knew that the agreements with foreign corporations were going to
entail not mere technical or financial assistance, but rather, foreign investment in and management of an
enterprise for large-scale exploration, development and utilization of minerals.

The framers spoke about service contracts as the concept was understood in the 1973 Constitution. It is
obvious from their discussions that they did not intend to ban or eradicate service contracts. Instead,
they were intent on crafting provisions to put in place safeguards that would eliminate or minimize the
abuses prevalent during the marital law regime. In brief, they were going to permit service contracts with
foreign corporations as contractors, but with safety measures to prevent abuses, as an exception to the
general norm established in the first paragraph of Section 2 of Article XII which reserves or limits to
Filipino citizens and corporations at least 60 percent owned by such citizens the exploration, development
and utilization of mineral or petroleum resources. This was prompted by the perceived insufficiency of
Filipino capital and the felt need for foreign expertise in the EDU of mineral resources.
Despite strong opposition from some ConCom members during the final voting, the Article on the
National Economy and Patrimony including paragraph 4 allowing service contracts with foreign
corporations as an exception to the general norm in paragraph 1 of Section 2 of the same Article was
resoundingly and overwhelmingly approved.
The drafters, many of whom were economists, academicians, lawyers, businesspersons and politicians
knew that foreign entities will not enter into agreements involving assistance without requiring measures
of protection to ensure the success of the venture and repayment of their investments, loans and other
financial assistance, and ultimately to protect the business reputation of the foreign corporations. The
drafters, by specifying such agreements involving assistance, necessarily gave implied assent to
everything that these agreements entailed or that could reasonably be deemed necessary to make them
tenable and effective including management authority with respect to the day-to-day operations of
the enterprise, and measures for the protection of the interests of the foreign corporation, at least to the
extent that they are consistent with Philippine sovereignty over natural resources, the constitutional
requirement of State control, and beneficial ownership of natural resources remaining vested in the State.
From the foregoing, it is clear that agreements involving either technical or financial assistance referred
to in paragraph 4 are in fact service contracts, but such new service contracts are between foreign
corporations acting as contractors on the one hand, and on the other hand government as principal or
"owner" (of the works), whereby the foreign contractor provides the capital, technology and technical
know-how, and managerial expertise in the creation and operation of the large-scale mining/extractive
enterprise, and government through its agencies (DENR, MGB) actively exercises full control and
supervision over the entire enterprise.
Such service contracts may be entered into only with respect to minerals, petroleum and other mineral
oils. The grant of such service contracts is subject to several safeguards, among them: (1) that the
service contract be crafted in accordance with a general law setting standard or uniform terms,
conditions and requirements; (2) the President be the signatory for the government; and (3) the
President report the executed agreement to Congress within thirty days.
Ultimate Test: Full State Control
To repeat, the primacy of the principle of the State's sovereign ownership of all mineral resources, and its
full control and supervision over all aspects of exploration, development and utilization of natural
resources must be upheld. But "full control and supervision" cannot be taken literally to mean that the
State controls and supervises everything down to the minutest details and makes all required actions, as
this would render impossible the legitimate exercise by the contractor of a reasonable degree of
management prerogative and authority, indispensable to the proper functioning of the mining enterprise.
Also, government need not micro-manage mining operations and day-to-day affairs of the enterprise in
order to be considered as exercising full control and supervision.
Control, as utilized in Section 2 of Article XII, must be taken to mean a degree of control sufficient to
enable the State to direct, restrain, regulate and govern the affairs of the extractive enterprises. Control
by the State may be on a macro level, through the establishment of policies, guidelines, regulations,
industry standards and similar measures that would enable government to regulate the conduct of affairs
in various enterprises, and restrain activities deemed not desirable or beneficial, with the end in view of
ensuring that these enterprises contribute to the economic development and general welfare of the
country, conserve the environment, and uplift the well-being of the local affected communities. Such a
degree of control would be compatible with permitting the foreign contractor sufficient and reasonable
management authority over the enterprise it has invested in, to ensure efficient and profitable operation.
ITCHSa
Government Granted Full Control by RA 7942 and DAO 96-40
Baseless are petitioners' sweeping claims that RA 7942 and its Implementing Rules and Regulations make
it possible for FTAA contracts to cede full control and management of mining enterprises over to fully
83
foreign owned corporations. Equally wobbly is the assertion that the State is reduced to a passive
regulator dependent on submitted plans and reports, with weak review and audit powers and little say in
the decision-making of the enterprise, for which reasons "beneficial ownership" of the mineral resources
is allegedly ceded to the foreign contractor.
As discussed hereinabove, the State's full control and supervision over mining operations are ensured
through the following provisions in RA 7942: Sections 8, 9, 16, 19, 24, 35[(b), (e), (f), (g), (h), (k), (l),
(m) and (o)], 40, 57, 66, 69, 70, and Chapters XI and XVII, as well as the following provisions of the
DAO 96-40: Sections 7[(d) and (f)], 35(a-2), 53[(a-4) and (d)], 54, 56[(g), (h), (l), (m) and (n)], 56(2),
60, 66, 144, 168, 171 and 270, and also Chapters XV, XVI and XXIV.
Through the foregoing provisions, the government agencies concerned are empowered to approve or
disapprove hence in a position to influence, direct, and change the various work programs and
the corresponding minimum expenditures commitments for each of the exploration, development and
utilization phases of the enterprise. Once they have been approved, the contractor's compliance with its
commitments therein will be monitored. Figures for mineral production and sales are regularly monitored
and subjected to government review, to ensure that the products and by-products are disposed of at the
best prices; copies of sales agreements have to be submitted to and registered with MGB.
The contractor is mandated to open its books of accounts and records for scrutiny, to enable the State to
determine that the government share has been fully paid. The State may likewise compel compliance by
the contractor with mandatory requirements on mine safety, health and environmental protection, and
the use of anti-pollution technology and facilities. The contractor is also obligated to assist the
development of the mining community, and pay royalties to the indigenous peoples concerned. And
violation of any of the FTAA's terms and conditions, and/or non-compliance with statutes or regulations,
may be penalized by cancellation of the FTAA. Such sanction is significant to a contractor who may have
yet to recover the tens or hundreds of millions of dollars sunk into a mining project.
Overall, the State definitely has a pivotal say in the operation of the individual enterprises, and can set
directions and objectives, detect deviations and non-compliances by the contractor, and enforce
compliance and impose sanctions should the occasion arise. Hence, RA 7942 and DAO 96-40 vest in
government more than a sufficient degree of control and supervision over the conduct of mining
operations.
Section 3(aq) of RA 7942 was objected to as being unconstitutional for allowing a foreign contractor to
apply for and hold an exploration permit. During the exploration phase, the permit grantee (and
prospective contractor) is spending and investing heavily in exploration activities without yet being able
to extract minerals and generate revenues. The exploration permit issued under Sections 3(aq), 20 and
23 of RA 7942, which allows exploration but not extraction, serves to protect the interests and rights of
the exploration permit grantee (and would-be contractor), foreign or local. Otherwise, the exploration
works already conducted, and expenditures already made, may end up only benefiting claim-jumpers.
Thus, Section 3(aq) of RA 7942 is not unconstitutional.
WMCP FTAA Likewise Gives theState Full Control and Supervision
The WMCP FTAA obligates the contractor to account for the value of production and sale of minerals
(Clause 1.4); requires that the contractor's work program, activities and budgets be approved by the
State (Clause 2.1); gives the DENR secretary power to extend the exploration period (Clause 3.2-a);
requires approval by the State for incorporation of lands into the contract area (Clause 4.3-c); requires
Bureau of Forest Development approval for inclusion of forest reserves as part of the FTAA contract area
(Clause 4.5); obligates the contractor to periodically relinquish parts of the contract area not needed for
exploration and development (Clause 4.6); requires submission of a declaration of mining feasibility for
approval by the State (Clause 4.6-b); obligates the contractor to report to the State the results of its
exploration activities (Clause 4.9); requires the contractor to obtain State approval for its work programs
for the succeeding two year periods, containing the proposed work activities and expenditures budget
related to exploration (Clause 5.1); requires the contractor to obtain State approval for its proposed
expenditures for exploration activities (Clause 5.2); requires the contractor to submit an annual report on
geological, geophysical, geochemical and other information relating to its explorations within the FTAA
area (Clause 5.3-a); requires the contractor to submit within six months after expiration of exploration
period a final report on all its findings in the contract area (Clause 5.3-b); requires the contractor after
conducting feasibility studies to submit a declaration of mining feasibility, along with a description of the
area to be developed and mined, a description of the proposed mining operations and the technology to
be employed, and the proposed work program for the development phase, for approval by the DENR
secretary (Clause 5.4); obligates the contractor to complete the development of the mine, including
construction of the production facilities, within the period stated in the approved work program (Clause
6.1); requires the contractor to submit for approval a work program covering each period of three fiscal
years (Clause 6.2); requires the contractor to submit reports to the secretary on the production, ore
reserves, work accomplished and work in progress, profile of its work force and management staff, and
other technical information (Clause 6.3); subjects any expansions, modifications, improvements and
replacements of mining facilities to the approval of the secretary (Clause 6.4); subjects to State control
the amount of funds that the contractor may borrow within the Philippines (Clause 7.2); subjects to State
supervisory power any technical, financial and marketing issues (Clause 10.1-a); obligates the contractor
to ensure 60 percent Filipino equity in the contractor within ten years of recovering specified
expenditures unless not so required by subsequent legislation (Clause 10.1); gives the State the right to
terminate the FTAA for unremedied substantial breach thereof by the contractor (Clause 13.2); requires
State approval for any assignment of the FTAA by the contractor to an entity other than an affiliate
(Clause 14.1).

In short, the aforementioned provisions of the WMCP FTAA, far from constituting a surrender of control
and a grant of beneficial ownership of mineral resources to the contractor in question, vest the State with
control and supervision over practically all aspects of the operations of the FTAA contractor, including the
charging of pre-operating and operating expenses, and the disposition of mineral products.
There is likewise no relinquishment of control on account of specific provisions of the WMCP FTAA.
Clause 8.2 provides a mechanism to prevent the mining operations from grinding to a complete halt as a
result of possible delays of more than 60 days in the government's processing and approval of submitted
work programs and budgets. Clause 8.3 seeks to provide a temporary, stop-gap solution in case a
disagreement between the State and the contractor (over the proposed work program or budget
submitted by the contractor) should result in a deadlock or impasse, to avoid unreasonably long delays in
the performance of the works.
The State, despite Clause 8.3, still has control over the contract area, and it may, as sovereign authority,
prohibit work thereon until the dispute is resolved, or it may terminate the FTAA, citing substantial breach
thereof. Hence, the State clearly retains full and effective control.
Clause 8.5, which allows the contractor to make changes to approved work programs and budgets
without the prior approval of the DENR secretary, subject to certain limitations with respect to the
variance/s, merely provides the contractor a certain amount of flexibility to meet unexpected situations,
while still guaranteeing that the approved work programs and budgets are not abandoned altogether.
And if the secretary disagrees with the actions taken by the contractor in this instance, he may also
resort to cancellation/termination of the FTAA as the ultimate sanction.
Clause 4.6 of the WMCP FTAA gives the contractor discretion to select parts of the contract area to be
relinquished. The State is not in a position to substitute its judgment for that of the contractor, who
knows exactly which portions of the contract area do not contain minerals in commercial quantities and
should be relinquished. Also, since the annual occupation fees paid to government are based on the total
hectarage of the contract area, net of the areas relinquished, the contractor's self-interest will assure
proper and efficient relinquishment.
Clause 10.2(e) of the WMCP FTAA does not mean that the contractor can compel government to use its
power of eminent domain. It contemplates a situation in which the contractor is a foreign-owned
corporation, hence, not qualified to own land. The contractor identifies the surface areas needed for it to
construct the infrastructure for mining operations, and the State then acquires the surface rights on
behalf of the former. The provision does not call for the exercise of the power of eminent domain (or
determination of just compensation); it seeks to avoid a violation of the anti-dummy law.
Clause 10.2(l) of the WMCP FTAA giving the contractor the right to mortgage and encumber the mineral
products extracted may have been a result of conditions imposed by creditor-banks to secure the loan
84
obligations of WMCP. Banks lend also upon the security of encumbrances on goods produced, which can
be easily sold and converted into cash and applied to the repayment of loans. Thus, Clause 10.2(l) is not
something out of the ordinary. Neither is it objectionable, because even though the contractor is allowed
to mortgage or encumber the mineral end-products themselves, the contractor is not thereby relieved of
its obligation to pay the government its basic and additional shares in the net mining revenue. The
contractor's ability to mortgage the minerals does not negate the State's right to receive its share of net
mining revenues.
Clause 10.2(k) which gives the contractor authority "to change its equity structure at any time," means
that WMCP, which was then 100 percent foreign owned, could permit Filipino equity ownership.
Moreover, what is important is that the contractor, regardless of its ownership, is always in a position to
render the services required under the FTAA, under the direction and control of the government.
Clauses 10.4(e) and (i) bind government to allow amendments to the FTAA if required by banks and
other financial institutions as part of the conditions of new lendings. There is nothing objectionable here,
since Clause 10.4(e) also provides that such financing arrangements should in no event reduce the
contractor's obligations or the government's rights under the FTAA. Clause 10.4(i) provides that
government shall "favourably consider" any request for amendments of this agreement necessary for the
contractor to successfully obtain financing. There is no renunciation of control, as the proviso does not
say that government shall automatically grant any such request. Also, it is up to the contractor to prove
the need for the requested changes. The government always has the final say on whether to approve or
disapprove such requests.
In fine, the FTAA provisions do not reduce or abdicate State control.
No Surrender of Financial Benefits
The second paragraph of Section 81 of RA 7942 has been denounced for allegedly limiting the State's
share in FTAAs with foreign contractors to just taxes, fees and duties, and depriving the State of a share
in the after-tax income of the enterprise. However, the inclusion of the phrase "among other things" in
the second paragraph of Section 81 clearly and unmistakably reveals the legislative intent to have the
State collect more than just the usual taxes, duties and fees.
Thus, DAO 99-56, the "Guidelines Establishing the Fiscal Regime of Financial or Technical Assistance
Agreements," spells out the financial benefits government will receive from an FTAA, as consisting of not
only a basic government share, comprised of all direct taxes, fees and royalties, as well as other
payments made by the contractor during the term of the FTAA, but also an additional government share,
being a share in the earnings or cash flows of the mining enterprise, so as to achieve a fifty-fifty sharing
of net benefits from mining between the government and the contractor.
The additional government share is computed using one of three (3) options or schemes detailed in DAO
99-56, viz., (1) the fifty-fifty sharing of cumulative present value of cash flows; (2) the excess profit-
related additional government share; and (3) the additional sharing based on the cumulative net mining
revenue. Whichever option or computation is used, the additional government share has nothing to do
with taxes, duties, fees or charges. The portion of revenues remaining after the deduction of the basic
and additional government shares is what goes to the contractor.
The basic government share and the additional government share do not yet take into account the
indirect taxes and other financial contributions of mining projects, which are real and actual benefits
enjoyed by the Filipino people; if these are taken into account, total government share increases to 60
percent or higher (as much as 77 percent, and 89 percent in one instance) of the net present value of
total benefits from the project.
The third or last paragraph of Section 81 of RA 7942 is slammed for deferring the payment of the
government share in FTAAs until after the contractor shall have recovered its pre-operating expenses,
exploration and development expenditures. Allegedly, the collection of the State's share is rendered
uncertain, as there is no time limit in RA 7942 for this grace period or recovery period. But although RA
7942 did not limit the grace period, the concerned agencies (DENR and MGB) in formulating the 1995
and 1996 Implementing Rules and Regulations provided that the period of recovery, reckoned from the
date of commercial operation, shall be for a period not exceeding five years, or until the date of actual
recovery, whichever comes earlier.
And since RA 7942 allegedly does not require government approval for the pre-operating, exploration and
development expenses of the foreign contractors, it is feared that such expenses could be bloated to
wipe out mining revenues anticipated for 10 years, with the result that the State's share is zero for the
first 10 years. However, the argument is based on incorrect information.
Under Section 23 of RA 7942, the applicant for exploration permit is required to submit a proposed work
program for exploration, containing a yearly budget of proposed expenditures, which the State passes
upon and either approves or rejects; if approved, the same will subsequently be recorded as pre-
operating expenses that the contractor will have to recoup over the grace period.
Under Section 24, when an exploration permittee files with the MGB a declaration of mining project
feasibility, it must submit a work program for development, with corresponding budget, for approval by
the Bureau, before government may grant an FTAA or MPSA or other mineral agreements; again,
government has the opportunity to approve or reject the proposed work program and budgeted
expenditures for development works, which will become the pre-operating and development costs that
will have to be recovered. Government is able to know ahead of time the amounts of pre-operating and
other expenses to be recovered, and the approximate period of time needed therefor. The aforecited
provisions have counterparts in Section 35, which deals with the terms and conditions exclusively
applicable to FTAAs. In sum, the third or last paragraph of Section 81 of RA 7942 cannot be deemed
defective.
Section 80 of RA 7942 allegedly limits the State's share in a mineral production-sharing agreement
(MPSA) to just the excise tax on the mineral product, i.e., only 2 percent of market value of the minerals.
The colatilla in Section 84 reiterates the same limitation in Section 80. However, these two provisions
pertain only to MPSAs, and have no application to FTAAs. These particular provisions do not come within
the issues defined by this Court. Hence, on due process grounds, no pronouncement can be made in this
case in respect of the constitutionality of Sections 80 and 84.


Section 112 is disparaged for reverting FTAAs and all mineral agreements to the old "license, concession
or lease" system, because it allegedly effectively reduces the government share in FTAAs to just the 2
percent excise tax which pursuant to Section 80 comprises the government share in MPSAs. However,
Section 112 likewise does not come within the issues delineated by this Court, and was never touched
upon by the parties in their pleadings. Moreover, Section 112 may not properly apply to FTAAs. The
mining law obviously meant to treat FTAAs as a breed apart from mineral agreements. There is
absolutely no basis to believe that the law intends to exact from FTAA contractors merely the same
government share (i.e., the 2 percent excise tax) that it apparently demands from contractors under the
three forms of mineral agreements.
While there is ground to believe that Sections 80, 84 and 112 are indeed unconstitutional, they cannot be
ruled upon here. In any event, they are separable; thus, a later finding of nullity will not affect the rest of
RA 7942. TDSICH
In fine, the challenged provisions of RA 7942 cannot be said to surrender financial benefits from an FTAA
to the foreign contractors.
Moreover, there is no concrete basis for the view that, in FTAAs with a foreign contractor, the State must
receive at least 60 percent of the after-tax income from the exploitation of its mineral resources, and that
such share is the equivalent of the constitutional requirement that at least 60 percent of the capital, and
hence 60 percent of the income, of mining companies should remain in Filipino hands. Even if the State is
entitled to a 60 percent share from other mineral agreements (CPA, JVA and MPSA), that would not
create a parallel or analogous situation for FTAAs. We are dealing with an essentially different equation.
Here we have the old apples and oranges syndrome.
85
The Charter did not intend to fix an iron-clad rule of 60 percent share, applicable to all situations,
regardless of circumstances. There is no indication of such an intention on the part of the framers.
Moreover, the terms and conditions of petroleum FTAAs cannot serve as standards for mineral mining
FTAAs, because the technical and operational requirements, cost structures and investment needs of off-
shore petroleum exploration and drilling companies do not have the remotest resemblance to those of
on-shore mining companies.
To take the position that government's share must be not less than 60 percent of after-tax income of
FTAA contractors is nothing short of this Court dictating upon the government. The State resultantly ends
up losing control. To avoid compromising the State's full control and supervision over the exploitation of
mineral resources, there must be no attempt to impose a "minimum 60 percent" rule. It is sufficient that
the State has the power and means, should it so decide, to get a 60 percent share (or greater); and it is
not necessary that the State does so in every case.
Invalid Provisions of the WMCP FTAA
Section 7.9 of the WMCP FTAA clearly renders illusory the State's 60 percent share of WMCP's revenues.
Under Section 7.9, should WMCP's foreign stockholders (who originally owned 100 percent of the equity)
sell 60 percent or more of their equity to a Filipino citizen or corporation, the State loses its right to
receive its share in net mining revenues under Section 7.7, without any offsetting compensation to the
State. And what is given to the State in Section 7.7 is by mere tolerance of WMCP's foreign stockholders,
who can at any time cut off the government's entire share by simply selling 60 percent of WMCP's equity
to a Philippine citizen or corporation.
In fact, the sale by WMCP's foreign stockholder on January 23, 2001 of the entire outstanding equity in
WMCP to Sagittarius Mines, Inc., a domestic corporation at least 60 percent Filipino owned, can be
deemed to have automatically triggered the operation of Section 7.9 and removed the State's right to
receive its 60 percent share. Section 7.9 of the WMCP FTAA has effectively given away the State's share
without anything in exchange.
Moreover, it constitutes unjust enrichment on the part of the local and foreign stockholders in WMCP,
because by the mere act of divestment, the local and foreign stockholders get a windfall, as their share in
the net mining revenues of WMCP is automatically increased, without having to pay anything for it.
Being grossly disadvantageous to government and detrimental to the Filipino people, as well as violative
of public policy, Section 7.9 must therefore be stricken off as invalid. The FTAA in question does not
involve mere contractual rights, but, being impressed as it is with public interest, the contractual
provisions and stipulations must yield to the common good and the national interest. Since the offending
provision is very much separable from the rest of the FTAA, the deletion of Section 7.9 can be done
without affecting or requiring the invalidation of the entire WMCP FTAA itself.
Section 7.8(e) of the WMCP FTAA likewise is invalid, since by allowing the sums spent by government for
the benefit of the contractor to be deductible from the State's share in net mining revenues, it results in
benefiting the contractor twice over. This constitutes unjust enrichment on the part of the contractor, at
the expense of government. For being grossly disadvantageous and prejudicial to government and
contrary to public policy, Section 7.8(e) must also be declared without effect. It may likewise be stricken
off without affecting the rest of the FTAA.
Epilogue
AFTER ALL IS SAID AND DONE, it is clear that there is unanimous agreement in the Court upon the key
principle that the State must exercise full control and supervision over the exploration, development and
utilization of mineral resources.
The crux of the controversy is the amount of discretion to be accorded the Executive Department,
particularly the President of the Republic, in respect of negotiations over the terms of FTAAs, particularly
when it comes to the government share of financial benefits from FTAAs. The Court believes that it is not
unconstitutional to allow a wide degree of discretion to the Chief Executive, given the nature and
complexity of such agreements, the humongous amounts of capital and financing required for large-scale
mining operations, the complicated technology needed, and the intricacies of international trade, coupled
with the State's need to maintain flexibility in its dealings, in order to preserve and enhance our country's
competitiveness in world markets.
We are all, in one way or another, sorely affected by the recently reported scandals involving corruption
in high places, duplicity in the negotiation of multi-billion peso government contracts, huge payoffs to
government officials, and other malfeasances; and perhaps, there is the desire to see some measures put
in place to prevent further abuse. However, dictating upon the President what minimum share to get
from an FTAA is not the solution. It sets a bad precedent since such a move institutionalizes the very
reduction if not deprivation of the State's control. The remedy may be worse than the problem it was
meant to address. In any event, provisions in such future agreements which may be suspected to be
grossly disadvantageous or detrimental to government may be challenged in court, and the culprits haled
before the bar of justice.
Verily, under the doctrine of separation of powers and due respect for co-equal and coordinate branches
of government, this Court must restrain itself from intruding into policy matters and must allow the
President and Congress maximum discretion in using the resources of our country and in securing the
assistance of foreign groups to eradicate the grinding poverty of our people and answer their cry for
viable employment opportunities in the country.
"The judiciary is loath to interfere with the due exercise by coequal branches of government of their
official functions." 99 As aptly spelled out seven decades ago by Justice George Malcolm, "Just as the
Supreme Court, as the guardian of constitutional rights, should not sanction usurpations by any other
department of government, so should it as strictly confine its own sphere of influence to the powers
expressly or by implication conferred on it by the Organic Act." 100 Let the development of the mining
industry be the responsibility of the political branches of government. And let not this Court interfere
inordinately and unnecessarily.
The Constitution of the Philippines is the supreme law of the land. It is the repository of all the
aspirations and hopes of all the people. We fully sympathize with the plight of Petitioner La Bugal B'laan
and other tribal groups, and commend their efforts to uplift their communities. However, we cannot
justify the invalidation of an otherwise constitutional statute along with its implementing rules, or the
nullification of an otherwise legal and binding FTAA contract.
We must never forget that it is not only our less privileged brethren in tribal and cultural communities
who deserve the attention of this Court; rather, all parties concerned including the State itself, the
contractor (whether Filipino or foreign), and the vast majority of our citizens equally deserve the
protection of the law and of this Court. To stress, the benefits to be derived by the State from mining
activities must ultimately serve the great majority of our fellow citizens. They have as much right and
interest in the proper and well-ordered development and utilization of the country's mineral resources as
the petitioners.
Whether we consider the near term or take the longer view, we cannot overemphasize the need for an
appropriate balancing of interests and needs the need to develop our stagnating mining industry and
extract what NEDA Secretary Romulo Neri estimates is some US$840 billion (approx. PhP47.04 trillion)
worth of mineral wealth lying hidden in the ground, in order to jumpstart our floundering economy on the
one hand, and on the other, the need to enhance our nationalistic aspirations, protect our indigenous
communities, and prevent irreversible ecological damage.

This Court cannot but be mindful that any decision rendered in this case will ultimately impact not only
the cultural communities which lodged the instant Petition, and not only the larger community of the
Filipino people now struggling to survive amidst a fiscal/budgetary deficit, ever increasing prices of fuel,
food, and essential commodities and services, the shrinking value of the local currency, and a
government hamstrung in its delivery of basic services by a severe lack of resources, but also countless
future generations of Filipinos.
For this latter group of Filipinos yet to be born, their eventual access to education, health care and basic
services, their overall level of well-being, the very shape of their lives are even now being determined
86
and affected partly by the policies and directions being adopted and implemented by government today.
And in part by this Resolution rendered by this Court today.
Verily, the mineral wealth and natural resources of this country are meant to benefit not merely a select
group of people living in the areas locally affected by mining activities, but the entire Filipino nation,
present and future, to whom the mineral wealth really belong. This Court has therefore weighed carefully
the rights and interests of all concerned, and decided for the greater good of the greatest number.
JUSTICE FOR ALL, not just for some; JUSTICE FOR THE PRESENT AND THE FUTURE, not just for the
here and now.
WHEREFORE, the Court RESOLVES to GRANT the respondents' and the intervenors' Motions for
Reconsideration; to REVERSE and SET ASIDE this Court's January 27, 2004 Decision; to DISMISS the
Petition; and to issue this new judgment declaring CONSTITUTIONAL (1) Republic Act No. 7942 (the
Philippine Mining Law), (2) its Implementing Rules and Regulations contained in DENR Administrative
Order (DAO) No. 9640 insofar as they relate to financial and technical assistance agreements referred
to in paragraph 4 of Section 2 of Article XII of the Constitution; and (3) the Financial and Technical
Assistance Agreement (FTAA) dated March 30, 1995 executed by the government and Western Mining
Corporation Philippines Inc. (WMCP), except Sections 7.8 and 7.9 of the subject FTAA which are hereby
INVALIDATED for being contrary to public policy and for being grossly disadvantageous to the
government.
SO ORDERED.























G.R. No. L-43938. April 15, 1988.
REPUBLIC OF THE PHILIPPINES (DIRECTOR OF FOREST DEVELOPMENT),
petitioner, vs. HON. COURT OF APPEALS (THIRD DIVISION) and JOSE Y. DE LA
ROSA, respondents.
G.R. No. L-44081. April 15, 1988.
BENGUET CONSOLIDATED, INC., petitioner, vs. HON. COURT OF APPEALS, JOSE Y.
DE LA ROSA, VICTORIA, BENJAMIN and EDUARDO, all surnamed DE LA ROSA,
represented by their father JOSE Y. DE LA ROSA, respondents.
G.R. No. L-44092. April 15, 1988.
ATOK-BIG WEDGE MINING COMPANY, petitioner, vs. HON. COURT OF APPEALS,
JOSE Y. DE LA ROSA, VICTORIA, BENJAMIN and EDUARDO, all surnamed DE LA
ROSA, represented by their father, JOSE Y. DE LA ROSA, respondents.
D E C I S I O N
CRUZ, J p:
The Regalian doctrine reserves to the State all natural wealth that may be found in the bowels of the
earth even if the land where the discovery is made be private. 1 In the cases at bar, which have been
consolidated because they pose a common issue, this doctrine was not correctly applied.
These cases arose from the application for registration of a parcel of land filed on February 11, 1965, by
Jose de la Rosa on his own behalf and on behalf of his three children, Victoria, Benjamin and Eduardo.
The land, situated in Tuding, Itogon, Benguet Province, was divided into 9 lots and covered by plan Psu-
225009. According to the application, Lots 1-5 were sold to Jose de la Rosa and Lots 6-9 to his children
by Mamaya Balbalio and Jaime Alberto, respectively, in 1964. 2
The application was separately opposed by Benguet Consolidated, Inc. as to Lots 1-5, Atok Big Wedge
Corporation, as to portions of Lots 1-5 and all of Lots 6-9, and by the Republic of the Philippines, through
the Bureau of Forestry Development, as to Lots 1-9. 3
In support of the application, both Balbalio and Alberto testified that they had acquired the subject land
by virtue of prescription. Balbalio claimed to have received Lots 1-5 from her father shortly after the
Liberation. She testified she was born in the land, which was possessed by her parents under claim of
87
ownership. 4 Alberto said he received Lots 6-9 in 1961 from his mother, Bella Alberto, who declared that
the land was planted by Jaime and his predecessors-in-interest to bananas, avocado, nangka and
camote, and was enclosed with a barbed-wire fence. She was corroborated by Felix Marcos, 67 years old
at the time, who recalled the earlier possession of the land by Alberto's father. 5 Balbalio presented her
tax declaration in 1956 and the realty tax receipts from that year to 1964, 6 Alberto his tax declaration in
1961 and the realty tax receipts from that year to 1964. 7
Benguet opposed on the ground that the June Bug mineral claim covering Lots 1-5 was sold to it on
September 22, 1934, by the successors-in-interest of James Kelly, who located the claim in September
1909 and recorded it on October 14, 1909. From the date of its purchase, Benguet had been in actual,
continuous and exclusive possession of the land in concept of owner, as evidenced by its construction of
adits, its affidavits of annual assessment, its geological mappings, geological samplings and trench side
cuts, and its payment of taxes on the land. 8
For its part, Atok alleged that a portion of Lots 1-5 and all of Lots 6-9 were covered by the Emma and
Fredia mineral claims located by Harrison and Reynolds on December 25, 1930, and recorded on January
2, 1931, in the office of the mining recorder of Baguio. These claims were purchased from these locators
on November 2, 1931, by Atok, which has since then been in open, continuous and exclusive possession
of the said lots as evidenced by its annual assessment work on the claims, such as the boring of tunnels,
and its payment of annual taxes thereon. 9
The location of the mineral claims was made in accordance with Section 21 of the Philippine Bill of 1902
which provided that:
"SEC. 21.All valuable mineral deposits in public lands in the Philippine
Islands both surveyed and unsurveyed are hereby declared to be free and
open to exploration, occupation and purchase and the land in which they
are found to occupation and purchase by the citizens of the United States,
or of said islands."
The Bureau of Forestry Development also interposed its objection, arguing that the land sought to be
registered was covered by the Central Cordillera Forest Reserve under Proclamation No. 217 dated
February 16, 1929. Moreover, by reason of its nature, it was not subject to alienation under the
Constitutions of 1935 and 1973. 10
The trial court * denied the application, holding that the applicants had failed to prove their claim of
possession and ownership of the land sought to be registered. 11 The applicants appealed to the
respondent court, ** which reversed the trial court and recognized the claims of the applicant, but
subject to the rights of Benguet and Atok respecting their mining claims. 12 In other words, the Court of
Appeals affirmed the surface rights of the de la Rosas over the land while at the same time reserving the
sub-surface rights of Benguet and Atok by virtue of their mining claims. cdll
Both Benguet and Atok have appealed to this Court, invoking their superior right of ownership. The
Republic has filed its own petition for review and reiterates its argument that neither the private
respondents nor the two mining companies have any valid claim to the land because it is not alienable
and registerable.
It is true that the subject property was considered forest land and included in the Central Cordillera
Forest Reserve, but this did not impair the rights already vested in Benguet and Atok at that time. The
Court of Appeals correctly declared that:
"There is no question that the 9 lots applied for are within the June Bug
mineral claims of Benguet and the 'Fredia and Emma' mineral claims of
Atok. The June Bug mineral claim of plaintiff Benguet was one of the 16
mining claims of James E. Kelly, an American and mining locator. He filed
his declaration of the location of the June Bug mineral and the same was
recorded in the Mining Recorder's Office on October 14, 1909. All of the
Kelly claims had subsequently been acquired by Benguet Consolidated,
Inc. Benguet's evidence is that it had made improvements on the June Bug
mineral claim consisting of mine tunnels prior to 1935. It had submitted
the required affidavit of annual assessment. After World War II, Benguet
introduced improvements on mineral claim June Bug, and also conducted
geological mappings, geological sampling and trench side cuts. In 1948,
Benguet redeclared the 'June Bug' for taxation and had religiously paid the
taxes.
"The Emma and Fredia claims were two of the several claims of Harrison
registered in 1931, and which Atok representatives acquired. Portions of
Lots 1 to 5 and all of Lots 6 to 9 are within the Emma and Fredia mineral
claims of Atok Big Wedge Mining Company. prcd
"The June Bug mineral claim of Benguet and the Fredia and Emma mineral
claims of Atok having been perfected prior to the approval of the
Constitution of the Philippines of 1935, they were removed from the public
domain and had become private properties of Benguet and Atok.
'It is not disputed that the location of the mining claim under consideration
was perfected prior to November 15, 1935, when the Government of the
Commonwealth was inaugurated; and according to the laws existing at
that time, as construed and applied by this court in McDaniel v. Apacible
and Cuisia (42 Phil. 749), a valid location of a mining claim segregated the
area from the public domain. Said the court in that case: 'The moment the
locator discovered a valuable mineral deposit on the lands located, and
perfected his location in accordance with law, the power of the United
States Government to deprive him of the exclusive right to the possession
and enjoyment of the located claim was gone, the lands had become
mineral lands and they were exempted from lands that could be granted to
any other person. The reservations of public lands cannot be made so as
to include prior mineral perfected locations; and, of course, if a valid
mining location is made upon public lands afterwards included in a
reservation, such inclusion or reservation does not affect the validity of the
former location. By such location and perfection, the land located is
segregated from the public domain even as against the Government.
(Union Oil Co. v. Smith, 249 U.S. 337; Van Mess v. Roonet, 160 Cal. 131;
27 Cyc. 546).

'The legal effect of a valid location of a mining claim is not only to
segregate the area from the public domain, but to grant to the locator the
beneficial ownership of the claim and the right to a patent therefor upon
compliance with the terms and conditions prescribed by law. Where there
is a valid location of a mining claim, the area becomes segregated from
the public domain and the property of the locator.' (St. Louis Mining &
Milling Co. v. Montana Mining Co., 171 U.S. 650; 655; 43 Law ed., 320,
322.) 'When a location of a mining claim is perfected it has the effect of a
grant by the United States of the right of present and exclusive possession,
with the right to the exclusive enjoyment of all the surface ground as well
as of all the minerals within the lines of the claim, except as limited by the
extralateral right of adjoining locators; and this is the locator's right before
as well as after the issuance of the patent. While a lode locator acquires a
vested property right by virtue of his location made in compliance with the
mining laws, the fee remains in the government until patent issues.' (18
R.C.L. 1152)' (Gold Creek Mining Corporation v. Hon. Eulogio Rodriguez,
Sec. of Agriculture and Commerce, and Quirico Abadilla, Director of the
Bureau of Mines, 66 Phil. 259, 265-266). cdll
"It is of no importance whether Benguet and Atok had secured a patent for
as held in the Gold Creek Mining Corp. Case, for all physical purposes of
88
ownership, the owner is not required to secure a patent as long as he
complies with the provisions of the mining laws; his possessory right, for
all practical purposes of ownership, is as good as though secured by
patent.
"We agree likewise with the oppositors that having complied with all the
requirements of the mining laws, the claims were removed from the public
domain, and not even the government of the Philippines can take away
this right from them. The reason is obvious. Having become the private
properties of the oppositors, they cannot be deprived thereof without due
process of law." 13
Such rights were not affected either by the stricture in the Commonwealth Constitution against the
alienation of all lands of the public domain except those agricultural in nature for this was made subject
to existing rights. Thus, in its Article XIII, Section 1, it was categorically provided that:
"SEC. 1.All agricultural, timber and mineral lands of the public domain,
waters, minerals, coal, petroleum and other mineral oils, all forces of
potential energy and other natural resources of the Philippines belong to
the State, and their disposition, exploitation, development, or utilization
shall be limited to citizens of the Philippines or to corporations or
associations at least 60% of the capital of which is owned by such citizens,
subject to any existing right, grant, lease or concession at the time of the
inauguration of the government established under this Constitution.
Natural resources with the exception of public agricultural lands, shall not
be alienated, and no license, concession, or lease for the exploitation,
development or utilization of any of the natural resources shall be granted
for a period exceeding 25 years, except as to water rights for irrigation,
water supply, fisheries, or industrial uses other than the development of
water power, in which case beneficial use may be the measure and the
limit of the grant."
Implementing this provision, Act No. 4268, approved on November 8, 1935, declared:
"Any provision of existing laws, executive order, proclamation to the
contrary notwithstanding, all locations of mining claim made prior to
February 8, 1935 within lands set apart as forest reserve under Sec. 1826
of the Revised Administrative Code which would be valid and subsisting
location except to the existence of said reserve are hereby declared to be
valid and subsisting locations as of the date of their respective locations."
The perfection of the mining claim converted the property to mineral land and under the laws then in
force removed it from the public domain. 14 By such act, the locators acquired exclusive rights over the
land, against even the government, without need of any further act such as the purchase of the land or
the obtention of a patent over it. 15 As the land had become the private property of the locators, they
had the right to transfer the same, as they did, to Benguet and Atok.
It is true, as the Court of Appeals observed, that such private property was subject to the "vicissitudes of
ownership," or even to forfeiture by non-user or abandonment or, as the private respondents aver, by
acquisitive prescription. However, the method invoked by the de la Rosas is not available in the case at
bar, for two reasons.
First, the trial court found that the evidence of open, continuous, adverse and exclusive possession
submitted by the applicants was insufficient to support their claim of ownership. They themselves had
acquired the land only in 1964 and applied for its registration in 1965, relying on the earlier alleged
possession of their predecessors-in-interest. 16 The trial judge, who had the opportunity to consider the
evidence first-hand and observe the demeanor of the witnesses and test their credibility was not
convinced. We defer to his judgment in the absence of a showing that it was reached with grave abuse
of discretion or without sufficient basis. 17
Second, even if it be assumed that the predecessors-in-interest of the de la Rosas had really been in
possession of the subject property, their possession was not in the concept of owner of the mining claim
but of the property as agricultural land, which it was not. The property was mineral land, and they were
claiming it as agricultural land. They were not disputing the rights of the mining locators nor were they
seeking to oust them as such and to replace them in the mining of the land. In fact, Balbalio testified that
she was aware of the diggings being undertaken "down below" 18 but she did not mind, much less
protest, the same although she claimed to be the owner of the said land.
The Court of Appeals justified this by saying there is "no conflict of interest" between the owners of the
surface rights and the owners of the sub-surface rights. This is rather strange doctrine, for it is a well-
known principle that the owner of a piece of land has rights not only to its surface but also to everything
underneath and the airspace above it up to a reasonable height. 19 Under the aforesaid ruling, the land
is classified as mineral underneath and agricultural on the surface, subject to separate claims of title. This
is also difficult to understand, especially in its practical application. cdll
Under the theory of the respondent court, the surface owner will be planting on the land while the mining
locator will be boring tunnels underneath. The farmer cannot dig a well because he may interfere with
the mining operations below and the miner cannot blast a tunnel lest he destroy the crops above. How
deep can the farmer, and how high can the miner, go without encroaching on each other's rights? Where
is the dividing line between the surface and the sub-surface rights?
The Court feels that the rights over the land are indivisible and that the land itself cannot be half
agricultural and half mineral. The classification must be categorical; the land must be either completely
mineral or completely agricultural. In the instant case, as already observed, the land which was originally
classified as forest land ceased to be so and became mineral and completely mineral once the
mining claims were perfected. 20 As long as mining operations were being undertaken thereon, or
underneath, it did not cease to be so and become agricultural, even if only partly so, because it was
enclosed with a fence and was cultivated by those who were unlawfully occupying the surface.
What must have misled the respondent court is Commonwealth Act No. 137, providing as follows:
"Sec. 3.All mineral lands of the public domain and minerals belong to the
State, and their disposition, exploitation, development or utilization, shall
be limited to citizens of the Philippines, or to corporations, or associations,
at least 60% of the capital of which is owned by such citizens, subject to
any existing right, grant, lease or concession at the time of the
inauguration of government established under the Constitution."
"SEC. 4.The ownership of, and the right to the use of land for agricultural,
industrial, commercial, residential, or for any purpose other than mining
does not include the ownership of, nor the right to extract or utilize, the
minerals which may be found on or under the surface."
"SEC. 5.The ownership of, and the right to extract and utilize, the minerals
included within all areas for which public agricultural land patents are
granted are excluded and excepted from all such patents."
"SEC. 6.The ownership of, and the right to extract and utilize, the minerals
included within all areas for which Torrens titles are granted are excluded
and excepted from all such titles."
This is an application of the Regalian doctrine which, as its name implies, is intended for the benefit of
the State, not of private persons. The rule simply reserves to the State all minerals that may be found in
public and even private land devoted to "agricultural, industrial, commercial, residential or (for) any
purpose other than mining." Thus, if a person is the owner of agricultural land in which minerals are
discovered, his ownership of such land does not give him the right to extract or utilize the said minerals
without the permission of the State to which such minerals belong.
The flaw in the reasoning of the respondent court is in supposing that the rights over the land could be
89
used for both mining and non-mining purposes simultaneously. The correct interpretation is that once
minerals are discovered in the land, whatever the use to which it is being devoted at the time, such use
may be discontinued by the State to enable it to extract the minerals therein in the exercise of its
sovereign prerogative. The land is thus converted to mineral land and may not be used by any private
party, including the registered owner thereof, for any other purpose that will impede the mining
operations to be undertaken therein. For the loss sustained by such owner, he is of course entitled to just
compensation under the Mining Laws or in appropriate expropriation proceedings. 21

Our holding is that Benguet and Atok have exclusive rights to the property in question by virtue of their
respective mining claims which they validly acquired before the Constitution of 1935 prohibited the
alienation of all lands of the public domain except agricultural lands, subject to vested rights existing at
the time of its adoption. The land was not and could not have been transferred to the private
respondents by virtue of acquisitive prescription, nor could its use be shared simultaneously by them and
the mining companies for agricultural and mineral purposes.
WHEREFORE, the decision of the respondent court dated April 30, 1976, is SET ASIDE and that of the
trial court dated March 11, 1969, is REINSTATED, without any pronouncement as to costs. LibLex
SO ORDERED.

You might also like